Pulmonology - PANCE Prep Pearls, Key Word Associations, PACKRAT Questions
*Acute Bronchitis* Tx
Supportive: Fluids, rest, cough suppressants, bronchodilators. Abx only if immunocompromised or conservative management failed.
Apical infiltrates, F, C, dry cough
TB
A contralateral mediastinal shift should make you think of what diagnosis?
Tension pneumothorax
*Asbestosis* Increased risk of (2)
1. Bronchogenic carcinoma (MC) 2. Malignant mesothelioma of the pleura (rare but specific)
*Croup* (Laryngotracheitis) Treatment of mild croup (no stridor or respiratory distress at rest) 1. Supportive (2) 2. Medical
1. Cool humidified air, hydration 2. *Dexamethasone*
Initial treatment for all pneumothorax patients?
100% oxygen
*Pulmonary HTN* Mean pulmonary arterial pressure over
25 mmHg
How high does pulmonary pressure need to be to diagnose pulmonary HTN?
25 mmHg at rest
Question: What is the gold standard diagnostic test for spontaneous pneumomediastinum?
The CT scan. Pneumomediastinum Spontaneous, trauma, Valsalva (crack users), Boerhaave syndrome (hx of vomiting) PE: crepitus, Hamman sign CXR: ring around right pulmonary artery Usually self-resolving
When performing a decompression for for a tension pneumothorax where do you put the needle?
2nd intercostal space at the midclavicular line
Zanamivir (Relenza) may be given to children above what age?
7 years old
Mesothelioma is most commonly found in what lung location?
80% are found in the pleural lining
Question: What is the blood pressure goal for patients with diabetes mellitus type 2?
< 140/90.
How many millimeters of induration is considered positive when reading a PPD result in an HIV positive patient?
> 5 mm
Gynecomastia is MC with what type of lung CA?
Adenocarcinoma
*Acute Laryngitis* Treatment
Air humidification and voice rest (predominantly viral)
Question: What type of precautions are necessary for a patient with active/suspected tuberculosis?
Airborne precautions.
First line agent of choice for acute asthma attack?
Albuterol
What medication must all asthma patients have regardless of the severity of their disease?
All asthma patients must have a short acting beta agonist (albuterol) as a rescue medication
What disorder should be considered in a young patient (20-30 years old) with severe emphysema?
Alpha 1 antitrypsin deficiency
Treatment for Histoplasma pneumonia?
Amphotericin B
When you step up the diagnosis from intermittent asthma to mild persistent asthma what medication should you add?
An inhaled steroid like fluticasone
One Step Further Question: True or false: Pneumocystis jiroveci is classified as a fungus based on ribosomal DNA and cell wall composition
Answer: True.
Insulation, ship building
Asbestosis
Ground glass appearance on CXR
Asbestosis (shipbuilders, building demolition)
Airway edema with eosinophils, neutrophils, lymphocytes
Asthma
Curschmann's spirals can be found in sputum
Asthma
Hyperinflation on a CXR as well as eosinophilia on CBC should make you think of what diagnosis?
Asthma
Paradoxical pulses may be found in what pulmonary disorder?
Asthma
Wheezing, reversible airway disorder, Samter's triad: Asthma, ASA allergy, nasal polyp. Reduced FEV1 to FVC ratio.
Asthma
Wheezing, reversible airway disorder. Samter's triad: Asthma, ASA allergy, nasal polyp. Reduced FEV1 to FVC ratio
Asthma
Where is a Pancoast tumor found on CXR?
At the apex of either the right or left lung
Question: Name two steroid-sparing alternative medications used for pulmonary sarcoidosis.
Azathioprine and methotrexate. Sarcoidosis African-Americans, females Primary target organ: lungs Parotid enlargement Hypercalcemia CXR: bilateral hilar adenopathy Biopsy: noncaseating granulomas Steroids
Aerospace, Electrical Plants
Beryllium
Question: In which population is Klebsiella pneumonia most commonly seen in?
COPD, Alcoholics and the elderly.
List three causes of decreased tactile fremitus.
COPD, asthma, pleural effusion, pneumothorax
What is the best test to diagnose bronchiesctasis?
CT
Tx of small cell lung Ca
Chemo +/- radiation
Deep breathing alternating w/ apnea
Cheyne-Stokes breathing - heart failure, brain damage
*PNA* If around poultry, pet shops
Chlamydia psittaci
Blue bloater refers to the characteristic physical exam findings of what disease?
Chronic bronchitis
*Asthma* Drugs not commonly used anymore d/t narrow tharapeutic index
Theophylline. Non-selective adenosine antagonist; may affect heart.
76-year-old woman with a history of chronic obstructive pulmonary disease presents to your office in December with complaints of fever, muscle aches, headache and malaise that started yesterday. Which of the following is the most appropriate next step in management? Advise supportive care measures including acetaminophen Begin a course of azithromycin Begin a course of oseltamivir Begin a course of prednisone
Correct Answer ( C ) Explanation: Seasonal influenza is caused by influenza A or B viruses and results in acute respiratory illness. Clinical presentation includes fever, myalgia, headache and malaise and high-risk patients can experience serious complications including death. Antiviral therapy can shorten the duration of illness and viral shedding and decrease the severity of symptoms. Patients with symptoms that began 24-48 hours prior to presentation should be evaluated for being at high risk of complications and those at high risk should start a course of oseltamivir. Groups considered to be at high risk include adults 65 years and older, children 2 years and younger, immunocompromised patients, pregnant women, residents of nursing homes or chronic care facilities, and individuals with certain chronic illnesses including chronic pulmonary disease.
Question: Carcinoid tumors are most likely to develop in which other organ system besides the pulmonary system?
These tumors tend to develop in the gastrointestinal tract and produce similar symptoms of carcinoid syndrome.
What is the first line antibiotic for epiglottis?
Third generation cephalosporin - ceftriaxone
Which of the following is the initial treatment of choice in an overweight patient with moderate obstructive sleep apnea? Albuterol nebulizer 30 minutes prior to sleep Oral appliances Tracheostomy Weight loss and continuous positive airway pressure
Correct Answer ( D ) Explanation: Weight loss and continue positive airway pressure remains the most proven and effective therapy for obstructive sleep apnea. The continuous positive airway pressure acts as a pneumatic splint, holding the airway open. In addition, even a small reduction in body weight is associated with clinically significant improvements, and weight loss alone may be curative in some patients. Other lifestyle or conservative measures may include curbing alcohol intake before bedtime and avoiding a supine posture if obstructive sleep apnea is position dependent
Question: What is the preferred form of management in patients with sarcoidosis?
Corticosteroids (eg, prednisone).
Barking cough. X-ray: Steeple sign
Croup
Child has barking cough and stridor?
Croup
Question: What is the most common complication of pneumonia in children?
Dehydration.
What CXR finding might make you think of berylliosis?
Diffuse infiltrates with hilar adenopathy
*Pulmonary HTN* Treatment (4)
Diuretics Sildenafil NO Ambrisentan
*Cor Pulmonale* Medical treatment (2)
Diuretics Vasodilators
What is it called when a patient says "ee" but on auscultation you hear "ay"?
Egophony
*COPD* Weight loss is more indicative of which type?
Emphysema
A barrel chest should make you think of what diagnosis?
Emphysema
Hyperinflation on CXR, tear drop heart
Emphysema
Pink Puffer?
Emphysema
Smoking, alpha 1 antitrypsin deficiency, barrel chest, decrease DLCO and FEV1.
Emphysema
Pink Puffer?
Emphysema - Consequence of destruction of alveolar septae = Pink puffers - The body's natural response to ↓ lung function is chronic hyperventilation
Hyperresonance to percussion should make you think of what diagnosis?
Emphysema, pneumothorax, asthma
Thumbprint sign
Epiglottitis
Clinical Intervention/Pulmonology A 21 year-old male presents to the ED with increasing dyspnea and pleuritic chest pain of sudden onset after getting hit in the left side of the chest during a bar fight. Examination reveals moderate respiratory distress with absence of breath sounds and hyperresonance to percussion on the left, with tracheal deviation to the right. Which of the following is the most appropriate next step? Answers A. order a V/Q scan B. order a chest x-ray C. administer a sclerosing agent D. insert large bore needle into left 2nd ICS stat
Explanations (h) A. A V/Q scan is indicated in suspected cases of pulmonary embolism. (u) B. Patients in respiratory distress and evidence of a tension pneumothorax, such as tracheal deviation, should have treatment initiated without waiting on a chest x-ray to be taken. (h) C. Pleurodesis by administration of a sclerosing agent is indicated in treatment of recurrent, not traumatic, pneumothorax. (c) D. Simple aspiration by insertion of a needle into the involved side will decompress the tension pneumothorax until a chest tube can be inserted.
Diagnosis/Pulmonology Which histologic type of lung cancer is typically centrally located? A. Adenocarcinoma B. Bronchoalveolar C. Large cell D. Squamous cell
Explanations (u) A. Adenocarcinoma of the lung typically presents as a peripheral lesion. (u) B. Bronchoalveolar carcinoma, actually a subset of adenocarcinoma of the lung, typically presents as a peripheral lesion. (u) C. Large cell lung cancers usually develop as peripheral lesions. (c) D. Most squamous cell lung cancers are centrally located.
Diagnosis/Pulmonology A 15 year-old male presents with a 1 week history of hacking non-productive cough, low grade fever, malaise and myalgias. Examination is unremarkable except for a few scattered rhonchi and rales upon auscultation of the chest. The chest x-ray reveals interstitial infiltrates and a cold agglutinin titer was negative. Which of the following is the most likely diagnosis? Answers A. acute bronchitis B. viral pneumonia C. mycoplasma pneumonia D. pneumococcal pneumonia
Explanations (u) A. While the patient's clinical symptoms of dry cough and rhonchi support this diagnosis, the chest x-ray would be normal or only show a mild increase in bronchovascular markings, not infiltrates. (c) B. The patient's clinical symptoms as well as chest x-ray findings and negative cold agglutinin titer are most consistent with viral pneumonia. (a) C. While the gradual onset of symptoms suggest mycoplasma, the negative cold agglutinin titer makes this less likely. (u) D. In older children the signs and symptoms of pneumococcal pneumonia are similar to an adult and consist of an abrupt onset of cough, fever and chills. The chest x-ray would reveal a lobar consolidation, not interstitial, picture.
*Pleural Effusion* Type cause by collagen vascular disease (RA)?
Exudative
*Pleural Effusion* Type cause by pancreatitis?
Exudative
In a patient with malignancy is the pleural effusion transudative or exudative?
Exudative
*Pleural Effusion* Malignancy, pulmonary embolism, infection, trauma/blood cause which type?
Exudative; result form an increase in vascular permeability
Inspiratory stridor
FB, viral croup (laryngotracheobronchitis)
*CF* Definitive test if sweat chloride test negative
Genetic testing
What is the most common organism causing pneumonia in COPD patients?
Haemophilus influenza
Cavitations on CXR
Infections - lung abscess, TB (Gohn focus)
*PNA* MC viral cause in adults
Influenza
*Asthma* Good sedating agent for intubation because it is also a bronchodilator
Ketamine
*PNA* Dx of Legionella
Legionella urine antigen
*Pneumoconioses* *Berylliosis* Treatment
Like the other pneumoconioses: supportive treatment with *corticosteroids*, bronchodilators, O2, vaccinations (influenza, pneumococcal). +/- *MTX* if steroids are unsuccessful
What is a bloody pleural effusion concerning for?
Malignancy
*Sarcoidosis* Can be used if steroids fail to Tx
Methotrexate
*COPD* Finding on EKG that is more specific for COPD'ers
Multifocal atrial tachcyardia
*Asthma* What is Samter's triad?
Nasal polyps, asthma, aspirin sensitivity
*TB* INH S/E? Tx?
Neuropathy; pyridoxine (vitamin B6) supplementation
*Pneumoconioses* *Silicosis* Treatment
None; supportive with bronchodilators, O2, vaccinations (influenza, pneumococcal), +/- steroids
*Pneumoconioses* *Asbestosis* Treatment?
None; supportive.
*Hypoventilation syndrome* 90% have co-existing:
OSA
Ethambutol side effect?
Optic neuritis
*COPD* The only medication therapy shown to dec mortality
Oxygen
Only medication that improves morbidity and mortality in COPD?
Oxygen
Virus that causes croup?
Parainfluenza virus type 1
Considered the *gold standard* for the diagnosis of Asthma
Peak expiratory flow rate
Dullness to percussion, decreased breath sounds, decreased tactile fremitus
Pleural effusion
Smoking, family history, males tall/skinny, pleuritic chest pain
Pneumothorax
*Bronchiectasis* MC etiology if d/t CF?
Pseudomonas
What does a positive whisper pectoriloquy represent?
Pulmonary consolidation
Should normal percussion over the lung fields sound dull or resonant?
Resonant
*PE* ABG will show
Respiratory alkalosis and hypoxia
*Pulmonary HTN* Dx
Right heart cath
*PNA* MCC CAP
S. Pneumoniae
A 30 year old female presents with a dry cough. Her chest X-ray shows bilateral hilar adenopathy with non-caseating granulomas. What is your suspected diagnosis?
Sarcoidosis
A noncaseating granuloma should make you think of what diagnosis?
Sarcoidosis
Bilateral hilar adenopathy?
Sarcoidosis
Class 5 pulmonary HTN cause?
Sarcoidosis
Sandblasting, Mines, Egg Shell Calcification
Silicosis
Virchow triad?
Stasis - Hypercoagulable - Trauma
What is the treatment for sarcoidosis?
Steroids
>35yo with PNA. Rusty colored or yellow-green sputum. Acute onset F/C
Strep. Pneumonia
Will evidence of TB be seen more frequently in the upper or lower lung lobes on CXR?
Upper
Where will newer pulmonary lesions from tuberculosis be found?
Upper lobe cavitary lesions
*PNA* Tx of Hospital Acquired - add what (2 options) if MRSA suspected
Vancomycin Linezolid
What are the antibiotics which may be used for MRSA pneumonia?
Vancomycin or linezolid + Levofloxacin or ciprofloxacin
What medications are recommended for MRSA pneumonia?
Vancomycin or linezolid + levofloxacin or ciprofloxacin
Class 1 pulmonary HTN causes?
Ventricular septal defect -Atrial septal defect -Patent ductus arteriosus
Most common cause of acute bronchitis?
Viral
Question: Can amiodarone be used in patients with heart failure?
Yes, one of the attractive features of using amiodarone over other antidysrhythmics.
Pediatric wheezing
lower respiratory FB, asthma
Pediatric with barking cough, stridor
viral croup (laryngotracheobronchitis); Tx w/ racemic epi and glucocorticosteroids if stridor at rest.
*Epiglottitis* Treatment 1. Most important 2. Medical
1. Secure airway 2. IV Ampicillin/sulbactam (Unasyn), cefotaxime, ceftriaxone
MCC lung CA? 2nd MCC?
1. Smoking 2. Asbestosis
*Bronchiolitis* Treatment 1. 1st line 2. Medical 3. If 1st line medical treatment fails
1. Supportive: *humidified air/oxygen, fluids* 2. Albuterol, palivizumab (?) 3. Racemic epi if albuterol failrs
*Pleural Effusion* Treatment 1. 1st line 2. For chronic
1. Thoracentesis (can alos be diagnostic - send for cultures/cytology) 2. Pleurodesis (with *talc*, doxy, minocycline)
Lung cancer screening is generally offered to people ____ and older who smoked heavily for many years and are otherwise healthy.
55
Question: What arterial blood gas finding is concerning for impending respiratory failure in an asthmatic?
A normal, elevated or rising carbon dioxide. Asthma Asthma: airway inflammation + bronchial hyperresponsiveness + reversible airflow obstruction PEF <50%: severe exacerbation Treatments: O2: maintain SpO2 > 88% ß-agonists: ↑ cAMP → bronchodilation Anticholinergics: ↓ bronchoconstriction Corticosteroids: ↓ inflammation, administer early Mg: severe exacerbations Non-invasive ventilation: ↓ work of breathing Mechanical ventilation: Objective: maximize expiratory time Low respiratory rate High inspiratory flow rate Maintain plateau pressure <30 cm H2O Permissive hypercapnia to avoid breath stacking
What will the likely findings be on a CXR of a patient with asbestosis?
Linear opacities at the bases with pleural plaques
A physical exam finding of egophony should make you think of what diagnosis?
Lobar pneumonia
Dullness to percussion should make you think of what diagnosis?
Lobar pneumonia
One Step Further Question: What are the local and systemic side effects of inhaled corticosteroids?
Local: Cough, dysphonia, oropharyngeal candidiasis. Systemic: Adrenal suppression, osteoporosis, skin thinning, easy bruising and cataracts.
Lofgren's syndrome may present like sarcoid. What major difference separates these diseases?
Lofgren's syndrome is short term and spontaneously resolves - Sarcoid is chronic
*SPN* Assessment of malignancy vs. benign Tests?
Look for other Sx; look at history: older? former smoker? environmental exposures? Can do serial CXR/CTs over time to monitor for growth. Can also Bx (transthoracic vs. scope).
A DVT in what location is the most common source of pulmonary embolism?
Lower extremity above the knee
Leading cause of CA death for men and women in the USA
Lung CA
*Asthma* Useful in asthmatics with allergic rhinitis or aspirin induced asthma?
Leukotriene modifiers: - Montelukast - Zafirlukast - Zileuton
*PNA* Tx of Hospital Acquired - add what (2 options) if Legionella suspected
Levofloxacin Azithromycin
Question: What are two clinical situations that can lead to a false-negative D-dimer?
Recent anticoagulation and subacute thrombosis (>7 days). Pulmonary Embolism 95% arise from deep leg veins Sudden onset of symptoms in 50% SOB, CP, tachypnea ECG: sinus tachycardia, nonspecific ST-T changes, right heart strain, S1Q3T3 (classic finding) CXR: nonspecific abnormalities, Hampton's hump (pleural-based wedge infarct), Westermark's sign (vascular cut-off sign) V/Q scan: usually nondiagnostic Low clinical suspicion: negative D-dimer excludes PE Dx of choice: CTPA Rx: anticoagulation, thrombolytics (if massive and HD unstable or submassive with shock, respiratory failure or evidence of moderate to severe RV strain), embolectomy (last resort)
*Carcinoid Tumor* Can cause what secondary endocrine abnormalities? (2)
SIADH Cushing's syndrome
List six extrapulmonary complications of lung cancer.
SPHERE - SVC syndrome, Pancoast tumor, Horner's syndrome, Endocrine problems, Recurrent laryngeal symptoms, Effusions (exudative)
Virus causing pneumonia after exposure to rodent feces (Western states)?
Hantavirus
*Bronchiolitis* MCC in infants or children (2)
1. *RSV* 2. Adenovirus
*Croup* (Laryngotracheitis) Diagnosis (2)
1. Clinical 2. XR: Steeple sign (*subglottic* narrowing)
Currant jelly sputum should make you think of what diagnosis?
Klebsiella pneumoniae
If you see a question with an alcoholic who has been diagnosed with pneumonia, what is the most likely organism?
Klebsiella pneumoniae
Cystic fibrosis w/ PNA
Pseudomonas aueroginosa causative agent
Question: What diagnostic test, although low sensitivity, is often used when influenza is suspected?
Rapid influenza test.
Which virus can lead to pneumonia after URI and also often causes diarrheal illness/GI symptoms?
Adenovirus
What is the most common cause of pneumonia in children less than 1 year old?
Respiratory syncytial virus (RSV)
*Bronchiectasis* If there is thick, brown sputum, what could it be? Tx?
Aspergillus. Corticosteroids + Itraconazole.
What antibiotic is most strongly associated with hypertrophic pyloric stenosis?
Clarithromycin (Note all macrolides can cause hypertrophic pyloric stenosis)
*Bronchiectasis* Treatment if its mycobacterium avium complex (MAC)
Clarithromycin + Ethambutol
Preferred treatment for pertussis?
Clarithromycin or azithromycin
Stqb wound, dullness to percussion, decreased breath sounds.
Hemothorax
Initial Treatment for pulmonary embolism?
Heparin
*PE* Treatment
Heparin: PTT 50-70 Coumadin: INR 2-3 +/- Thrombolytics (streptokinase, alteplase, urokinase)
What are the main side effects of isoniazid?
Hepatitis and peripheral neuropathy
What is the sputum test for tuberculosis?
AFB smear and cultures
Charcot-leyden crystals can be found in sputum
Asthma
What is given to prevent neuropathy in a patient receiving isoniazid (INH) therapy?
B6 (pyridoxine)
Meconium ileus at birth
CF
Diarrhea, flushing, bronchospasm
Carcinoid Syndrome
*Asthma Classification* FEV >80% predicted
Intermittent
Agent of choice for COPD exacerbation?
Ipratropium
*Asthma Classification* FEV over or equal to 80% predicted
Mild
Smoking, family history, males, tall/skinny, pleuritic chest pain
Pneumothorax
Three dishwashers started working at 8:30 a.m. By 1:00 p.m., 11 workers were involved in washing the dishes since there was a huge banquet the previous night, leaving lots of dirty dishes. The dishwashing procedures consisted of: Pre-rinsing the dirty dishes by hand in a water tank. Washing the pre-rinsed dishes in the automatic dishwasher. Draining the washed dishes and placing them in cupboards. Starting around 2:00 p.m., workers began complaining of such symptoms as headaches and dizziness. By 2:30 p.m., all workers showed the same symptoms. Two seriously affected workers were taken to a local hospital by ambulance; the other 9 workers were also eventually taken to the hospital for treatment. One of the workers arriving in the emergency department is a 54-year-old woman whose last memory was washing asparagus off a plate. The next thing she remembers was riding in the ambulance. PAST MEDICAL HISTORY: Peptic ulcer disease, fibromyalgia MEDICATIONS: Unknown anxiolytic, estrogen FAMILY HISTORY: Unknown SOCIAL HISTORY: Divorced; lives with her boyfriend in a trailer park; smokes 3 ppd; drinks a 6-pack of beer daily ROS: Positives: Headache; dizziness; weakness; nausea; difficulty concentrating; dyspnea; visual changes PHYSICAL EXAMINATION: BP 126/91, HR 86, RR 30, T 98.8° F MS: Oriented to name only; speech without dysarthria; 2/3 recall at 5 minutes General: Erythema to face and trunk HEENT: PERRLA, EOMI, Throat clear Neck: No masses Heart: RRR without murmurs Lungs: CTA Abdomen: +BS, soft, nontender Neuro: CN grossly intact; reflexes equal and symmetrical Motor: Full strength throughout with normal muscle tone and bulk Sensory: Unremarkable ABG: 7.41/30/370 with O2 Sat 98% on 100% FiO2; carboxyhemoglobin level is 26% Which of the following is the most likely diagnosis based on the epidemiology of multiple people being ill and this woman's ABG results? a) Allergic reaction to dishwasher detergent b) Meningococcemia c) Influenza A outbreak d) Carbon monoxide poisoning e) Asthma
Pulmonary Medicine Answer D. Carbon monoxide poisoning Explanation All of the workers were diagnosed as suffering from carbon monoxide poisoning. CO has been called the silent killer and is the cause of approximately 5,000 deaths annually, with 2 to 5 times that number requiring treatment. Even with treatment, the devastating sequelae that can accompany CO poisoning can be life-changing. These include chorea, rigidity, dementia, myoclonus, impaired sensory function, seizures, and gait dysfunction. There can also be permanent cardiac damage due to the hypoxia involved in the poisoning process. CO is an odorless, colorless, and tasteless gas that results from incomplete combustion of fuels (i.e., coal, wood, gasoline). Once inhaled, it binds quickly and tightly to the hemoglobin (Hgb) and crowds out the oxygen; studies have shown CO can bind 200 times stronger than oxygen. Since the Hgb can no longer carry the oxygen, the patient becomes hypoxemic and anoxic. Also, the CO binds with myoglobin in the muscles and interferes with cellular metabolism, causing metabolic acidosis. Normal carboxyhemoglobin (HbCO) levels are 0-3% for nonsmokers and 3-8% for smokers. A level of 10-20% causes headaches, nausea, vomiting, and dyspnea. A level of 30-40% causes severe headaches, syncope, and tachyarrhythmias. Levels greater than 40% cause Cheyne-Stokes respiration or respiratory failure, seizures, unconsciousness, permanent brain damage, cardiac arrest, and even death. Because of the vagueness of the symptoms and their similarity to flu-like symptoms (nausea, vomiting, dizziness, headache, etc.), CO poisoning is often misdiagnosed. Also, even if HbCO is present, it cannot be diagnosed with a simple pulse oximetry device—because the displayed saturation level equals the sum of the oxyhemoglobin and carboxyhemoglobin. So if you suspect a patient has CO poisoning, what should you do? First and foremost, the patient needs high-flow, high-concentration O2, preferably a non-rebreather mask at 15 liters per minute, as well as a large bore IV. Prepare for blood draws (ABG, CBC, electrolytes, CPK, lactate, and carboxyhemoglobin). A urine sample is also useful to rule out rhabdomyolysis (cardiac muscle breakdown secondary to the myoglobin damage from CO). Other treatment modalities may include a CXR, cardiac monitoring, and possibly mannitol to help decrease the cerebral edema accompanying the CO poisoning. Finally, probably the most effective treatment is transfer of the patient to a hyperbaric oxygen unit (HBO). This is clearly indicated when the patient is very symptomatic and/or the HbCO is 25% or greater.
Cushing's syndrome (d/t ectopic ACTH production) is MC with what type of lung CA?
Small cell
Virchow's triad
Stasis, hypercoagulability, endothelial damage
Question: What is aspiration pneumonia?
An alveolar space infection resulting from the inhalation of pathogenic material from the oropharynx. Aspiration Pneumonitis Inflammatory chemical injury Inhalation of gastric contents Can lead to aspiration pneumonia Depressed level of consciousness Unilateral focal/patchy consolidations in dependent lung segments Prophylactic antibiotics NOT recommended
Wheezing, prolonged expiration
Asthma
*Tobacco use/dependence* Treatments (3)
Buproprion Varenicline Nicotine replacement (patch, nasal spray, gum, etc.)
Oseltamivir (Tamiflu) may be given to children above what age?
1 year old
*COPD* General treatment (3)
1. *BRONCHODILATORS*: - **Anticholinergics** (preferred): Ipratropium, Tiotropium - B-agonists: Albuterol, Terbutaline, Salmeterol (L.A.) 2. *CORTICOSTEROIDS*. 3. *OXYGEN* if SaO2 ≤88%, PaO2≤55mm
*Sarcoidosis* Tx (1st, 2nd)
1. Observation. Majority of patients have spontaneous remission in 2 years. 2. *Oral corticosteroids* the Tx of choice when Tx is needed.
*ARDS* DDx from CHF (2)
1. The diffuse bilateral pulmonary infiltrates *spare* the costophrenic angles on CXR 2. PCWP is under 18 mmHg
At what size induration would a PPD be positive for an HIV patient?
5 mm
At what age should vaccinations for pneumococcal pneumonia begin?
65 years old
*Influenza* Which serotype is associated with more extensive, severe outbreaks?
A
Question: What is the significance of a high amylase level in a pleural fluid sample?
A high amylase level usually indicates the presence of pancreatitis, esophageal rupture or malignancy. Pleural Effusion Transudate: CHF (most common) Exudate: infection > malignancy, PE ↓ Breath sounds + dull percussion + ↓ tactile fremitus CXR: blunting of the costophrenic angle
What dermatologic condition is associated with sarcoidosis?
Erythema nodosum
Clinical Intervention/Pulmonology A 60 year-old female with a 30 pack year smoking history complains of new onset shortness of breath. On physical examination, dullness is noted on percussion with dimished breath sounds over her left base. Chest x-ray shows a new left pleural effusion. Which of the following is the next step in the management of this patient? A. Repeat chest x-ray in two months B. Perform diagnostic thoracentesis C. Order MRI of the chest D. Treat with antibiotic
Explanations (u) A. See B for explanation. (c) B. Diagnostic thoracentesis should be performed whenever there is a new pleural effusion and no clinically apparent cause. (u) C. See B for explanation. (u) D. See B for explanation.
Viral, cough, negative CXR
Acute Bronchitis
*PNA* Aspiration PNA, severe periodontal disease → R lower lobe
Anaerobes (Peptosreptococcus, bacteroides, fusobacterium)
How is RSV diagnosed?
Analysis/culture of Respiratory secretions
Which of the following organisms can be spread from person to person in respiratory droplets from a person with pneumonia and is useful as a bioterrorist agent because of this characteristic? a)Neisseria meningitidis b) Clostridium botulinum c) Arcanobacterium haemolyticum d) Bacillus anthracis e) Yersinia pestis
Answer E. Yersinia pestis Explanation Yersinia pestis is the only agent listed that can cause a necrotizing pneumonia that is infectious by spread of coughing or sneezing respiratory droplets. Pneumonia due to anthrax (from Bacillus anthracis) is not contagious. Botulism does not cause pneumonia. Neisseria meningitidis is not a potential bioterrorism agent, and Arcanobacterium haemolyticum just causes sore throats in adolescents and young adults.
Piperacillin/tazobactam, cefepime, imipenem, meropenem, ceftazidime are all examples of
Antipseudomonal B-lactams
The "5-A" framework is helpful for smoking cessation counseling:
Ask about tobacco use. Advise to quit through clear, individualized messages. Assess willingness to quit. Assist in quitting. Arrange follow-up and support sessions.
PNA & decreased mental status, poor dental hygiene, dentures, foul smelling sputum, bronchiectasis. Patchy infiltrates in dependant lung zones
Aspiration PNA
Question: What is the most common cause of lung abscess?
Aspiration pneumonia. Lung Abscess Patient will be complaining of several weeks of cough, fever, pleuritic chest pain, weight loss, and night sweats CXR will show area of dense consolidation with an air-fluid level inside a thick-walled cavitary lesion Most commonly caused by aspiration pneumonia Treatment is clindamycin
< 2 days post-op with fever
Atelectasis
*PNA* Shows up on CXR as diffuse, patchy interstitial or reticulonodular infilatrates. S/Sx include *low grade* fever, *dry*/nonproductive cough, extrapulmonary symptoms including myalgias, malaise, ST, HA, N/V/D. Normal PE; signs of consolidation usually absent. General class? Etiologies (4)?
Atypical: 1. Mycoplasma pneumoniae 2. Chlamydophila pneumoniae 3. Legionella pneumophila 4. Viruses
Question: What pulmonary finding is pathognomonic for sarcoidosis?
Bilateral hilar adenopathy.
Common in patients with *cystic fibrosis*, CXR shows *plate like atelectasis*
Bronchiectasis
*CF* CXR
Bronchietasis
RSV
Bronchiolitis
Lung disease d/t cotton exposure in the textile industry. Sx characterized by dyspnea, wheezing, cough, chest tightness that are *worse at the beginning of the work week* and improve later on, or during the weekend. This gives it the nickname of *Monday fever*.
Byssinosis
What situations predispose an infant to developing type 2 IRDS?
C-section or diabetic mother
Class 3 pulmonary HTN cause?
COPD
Smoker, DOE, cough
COPD
Diarrhea, flushing, bronchospasm.
Carcinoid Syndrome
*PNA* Fungal pneumonia found in the soil of southwest US (california, new mexico). "Valley fever".
Coccidioides
A 26-year-old gravida 3 para 2 at 12 weeks gestation presents with fever, myalgias, headache, and malaise. There have been multiple cases of influenza in the community and her influenza swab is positive. Which one of the following is recommended by the Centers for Disease Control and Prevention in this situation? Acyclovir Oseltamivir Rimantadine Supportive therapy only
Correct Answer ( B ) The Center for Disease Control recommends that neuraminidase inhibitors be prescribed for pregnant women and for those up to two weeks postpartum who have suspected or confirmed influenza. Women can continue to breastfeed while being treated with antivirals. Treatment with antiviral agents, should be started within 48 hours of symptom onset. The recommended treatment of influenza in pregnancy is one of the neuraminidase inhibitors, such as oseltamivir. Neuraminidase inhibitors have modest effectiveness in reducing influenza-related symptoms in patients and they protect against both strains of influenza A and influenza B.
When diagnosing pneumonia in children, which of the following findings has the highest odds ratio? Crackles Oxygen saturation < 92% Retractions Temperature >38°C
Correct Answer ( B ) Explanation: A study of more than 2,500 pediatric patients who presented to an ED showed hypoxia with oxygen saturation at or below 92% to have an odds ratio (OR) of 3.7 for pneumonia. This was higher than the OR for any other measured parameter. Pediatric Pneumonia 0-3 weeks: GBS 3 weeks-3 months: C. trachomatis 1 month-5 years: RSV 6-8 years: M. pneumoniae C. trachomatis: staccato cough, eye findings S. pneumoniae: rust-colored sputum M. pneumoniae: bullous myringitis ((classically associated together however middle ear aspirate culture results show that typical acute otitis media pathogens are the true cause)
A 19-year-old man presents with a sore throat and difficulty swallowing. He has had 4 days of worsening sore throat and fever but today he was unable to swallow any liquids. Physical examination reveals a muffled voice, difficulty tolerating saliva, and minimal pharyngeal erythema with a midline uvula. He also has tenderness over the hyoid bone. Which of the following managements is most likely indicated? Inhaled racemic epinephrine and discharge home Intravenous antibiotics and admission Intravenous corticosteroids and discharge home Needle aspiration of peritonsilar area
Correct Answer ( B ) Explanation: This patient presents with symptoms and signs concerning for epiglottitis and should have antibiotics started and admission for further monitoring. Epiglottitis is a localized cellulitis of the supraglottic structures including the aryepiglottic folds, arytenoids, lingual tonsils, epiglottis and base of the tongue. It may be caused by viral or bacterial agents (Haemophilus influenzae type b is the most common). Although incidence in pediatric populations has decreased with the H. influenzae vaccine, it is becoming more common in adults and can lead to rapid airway obstruction. Patients usually present with dysphagia, odynophagia and sore throat. Pain may be severe in relation to physical exam findings. Additionally, patients frequently complain of a muffled voice. Fever is present in half of cases. Physical examination typically reveals mild inflammation in the oropharynx and may reveal tenderness to the neck over the hyoid bone. Epiglottitis is a clinical diagnosis but may be aided by plain radiographs of the neck, which shows edema of the epiglottis (thumb sign). An epiglottic width greater than 8 mm or an aryepiglottic fold greater than 7 mm suggests epiglottitis. Definitive diagnosis can be made with indirect laryngoscopy. Management in patients focuses on protecting the airway when necessary starting IV antibiotics (ceftriaxone and ampicillin-sulbactam are first-line) and admission for monitoring. Supportive care with hydration is important as well
Steeple sign on a frontal chest X-ray showing tracheal narrowing?
Croup
What is laryngotracheo-bronchitis also known as?
Croup
Barking cough, stridor, hoarseness, dyspnea that is worse at night. +/- preceding or concurrent URI symptoms.
Croup (Laryngotracheitis)
What is a Hallmark sign seen with carcinoid tumors?
Cutaneous Flushing
Autosomal recessive, infertility, sweat chloride test
Cystic Fibrosis
What is the most common cause of bronchiectasis?
Cystic Fibrosis
Abnormal *sweat chloride test*
Cystic fibrosis
Salty baby?
Cystic fibrosis
Sweat chloride test
Cystic fibrosis
What condition is bronchiectasis often associated with?
Cystic fibrosis
Pediatric with Hx recurrent lung infections, pancreatitis, reproductive problems, FTT
Cystic fibrosis (Staph & Pseudomonal infections usually cause of death)
Venous stasis, vessel wall injury, hypercoagulability
DVT/PE (Virchow's triad)
A thin looking heart and flattened diaphragm on CXR should make you think of what diagnosis?
Emphysema
Smoking, alpha 1 antitrypsin deficiency, barrel chest, decreased DLCO and FEV1
Emphysema
Pertussis is the most commonly treated with what antibiotic?
Erythromycin
*Pleural Effusion* Light's Criteria
Exclusive to exudates; presence of any of the 3 indicative: 1. Pleural fluid protein: serum protein >0.5 2. Pleural fluid LDH: serum LDH >0.6 3. Pleural fluid LDH > 2/3 upper limit of normal LDH
Scientific Concepts/Pulmonology Which of the following types of pleural effusion result from increased production of fluid due to underlying inflammatory conditions? A. Exudate B. Empyema C. Transudate D. Hemothorax
Explanations (c) A. Increased production of fluid due to inflammatory or malignant processes results in an exudative pleural effusion. (u) B. Infection in the pleural space causes an empyema. (u) C. Transudates result from increased hydrostatic or decreased oncotic pressures across normal capillaries. (u) D. Bleeding into the pleural space causes a hemothorax.
*Asthma* Extrinsic vs. intrinsic triggers
Exstrinsic: Allergens (pollen) Intrinsic: Infection, exercise, cold air, medications (BB)
*Pleural Effusion* Type cause by Chylothorax?
Exudative; will have *TGs* in the fluid
Steeple sign
FB, viral croup (laryngotracheobronchitis)
In a patients with asthma the FEV1 to FVC will be <______ ?
FEV1 to FVC < 75% = diagnosis of asthma in pt with chronic wheezing/cough
*COPD* Per GOLDs criteria, what is considered severe and needs O2 therapy?
FEV1 under 30% predicted
*PNA* Treatment of CAP, ICU if ALLERGY to Beta-lactam
FQ +/- Aztreonam (monobactam) or Clinamycin + Aztreonam
*PNA* Tx of OP CAP if comorbid conditions or recent abx use?
FQs - Levofloxacin, Moxifloxacin
One Step Further Question: What is the most common inherited risk factor for thromboembolic disease?
Factor V Leiden
True or false: Bacterial infections are the most common cause of acute bronchitis?
False.
Rapid Review Foreign Body Aspiration
Foreign Body Aspiration Patient will be a child Complaining of a sudden episode of coughing, wheezing, or stridor Comments: most common is the right main bronchus
Pneumonia associated with inhalation of bat or bird droppings?
Histoplasma
What should be considered in a patient with pneumonia who was exposed to bird or bat droppings in the Mississippi to the Ohio river area?
Histoplasma capsulatum
You suspect a pneumothorax. What instructions should go along with the order for a CXR?
It should be an end expiratory view to look for visceral pleural air.
What lab is elevated in PJP pneumonia?
LDH
Tympany to percussion should make you think of what diagnosis?
Large pneumothorax
A thumbprint sign on x-ray should make you think of what two diagnoses?
Lateral C-spine = epiglottis, abdominal = intestinal ischemia
PNA w/ Water, late summer, construction site. Diarrhea. Toxic looking
Legionella
What pneumonia associated with diarrhea and low sodium levels?
Legionella
Where will older pulmonary lesions from tuberculosis be found?
Lower lobes
Hilar mass on CXR
Lung Ca
What is the likely Dx? Solitary pulmonary nodule of 2 cm in size found on chest X-ray incidentally.
Lung cancer
What is the most likely Dx? Smoker with hemoptysis weight loss and new DVT?
Lung cancer
What should be considered with a localized ground glass appearance and a pulmonary nodule on chest X-ray?
Lung cancer
*PNA* Common in patients w COPD
M. catarrhalis
Organisms responsible for atypical pneumonia?
Mycoplasma pneumoniae- Chlamydia pneumoniae- Legionella
Question: Name 2 clinical prediction rules that may be used to help determine whether a patient with CAP requires inpatient or outpatient management?
PORT/PSI (Pneumonia Severity Index) and the CURB 65.
Sudden onset of dyspnea and pleuritic chest pain that is non-exertional. *Hyperresonance* to percussion, decreased fremitus, decreased breath sounds.
PTX Tension: Increased JVP, pulsus paradoxus, hypotension
"Pack-years" formula
Packs per day x years smoked
Stab wound, hyperresonance to percussion, decreased breath sounds, tympany
Pneumothorax
Which of the following values is consistent with emphysema? a) %TLC %DLCO Low Normal b) %TLC %DLCO Normal Normal c) %TLC %DLCO Low High d) %TLC %DLCO Low Low e) %TLC %DLCO High Low
Pulmonary Medicine Answer E. %TLC %DLCO High Low Explanation The high TLC and disproportionately low DLCO is classic for emphysema. The other values are not consistent with emphysema.
A Westermark sign on CXR should make you think of what diagnosis?
Pulmonary embolism
Question: What are the mechanisms of pulmonary bleeding?
Pulmonary hypertension, erosion into a blood vessel, and coagulopathy. Hemoptysis Diastolic murmur: mitral stenosis Sudden SOB, CP: PE Trauma: pulmonary contusion Immunodeficiency, immigrant: TB Hx of TB or sarcoidosis: aspergilloma Renal dysfunction: Goodpasture's syndrome or Wegener's granulomatosis (Granulomatosis with polyangiitis) Hx of tobacco use, weight loss: malignancy Massive hemoptysis: > 600 mL of blood/24 hrs Massive hemoptysis Rx: patient in bleeding side down position, mainstem bronchus intubation
Incomplete lung development due to congenital malformation?
Pulmonary hypoplasia
*PNA* Rusty, blood-tinged sputum
S. pneumoniae
Lung CA associated with cavitary lesions, hypercalcemia, and pancoast syndrome? (CCCP)
SCC
Question: What diagnostic criteria is used in Light's criteria?
Serum protein and pleural fluid lactate dehydrogenase are used in Light's criteria, which is used to distinguish between transudative and exudative pleural effusions.
*Asthma Classification* FEV under 60% predicted
Severe
Eggshell opacities on the CXR should make you think of what diagnosis?
Silicosis
Metastasizes early; typically located *centrally* and is *aggressive*. Less common type, almost always in smokers.
Small Cell (Oat Cell) Carcinoma
Lambert-Eaton syndrome is associated with what type of lung CA?
Small cell
What is the cause of hyaline membrane disease in infants?
Surfactant deficiency
What is erythema nodosum?
Tender red nodules usually found on the shins.
Stab wound to chest. Hypotension, tracheal shift
Tension PTX
*COPD* Bronchodilator used only in refractory cases
Theophylline
Most common mediastinal tumor?
Thymoma
Which of the following interstitial lung diseases has upper lobe predominance? a) Silicosis b) Asbestosis c) Lymphangitic carcinomatosis d) Rheumatoid fibrosis Pulmonary infiltrates related to scleroderma
Topic Pulmonary Medicine Answer A. Silicosis Explanation Silicosis has an upper lobe predominance. The other choices more commonly occur in the lower lobes. Other upper lobe predominant diseases include ankylosing spondylitis and eosinophilic granuloma.
A 75-year-old Caucasian male with recent fracture of his right femur 3 hours ago presents by ambulance and is seen by an orthopedist, who places him in a cast and admits him for observation. Three hours later, you are called by the doctor because his patient is now short of breath and confused. He thinks the patient might have pneumonia because he is breathing fast. He is consulting you for antibiotic choices. You tell him you will take over the care for the respiratory disorder. On evaluation, you find a disheveled elderly man lying in the bed in traction. He is having some difficulty breathing. He has supplemental oxygen with a 40% FiO2 facemask that is keeping his pulse oximetry at 95%. He is unable to answer any questions due to his dyspnea, and he is also quite confused. Eventually, he says the year is 1960 and that you are his 5th grade school teacher. PHYSICAL EXAMINATION: BP 130/70, P 99,Temp 99° F, RR 24 HEENT: PERRLA, EOMI; conjunctival petechiae TMs clear Throat clear Neck: Supple, no masses; petechiae on neck Heart: RRR without murmurs, rubs, or gallops Lungs: Coarse breath sounds; few basilar scattered crackles Abdomen: Bowel sounds present; no hepatosplenomegaly Extremities: No cyanosis, clubbing, some edema on the fractured leg at the ankle; exam of fractured extremity limited but neuro-vasculature looks to be grossly intact LABORATORY: Pending Based on the history and physical findings, which of the following is the most likely diagnosis? a) Cerebrovascular accident b) Fat embolism c) Drug toxicity d) Hospital psychosis e) Aspiration pneumonia
Topic Pulmonary Medicine Answer B. Fat embolism Explanation Based on the findings of dyspnea, confusion, and petechiae (particularly neck, conjunctiva, and axilla [not described in this patient]), you can make the diagnosis of fat emboli in the setting of a recent large bone fracture. Fat emboli can also occur after aggressive CPR and with sickle cell bone occlusive crisis. The other answer options do not cause this combination of findings. Treatment is supportive; unless the patient has secondary ARDS, steroids have not proven of benefit. We don't give you a lot of lab values here, and you shouldn't need them at this point. Again, the triad of dyspnea, confusion, and petechiae in the setting of recent fracture ... think of fat embolism!
A 74-year-old male with a known history of COPD presents to the emergency department cyanotic and in severe respiratory distress. He is a thin man weighing 70 kg. An arterial blood gas indicates severe respiratory acidosis and hypoxemia on room air (pH 7.00; pCO2 120 mmHg; and pO2 40 mmHg). He is intubated immediately and placed on mechanical ventilation in the assist/control mode at the following settings: Rate: 25/min Tidal volume: 1,000 cc Peak inspiratory flow rate: 50 L/min When you arrive in the emergency department to see your patient, you note that his blood pressure is 60/30 and his heart rate is 140/min. His pulse is thready, his neck veins are full, and his trachea is midline. There are equal breath sounds bilaterally. There are many alarm lights going off on the ventilator. You yell for someone to page Respiratory Therapy, disconnect the patient from the ventilator for a few minutes, and watch his blood pressure and heart rate improve. The respiratory therapist arrives. Which of the following orders do you give the respiratory therapist for mechanical ventilation? a) Assist/control mode, rate 28/min, tidal volume 600 cc, peak flow 40 L/min b) Assist/control mode, rate 18/min, tidal volume 1,000 cc, peak flow 60 L/min c) Assist/control mode, rate 12/min, tidal volume 700 cc, peak flow 80 L/min d) T-bar with 80% oxygen on blow by
Topic Pulmonary Medicine Answer C. Assist/control mode, rate 12/min, tidal volume 700 cc, peak flow 80 L/min Explanation We all recognize that the hemodynamic demise in this patient was from dynamic hyperinflation and the creation of auto-PEEP. Just think of air trapping and breath "stacking." This patient's underlying problem is expiratory airflow obstruction. So, what did the emergency department doctor do? In an effort to immediately reverse the acute hypercapnic respiratory failure, he hyperventilated the patient and provided a large tidal volume at a low peak flow rate. (Peak flow determines how fast the gas is delivered for each breath—a higher peak flow results in the breath being delivered quickly [short inspiratory time], allowing for a longer expiratory phase.) This patient could not possibly have had enough time to empty his lungs with each breath. Ultimately, he builds up such a large pressure in his chest (auto-PEEP) that blood flow return is compromised and hypotension ensues. In removing the patient from mechanical ventilation, you allowed the trapped gas to be released and hemodynamic stability returned. While it is tempting to leave the patient on a T-bar without any positive pressure, be assured that he will quickly tire and have a respiratory arrest. Therefore, you return the patient to mechanical ventilation (assist/control mode is as good as any) with the understanding that you want a tidal volume of 5-10 mL/kg, a high peak flow rate, and a lower respiratory rate (8-14). Even without sophisticated monitoring, you should be able to see that you are not trapping air by auscultating the lungs through a few respiratory cycles and see that the patient has finished his expiration phase before the next breath by the ventilator is delivered. Recognize that this method of ventilating the patient may not fully correct his respiratory acidosis. That's OK; a moderate amount of respiratory acidosis is well tolerated (permissive hypercapnia), so aim to get a pH > 7.20 as your initial goal and let the bronchodilators, steroids, etc., start to work.
An 18-year-old college freshman developed a sore throat approximately 8 days ago that was cultured and did not grow Streptococcus pyogenes. However, her throat culture did grow Staphylococcus aureus. She presents today with complaint of fever and cough that began last night. She also describes hoarseness since she has had her sore throat symptoms. The sore throat symptoms resolved after 3 days without specific antimicrobial therapy. PAST MEDICAL HISTORY: Attention deficit disorder diagnosed at age 8; on no medication since 4 years ago SOCIAL HISTORY: Recently moved here for college Doesn't smoke or drink alcohol Not sexually active Has a pet parrot back home FAMILY HISTORY: Mother 40, healthy Father 42, in prison for securities fraud Sister 15, pregnant REVIEW OF SYSTEMS: Negative for other symptoms; no rash, no joint manifestations PHYSICAL EXAMINATION: BP 110/70, P 88, Temp 100° F, RR 18 Ill-appearing woman in no acute distress HEENT: PERRLA, EOMI, wears contact lenses TMs clear Throat: slightly erythematous; no exudates Neck: Supple, no meningismus Heart: RRR without murmurs, rubs, or gallops Lungs: Coarse breath sounds with crackles heard on the left base Abdomen: Bowel sounds present in all 4 quadrants, nontender; no hepatosplenomegaly Extremities: No cyanosis, clubbing, or edema LABORATORY: CXR: Left lower lobe infiltrate Besides Streptococcus pneumoniae, which of the following is a likely etiology for her pneumonia? a) Chlamydia trachomatis b) Staphylococcus aureus c) Chlamydophila psittaci d) Chlamydophila pneumoniae e) Haemophilus influenzae type b
Topic Pulmonary Medicine Answer D. Chlamydophila pneumoniae Explanation With the history of prior sore throat and now a febrile illness with documented pneumonia by chest x-ray, Chlamydophila pneumoniae is the most likely. The parrot is a red herring here because she had no signs of C. psittaci infection—look for a pneumonia with an extensive interstitial pattern that appears to be much worse than the physical exam would indicate. C. trachomatis, remember, causes genitourinary and eye disease. S. aureus would be seen after an influenza-like illness or if she were immunocompromised, and she would be much sicker. Don't be thrown by the positive throat culture for this organism—it can be found frequently in asymptomatic people. H. influenzae should not cause pneumonia in her age group.
A 30-year-old non-smoker presents with worsening shortness of breath. A CXR is done and shows emphysematous bullae in the bases consistent with early-onset COPD. Which of the following is a likely etiology for his condition? a) Eosinophilic granuloma b) Sarcoidosis AIDS c)Homozygous α1-antitrypsin deficiency d) Berylliosis
Topic Pulmonary Medicine Answer D. Homozygous α1-antitrypsin deficiency Explanation Think of this in a non-smoker who presents at a young age with COPD and has changes on CXR consistent with that diagnosis (as we see here with emphysematous bullae). IV therapy is available but controversial in efficacy. If the emphysema is severe, then lung transplant is indicated. The other diseases mentioned would not be common in a person this age, except for AIDS, which would not cause emphysematous bullae.
A 45-year-old female presents to your office complaining of fever and shortness of breath. This is the 4 th Monday in the past 2 months that she has come to see you for this complaint. She began to wheeze and cough this morning, and as the day progressed, she developed fever to 101.5° F and shortness of breath. She bought a parakeet in the last few months. Meds: HCTZ and lovastatin PE: T 101.7° F, RR 22, HR 89, BP 136/82. Mild respiratory distress. No jugular venous distention. Lungs show symmetric expansion, normal resonance, diffuse wheezing, and inspiratory crackles audible at the bases. Remaining exam is normal. CXR: Diffuse alveolar filling pattern and variable interstitial streaks. For which of the following is this patient at risk if the correct diagnosis is missed today? a) Diffuse alveolar hemorrhage b) Sudden cardiac death c) Lymphoma Interstitial fibrosis d) Obstructive lung disease
Topic Pulmonary Medicine Answer D. Interstitial fibrosis Explanation This question gives you the historical information you need to diagnose the patient with hypersensitivity pneumonitis. The patient is a generally healthy woman who episodically develops acute wheezing and dyspnea with fever. The new bird could certainly be a cause of hypersensitivity pneumonitis due to a reaction to avian proteins. The PE directs your attention to the bases of her lungs, and the radiograph shows classic interstitial abnormalities. The question queries whether you know that fibrosis is a long-term consequence of recurrent hypersensitivity pneumonitis. Her physical exam is not concerning for lymphoma, nor does the clinical history suggest that diagnosis. Sudden cardiac death is a concern in patients with pulmonary hypertension and structural heart disease, but this patient has no indication of either on physical exam. Diffuse alveolar hemorrhage is a complication of the pulmonary-renal vasculitides, especially Goodpasture syndrome; but the clinical presentation here does not match that of a vasculitis.
A 75-year-old male is admitted to the hospital with an exacerbation of COPD. He is treated with oxygen, antibiotics, and bronchodilators. On the 4th day, he is feeling better and he wants to go home. A pulse oximetry on room air today reveals a saturation of 89%, which was confirmed by a simultaneous ABG on room air, which demonstrated a PaO2 of 58 and a saturation of 89%. With ambulation, the oxygen saturation falls to 86%, and the patient becomes dyspneic. His Hct on admission was elevated to 52%. You request a referral from Social Services for home oxygen for your patient, and you are told that "he doesn't qualify but he could pay for it himself." Your patient, and his family, are quite distressed to learn that they will have to pay for the oxygen out of their own pockets. You are determined to see if you can get your patient qualified for home oxygen. Which of the following is the best plan of action? a) Secretly tell your patient to ambulate and repeat the ABG. b) Take up a collection for your patient. c) Resubmit the application on the basis of his erythrocytosis. d) Order an ECG and echocardiogram to evaluate for cor pulmonale.
Topic Pulmonary Medicine Answer D. Order an ECG and echocardiogram to evaluate for cor pulmonale. Explanation For insurance to cover this man's home oxygen, you would need to demonstrate evidence of cor pulmonale in addition to his blood gas results. His Hct does not qualify him on the basis of erythrocytosis. Only objective data from an arterial blood gas is acceptable to Medicare for supplemental oxygen. History and physical examination will not help get this funded. He needs to meet one of the following criteria: Resting PaO2 ≤ 55 mmHg, or O2 saturation ≤ 88%, or PaO2 ≤ 59 mmHg (O2 sat ≤ 89%) with evidence of cor pulmonale Evidence of cor pulmonale in these considerations is: Clinical evidence of right heart failure Pulmonale on ECG (P wave height > 2.5 mm in II, III, and AVF) Hct > 56 (due to polycythemia from chronic hypoxia 2° cor pulmonale) You can tell him that being on supplemental oxygen is likely to improve his symptoms and there is a possibility that, in several months, he may not need the oxygen anymore. Medicare guidelines require you to bring him back between 61 and 90 days and retest him. Note that the use of supplemental oxygen does not depress the respiratory drive and cause an elevated CO2. This is due to several factors, including an easing of pulmonary vasoconstriction, which leads to perfusion of previously underperfused spaces and possibly due to ventilation of previous dead space.
A 54-year-old male who resides in Ohio presents with a 6-month history of recurrent cough that is nonproductive. He reports that the cough is persistent and worse during the day and seems to improve during the night. It has been persistent, but it seemed to improve when he took a vacation to Florida 2 months ago and recurred on his return. He denies any sputum, wheezing, reflux symptoms, chest pain, edema, orthopnea, paroxysmal nocturnal dyspnea, or palpitations. He does endorse mild dyspnea on exertion, as well as occasional fevers without chills. He has no recent sick contacts or travel exposure. He denies any smoking or illicit drug use. He began working for a mushroom farm 9 months ago. Prior to that, he worked on various construction jobs and was told that there was some asbestos material exposure, but he is not sure how much. Chest radiograph is ordered and reveals patchy infiltrates in the right upper lobe and left lingula region, without any effusions or masses. Which of the following is the next best step in management for this patient? a) Initiation of corticosteroids at 1 mg/kg daily. b) Bronchoscopy with transbronchial biopsy. c) Start empiric treatment with oral fluoroquinolone. d) Prolonged duration away from his work environment. e) Transthoracic echocardiogram.
Topic Pulmonary Medicine Answer D. Prolonged duration away from his work environment. Explanation This patient has signs and symptoms most consistent with hypersensitivity pneumonitis that is most likely related to his exposure to toxins from working with mushrooms. Hypersensitivity pneumonitis (HP) is an immune-mediated reaction to an antigen that can result in acute, subacute, and chronic forms of HP. The acute form often presents with cough, fever, dyspnea, and fleeting infiltrates. A history of recurrent "pneumonias" and a temporal relation to an organic trigger is often elicited. In this patient, the temporal history of symptoms after starting work in a mushroom farm associated with improvement when away from work is very consistent with subacute HP, aka mushroom farmer's lung. Antigen avoidance is the mainstay of treatment. For those who have severe symptoms without improvement with antigen avoidance, corticosteroids may be effective. Bronchoscopy is not indicated because the tissue obtained from transbronchial biopsy is often not sufficient to make an adequate diagnosis. There are no signs of congestive heart failure or other cardiovascular issues, so echocardiogram is not indicated at present. The current presentation is not consistent with acute bacterial pneumonia, so antibiotics would not be an appropriate first choice.
An asthmatic presents in respiratory distress. He requires mechanical ventilation. Which of the following parameters is best for an asthmatic? A) Rate Tidal Volume Flows High High High b) Rate Tidal Volume Flows High Low Low c) Rate Tidal Volume Flows Low High Low d) Rate Tidal Volume Flows Low Low High e) Rate Tidal Volume Flows Low High High
Topic Pulmonary Medicine Answer D. Rate Tidal Volume Flows Low Low High Explanation These patients need a prolonged expiratory phase.
What do "egg-shell" calcifications make you think of? a) Blastomycosis b) Sarcoidosis c) Histoplasmosis d) Silicosis e) Asbestosis
Topic Pulmonary Medicine Answer D. Silicosis Explanation These usually are hilar in location. None of the other choices have "egg-shaped" calcifications associated with them.
A 57-year-old man with a history of obesity and hypertension presents to the emergency department complaining of dizziness, weakness, and dyspnea on exertion. This has been occurring for one day but continues to escalate. He denies chest pain or calf tenderness. He recently traveled overseas on a 6-hour flight and returned seven days prior to presentation. Physical examination reveals: HR: 105, BP: 132/76, RR: 22, 87% oxygen saturations on room air. Chest radiograph is unremarkable, and the ECG reveals sinus tachycardia. Chest CT angiogram demonstrates a saddle pulmonary embolus in the main pulmonary arterial trunk. Transthoracic echocardiogram reveals evidence of acute right heart strain, with moderate reduction in the right ventricular systolic function. There is also tricuspid regurgitation with pulmonary hypertension. Computed tomography of the head is normal and reveals no cause for dizziness. He has no known contraindications to anticoagulation. How should the echocardiographic findings of right ventricular strain alter management of this patient? a) The patient is at increased risk for recurrent PE and should receive LMWH rather than warfarin. b) The patient is at increased risk for shock or death from PE and should receive thrombolytic therapy. c) The patient is at increased risk for ongoing embolization and should receive a vena caval interruption device. d) The patient is at increased risk for shock or death from PE and should receive more intensive monitoring.
Topic Pulmonary Medicine Answer D. The patient is at increased risk for shock or death from PE and should receive more intensive monitoring. Explanation Most patients with pulmonary embolus will have a normal echocardiogram. When evidence of right ventricular strain is present on echocardiogram, this is associated with an increased risk of death from PE. Even if patients are hemodynamically stable when the echocardiogram reveals right ventricular compromise, 10% of such patients will develop shock, and 5-10% will die in the hospital. Given the risk for clinical worsening, these patients should be closely monitored in high levels of care such as in an intensive care unit. Although some physicians advocate more aggressive treatment for such patients, such as thrombolysis or venal caval filter, these have not been proven to improve mortality. There is some evidence to suggest that those patients who present with RV strain have a higher risk for recurrent VTE; different regimens have not been thoroughly studied to show a difference in recurrence between warfarin and LMWH. The exception is for the patient population with solid tumor malignancy.
A 26-year-old female with asthma presents to your office for routine follow-up. She was diagnosed with asthma at age 16 and required ICU admission once at age 17. She has never been on a ventilator. She is adherent to her asthma regimen. Review of systems is positive for daily wheezing, requiring inhaled bronchodilators, and nocturnal coughing 3x/week that wake her from sleep. Meds: Beclomethasone 80 mcg bid Albuterol MDI prn PE: Normal vital signs No jugular venous distention Lungs with symmetric expansion, resonant to percussion, clear to auscultation Which of the following is the most appropriate next step in patient care? a) Add ipratropium bromide bid. b) Add montelukast daily. c) Add theophylline. d) Increase beclomethasone to 320 mcg bid. e) Add salmeterol bid.
Topic Pulmonary Medicine Answer E. Add salmeterol bid. Explanation This is a common question on Board exams requiring you to determine whether a patient's asthma is controlled. Because the patient is on long-standing asthma medications, you will assess her level of "control" (remember that "severity" assessments are made at initial presentation in patients not yet prescribed long-term control medications). First determine the treatment "step" the patient is receiving now: low-dose ICS +prn SABA (Step 2 treatment). Next, determine how well the Step 2 treatment is controlling her asthma. From the asthma guidelines, note that daily symptoms indicate "very poorly controlled" disease, and nocturnal symptoms 3x/week indicate "not well-controlled" disease. Therefore, assign her the most severe classification of "very poorly controlled." Next determine how to manage a patient whose asthma is very poorly controlled. The recommendation is to consider a short course of systemic steroids, and step up her treatment regimen 1-2 steps. Increasing this patient's regimen by 1-2 steps would include adding either a LABA and/or increasing her inhaled steroids to medium-dose. Of the choices listed, adding salmeterol (a LABA) is the only correct answer. Theophylline and montelukast additions to low-dose ICS are only alternatives to the preferred Step 3 treatment, and alternative options should not take precedence over the preferred Step 3 recommendation. 360 mcg of beclomethasone is the highest available dose of this medication and would be considered Step 5 treatment. A LABA should be added prior to increasing an ICS to the highest dose. Ipratropium is not used in the outpatient control of persistent asthma. Certainly, you should consider whether this patient needs a dose of oral corticosteroids. You could make this assessment by performing a post-bronchodilator FEV 1 measurement. However, prescribing corticosteroids is not a choice in the list of answers.
You are presented with an HIV-infected patient with possible tuberculosis versus atypical mycobacterium lung infection. Which of the following would make you lean more toward tuberculosis as an etiology? a) HIV for 20 years. b) CD4 of 20. c) Taking trimethoprim/sulfamethoxazole for prophylaxis. d) Acid fast smears of the sputum are negative. e) CD4 of 450.
Topic Pulmonary Medicine Answer E. CD4 of 450. Explanation Having a CD4 count that high would go very much against MAI/MAC as an etiology. Tuberculosis would be much more likely. None of the other factors by themselves would help differentiate tuberculosis from MAI/MAC.
A 62-year-old male presents with a 3-year history of dyspnea on exertion. He has a chronic dry cough without any specific triggers. He is a lifelong nonsmoker and spent most of his working life in a chemical plant dealing with petroleum. His oxygen saturation at rest is 93% but decreases to 86% with exertion after 3 minutes. He has digital clubbing but no cyanosis or edema. His lung exam reveals dry, coarse inspiratory crackles without wheezing. A chest CT is performed. It reveals bibasilar fibrosis with traction bronchiectasis and subpleural blebs without any ground glass abnormalities. The patient is sent for surgical lung biopsy, and the pathology is read as "UIP pattern with fibroblastic foci, temporal heterogeneity, honeycombing appearance, and minimal inflammation." What is the most likely diagnosis? a) Asbestosis b) Acute respiratory distress syndrome c) Sarcoidosis d) Emphysema e) Idiopathic pulmonary fibrosis
Topic Pulmonary Medicine Answer E. Idiopathic pulmonary fibrosis Explanation Idiopathic pulmonary fibrosis (IPF) is a diagnosis of exclusion with histopathologic findings of usual interstitial pneumonitis (UIP). Heterogenous changes are noted on biopsy ranging in stages from early with inflammation to late with severe fibrosis. Patients generally present in middle age with dry cough and progressive dyspnea. Biopsy will confirm the diagnosis of UIP or exclude other diagnosis. However, in the absence of any evidence to support an alternative diagnosis (serologic tests, drug/toxin/occupational exposure, other history), and then radiographic evidence on chest CT, this may be adequate to make a diagnosis without surgical lung biopsy. Characteristic radiographic findings include bibasilar reticular opacities, minimal ground glass opacities, honeycombing, and traction bronchiectasis. Sarcoidosis can present with areas of fibrosis along with areas of inflammation, but biopsy typically will reveal non-caseating granulomas. Acute respiratory distress syndrome is a rapid onset disorder that leads to respiratory failure. This patient's history is more insidious in onset, so it is inconsistent. Emphysema will not have the chest CT findings or physical exam findings of crackles. A biopsy would reveal emphysematous areas of lungs with destroyed alveolar walls. Asbestosis does cause pulmonary fibrosis with similar radiographic and pathologic findings of IPF. However, there is no history to suggest asbestos exposure, so asbestosis would be unlikely.
A usually healthy 38-year-old woman comes to you because of a 3-day history of pain and swelling of her right leg. She has smoked 1 pack of cigarettes daily for 20 years. Her only medication is oral contraceptives. Her pulse is 78 bpm and regular. Her right calf is edematous and tender. The peripheral arterial pulses are present and normal in upstroke. Venous duplex ultrasonography shows that the superficial and deep femoral veins are not compressible. Which of the following is the most appropriate next step in management? a) V/Q scan b) Insertion of an inferior vena cava (IVC) filter c) Observation d) Thrombolytic therapy e) Low-molecular-weight heparin therapy
Topic Pulmonary Medicine Answer E. Low-molecular-weight heparin therapy Explanation This patient has symptomatic calf thrombosis with propagation of the clot across the popliteal fossa into the deep circulation. Treatment goals should be aimed at stabilizing the clot, preventing it from becoming larger, and preventing new clots from forming in other vessels. Some form of heparin followed by the oral administration of warfarin remains the mainstay of treatment. Subcutaneously administered low-molecular-weight (LMW) heparin and unfractionated heparin are equally effective. Heparin may be discontinued after the warfarin has maintained the INR in a therapeutic range, between 2 and 3, for 48 hours. The duration of warfarin therapy depends on the individual patient and is usually continued for at least 3 months. Alternatively, LMW heparin may be continued for the full duration of treatment and is preferred therapy in the pregnant female and in patients with cancer. Observation is incorrect because the DVT is causing symptoms. A V/Q scan is unnecessary because 1) she has no symptoms of a pulmonary embolism, and 2) even if a PE is present, management will be the same. The patient doesn't meet criteria for an IVC filter. Thrombolytics are used only in unstable patients.
A 30-year-old health care worker was recently found to have a PPD of 12 mm on his routine screen. He is about to be started on INH therapy of 300 mg daily for 9 months. He is otherwise healthy and has no complaints. His CXR was normal. PAST MEDICAL HISTORY: Negative; except for gonorrhea at age 18 SOCIAL HISTORY: Lives with mother in a motel Drinks on occasion; 2 beers at most on a weekend FAMILY HISTORY: Mother 60, healthy; runs the motel Father died at age 30; murdered at the motel No siblings REVIEW OF SYSTEMS: Negative Denies fever, chills, sore throat Occasionally hears voices—knows they are not real PHYSICAL EXAMINATION: BP 110/70, P 55, RR 14, Temp 98.5° F Well-developed, well-nourished man in no distress HEENT: PERRLA, EOMI TMs clear Throat clear Neck: Supple, no masses Heart: RRR with S4 gallop (runs marathons) Lungs: Scattered rhonchi; cleared with cough Abdomen: No masses; no hepatosplenomegaly; nontender Extremities: No cyanosis, clubbing, or edema You are about to start him on INH. Of the following options, when should you check screening laboratory (AST, ALT, and bilirubin)? a) Today, then q month for 3 months; none after that unless has problems b) Today, then q month for 3 months, then every other month until completes therapy c) Today, then q month until therapy complete d) Today, then in 1 week, then q month until finishes therapy e) None needed unless clinical symptoms/problems develop
Topic Pulmonary Medicine Answer E. None needed unless clinical symptoms/problems develop Explanation He is a healthy person without evidence of liver disorder. If he had evidence during the initial evaluation to suggest a liver disorder, then you would draw baseline serum alanine aminotransferase (ALT) and bilirubin. Baseline testing is also indicated for patients with HIV infection treated with HAART, pregnant women, women in the immediate postpartum period (within 3 months of delivery), persons with a history of chronic liver disease (hepatitis B or C, alcoholic hepatitis, or cirrhosis), persons who use alcohol regularly, and persons at risk for chronic liver disease. Baseline testing is no longer absolutely indicated in older persons (> 35 years old), although some experts recommend that baseline and scheduled ALT testing be done in those older than 35 years. For patients with chronic conditions on medications that could cause problems, testing may be warranted. After baseline testing, individuals may be followed every 2-4 weeks depending on the severity of their clinical condition or if baseline laboratory is abnormal. All patients regardless of age or health status require clinical monitoring! This includes educating patients about signs and symptoms that might indicate a problem with the medication. These include any of the following: unexplained anorexia, nausea, vomiting, dark urine, icterus, rash, persistent paresthesias of the hand and feet, persistent fatigue, weakness or fever lasting 3 or more days, abdominal tenderness (especially right upper quadrant discomfort), easy bruising or bleeding, and arthralgia. Clinical monitoring begins at the first visit and should be done monthly.
A 65-year-old man presents with progressive dyspnea on exertion associated with a dry cough. This has been occurring for over two years. He has a 25 pack-year history of smoking. Occupational history includes over 30 years working in a factory that manufactured glass bowls and vases, as well as 2 years working in an auto body shop doing brake repair jobs. His chest radiograph is described as: "bilateral apical regions of parenchymal fibrosis with coalescent areas of consolidation." Previous testing indicates an FEV1/FVC ratio of 80%, FEV1 that is 1.5 L (56% predicted), and a TLC of 3.50 L (50% predicted). Which of the following occupational causes of interstitial lung disease is associated with a higher risk of tuberculosis? a) Berylliosis b) Byssinosis c) Coal worker's pneumoconiosis d) Asbestosis e) Silicosis
Topic Pulmonary Medicine Answer E. Silicosis Explanation Silicosis is an occupational lung disease characterized by restrictive lung function. Occupational exposures include any exposure to silica dust, which can occur in mining, stone cutting, quarrying, road and building construction, glass manufacturing, and sand blasting. Typically, patients will have biapical distribution of parenchymal lung disease but often will have areas of "eggshell calcification" of hilar lymph nodes. Silicosis is associated with a higher risk of contracting tuberculosis and developing lung cancer as well as strong association with scleroderma and rheumatoid arthritis. Coal worker's pneumoconiosis will have similar radiographic and PFTS findings to silicosis. Eggshell calcification is not associated with this, and there is no increased risk of TB or cancer. Berylliosis is seen in patients with exposure to electronics, nuclear material, or ceramics. Radiographically, it presents with bilateral hilar lymphadenopathy, similar to the appearance of sarcoidosis. Asbestos exposure does carry an increased risk of developing lung cancer, but this association is not seen for tuberculosis. Byssinosis is related to exposure to impurities in cotton, flax, and hemp dusts. It often develops after >10 years of exposure and is associated with a temporal relationship to exposures. For example, patients may complain of difficulty of breathing on the first day of the workweek. It is associated with a gram-negative exotoxin but not tuberculosis.
*Pertussis* Second-line therapy if allergic to macrolides (Erythromycin, Azithromycin)
Trimethoprim-sulfamethoxazole
Treatment for PJC pneumonia?
Trimethoprim-sulfamethoxazole or pentamidine
Question: In a patient with AIDS, what medication is given prophylactically at a CD4 count of less than 100 cells/μL to prevent infection with toxoplasmosis gondii?
Trimethoprim/sulfamethoxazole
Coal miners lungs often shows nodular opacities in what lung fields?
Upper lung fields
Which vitamin should be given along with isoniazid (INH) therapy?
Vitamin B6 (pyridoxine)
*SPN* Tx
If can't determine it as benign, surgical removal.
Nodular opacities in the upper lung fields should make you think of what diagnosis?
Silicosis
What is the Dx? Child presents with dysphagia, drooling and respiratory distress. Lateral neck X-ray shows "thumbprint sign".
Acute Epiglottitis
A 60-year-old man presents to your office for routine yearly follow-up. He has mild hypertension for which he takes lisinopril. He is otherwise healthy. He denies cough or shortness of breath. He used to smoke 1 ppd of cigarettes for 20 years but quit 6 years ago. He works in a manufacturing plant, and as part of the yearly physical, he underwent Pulmonary Function Tests. The results are as follows: FEV1/FVC 65%, FEV1 2.4 L (82% predicted). There was no bronchodilator response. A chest radiograph was read as "unremarkable." Which one of the following recommendations would be most appropriate for this patient? a) Start nocturnal oxygen. b) Start long-acting anti-cholinergic agent once daily. c) Start long-acting beta-agonist twice daily. d) Initiate pulmonary rehab program. e) Vaccinate for influenza annually.
Topic Pulmonary Medicine Answer E. Vaccinate for influenza annually. Explanation This case scenario outlines a patient with asymptomatic fixed airflow obstruction with a significant smoking history, most consistent with a diagnosis of chronic obstructive pulmonary disease (COPD). Based on his spirometry, he has GOLD Criteria Stage I COPD. Guidelines would recommend yearly influenza vaccination, short-acting bronchodilator as needed, and routine spirometric follow-up annually. Long-acting beta-agonists and anti-cholinergics are considered for patients with symptoms and Stage II or more severe COPD. They have been shown to improve symptoms and reduce the number of exacerbations in this patient population. Pulmonary rehabilitation should also be considered in this same group and has been shown to improve exercise capacity, decrease hospitalizations, and enhance quality of life. This scenario doesn't provide data regarding oxygen testing, so this therapy would not be initiated based on the available information. However, oxygen therapy has been shown to increase survival in those patients with documented oxygen desaturations if used for > 15 hours/day.
What does positive egophony on auscultation in a patient with pneumonia mean?
When listening over the area of consolidation, patient will say "eee" and it sounds like "aaa"
Hamptom's hump
Peripheral wedge shaped opacity representing pulmonary infarct.
istory & Physical/Pulmonology Upon auscultation of a patient's lungs, there are harsh, hollow breath sounds which have a long inspiratory component in the region of the suprasternal notch. Throughout the periphery of the lung fields, softer breath sounds are heard. Which of the following best describes these findings? A. Normal B. Asthmatic C. Atelectasis D. Foreign body Explanations
(c) A. Bronchial breath sounds are normally heard near the sternum and vesicular breath sounds are heard over the periphery of the lungs in a healthy, normal patient. (u) B. Breath sounds in an asthmatic patient are usually obscured by wheezing. (u) C. Breath sounds are usually absent over an area of atelectasis. (u) D. Foreign body aspiration can present with stridor, wheezing or decreased breath sounds depending on where it has lodged.
Clinical Therapeutics/Pulmonology A patient is brought to the emergency room with acute onset of dyspnea and tachypnea. He has a long history of alcoholism and was involved in a motor vehicle accident two days ago. He is hypoxic with crackles auscultated bilaterally. Chest radiography reveals diffuse bilateral infiltrates which spare the costophrenic angle and air bronchograms, there was no cardiomegaly or pleural effusion noted. Oxygen saturation is 70%. Which of the following is the most important initial treatment? A. Tracheal intubation B. Bilateral chest tube insertion C. Type-specific packed cells D. Colloid solutions
(c) A. Tracheal intubation with lowest level of PEEP is required to maintain the PaO2 above 60mmHg or SaO2 above 90% in a patient with ARDS. (u) B. See A for explanation. (u) C. Fluids are the preferred treatment initially for hypovolemia. Type-specific packed cells are given when the patient's blood type is identified. Until then O negative packed cells are administered. (u) D. Use of crystalloid solutions are preferred to avoid pulmonary edema.
*PNA* Tx of Hospital Acquired
**Anti-pseudomonal B-lactam and anti-pseudomonal AG or FQ** - Cefepime - Ceftazidime - Piperacillin/tazobactam
*TB* Tx:
*2 months* of "RIPE" therapy; 4 month continuation with INH and Rifampin.
*Pneumoconioses* Which type affects the lower lobes? The upper? (2)
*Asbestosis* affects the lower lobs. *Coal worker's* and *silicosis* affects the upper lobes.
*PNA* Treatment of CAP, ICU (2 options)
*Beta-lactam + macrolide* (Ceftriaxone + Azithromycin) *Beta-lactam + broad spectrum FQ* (Ceftriaxone + Levofloxacin, Moxifloxacin)
Pulm HTN can be secondary to (3) pulmonary diseases:
*COPD* OSA PE
*TB* Imaging of choice? Findings?
*Chest x-ray (CXR)* is the diagnostic test of choice and classically shows *upper lobe infiltrates with CAVITATION and/or lymphadenopathy*; may also show *a solitary nodule (Ghon complex)* and diffuse infiltrates that may represent bronchogenic spread
What are the chest X-ray findings with acute respiratory distress syndrome (ARDS)?
Bilateral infiltrates/white out (may look like CHF)
*Asthma* Additional Tx (2) to SABA during exacerbation
*Ipratropium*, an anticholinergic (central bronchodilator) *Corticosteroids* - send home on Prednisone, methylprednisolone (Solumedrol), Prednisolone (Prelone)
*Sarcoidosis* What physical exam finding is pathognomonic for sarcoidosis?
*Lupus pernio*: Violaceous, raised discoloration of the nose, ear, cheek, and chin.
*COPD* Gold standard Dx
*PFTs.* May show obstruction (decreased FEV1, FEV1/FVC) or hyperinflation (increased lung volumes, residual volumes).
Inflammation of the trachea/bronchi following a URI. Cough x1-3 weeks. CXR negative for PNA.
Acute bronchitis
RSV is part of the ___ family
Paramyxovirus
*COPD* CB may have what abnormality on CBC?
*Polycythemia* (increased HCT/Hgb) secondary to compensation for chronic hypoxia
Types of lung CA (4)
1. *Small Cell (Oat Cell) Carcinoma* 2. *Non-Small Cell Carcinoma* - Adenocarcinoma - Squamous Cell - Large Cell (Anaplastic) Carcinoma
*TB* Diagnosis (3 ways)
1. *Sputum*: Acid-fast stain, NAAT 2. *Skin*: Mantoux test, +PPD 3. *Blood*: Quantiferon
At least how many antibiotics should be used at the same time for pseudomonas pneumonia?
Always treat with at least two antibiotics
Associated with cavitary lesions, hypercalcemia, and pancoast syndrome
*Squamous Cell Carcinoma* Think CCCP: *C*entrally located, *c*avitary lesions, hyper*c*alcemia, *p*ancoast syndrome
*PNA* *Fungal* Treatment (2)
Amphotericin B Itraconazole
*Sarcoidosis* Dx (5)
1. 50% of cases are Asx and found incidentally on *CXR* as bilateral hilar lymphadenopathy. There may also be interstitial lung disease. Can also do a CT. 2. *Tissue Bx* showing noncaseating granulomas. 3. PFTs. Restrictive pattern seen in advanced disease. 4. Gallium scan. Inc uptake in affected areas. Parotid/salivary glands. 5. Bronchoalveolar lavage to r/o infectious causes.
One Step Further Question: What is a Westermark sign on X-ray?
Answer: Westermark sign is a rare indication of pulmonary embolus (PE) on X-ray. The X-ray would show a sharp cut-off of pulmonary vessels with distal hypoperfusion.
What are the combinations of antibiotics which may be used for pseudomonas pneumonia?
Anti pseudomonal beta-lactam PLUS an antipseudomonal quinolone or aminoglycoside - Antipseudomonal quinolone PLUS an aminoglycoside
*COPD* Treatment of exacerbations (4 options) in UNCOMPLICATED COPD
Antibiotics: 1. Doxycycline 2. TMP-SMX 3. Macrolide (Azithromycin, Clarithromycin) 4 Cephalosporins: Cefuroxime, Cefpodoxime, Cefdinir
Lambert-Eaton syndrome
Antibodies against calcium-gated channels at NMJ, causing weakness similar to Myasthenia Gravis, but LE weakness improves with continued use (MG worsens)
*Epiglottitis* Diagnosis 1. Most often made ____ 2. Definitive 3. Can assist
1. Clinically 2. Direct laryngoscopy *in a controlled environment 3. Lateral neck x-ray (thumbprint sign)
*PNA* 1st-line Tx of CAP, OP 1. Class of medications
1. Macrolides (Azithromycin, Clarithromycin) 2. Doxycycline
Horner's syndrome (3)
1. Miosis 2. Ptosis 3. Anhydrosis D/t cervical cranial sympathetic compression
*PNA* Atypical bugs (4)
1. Mycoplasma pneumonia 2. Chlamydophila pneumoniae 3. Legionella pneumophilia 4. Viruses
*Asthma* When do admit? (4)
1. PEFR <50% of predicted 2. ER visit within 3 days of prior exacerbation/post-treatment failure 3. AMS 4. Status asthmatics
*Carcinoid Tumor* Tx 1. Definitive 2. Symptomatic
1. Surgical resection. Often resistant to radiation/chemo. 2. Octreotide (decreases active hormones).
At what induration, would a PPD be positive for a healthcare worker?
10 mm
Question: What is the most useful test to confirm a diagnosis of sarcoidosis?
Biopsy of the affected organ. Sarcoidosis African-Americans, females Primary target organ: lungs Parotid enlargement Hypercalcemia CXR: bilateral hilar adenopathy Biopsy: noncaseating granulomas Steroids
Flu can be treated with oseltamivir within how many hours of onset of symptoms?
48 hours
How long after administering a PPD should it be read?
48-72 hours
Normal FEV1 is
>80% of FVC (exhaling as much as you can after inhaling as much as you can)
*Influenza* In the United States, the recommendation is that all individuals ____ receive inactivated influenza vaccine yearly
>6 months
*TB* Treatment active
>6 months *RI*PE therapy
*Asthma* RF
Atopy
*Bronchiolitis* MC acute complication (of RSV)
AOM
Acute onset of respiratory failure d/t increased permeability of alveolar capillary membranes → pulmonary edema, hypoxia, and dyspnea. ABG shows respiratory acidosis. Can be caused by *sepsis (#1)*, multiple trauma, aspiration, DIC, shock, blood transfusion, pancreatitis.
ARDS
Severe refractory hypoxemia is hallmark symptom.
ARDS
A very sick patient with diffuse infiltrates that spare the costophrenic angles should make you think of what diagnosis?
Acute respiratory distress syndrome (ARDS)
What is the Dx? Looks like CHF on chest X-ray but pulmonary wedge pressure in normal.
Acute respiratory distress syndrome (ARDS)
*PNA* *Viral* Treat HSV with
Acyclovir
MC lung CA type in smokers, women, and nonsmokers?
Adenocarcinoma
MC type of lung Ca? (Also MC in non-smokers)
Adenocarcinoma
This type of CA is typically located *peripheral* and arises from mucous glands.
Adenocarcinoma
What is the most common type of lung cancer in nonsmokers?
Adenocarcinoma
What two lung cancers are typically found peripherally on CXR?
Adenocarcinoma and Large Cell
One Step Further Question: What is the most common form of lung cancer?
Adenocarcinoma.
What occupations are associated with berylliosis?
Aerospace and nuclear plant workers
Question: What is the prognosis for a patient with dilated cardiomyopathy?
Approximately 1/3 worsen and die, 1/3 recover completely, and 1/3 have residual cardiac dysfunction.
Insultation, Ship Building
Asbestosis
Patient with a history of building renovation/insulation, ship building, or pipe fitting presents with DOE, nonproductive cough. CXR shows *pleural plaques* (thickening), and *interstitial fibrosis* (honeycombing).
Asbestosis
What is the Dx? Patient worked with insulation. Chest X-ray shows thickened pleura and basilar lesions.
Asbestosis
Lung biopsy may show "brown rods" due to iron/protein deposits.
Asbestosis. The "brown rods" are linear asbestos bodies.
*PNA* *Fungal* Occurs in immunocompromised. Waterfowl, fungal ball on CXR. Dx/Tx?
Aspergillosis, treat with any antifungal
Which of the following is the most common dysrhythmia associated with the diagnosis of pulmonary embolism? Atrial fibrillation AV-nodal reentrant tachycardia Multifocal atrial tachycardia Sinus tachycardia
Atrial fibrillation (A) can be seen with pulmonary embolism but is less common than sinus tachycardia. Atrioventricular nodal reentry tachycardia (AVNRT) (B) is the most common type of reentrant supraventricular tachycardia. Pulmonary embolism is an unlikely cause of AVNRT Multifocal atrial tachycardia (MAT) (C) is a dysrhythmia that is seen in a variety of clinical disorders. It is more common in the elderly and is associated with COPD and hypoxia. In addition to a heart rate greater than 100 beats per minute, the characteristic ECG feature is variability in P wave morphology, with each unique P wave morphology felt to indicate a different site of atrial origin. Patients with multiple P wave morphologies but a normal heart rate are considered to have a wandering atrial pacemaker because the heart rate does not meet criteria for a tachycardia.
What is the treatment for pneumocystis jiroveci pneumonia?
Bactrim
Question: What is a physiologic complication of noninvasive positive pressure ventilation?
Barotrauma including pneumothorax.
*SPN* Calcification is usually seen in benign or malignant lesions?
Benign
What is the Dx? Nuclear plant worker with shortness of breath?
Berylliosis
Which pneumoconiosis has diffuse infiltrates and hilar lymphadenopathy?
Berylliosis
Patient may have a history of exposure to electronics, aerospace, ceramins, tool and dye manufacturing, fluorescent light bulbs. About 50% have normal CXR. Other 50% will either have hilar lymphadenopathy or increased interstitial lung markings. Disease? How to diagnose? (3)
Berylliosis Diagnosis based of: 1. History 2. *Positive beryllium lymphocyte proliferation test* 3. Biopsy demonstrating *noncaseating* granuloma
Aerospace, electrical plants
Beryllium
List 4 medications which may cause asthma.
Beta blockers, NSAIDs, ACEI, ASA
Question: What is the prevalence of idiopathic pulmonary fibrosis in the U.S.A?
Between 14 and 43 per 100,000 persons in the US. Idiopathic Pulmonary Fibrosis Patient will be a male With a history of smoking Complaining of chronic dry cough and dyspnea Labs will show restrictive characteristics - decrease FVC and FEV1 but a near normal FEV1/FVC ratio CXR will show "honeycombing" Treatment is O2, pulmonary rehabilitation
*Cor Pulmonale* Surgical treatment (3)
Biventricular pacing VAD Transplant
Which pneumoconiosis has nodular opacities in the upper lung fields on chest X-ray?
Black lung from coal mining
*PNA* Endemic in North America around the Great Lakes, Ohio river basin, and Mississippi River. Broad base budding organism; extrapulmonary lesions in skin, bone, prostatitis. Diagnosis and Treatment?
Blastomycosis; oral Itraconazole
List the four most common places lung cancer will metastasize to.
Bone, Brain, Adrenals and Liver
Organism that causes whooping cough?
Bordetella pertussis
What are the three most common cancers to metastasize to the lungs?
Breast, liver and colon
Smoker, chronic productive cough. NO hemoptysis, wt. loss.
Brochitis (COPD)
A rare neuroendocrine (enterochromaffin cell) tumor, that may secrete serotonin, ACTH, ADH, melanocyte stimulating hormone.
Bronchial carcinoid tumors
Child with chronic persistent productive cough foul smelling sputum?
Bronchiectasis
Chronic aspiration in kids leads to plate like atelectasis on X-ray and what condition?
Bronchiectasis
MCC massive hemoptysis
Bronchiectasis
A CXR shows vascular redistribution and blurred vascular outlines. What is the most likely diagnosis?
CHF
What conditions is a right sided pleural effusion often associated with?
CHF or cirrhosis
*PNA* Viral PNA that occurs in transplant patients, and patients with AIDs
CMV
*COPD* Testing other than PFTs (2)
CXR/CT: hyperinflation/flat diaphragm in emphysema; increase in AP diameter in both. *ECG* may show signs of cor pulmonale such as RAH, RVH, RAD, and R-CHF.
*Pleural Effusion* Diagnosis? Which is best?
CXR: left lateral decubitus films the best (can detect smaller ones)
*PTX* Type that is seen during menstruation d/t ectopic endometrial tissue in the pleura
Catamenial
*Pertussis* Most contagious during which phase?
Catarrhal (1)
*TB* Appearance of reactivation TB on CXR?
Cavitary lesions in upper lobes
Sputum cytology can Dx which types of lung CA?
Central lesions like squamous or SCLC
List three classes of antibiotics which may be used for Klebsiella pneumoniae.
Cephalosporins - Aminoglycosides - Fluoroquinolones
List four indications for the flu vaccine.
Child 6-59 months; Pregnant female; Healthcare worker; Age greater than 50
A college student presents with pneumonia. What two organisms should you be thinking of?
Chlamydia and mycoplasma
Reticular to nodular pattern on CXR
Coal Miner's lung
Coal
Coal Workers Pneumoconiosis
Smoking, decreased FEV1
Chronic Bronchitis
*Blue bloaters (hypoxia)*, treat with ipratropium inhaler
Chronic bronchitis
*Productive cough* over 3 months x 2 consecutive years. Rales, rhonchi, wheezing. Signs of *cor pulmonale* (peripheral edema, cyanosis).
Chronic bronchitis
Blue bloater?
Chronic bronchitis
Is chronic bronchitis or emphysema associated with an elevated hemoglobin?
Chronic bronchitis
What are the two types of COPD?
Chronic bronchitis and emphysema
*PNA* Tx Aspiration PNA (3 options)
Clindamycin or Metronidazole or Augmentin (covers anaerobes)
Caplan syndrome
Coal worker's pneumoconiosis + rheumatoid arthritis
Transudative pleural effusions should make you think of what diagnosis?
Congestive heart failure
Right sided heart failure due to pulmonary HTN?
Cor Pulmonale
Right heart failure caused by long term COPD history?
Cor pulmonale
Right heart failure d/t lung disease. Sx based on fluid back-up to body.
Cor pulmonale
A 55-year-old obese man presents to the clinic complaining of fatigue and poor concentration for the past six months. His wife reports that he snores nightly and often sounds like he gasping for air. Which of the following would most likely be found on physical exam? A large neck circumference Coarsening of facial features Increased skin pigmentation Moon facies
Correct Answer ( A ) Explanation: A large neck circumference would most likely be found on physical exam in this man with symptoms concerning for obstructive sleep apnea. Cardinal features of obstructive sleep apnea include irregular and abnormal respiratory patterns, loud snoring and snorts during sleep. Symptoms attributable to disrupted sleep include sleepiness, fatigue and poor concentration. Well-defined risk factors for obstructive sleep apnea include older age, male gender, obesity, craniofacial abnormalities, and upper airway soft tissue abnormalities. Tonsillar hypertrophy, macroglossia, retrognathia/micrognathia, and upper airway mass lesions can contribute to upper airway narrowing. The physical exam can be normal, although obesity, elevated blood pressure, a narrow airway, and a large neck circumference are common. Obstructive sleep apnea is more strongly correlated with an increased neck size or waist circumference than general obesity and is particularly prominent among men who have a collar size greater than 17 inches and women who have a collar size greater than 16 inches. Moon facies (D) is associated with Cushing syndrome, not obstructive sleep apnea. Increased skin pigmentation (C) is sometimes seen in Addison disease and coarsening of facial features (B) is seen with acromegaly. Neither of these physical exam findings are associated with obstructive sleep apnea.
What childhood vaccine protects children from pertussis?
DTaP
Dyspnea after surgery, travel (airplane), LE Fx. May have c/o calf pain also.
DVT/PE
One Step Further Question: Which antibiotic should be used if macrolide resistance is suspected or documented in a patient with atypical pneumonia?
Doxycycline or a fluoroquinolone.
Abnormal, permanent enlargement of terminal airspaces. Loss of elastic recoil, increased compliance, and air trapping/hyperinflation. *Dyspnea is the MC Sx*
Emphysema
Bullae and blebs on CXR should make you think of what diagnosis?
Emphysema
Consequence of destruction of alveolar septae = *Pink puffers* - The body's natural response to ↓ lung function is chronic hyperventilation. Minimal cough, quite lungs, *thin, barrel chest*. Chest X-ray will reveal flattened diaphragm, hyperinflation and small, thin appearing heart.
Emphysema
A patient without a functioning spleen is more susceptible to what type of organisms?
Encapsulated organisms most commonly Strep pneumoniae and H. Flu.
HIB. Hot potato voice, sniffing position, drooling. X-ray: Thumb sign
Epiglotitis
A young child presents to ER and you see him waiting in tripod position. What is the most likely diagnosis?
Epiglottis
Drooling, sniffing position, tripod, toxic
Epiglottitis
HIB. Hot potato voice, sniffing position, drooling. X-ray: Thumb sign
Epiglottitis
A very aggressive type of non-small cell carcinoma
Large cell (anaplastic) carcinoma
Patchy fibrosis on CXR
Farmers lung
s/p thoracic trauma. Multiple rib fractures. Chest wall moves in with inspiration, out with expiration.
Flail chest (pain control, incentive spirometry, pulmonary toilet, intubation)
*PNA* *Fungal* Treatment of coccidiodes or "valley fever" (2)
Fluconazole Amphotericin B
List the antibiotics classes for outpatient treatment in a healthy patient with pneumonia caused by S. pneumoniae or non MRSA staph.
Fluoroquinolone doxycycline or macrolide
List the antibiotics classes for outpatient treatment in a patient with comorbidities and a pneumonia caused by Strep pneumoniae or non MRSA staph.
Fluoroquinolone doxycycline or macrolide PLUS a beta lactam
Treatment to prevent Hyaline membrane disease in the newborn?
Give antenatal corticosteroids
*PNA* S. Aureus gram stain
Gram + cocci in clusters
*PNA* S. pneumoniae gram stain
Gram + cocci in pairs
*PNA* H. influenzae gram stain
Gram - rods (bacilli)
*PNA* Klebsiella pneumoniae gram stain
Gram - rods (bacilli)
*PNA* Pseudomonas aeruginosa gram stain
Gram - rods (produces slime coat)
*Pertussis* *Pertussis* Bordetella pertussis gram stain
Gram negative coccobaccilus
*TB* Bx of TB will reveal:
Granulomas with CASEATING necrosis.
*PNA* MCC atypical (walking PNA); common in school-aged, college students, military recruits
H. influenzae
Does FEV1 go up or down with asthma?
It goes down
Question: What organism is associated with pneumonia in patients with COPD?
Haemophilus influenzae,Pseudomonas aeruginosa. Bacterial Pneumonia S. pneumonia: most common, rusty colored sputum, rigors, gram+ paired lancets Klebsiella: alcoholics, currant jelly sputum, bulging fissures, S. aureus: IVDA, postinfluenza, elderly, gram+ cocci in clusters H. influenzae: COPD, gram negative pleomorphic rods Pseudomonas: cystic fibrosis, nursing home resident and cyanosis Health care associated pneumonia: pseudomonas, MRSA Outpatient, healthy: macrolide or doxycycline Outpatient, comorbidity: respiratory tract fluoroquinolone (RTF) Inpatient: RTF ICU: antipneumococcal ß-lactam (ceftriaxone or cefotaxime) + either azithromycin or an RTF
What must be considered with a young patient who has been exposed to rodent feces and has a "CHF like" presentation?
Hanta virus
What happens to total lung capacity in sarcoidosis?
It goes down
Does total lung capacity go up or down in COPD?
It goes up.
*Pulm HTN* Definitive management
Heart-lung transplant
Does patient with sarcoidosis tend to have high or low serum calcium levels?
High (hypercalcemia)
*PNA* Found in caves, Ohio valley, lower Mississipi region; grows in soil with bird or bat droppins.
Histoplasmosis
*Croup* (Laryngotracheitis) Treatment of severe (stridor at rest, marked retractions)
Hospitalize... 1. Dexamethasone 2. Nebulized epinephrine
Preemie CXR w/ hypoexpansion (ATX), air bronchograms
Hyaline Mb Disease
Premature infant with respiratory distress
Hyaline Mb Disease
*Sarcoidosis* Treatment for chronic disfiguring skin lesions
Hydroxychloroquine
*Pulm HTN* Classification
I: Idiopathic/primary II: Secondary to left heart disease III: d/t hypoxemic or chronic lung disease (COPD) IV: d/t chronic thromboembolic disease
What is the initial treatment for active TB?
INH + RIF + PZA + EMB (RIPE) x several months typically followed by INH + RIF once sensitivities come back
*TB* Treatment of latent TB?
INH daily x9 months
*Asthma* Adjunctive therapy
IV magnesium (for severe cases)
Rapid Review Idiopathic Pulmonary Fibrosis
Idiopathic Pulmonary Fibrosis Patient will be a male With a history of smoking Complaining of chronic dry cough and dyspnea Labs will show restrictive characteristics - decrease FVC and FEV1 but a near normal FEV1/FVC ratio CXR will show "honeycombing" Treatment is O2, pulmonary rehabilitation
A 40-50 y/o man presents with gradual onset dyspnea and non-productive cough. PFTs show *restrictive pattern*: decreased lung volumes with *normal FEV1/FVC*. CXR/CT shows diffuse reticular opacities, or *honeycombing*.
Idiopathic fibrosing interstitial pneumonia (pulmonary fibrosis)
What is the Dx? Chronic dry cough dyspnea fatigue and clubbing. Chest X-ray shows fibrosis and CT chest shows honeycombing.
Idiopathic pulmonary fibrosis
*Pulm HTN* Tx of PRIMARY pulmonary HTN What 2 treatments long-term regardless?
If vasoreactive to NO, IV adenosine, or CCBs; *CCBs are first line Tx* If not: Prostacylcines, *PDE5 inhibitors*, or endothelial receptor antagonists (Bosentan) *O2 therapy (only one to dec mortality), long term anticoagulation
Question: Why is airway compromise in epiglottitis less common in adults than in children?
In adults, there is an increased trachea diameter to epiglottis diameter making airway compromise unusual. Epiglottitis Patient will be complaining of rapid onset of fever and dysphagia PE will show patient leaning forward, drooling, inspiratory stridor Imaging will show "thumbprint" sign Most commonly caused by H. influenzae, Streptococcus Treatment is IV antibiotics and airway management
*Asthma* What to do or add if an inhaled steroid is not effectively controlling asthma alone?
Increase ICS dose OR add LABAs: - Salmeterol Can do a ICS/LABA combo like Symbicort or Advair. Never use LABAs for acute management.
Tactile fremitus is increased with (2) Decreased with (2)
Increased with: 1. PNA 2. Nodules Decreased with: 1. Pleural effusion 2. PTX
What should be considered with a diffuse ground glass appearance and no pulmonary nodules on chest X-ray?
Infiltrative lung disease
Most common cause of viral pneumonia in adults?
Influenza
*Asthma* Drug of choice for longer term management
Inhaled corticosteroids - Beclomethasone - Flunisolone - Triamcinolone
*Asthma* Drug of choice for longer term management; mainstay of treatment for persistent asthma
Inhaled corticosteroids - Beclomethasone - Flunisolone - Triamcinolone
Types of asthma classifications?
Intermittent - Mild persistent - Moderate persistent - Severe persistent
*Asthma Classification* Use of SABA for Sx under twice a day, or twice a week total. Night awakenings under 2x per month. Tx?
Intermittent. SABA PRN.
*PNA* Legionella pneumophilia gram stain
Intracellular GNR; lives in aquatic environment
What is the recommended treatment for a healthcare worker with a first time positive PPD?
Isoniazid (INH) for 6 months
*TB* Treatment latent
Isoniazid x9 months
Question: What is the CURB-65 score?
It attempts to predict the severity of pneumonia to help determine inpatient vs. outpatient treatment. Legionella Pneumonia Patient will be complaining of fevers, malaise, myalgias, cough and GI symptoms Labs will show leukocytosis, elevated liver transaminases and hyponatremia CXR will show unilateral patchy alveolar lower lobe infiltrates Most commonly caused by gram-negative bacillus and is found in aquatic environments Treatment is azithromycin
*PNA* Consolidates in upper lobe (especially R); may cause bulging fissure
Klebsiella
*PNA* Currant jelly sputum
Klebsiella
Organism usually causing pneumonia after aspiration?
Klebsiella
PNA w/ DM, immunocompromised, EtOH. Currant color sputum.
Klebsiella
Pt with suspected pneumonia has "currant jelly colored" sputum. Most likely organism?
Klebsiella
Rapid, deep labored breathing
Kussmaul breathing - DKA, Metabolic acidosis
Question: What is the most common location of extrapulmonary TB?
Lymph nodes. Pulmonary Tuberculosis (TB) RFs: immunodeficiency, immigrant, close contact Latent/primary TB: asymptomatic Active/reactivation TB: fever, night sweats, weight loss, productive cough, hemoptysis Erythema nodosum Primary TB CXR: Ghon focus Active/reactivation TB: upper lobes, cavitary lesions Dx: sputum smears for acid-fast bacilli (AFB), sputum/tissue culture for AFB (gold standard) PPD: gold standard for latent TB dx Latent TB rx: 9 months of INH Primary TB rx: rifampin, INH, pyraziniamide, ethambutol (RIPE)
*Pertussis* Lab value
Lymphocytosis (60-80% lymphocytes on differential)
Name 2 antibiotics you can use to treat Legionella.
Macrolide or Fluoroquinolone
*PNA* *Atypical* Treat mycoplasma with:
Macrolides
*Pertussis* Treatment
Macrolides (Azithromycin, Clarithromycin)
What two antibiotic classes may be considered to treat legionella pneumonia?
Macrolides and fluoroquinolones
Question: What are the most common sites of hemorrhage in massive hemoptysis?
Massive hemoptysis typically involves the bronchial or pulmonary arteries.
*Asthma* What drugs can be used as prophylaxis only?
Mast cell modifiers - Cromolyn - Nedocromil
One Step Further Question: What are the contraindications to using antenatal corticosteroid therapy?
Maternal systemic infections, namely chorioamnionitis. Infant (Neonatal) Respiratory Distress Syndrome ↓ Fetal surfactant Risk factors: prematurity (most common), maternal diabetes, c-section Respiratory difficulty within a few hours after birth CXR: ground glass appearance Rx: O2, intubation, CPAP, surfactant Concern for premature birth: prenatal glucocorticoids O2 complications: blindness, bronchopulmonary dysplasia
Pleural thickening on CXR
Mesothelioma
What cancer is a patient with asbestos exposure at increased risk for.
Mesothelioma
Where does the cancer associated with asbestosis tend to locate in the lung?
Mesothelioma locates to the pleural lining at the base of the lung
Question: What is a side effect of dapsone that can lead to hypoxia?
Methemoglobinemia. Pneumocystis Pneumonia (PCP) Patient with a history of HIV Complaining of gradual onset of non-productive cough Labs will show CD4 < 200, increased LDH CXR will show bilateral infiltrates (bat wing pattern) Most commonly caused by Pneumocystis jirovecii Treatment is TMP-SMX
*Symptoms ≤2 days/week, nighttime awakenings ≤2×/month*, short-acting β-agonist use ≤2 days/week, no interference with normal activity
Mild intermittent asthma
*Symptoms >2 days/week but not daily, nighttime awakenings 3-4×/month*, short-acting β-agonist use >2 days/week but not daily, minor limitations in normal activity
Mild persistent asthma
*Asthma Classification* Use of SABA for Sx over twice a week, but not every day. Night awakenings under 3-4x per month. Minor limitation of activity. Tx?
Mild. SABA PRN + low dose ICS.
Class 2 pulmonary HTN causes?
Mitral stenosis - Aortic stenosis -Left ventricular hypertrophy - Left heart failure
*Asthma Classification* FEV 60-80% predicted
Moderate
*Daily symptoms, nighttime awakenings ≥1×/week but not nightly*, daily use of short-acting β-agonist, some limitation in normal activity
Moderate persistent asthma
*Asthma Classification* Use of SABA for Sx daily. Night awakenings under once or more per week, but not nightly. Some limitation of activity. Tx?
Moderate. SABA PRN + medium dose ICS *or* SABA PRN + low dose ICS + LABA.
SCLC Tx
Most are treated w chemo
What organism is associated with positive cold agglutinins?
Mycoplasma
+/- sputum, chills, muscle aches"
Mycoplasma pneumonia
Bullous myringitis
Mycoplasma pneumonia
*Bronchiolitis* Diagnosis
Nasopharyngeal swab (monoclonal Ab testing)
*Influenza* Diagnosis
Nasopharyngeal swab for PCR (best sensitivity/specificity) vs. rapid antigen detection (sensitivity 50-70%)
Where are squamous cell lung cancers typically located? What could this lead to?
Near/in central airways; can lead to atelectasis or post-obstructive PNA
*Influenza* Treatment
Neuramidase inhibitors: Oseltmavir. Best within 48 hours of Sx onset.* *Give to sick patients in hospital regardless of timing
What is a paraneoplastic syndrome?
Neurological symptoms (i.e. difficulties walking or swallowing, loss of muscle tone/fine motor skills, slurred speech, memory loss, vision changes, seizures, vertigo, and more) that develop as a result of the *patient's own immune response to the neoplastic process*
One Step Further Question: What histopathological biopsy finding is typically seen in patients with sarcoidosis?
Noncaseating granulomas.
*IPF* Tx
None. Smoking cessation, oxygen, steroids in some acute exacerbations. *Lung transplant is the only cure*.
*PTX* Treatment of a small PTX (15-20% of diameter of hemithorax)
Observe; serial CXR to confirm not enlarging and F/U in 24-48 hours
Poor sleeping, obese, daytime fatigue & drowsy, snoring, HTN, PM wakening
Obstructive sleep apnea
*Asthma* Biologic medication used in severe, uncontrolled asthma
Omalizumab
List the three most commonly used medical treatments for COPD?
Oxygen, ipratropium and albuterol
Ethambutol has what two major side effects?
Optic neuritis, red green vision loss
What is the common side effect of rifampin?
Orange discoloration of body fluids. Pulmonary Tuberculosis (TB) RFs: immunodeficiency, immigrant, close contact Latent/primary TB: asymptomatic Active/reactivation TB: fever, night sweats, weight loss, productive cough, hemoptysis Erythema nodosum Primary TB CXR: Ghon focus Active/reactivation TB: upper lobes, cavitary lesions Dx: sputum smears for acid-fast bacilli (AFB), sputum/tissue culture for AFB (gold standard) PPD: gold standard for latent TB dx Latent rx: 6-9 months of INH Primary rx: Rifampin, INH, Pyrazinimide, Ethambutol (RIPE)
*PNA* *Atypical* Treat influenza with
Oseltmavir Zanamivir
*Pertussis* Diagnosis
PCR of nasopharygneal swab (gold standard, perform w/in 3 weeks onset)
Lung scan with perfusion defects
PE
Virchow's Triad (Stasis, hypercoagulability, endothelial damage), Sinus Tachycardia, Pleuritic Chest Pain, S1Q3T3, Westermark Sign, Hampton Hump
PE
Virchow's triad, sinus tachycardia, pleuritic chest pain, S1Q3T3, Westermark sign, Hampton hump
PE
*Asthma* Gold standard Dx
PFT - reversible obstruction
Pneumonia seen in HIV patients with low CD4 counts?
PJC= Pneumocystis jiroveci (formerly called PCP)
What is the most likely Dx? Young IV drug user with fever severe hypoxia and diffuse infiltrates on X-ray?
PJP pneumonia
*Pertussis* Complications (3)
PNA Encephalopathy Seizures
Thoracic needle Bx can Dx which types of lung CA?
Peripheral lesions like Adenocarcinoma
Adenocarcinoma location in the lung
Periphery
Question: What are the classic symptoms of active tuberculosis?
Persistent malaise, anorexia, weight loss, fever, night sweats, and a chronic cough are classic symptoms
Inspiratory whoop
Pertussis
*Pleural Effusion* Emphyema diagnostic criteria (3)
Pleural fluid pH <7.2 Glucose <40 + gram stain
What CXR finding might make you think of mesothelioma?
Pleural thickening, remember 80% are found in the pleura lining
One Step Further Question: What are the three laboratory criteria for an exudative effusion?
Pleural:Serum Protein >0.5; Pleural:Serum LDH >0.6; Pleural fluid LDH greater than 2/3 upper limit of normal for serum LDH Pleural Effusion Transudate: CHF (most common) Exudate: infection > malignancy, PE ↓ Breath sounds + dull percussion + ↓ tactile fremitus CXR: blunting of the costophrenic angle
Pneumocystis Pneumonia (PCP)
Pneumocystis Pneumonia (PCP) Patient with a history of HIV Complaining of gradual onset of non-productive cough Labs will show CD4 < 200, increased LDH CXR will show bilateral infiltrates (bat wing pattern) Most commonly caused by Pneumocystis jirovecii Treatment is TMP-SMX
Immunocompromised. Sx out of proportion to exam. Diffuse interstitial & alveolar infiltrates"
Pneumocystis jerovecii; TMP-SMX = Drug of choice
Middle-aged woman presents with dyspnea, especially with exertion. On exam she has an accentuated S2, and some swelling in her legs. CXR shows enlarged pulmonary arteries.
Pulm HTN
*PTX* Type that has no underlying lung disease; mainly affects tall, thin men 20-40 years old, smokers. May have +FHx.
Primary
A 20-year-old man from Mexico presents to the emergency room in a South Texas hospital with a persistent cough for 3 weeks, low-grade fever, and night sweats. His chest x-ray shows mediastinal and right hilar lymphadenopathy and right upper lobe consolidation concerning for:
Primary TB
Most common ventilator associated bacterial infection?
Pseudomonas
What organisms is the most common cause of pneumonia in a patient with cystic fibrosis?
Pseudomonas
Class 4 pulmonary HTN cause?
Pulmonary embolism
Are the lungs noise or quiet in COPD?
Quiet
*PNA* *Atypical* Treat Legionella with (3)
Quinolones Ketolides (Telithromycin) Macrolides
*Pulm HTN* Definitive Dx
R heart catheterization; mean pulmonary pressure over 25
*Cor Pulmonale* ECG shows
RAD; transient RBB; *flat or inverted T waves in the right ventricular leads (V1, V2)*
Name the commonly used meds to treat active tuberculosis infection.
RIPE: Rifampin - isoniazid (INH) - Pyrazinamide - Ethambutol
*PNA* MC viral cause in infants/small children
RSV and parainfluenza
Treatment for infant with Hyaline membrane disease (AKA: infant respiratory distress syndrome - IRDS)?
Respiratory support - Ventilatory support - Exogenous surfactant
Most common cause of acute bronchiolitis in an infant?
Respiratory syncytial virus (RSV)
*Hyaline Membrane Disease* CXR
Reticular ground-glass opacities + air bronchograms, domed diaphragms
Flu + aspirin in children can cause what condition?
Reye syndrome
*Bronchiolitis* What is administered if there is severe lung or heart disease or in immunosuppressed patients?
Ribavirn
*PNA* *Viral* Treat RSV with
Ribavirn
*Epiglottitis* If the household contacts of a patient with H. influenzae epiglottitis include an unvaccinated child aged <4 years, all members of the household (including the patient) should receive prophylactic
Rifampin
Red orange urine occurs with which tuberculosis medication?
Rifampin
You are treating a patient for TB. He has noticed that his body fluids now have an orange tint. What drug is most likely causing this?
Rifampin
Question: What are the classic ECG changes in a patient with cor pulmonale?
Right axis deviation, R/S ratio >1 in V1 and <1 in V6, P-pulmonale (increased P wave amplitude in leads 2, 3 and aVF). Cor Pulmonale Pulmonary HTN + RVH → right heart failure MC chronic cause: COPD MC acute cause: PE Right heart catheterization
Gold standard test used to diagnose Pulmonary HTN?
Right heart cardiac catheterization
*PNA* Often seen after viral illness, like the Flu. Usually bilateral with multi lobar infiltrates or abscesses
S. Aureus
Granulomas and inflammation of alveoli, small bronchi and small blood vessels
Sarcoidosis
If you come across a question where the patient has erythema nodosum and an enlarged parotid gland, what is the most likely diagnosis?
Sarcoidosis
Increase in serum ACE. Hypercalciuria/hypercalcemia. Eosinophilia. Cutaneous anergy (diminished reactivity to allergens).
Sarcoidosis
Non-caseating granulomas, erythema nodosum, bilateral hilar adenopathy, elevated ACE
Sarcoidosis
What is the Dx? Chest X-ray shows bilateral hilar adenopathy and non caseating granulomas?
Sarcoidosis
Young patient w respiratory Sx (cough, dyspnea, CP), constitutional Sx (fever), blurred vision, and erythema nodosum.
Sarcoidosis
Question: What is a potential neurologic complication of pertussis infection?
Seizures. Pertussis (Whooping Cough): Patient with a history of nasal congestion, cough, and low-grade fever Complaining of "rapid fire," repetitive coughing followed by an inspiratory "whoop" and post-tussive emesis Most commonly caused by Bordetella pertussis Treatment is a macrolide - azithromycin
*PNA* Dx of mycoplasma PNA
Serum cold agglutinins (lacks cell wall)
Patient with a history of mining; quarry work with granite, slate, quartz, or pottery sandblasting. May be asymptomatic or have DOE, non-productive cough. CXR shows multiple small (<10mm) round nodular opacities seen in the *upper lobes*. *Egg shell calcifications* of the hilar and mediastinal nodes. (uncommon)
Silicosis
What is the Dx? Pulmonary fibrosis in a patient who was a sandblaster. Chest X-ray shows egg shell calcifications.
Silicosis
Which pneumoconiosis has egg shell calcification on chest X-ray?
Silicosis
Eggshell pattern on CXR
Silicosis (Sandblasters)
*PNA* Pneumocystis jirovecii (PCP) stain?
Silver stain
*Hyaline Membrane Disease* Prevention
Steroids to mature lungs if premature delivery expected (24-36 weeks)
Tall, skinny, male, band student, acute onset one-sided chest pain, dyspnea
Spontaneous PTX
What is the Dx? Thin young healthy male runner who develops acute onset of chest pain and dyspnea?
Spontaneous pneumothorax
*TB* How to Dx active pulmonary TB?
Sputum culture/acid-fast stain, 3 sets of samples collected 8 to 24 hours apart. M. tuberculosis is slow growing and may take 4 to 8 weeks to identify.
*TB* Monitoring response to treatment:
Sputum cultures monthly. Should be negative within 2 mo; if still positive at month 3, consider treatment failure
Hypercalcemia is MC with what type of lung CA?
Squamous cell
What is the most common type of lung cancer in smokers?
Squamous cell
Name 3 types of non small cell (bronchogenic) cancers.
Squamous cell - Adenocarcinoma - Large cell
A pearl formation on CXR should make you think of what diagnosis?
Squamous cell lung cancer
Bacterial pneumonia which occurs in a young patient after flu is most likely caused by what organism?
Staph aureus
Pt with suspected pneumonia/URI has salmon/pink colored sputum. Most likely organism?
Staph aureus
PNA from Nursing homes, chronic care facility. Purulent sputum
Staphylococcus aureus
A patient has pneumonia with rust colored sputum. What is the most likely diagnosis?
Strep pnemonia
Chest X-ray shows lobar pneumonia. Most likely organism?
Strep pneumoniae
Pt with suspected pneumonia/URI has rusty colored sputum. Most likely organism?
Strep pneumoniae
Name the organisms that cause typical pneumonia.
Strep pneumoniae - Staph aureus - Group A Strep
Rusty sputum?
Strep pneumoniae pneumonia
One Step Further Question: What is the treatment of asbestosis?
Supportive care: oxygen, pulmonary rehabilitation, pneumococcal and influenza immunization and pulmonary function monitoring. Asbestosis Shipping, roofing, plumbing SOB + nonproductive cough + chronic hypoxia CXR: "ivory white" calcified pleural plaques Ferruginous bodies Bronchogenic carcinoma > mesothelioma From the roof, but affects the base (lower lobes)
Caseating granulomas should make you think of what diagnosis?
TB
Ghon or Ranke complexes on CXR should make you think of what diagnosis?
TB
Night sweats, TNF inhibitors, Immigration
TB
Posttussive rales should make you think of what diagnosis?
TB
An exudative pleural effusion should make you think of what diagnosis?
TB, lung cancer
*PNA* *Atypical* Treat Chlamydia with
Tetracycline
*Acute Bronchitis* MC etiology?
Viruses; *Adenovirus*
A 67-year-old man with Type 2 diabetes mellitus, obesity, and hypertension is admitted for elective colonoscopy for chronic iron deficient anemia. He undergoes a prepped colonoscopy and receives midazolam 1 mg and fentanyl 25 µg. His eyelids close, and he does not respond to voice. Within a few minutes, the nurse reports that the patient's pulse oximetry reading is falling. He is making an effort to breathe, but there are paradoxical movements of the chest and abdomen. Vital signs, including temperature and blood pressure, remain stable. The patient is suctioned, but scant material is recovered. Chest radiograph does not reveal acute infiltrates. What is the most likely cause of this patient's response to conscious sedation? a) Aspiration pneumonitis b) Decreased respiratory drive due to sedation c) Hemorrhagic shock d) Obstructive sleep apnea (OSA) e) Sepsis
Topic Pulmonary Medicine Answer D. Obstructive sleep apnea (OSA) Explanation When providing conscious sedation, providers should be cognizant of the possibility of OSA syndrome. The prevalence of OSA in patients presenting for surgery has been estimated at 1-9%; although, it may be more common but undiagnosed in certain populations. Usual medications for conscious sedation include benzodiazepines (e.g., midazolam), which act as amnestics and anxiolytics, and opiates (e.g., fentanyl), which provide pain relief. These medications not only suppress respiratory drive centrally but also lower the tone of pharyngeal structures. This change in tone may provoke obstructive apneas or hypopneas. While central hypoventilation, or central apnea, is characterized by a decreased central effort to breathe, the hallmark of obstructive apnea is obstruction at the palatal-glossopharyngeal level. Often, the apneas can be mixed. Falling oximetry, continued effort to breathe, and paradoxical movements of the chest and abdomen suggest obstruction of upper airway muscles, most consistent with OSA. In the absence of fever or hypotension, hemorrhagic shock or sepsis syndrome is unlikely. There is no history of aspiration given lack of secretions and normal chest radiograph.
Transfusion Complications
Transfusion Complications Massive transfusion: coagulopathy, hypothermia, hypocalcemia Febrile reaction: most common complication, fever/chills, Hemolytic reaction: ABO incompatibility, immediate fever/chills, HA Rx: stop transfusion, IVF, diuretics Allergic reaction: urticaria or hives TRALI: like ARDS Rx: stop transfusion Delayed reaction: 3-4 weeks after transfusion, decreased Hb, increased Br GVHD: immunocompromise, rash, pancytopenia, increased LFTs Prevention: irradiated blood products in immunocompromised
*Pleural Effusion* Type cause by Myxedema?
Transudative
In a patient with CHF is the pleural effusion transudative or exudative?
Transudative
*TB* How to Dx latent TB?
Tuberculin skin test (TST) with purified protein derivative (PPD) Positive result is 10mm+ induration, or 5mm+ in someone who is immunocompromised.
A "cinnamon breath smell" is associated with what pulmonary infection?"
Tuberculosis
Fever, night sweats, hemoptysis. AFB/culture x 3 days. RIPE therapy. Screen with TNF inhibitors, immigration
Tuberculosis
What is the Dx? Recent international travel now with fever night sweats cough hemoptysis SOB and weight loss.
Tuberculosis
Any question about rabbits and pneumonia should make you think of what diagnosis?
Tularemia
*PNA* If rabbit exposure (2)
Tularemia Francisella
Pancoast syndrome (3)
Tumors at superior sulcus causes: 1. Shoulder pain 2. Horner's syndrome 3. Atrophy of hand/arm muscles
What type of Infant respiratory distress syndrome (IRDS) occurs in a preterm infant? (usually born before 30 weeks)
Type 1 IRDS
What type of Infant respiratory distress syndrome (IRDS) occurs in a near or full term infant?
Type 2 IRDS
*PNA* CXR: lobar consolidation S/Sx: fever, productive cough, pleuritic CP, *rigors*. PE: Signs of consolidation - bronchial breath sounds, dullness on percussion, increased tactile fremitus. Class of PNA? Etiologies (4)
Typical: 1. S. pneumoniae 2. H. influenze 3. Klebsiella pneumoniae 4. S. aureus
Question: How soon after blood transfusion initiation will anaphylaxis occur?
Within minutes. It is triggered by presence of IgA antibodies in IgA-deficient patients.
If you see a question with anything to do with rats and the patient has pneumonia, what organism should you be thinking of?
Yersinia pestis
What is the most common causative organism in community acquired pneumonia? Klebsiella pneumoniae Mycoplasma pneumoniae Staphylococcus aureus Streptococcus pneumoniae
orrect Answer ( D ) Explanation: Streptococcus pneumoniae is a gram positive cocci and is the most common organism causing community acquired pneumonia. Symptoms of community-acquired pneumonia include fever, tachycardia, chills, shortness of breath, cough, myalgias, and pleuritic chest pain. Some patients may also present with GI symptoms such as nausea, vomiting or diarrhea. Physical exam findings may include increased respiratory rate, crackles, increased tactile fremitus and dullness on percussion.
Clinical Intervention/Pulmonology A 32 year-old male with a history of Tetralogy of Fallot with poor right ventricular function presents for evaluation of sleep apnea. There is no evidence of deviated septum. Polysomnography reveals apneic episodes of 60 seconds in duration. Oxygen saturation falls to low levels. Which of the following is the first-line treatment in the management of this patient? A. Uvulopalatopharyngoplasty B. Nasal septoplasty C. Continuous positive airway pressure D. Antidepressants and oxygen
(u) A. Uvulopalatopharyngoplasty (UVVP) is a surgical procedure with resection of pharyngeal soft tissue and partial amputation of the soft palate and uvula. It is a procedure of last resort in most cases of sleep apnea. (u) B. Nasal septoplasty is performed if gross anatomic nasal septal deformity is present. (c) C. Continuous positive airway pressure prevents hypoxemia and maintains patency of the airway. (u) D. Antidepressants have no role in the treatment of sleep apnea.
*ARDS* Treatment (3)
1. Fix underlying cause (i.e. broad spectrum antibiotics if septic) 2. O2 - noninvasive vs. mechanical ventilation 3. Diuretics
Which of the following is true regarding active tuberculosis? A cavitary lesion on CT of the chest is pathognomonic Isoniazid treatment for six months is adequate therapy It may have varied appearance on chest X-ray Patients with active tuberculosis need droplet precautions
.The classic X-ray finding on chest radiograph is a cavitary lesion in the upper lobe of the lung. However, tuberculosis can cause varied abnormalites on the X-ray including infiltrate in any portion of the lung. Lymphadenopathy is commonly seen in the hilum on X-ray. Cough is the most common symptom of pulmonary tuberculosis. Additionally, patients may develop fever (more common in the afternoon or evening), night sweats and hemoptysis. Patients with active tuberculosis need airborne precautions, not droplet precautions (D). The size of the droplets is quite small necessitating airborne precautions in a negative pressure room with recirculating air. A cavitary lesion on CT of the chest (A) is not pathognomonic for tuberculosis. There are multiple other infectious causes of cavitary lesions in the chest including Staph aureus, Klebsiella pneumoniae, Pseudomonas aeruginosa, and fungal lesions.
*PNA* Treatment of CAP that is *inpatient* (2 options/groups)
1. *Beta-lactam + macrolide* or Doxycycline (Ceftriaxone + Azithromycin) 2. Broad spectrum FQ
*Carcinoid Tumor* Dx
1. *Bronchoscopy - pink to purple well-vascularized central tumor* 2. Tumor localization w CT scan or octreotide scintography
*Bronchiectasis* Dx (In order, 2)
1. *CT scan* study of choice. Lack of tapering of the bronchi; "tram-tracks" or "tortuous airways". Signet ring sign (pulm artery next to a dilated bronchus). 2. *PFTs*. Obstructive pattern.
*Croup* (Laryngotracheitis) Treatment of moderate (stridor at rest, mild-mod retractors) (2)
1. *Dexamethasone* PO vs. IM 2. +/- *Nebulized epinephrine*
*Asthma* 3 tests that can be used if the PFTs are non-diagnostic
1. *Methacholine* challenge test: 20%+ decrease in FEV1 2. *Bronchodilator* challenge test: 12%+ increase in FEV1 3. Exercise challenge test (drop in 15%+)
*SPN* DDX (5)
1. CA 2. Inflammation (granuloma) 3. Infection 4. Congenital (AVM) 5. Misc (Round atelectasis, mucus plug)
*Pleural Effusion* 1. Most common cause of transudative? 2-3. 2 others?
1. CHF 2. Nephrotic syndrome 3. Cirrhosis Result from increased hydrostatic and/or decreased oncotic pressure
*Hyaline Membrane Disease* Treatment (2)
1. Exogenous surfactant 2. CPAP
*COPD* Treatment of exacerbations in COMPLICATED COPD (>65, FEV1 <50% predicted, over 3 exacerbations/year) (2)
1. FQ: Moxifloxacin, Levofloxacin, Ciprofloxacin (if Pseudomonas expected) 2. Amoxicillin/clavulanate
*PNA* No person-to-person transmission. Outbreaks related to contaminated water supplies such as air conditioners, cooling towers, etc.
Legionella pneumophilia
*PNA* Associated with GI symptoms, increased LFTs, hyponatremia. Which type? Diagnosis
Legionella; diagnosed by urine antigen or PCR
Hypoventilation syndrome (AKA Pickwickian syndrome)
A condition in which severely overweight people fail to breathe rapidly enough or deeply enough, resulting in low blood oxygen levels and high blood carbon dioxide (CO2) levels.
Question: What is the choroidal tubercle?
A granuloma in the choroid of the retina that is specific for disseminated TB. Pulmonary Tuberculosis (TB) RFs: immunodeficiency, immigrant, close contact Latent/primary TB: asymptomatic Active/reactivation TB: fever, night sweats, weight loss, productive cough, hemoptysis Erythema nodosum Primary TB CXR: Ghon focus Active/reactivation TB: upper lobes, cavitary lesions Dx: sputum smears for acid-fast bacilli (AFB), sputum/tissue culture for AFB (gold standard) Latent TB dx: PPD or interferon-gamma release assay Latent rx: 9 months of INH Primary rx: Rifampin, INH, Pyrazinamide, Ethambutol (RIPE)
Lung CA presenting as radicular-like pain and paresthesias radiating down the arm could be what type of tumor?
A pancoast tumor; or a tumor located in the apical lung/chest wall that has the potential to erode into the brachial plexus or sympathetic cervical chain (Horner's)
One Step Further Question: What is Caplan's syndrome?
A patient, typically a miner, with rheumatoid arthritis who acquires any of the pneumoconioses.
What criteria determine transudate from exudate?
Light's criteria
Question: What is a saddle pulmonary embolism?
An embolus that lodges in the bifurcation formed by the main pulmonary artery and right and left pulmonary arteries. Pulmonary Embolism 95% arise from deep leg veins Sudden onset of symptoms in 50% SOB, CP, tachypnea ECG: sinus tachycardia, nonspecific ST-T changes, right heart strain, S1Q3T3 (classic finding) CXR: nonspecific abnormalities, Hampton's hump (pleural-based wedge infarct), Westermark's sign (vascular cut-off sign) V/Q scan: usually nondiagnostic Low clinical suspicion: negative D-dimer excludes PE Dx of choice: CTPA Treatment: Anticoagulation Thrombolytics (if massive and HD unstable or submassive with shock, respiratory failure or evidence of moderate to severe RV strain) Embolectomy (last resort)
What lab is elevated in sarcoidosis?
Angiotensin converting enzyme levels 4x higher than normal - Elevated ESR
A 20-year-old woman comes to the office for her routine checkup. She has a past history of asthma and presently is being treated with albuterol MDI on prn basis. She states she uses the inhaler at least once a day for wheezing. She admits to awakening from sleep 3-4 times per week for cough/wheeze. Past History: Hospitalized 1-2 times/year; emergency department visits ~ 6 times/year; 2 prior ICU admits at ages 6 and 8. Previously treated with inhaled steroids for 4-6 weeks around the time of hospital discharge (noticed no benefit). Requires oral steroid bursts every 6-8 weeks for 3-5 days. PE: Essentially normal Office spirometry: FVC 88%, FEV1 59%, FEF25-75 48% Based on the guidelines, how would this patient be classified? a) Mild persistent b) Moderate persistent c) Severe persistent d) Mild intermittent
Answer C. Severe persistent Explanation This is straight from the NHLBI-asthma guidelines. Severe persistent asthma is defined as: continual symptoms, frequent exacerbations, frequent nighttime asthma symptoms, physical activities limited by asthma symptoms, FEV1 or PEF ≤ 60% predicted, and/or PEF variability > 30%. Based on this definition, she has daily use of inhaled bronchodilators and FEV1 less than 60%. Classification is based on the worst symptoms in any category. A patient of yours is concerned about bioterrorism agents and asks if he should be worried about a "severe pneumonia" diagnosed in one of his colleagues.
One Step Further Question: A completely occluding foreign body will, over time, cause what finding on chest radiograph?
Answer: Atelectasis.
One Step Further Question: What medication is recommended for acute exacerbations of idiopathic pulmonary fibrosis?
Answer: Corticosteroids.
One Step Further Question: What is the patient's forced expiratory volume (FEV1) in a moderate to severe COPD exacerbation versus a mild exacerbation?
Answer: FEV1 is less than 50 percent in moderate to severe exacerbations and greater than 50 percent in mild cases.
Pneumoconiosis
Asbestosis: shipping, roofing, plumbing Berylliosis: aerospace, fluorescent bulbs Byssinosis: cotton Silicosis: foundries, sandblasting, mines Coal worker's lung: coal Siderosis: iron Stannosis: tin SOB + nonproductive cough + chronic hypoxia PFTs: reduced lung volumes CXR: interstitial fibrosis Asbestos: from the roof, but affects the base (lower lobes) Silica, coal: from the base (earth), but affect the roof (upper lobes) Cor pulmonale
One Step Further Question: What is the difference between aspiration pneumonitis and aspiration pneumonia
Aspiration pneumonitis is an inflammatory reaction which results from the aspiration of gastric acidic fluid from the stomach. Aspiration pneumonia is a bacterial pneumonia caused by aspiration. Bacterial Pneumonia S. pneumonia: most common, rusty colored sputum, rigors, gram+ paired lancets Klebsiella: alcoholics, currant jelly sputum, bulging fissures, S. aureus: IVDA, postinfluenza, elderly, gram+ cocci in clusters H. influenzae: COPD, gram negative pleomorphic rods Pseudomonas: cystic fibrosis, nursing home resident and cyanosis Health care associated pneumonia: pseudomonas, MRSA Outpatient, healthy: macrolide or doxycycline Outpatient, comorbidity: respiratory tract fluoroquinolone (RTF) Inpatient: RTF ICU: antipneumococcal ß-lactam (ceftriaxone or cefotaxime) + either azithromycin or an RTF
Prolonged expiration (low FEV1) should make you think of what diagnosis?
Asthma or COPD
List three things that should be in your differential if you hear wheezes on a lung exam?
Asthma, COPD, Bronchitis
List the antibiotics classes for inpatient ICU treatment in a patient with a pneumonia caused by Strep pneumoniae or non MRSA staph.
Beta-lactam + macrolide or fluoroquinolone
List the antibiotics classes for inpatient non ICU treatment in a patient with a pneumonia caused by Strep pneumoniae or non MRSA staph.
Beta-lactam PLUS a macrolide
Cystic Fibrosis. Tram lines. Obstructive pattern.
Bronchiectasis
Cystic fibrosis. Tram lines. Obstructive pattern.
Bronchiectasis
Wet cough and foul smelling sputum in a child?
Bronchiectasis
Honeycombing and tram tracks on the CXR should make you think of what diagnosis?
Bronchiectasis and idiopathic pulmonary fibrosis
Irreversible bronchial dilation d/t transmural inflammation. The airways collapse and trap mucus. This leads to infection.
Bronchietasis
Fever, URI symptoms that progress to cough and dyspnea, commonly under 2 y/o (especially under 2 mo). Obstructive pattern on PFTs. *Minimal findings on CXR* - hyperinflation, peribronchial cuffing. Can be associated with organ transplantation, toxic fume inhalation, connective tissue disease.
Bronchiolitis
Rhonci or wheezes that clear after coughing suggest what diagnosis?
Bronchitis or atelectasis
*Bronchiectasis* General treatment (4)
Bronchodialtors *ANTIBIOTICS* Empiric: Ampicillin, Amoxicillin, Bactrim Pseudomonas: FQ, Piperacillin/tazobactam (Zosyn), AMG, Cephalosporin. Oxygen Chest physiotherapy
Long term inflammation and eventual scarring after episodes of severe respiratory distress and mechanical ventilation?
Bronchopulmonary dysplasia
Diarrhea (d/t increased serotonin). Flushing, tachycardia, bronchoconstriction (wheezing) d/t increased bradykinin and histamine. Hemodynamic instability (hypotension) and acidosis.
Carcinoid syndrome, a manifestation of a carcinoid tumor
62-year-old man with a history of chronic obstructive pulmonary disease presents with cough, headache, dyspnea, and watery diarrhea that started six days ago. He was seen at a local urgent care four days ago and prescribed amoxicillin-clavulanate without improvement. He is ill-appearing with a fever of 38.7°C and inspiratory rales on auscultation. Which of the following results would be most consistent with his diagnosis? Right upper lobe infiltrate with bulging fissure on chest X-ray Serum potassium 6 mEq/L Serum sodium 128 mEq/L Sputum gram stain with gram positive cocci in pairs
Correct Answer ( C ) Explanation: This patient is presenting with Legionella pneumonia. It was first identified in 1976 after an outbreak at the American Legion Convention in Philadelphia. Legionella is a gram-negative facultative intracellular bacillus and is found in aquatic environments. Transmission is from inhalation of contaminated aerosols. It is a more common cause of pneumonia in the summer months when other pathogens are less frequently seen. Patients at risk include smokers, those with underlying respiratory disease (e.g. COPD) or immunosuppression, and men > 50 years of age. Patients present with fevers, malaise, and myalgias. The cough is initially dry, but becomes productive of purulent sputum as the illness progresses. Gastrointestinal symptoms are frequently seen, including diarrhea in about 50% of patients. Hyponatremia is also commonly seen. Typical chest X-ray findings include unilateral patchy alveolar lower lobe infiltrates occasionally accompanied by pleural effusions. Laboratory studies include leukocytosis, elevated liver transaminases, and hyponatremia. Diagnosis is made based on clinical findings and urinary antigen assay. The antibiotic of choice is azithromycin. Alternative agents include trimethoprim-sulfamethoxazole and fluoroquinolones. Upper lobe infiltrates with a bulging fissure (A) are seen in Klebsiella pneumoniae. The gram stain in Legionella typically shows numerous polymorphonuclear lymphocytes (PMNs), but no predominant organism. Sputum gram stain with gram positive cocci in pairs (D) is consistent with pneumococcal pneumonia. Diarrhea typically leads to hypokalemia, not hyperkalemia (B). Patients with Legionella may have hyponatremia, but not hypernatremia.
Treatment for Berylliosis?
Chronic steroids
CXR shows small *upper lobe nodules* and *hyperinflation* resembling centrilobar emphysema (similar to smoking-related emphysema). May have *obstructive* pattern on PFTs. Diagnosis/Treatment?
Coal worker's pneumoconiosis; black lung disease Like the other pneumoconioses: supportive treatment with bronchodilators, O2, vaccinations (influenza, pneumococcal), +/- steroids
A 16-year-old boy with a history of asthma presents complaining of increasing episodes of evening and daytime symptoms. He is on a short acting inhaled beta agonist on an as needed basis. He presently needs to use his short acting beta agonist daily. Which of the following is the most appropriate addition to this patient's medication regimen? Inhaled corticosteroid Leukotriene inhibitor Long acting beta agonist inhaler Methylxanthine oxidase inhibitor
Correct Answer ( A ) Explanation: According to the stepwise approach for managing asthma by the National Asthma Education and Prevention Program, inhaled corticosteroids are indicated for mild to moderate persistent asthma. For most patients twice-daily dosing provides adequate control of asthma symptoms. Short acting beta agonists are helpful with controlling acute exacerbations by acting as bronchodilators; however they are not helpful in the long term management in patients with persistent asthma. Using an inhalation chamber, also known as a "spacer," along with rinsing of the month after inhaled corticosteroid use decreases local side effects and systemic absorption.
What is the most common cause of minor hemoptysis in the Emergency Department? Acute bronchitis Pneumonia Pulmonary embolism Tuberculosis
Correct Answer ( A ) Explanation: Acute bronchitis is the most common cause of minor hemoptysis in the Emergency Department. Hemoptysis describes the presence of blood in sputum expectorated from the respiratory tract. The majority of episodes of hemoptysis are minor episodes with small amounts of blood. Less than 5% of patients with hemoptysis have life-threatening hemorrhage. Bronchitis results from inflammation in the bronchial tree and can lead to hemoptysis. Additionally, forceful coughing further irritates the inflamed mucosal surfaces and can result in bleeding. Hemoptysis in bronchitis is typically self-limited. Acute Bronchitis Viruses > bacteria Most common cause of minor hemoptysis Hallmark: cough (usually productive), <1week Symptomatic treatment, bronchodilators Routine ABX therapy not indicated
A 43-year old man presents complaining of a two-week history of gradually worsening dry cough, fatigue and occasional shortness of breath. He has "felt warm" but has not checked his temperature. Review of systems is notable for mild diarrhea and decreased appetite, though he is drinking fluids well. He denies chronic medical problems and takes no medications. He does not smoke. His temperature is 100.6°F; pulse 112; BP 122/78; RR 24; pulse oximetry is 92% on room air. He appears tired, though is not ill-appearing. Lung-fields sound clear on auscultation, though a chest X-ray is obtained which demonstrates diffuse infiltrates. What is the most appropriate treatment for his condition? Azithromycin Metronidazole Olsetamivir Piperacillin and Tazobactam
Correct Answer ( A ) Explanation: Azithromycin, a macrolide antibiotic, is the most appropriate choice for this patient with community-acquired bacterial pneumonia (CAP). The most common side effects of azithromycin include nausea, vomiting and diarrhea, though other side effects such as headache and fatigue may occur. Other reasonable choices for treatment would include doxycycline or a fluoroquinolone. The most common organism responsible for "atypical" bacterial pneumonia is Mycoplasma pneumoniae, though suspicion for a particular pathogen should be guided by the patient's co-morbidities as well as setting. In contrast to CAP caused by typical pathogens such as S. pneumoniae or H.influenzae, this type of bacterial pneumonia usually presents more gradually with symptoms of dry cough, dyspnea, and extra-pulmonary symptoms such as headache, myalgias, fatigue, and GI disturbance. Other bacterial organisms causing pneumonia in certain hosts and settings may include Legionella, Moraxella, Staphylococcus, Chlamydia, Pseudomonas, Klebsiella, Acinetobacteria, and Mycobacterium sp.
A 53-year-old man comes to the emergency department complaining of a 5-day history of a cough and shortness of breath. His temperature is 37.6°C (99.8°F). Auscultation of the lungs shows rhonchi and wheezing. Chest X-ray shows thickening of the bronchial walls in both lower lobes. Laboratory studies show a slightly elevated white blood count. Which of the following is the most likely diagnosis? Acute bronchitis Asthma Influenza Pneumonia
Correct Answer ( A ) Explanation: Based on the constellation of findings, this patient most likely has acute bronchitis. Acute bronchitis is a self-limited inflammation of the bronchi and typically presents with mucopurulent cough for more than 5 days. Chest X-ray will show thickening of the bronchial walls in the lower lobes and auscultation of the lungs usually reveals wheezing and rhonchi. Acute bronchitis is typically caused by viruses (eg, respiratory syncytial virus, rhinovirus, influenza), therefore, antibiotics are not indicated. Most cases of acute bronchitis are self-limited and resolve on their own within a few weeks. Although influenza (C) can cause acute bronchitis, it is not likely this patient has influenza. Influenza typically presents with cough, fever, sputum production, and constitutional symptoms. Pneumonia (D) presents similarly to influenza. Radiography typically shows lung consolidation (ie, lobar pneumonia). It is unlikely this patient has influenza or pneumonia due to the lack of fever and other constitutional symptoms. New onset asthma (B) is not likely in this patient because it is less frequent in older adults and usually presents before the age of 7.
A patient presents with acute dyspnea and pleuritic chest pain. You suspect pulmonary embolism. Which of the following is the most appropriate test to confirm the diagnosis of a pulmonary embolus? Computed tomography angiography D-dimer assay Echocardiography Ventilation-perfusion scan
Correct Answer ( A ) Explanation: Clinical impression alone does not suffice for the diagnosis of acute pulmonary embolism. Diagnosis is difficult due to the low specificity of the common presenting symptoms of acute dyspnea, cough and pleuritic chest pain. Therefore, supplemental testing is used to effectively rule-in or rule-out pulmonary embolism (PE). Routine laboratory tests are nonspecific. Arterial blood gas measurements have a limited role in diagnosing PE. Electrocardiogram and chest radiography abnormalities found in patients with PE are also commonly found in patients without PE, which limits the diagnostic usefulness of these tests. Pulmonary angiography is the definitive, or "gold standard", test for the diagnosis of PE. It is performed by injecting contrast intravenously via the femoral vein. The radiologist looks for a filling defect or abrupt cutoff in the pulmonary vasculature which concretely indicates the presence of an embolus. Helical CT with pulmonary angiography is being used more and more as the diagnostic test for PE as it can detect not only an embolus, but also alternative pulmonary abnormalities which may be causative of the patient's symptoms. However, one must take into account the radiation risk of CT versus the diagnostic benefit or necessity of this test.
Which of the following is the most common cause of cor pulmonale? Chronic obstructive pulmonary disease Connective tissue disorders Left-sided cardiac disease Right ventricular hypertrophy
Correct Answer ( A ) Explanation: Cor pulmonale is a condition of altered right heart function due to respiratory disease, and is estimated to account for 7% of all types of adult heart disease in the U.S. Acute forms are due to pulmonary emboli and acute respiratory distress syndrome. On the other hand, cor pulmonale more commonly has a chronic progressive course. Some pathophysiologic mechanisms include pulmonary vasoconstriction, pulmonary vascular bed compromise due to pulmonary interstitial or alveolar disease, connective tissue disease and high blood viscosity. There are several underlying etiologies, the most common being COPD (>50%). Others include pulmonary hypertension, rheumatoid disease, pulmonary thromboemboli, polycythemia vera, and sickle cell disease. Cor Pulmonale Pulmonary HTN + RVH → right heart failure MC chronic cause: COPD MC acute cause: PE Right heart catheterization
Which of the following HIV-positive patients suspected of having Pneumocystis pneumonia (PCP) should receive prednisone before treatment with trimethoprim/sulfamethoxazole? A 10-year-old with a normal chest x-ray and a PaO2 of 65 mm Hg A 15-year-old with diffuse interstitial infiltrates on chest x-ray, a pulse oximetry of 92% and PaO2 of 80 mm Hg A 20-year-old with diffuse interstitial infiltrates on chest x-ray and an A-a gradient of 25 mm Hg An 18-year-old with diffuse interstitial infiltrates on chest x-ray, a pulse oximetry of 94% on room air, PaO2 of 75 mm Hg, and an A-a gradient of 10 mm Hg
Correct Answer ( A ) Explanation: Corticosteroids are used as adjunct therapy in HIV-positive patients with moderate to severe PCP (now known as Pneumocystis jiroveci pneumonia), defined by a room air arterial oxygen partial pressure (PaO2) of less than 70 mm Hg (severe disease is less than 60 mm Hg) or an alveolar-arterial oxygen gradient that exceeds 35 mm Hg (exceeding 45 mm Hg for severe disease). When administered, steroids should be given before trimethoprim/sulfamethoxazole or pentamidine because microbial degradation and clearance caused by antibiotics may trigger a severe inflammatory response. Corticosteroid therapy can blunt this inflammatory response, improve oxygenation, and reduce the incidence of respiratory failure.
17-year-old girl with a history of well-managed cystic fibrosis is being evaluated for a steadily worsening chronic cough with shortness of breath and wheezing. She is producing copious purulent malodorous sputum and occasional hemoptysis. Crackles are heard at her bilateral lung bases. Which of the following findings would be most expected on this patient's chest radiograph? Dilated, thickened bronchi with "tram-track" marks Low lung volumes and ground glass opacities Lung hyperinflation with flattening of the diaphragm Normal chest radiograph
Correct Answer ( A ) Explanation: Dilated, thickened bronchi are classic radiograph findings in patients with bronchiectasis, the most likely disease in this patient. The bronchi are often described as having "tram-track" or ring-like markings. Scattered, irregular opacities, atelectasis, or focal consolidations may also be noted. A suspicion of bronchiectasis based on radiographic findings and patient symptoms warrants a high-resolution CT scan which is a more precise diagnostic study. Bronchiectasis is suspected in patients who have a chronic cough, dyspnea, wheezing, and heavy production of purulent, foul-smelling sputum. Pleuritic chest pain, weight loss, and anemia are commonly associated. An exam usually reveals crackles at the lung bases and nail clubbing in severe disease. In the United States, the greatest percentage of patients with bronchiectasis develop it secondary to cystic fibrosis. Lung infections, tumor presence, and immunodeficiency states are other less common causes. Antibiotics are usually needed with the choice being guided by sputum cultures. Haemophilus influenza, Streptococcus pneumonia, and Staphylococcus aureus are often isolated. In addition to proper antibiotic coverage, bronchiectasis should be treated with daily chest physiotherapy and inhaled bronchodilators.
A patient with acute pancreatitis is noted to have a pleural effusion on chest radiography. Which of the following findings would you expect to find on pleural fluid analysis? Elevated amylase concentrations Elevated triglyceride concentrations Low glucose concentrations Low pH of pleural fluid
Correct Answer ( A ) Explanation: Elevated amylase concentrations would be expected on pleural fluid analysis in the setting of acute pancreatitis. Although not routinely tested in pleural fluid samples, amylase measurements can assist when a pleural effusion is caused by a pancreatic or esophageal etiology. The finding of an amylase-rich pleural effusion, defined as either a pleural fluid amylase greater than the upper limits of normal for serum amylase or a pleural fluid to serum amylase ratio greater than 1.0, narrows the differential diagnosis of an exudative effusion to the following major possibilities: acute pancreatitis, chronic pancreatic pleural effusion, esophageal rupture and malignancy. Acute Pancreatitis Patient will be complaining of epigastric pain radiating to the back, nausea, and vomiting PE will show ecchymosis of left flank (GreyTurner sign), umbilical ecchymosis (Cullen sign) Labs will show elevated lipase (best) and amalyse Diagnosis is made by US and Ranson's criteria Most commonly caused by gallstones > alcohol Treatment is IV fluids
65-year-old undomiciled man presents to your emergency department with complaints of fever, productive cough, and shortness of breath. He has a 40-pack-year history of smoking and daily alcohol consumption. His chest radiograph demonstrates a right upper lobe lung infiltrate with an air-fluid level. What is the most likely etiology for this finding? Klebsiella pneumoniae Legionella pneumophilia Mycoplasma pneumoniae Streptococcous pneumoniae
Correct Answer ( A ) Explanation: Klebsiella pneumoniae is a gram-negative encapsulated organism. It occurs most commonly in alcoholic or chronically debilitated patients. Patients with klebsiella pneumonia commonly present with shaking chills, cyanosis, pleuritic chest pain, and a productive cough with characteristic currant-jelly sputum. Chest radiography reveals an infiltrate that is often in the upper lobes (most commonly the right) and is associated with a bulging fissure. If untreated, the infiltrate will progress into a necrotizing lesion with air-fluid levels, and can ultimately lead to development of an empyema. Legionella pneumophila (B) is the most common causative organism of Legionnaire's disease. Legionella is a gram-negative bacillus that lives in the water supply. Transmission occurs through inhalation of contaminated aerosolized droplets emanating from equipment such as cooling towers, evaporative condensers, and shower heads. Legionnaire's disease tends to occur in clusters, with common exposure to the same contaminated source. Spread does not occur from person to person. Infection is associated with hyponatremia, elevated liver enzymes, GI symptoms (watery diarrhea, nausea, vomiting, and abdominal pain), and neurologic signs (seizures, altered mental status, and gait disturbances). Approximately half of patients with Legionnaire's disease will have relative bradycardia. Mycoplasma pneumoniae (C) is considered one of the atypical bacterial causes of pneumonia. It is often referred to as "walking pneumonia," where the patient tends to look better than you would expect, given the diffuse interstitial pattern commonly seen on chest X-ray. Mycoplasma pneumonia is associated with extrapulmonary manifestations such as conjunctivitis, pharyngitis, rash, and pericarditis. Staph aureus pneumonia often affects IV drug abusers, nursing home patients, and those recovering from an influenza infection. Spread to the lungs occurs hematogenously. Chest radiograph reveals a patchy infiltrate that is initially multicentric or peripheral and ultimately progresses to lobar consolidation and abscess formation. Overall, Streptococcus pneumonia (D) remains the most common cause of community-acquired pneumonia. Patients often look ill and tend to have abrupt onset of symptoms, including rigors, before progressing to cough with rust-colored sputum. The chest radiograph of these patients tends to have a single lobar infiltrate, often in the lower lobes of the lun
A 45-year-old woman presents to the Emergency Department with a fever, productive cough, and diarrhea. Her chest X-ray shows a patchy unilobar infiltrate and her sodium is 127 mmol/L. Which of the following organisms is most likely to be responsible for these symptoms? Legionella pneumophila Pseudomonas aeruginosa Staphylococcus aureus Streptococcus pneumoniae
Correct Answer ( A ) Explanation: Legionella pneumophila is the most likely organism and causes Legionnaires' disease. It is responsible for about 5% of all pneumonias. Common symptoms include a fever, cough, diarrhea, and confusion. Chest X-ray findings vary but often show a patchy unilobar infiltrate. Laboratory abnormalities are common with hyponatremia being the classic finding. First-line treatment includes either respiratory quinolones (e.g. levofloxacin) or macrolides (e.g. azithromycin). Streptococcus pneumoniae (D) is the most common cause of community-acquired pneumonia (CAP). It presents classically with fever, chills, productive cough, and pain and a lobar opacity on chest X-ray. First-line treatment for inpatient management is a third-generation cephalosporin (e.g. ceftriaxone or cefotaxime). Outpatient pneumococcal pneumonia can be treated with amoxicillin or a macrolide. Pseudomonas aeruginosa (B) is a common cause of hospital-acquired pneumonia. It should be considered in patients with cystic fibrosis, bronchiectasis, or an immunocompromised state. Treatment can include an antipseudomonal penicillin (e.g. piperacillin-tazobactam) or an antipseudomonal cephalosporins (e.g. cefepime). Staphylococcus aureus (C) is a common cause of healthcare-acquired pneumonia and is seen following influenza infections. Treatment includes either vancomycin or linezolid since methicillin resistance is common.
17-year-old girl with a history of asthma presents to your office with complaints of wheezing and shortness of breath. She says that in the past month she has experienced symptoms approximately 3-4 times per week requiring use of her short-acting beta agonist inhaler. She has also woken up at night four times during the month with shortness of breath and occasionally gets dyspneic on her daily walk. Which of the following is the most effective management? Add a low dose inhaled glucocorticoid Add a low dose inhaled glucocorticoid plus long-acting beta agonist Continue use of short-acting beta agonist only Start an oral course of glucocorticoids
Correct Answer ( A ) Explanation: Mild persistent asthma is defined as having symptoms more than twice per week but less than daily, 3-4 nocturnal awakenings per month, symptoms requiring the use of a short-acting beta agonist rescue inhaler more than two times per week, minor interference with activities of daily living, FEV1 measurements and FEV1/FVC ratio within normal limits and two or more asthma exacerbations requiring an oral course of corticosteroids per year. Treatment of asthma follows a step-wise plan depending on the severity of symptoms. Mild persistent asthma requires daily use of a long-term controlling medication and a low dose inhaled glucocorticoid. Use of a short-acting beta agonist rescue inhaler should also continue as needed.
65-year-old obese man presents to your office with complaints of snoring and fatigue. His wife makes him sleep in a separate room because of his loud snoring and gasping for air in his sleep. Which of the following is the most appropriate therapy? Continuous positive airway pressure therapy Modafinil Oxygen therapy Theophylline
Correct Answer ( A ) Explanation: Obstructive sleep apnea (OSA) is a disorder characterized by a decrease or cessation of airflow during sleep. It is more common in men. The prevalence increases with age and obesity is the most well-documented risk factor. The classic symptoms of OSA are snoring, spousal report of apnea, daytime sleepiness, fatigue and hypoxia. Diagnosis of OSA is confirmed by a sleep study. All patients with OSA should be counseled on behavioral modifications, including weight loss, sleeping in the lateral recumbent position, smoking cessation, and avoiding alcohol for 4-6 hours prior to bedtime. The cornerstone of OSA treatment is continuous positive airway pressure (CPAP) therapy. Over time, untreated OSA may lead to increased morbidity and mortality due to hypertension, heart disease, stroke, diabetes or metabolic syndrome.
33-year-old man presents to the ED with several weeks of cough, pleuritic chest pain, weight loss, and night sweats. The patient drinks a 6-pack of beer daily. Vital signs are BP 145/75, HR 88, RR 18, and T 37.7°C. Pulmonary exam reveals crackles and decreased breath sounds on auscultation. You obtain the radiograph seen above. Which of the following is the most likely diagnosis? ALung abscess BNeoplasm CSeptic pulmonary emboli DTuberculosis
Correct Answer ( A ) Explanation: Patients with lung abscess classically present with several weeks of cough, fever, pleuritic chest pain, weight loss, and night sweats. There may be cough productive of putrid sputum. Because the progression of lung abscess is indolent, tachycardia, tachypnea, or fever may be absent. The chest radiograph often shows an area of dense consolidation with an air-fluid level inside a thick-walled cavitary lesion. Those who abuse alcohol or have other conditions associated with the potential for aspiration are at greatest risk for lung abscess development.
A 48-year-old man with a history of HIV presents to your office with complaints of fever, cough and shortness of breath. Laboratory testing reveals a CD4 count of 130 cells/microL. Which of the following findings is most likely to be seen on chest X-ray? Ground glass opacification Hampton's hump Honeycombing Kerley B lines
Correct Answer ( A ) Explanation: Pneumocystis jirovecii pneumonia (PCP), previously called Pneumocystis carinii pneumonia, is an opportunistic infection found in individuals infected with HIV who have CD4 counts of less than 200 cells/microL. Symptoms of PCP develop gradually, with fever, cough and shortness of breath being the most common complaints. Patients may also present with fatigue, weight loss, chest pain and chills. Chest X-ray will reveal bilateral, fine, diffuse opacification, often with a ground glass appearance. Patients with HIV whose CD4 counts drop below 200 cells/microL should be given prophylaxis with trimethoprim-sulfamethoxazole and treatment of the infection is with the same agent. Kerley B lines (D) are caused by a thickening of the subpleural interstitium and are often seen in patients with pulmonary edema. Hampton's hump (B) is a wedge of airspace opacity on the periphery of the lung and is seen with pulmonary embolism. Honeycombing (C) is found with end-stage interstitial lung disease such as pneumoconiosis.
A 72-year-old woman was hospitalized for atrial fibrillation with rapid ventricular rate. She is being discharged today on amiodarone for long term control of her dysrhythmia. Which of the following diagnostic studies should be followed as an outpatient? Chest X-ray Coagulation studies Complete blood counts Renal panels
Correct Answer ( A ) Explanation: Pulmonary function tests should be monitored for patients on chronic amiodarone therapy. Amiodarone is a class III anti-arrhythmic drug used to treat many common dysrhythmias. Annual chest radiograph is are recommended when patients are on chronic Amiodarone use. Pulmonary toxicity generally correlates more closely with the total cumulative dose than the serum levels. It usually occurs months to years after initiation. Several types of pulmonary toxicity may result from chronic amiodarone therapy; however, the most common is a chronic interstitial pneumonitis. A non-productive cough and dyspnea are present in the majority of affected individuals. If signs or symptoms develop the patient should be referred for pulmonary function testing and amiodarone should be immediately discontinued. Other complications of chronic amiodarone therapy include, but are not limited to, thyroid dysfunction, both hypo- and hyperthyroidism, symptomatic hepatitis, corneal microdeposits, optic neuropathy, or dermatologic manifestation such as photosensitivity and blue-gray skin discoloration. Thyroid function tests and hepatic panels are recommended every six months. Dermatologic physical exams and ophthalmologic eye evaluations are recommended as needed for signs or symptoms. These complications are treated by either reducing the dose and discontinuation.
What lab abnormality is commonly seen in sarcoidosis? Hypercalcemia Hypocalcemia Hypophosphatemia Neutropenia
Correct Answer ( A ) Explanation: Sarcoidosis has a number of lab abnormalities associated with it including hypercalcemia. Sarcoidosis is a multisystem inflammatory disease that involves non-caeseating granulomas composed of collections of T-helper cells and other inflammatory cells. It predominantly affects the lungs and intrathoracic lymph nodes but can involve a number of systems including cardiac, skin and neurologic. Chest radiography often reveals hilar adenopathy in early stages and progresses to pulmonary fibrosis. Patients may present with a host of non-specific symptoms including fever, fatigue, weight loss and polyarthritis. Other symptoms depend on end organ involvement. The granulomas secrete 1,25 vitamin D leading to hypercalcemia and hypercalciuria.
A 55-year-old man presents complaining of a 2-day history of cough, fatigue and fever which came on suddenly. He denies chronic medical problems and takes no medications. He does not smoke. He is ill-appearing, though in no distress. His temperature is 101.6°F; pulse 112; BP 122/78; RR 24; and pulse oximetry 92% on room air. Physical exam is remarkable for rales along the right side of his lung fields. What is the most appropriate next step in evaluating his condition? AAcid-fast bacillus smear BChest X-ray CComplete blood count DSputum Gram stain and culture
Correct Answer ( B ) Explanation: Chest X-ray is the most appropriate next step in the evaluation of this patient who has a history and physical exam suggestive of community-acquired bacterial pneumonia (CAP). It is the most common infectious cause of death in the United States. Patients may present with productive cough, fever, shortness of breath, and sometimes pleuritic chest pain. Symptoms may came on suddenly. Clinical findings include tachypnea, tachycardia, fever or hypothermia, and altered breath sounds such as the auscultation of rhonchi or rales, dullness, increased fremitus, or egophony.
A 16-year-old man presents to the ED complaining of 3 days of nasal rhinorrhea, cough, myalgias, and generalized malaise. After a coughing episode yesterday, he developed pleuritic chest pain radiating to the left neck. His vital signs are BP 130/70 mm Hg, HR 76, RR 16, T 36.6°C and pulse oximetry 98% on room air. You obtain the chest radiograph seen above. Which of the following is the most appropriate next step in management? Discharge home with close follow-up Endoscopy Needle decompression Tube thoracostomy
Correct Answer ( A ) Explanation: Spontaneous pneumomediastinum is the presence of air or gas in the mediastinum. The source of air usually originates from the respiratory or gastrointestinal tract. Spontaneous pneumomediastinum usually results from rupture of alveoli, particularly after a strenuous Valsalva maneuver, leading to air dissecting along peribronchovascular sheaths and spreading into the mediastinum. The air then moves through mediastinal fascia planes and spreads to subcutaneous tissues of the thorax, upper limbs, and cervical region. Patients commonly report transient stabbing chest pain that may radiate to the shoulders, arms, or back. The finding of Hamman's sign (crunching sound heard on auscultation of the mediastinum with each heartbeat) is suggestive of pneumomediastinum but has a low sensitivity. Spontaneous pneumomediastinum is generally a benign, self-limited condition that usually does not require hospital admission. Secondary causes of pneumomediastinum such as Boerhaave's syndrome are more serious, and treatment is aimed at the underlying disorder.
An eight-year-old boy comes in your clinic for follow-up. Two days ago, he was diagnosed with pneumonia and started on amoxicillin-clavulanic acid. However, fever and cough persisted and is accompanied by decreased appetite, chest pain, and difficulty breathing. You suspect a complicated pneumonia. Which of the following physical exam findings is consistent with pleural effusion? Dullness to percussion Early inspiratory crackles Trachea deviates away from affected side Vesicular breath sounds
Correct Answer ( A ) Explanation: The boy has history of pneumonia that failed treatment and developed a paraneumonic effusion. Bacterial pneumonias can be associated with complications involving the respiratory tract, which include pleural effusion, empyema, pneumatocoeles, necrotizing pneumonia, and lung abscesses. The incidence of paraneumonic effusions appears to be increasing in the United States. The causative organisms have changed over time with Streptococcus pneumoniae as the predominant organism. In addition, community-acquired Staphylococcus aureus is also becoming more common. The presentation of pleural effusion or empyema may depend upon when the child presents for medical attention. Some children present with symptoms related to empyema while others have been seen earlier in the course and appropriately treated for pneumonia but fail to respond. On physical examination, children may appear ill or occasionally be toxic-appearing. The majority are tachypneic, with fever and cough present in approximately 90 percent of patients. Chest examination may reveal a small degree of new scoliosis that is related to the patient's splinting toward the affected side. There may also be dullness to percussion, decreased air exchange, and possibly a pleural rub, on the side of the fluid collection. Early inspiratory crackles (B) is an exam finding that suggests small airway disease like bronchiolitis. Trachea deviating away from affected side (C) is a finding seen in tension pneumothorax. Vesicular breath sounds (D) is a normal physical examination finding.
A two-year-old boy who is new to your practice is brought to the clinic because of coughing. For the past two weeks, he has had rhinorrhea and coughing. However, the coughing has become worse with more frequent coughing fits and episodes of vomiting. You review his previous records and note that the parents have refused immunizations. On physical examination, his temperature is 38 degrees Celsius, respiratory rate is 35, and lungs are clear to auscultation. During the examination, you note that the boy has a long series of coughs followed by a whoop. Which of the following is the most likely etiologic agent causing the illness? Bordetella pertussis Human metapneumovirus Mycoplasma pneumoniae Respiratory syncytial virus
Correct Answer ( A ) Explanation: The boy has signs and symptoms consistent with Bordetella pertussis infection or simply known as pertussis. The classic presentation of pertussis includes paroxysms of coughing, an inspiratory whoop, and posttussive vomiting. The classic presentation typically occurs as a primary infection in unvaccinated children younger than 10 years of age. There are three stages with the first stage being the catarrhal stage. In this stage the patient develops mild cough and coryza. This may last from one to two weeks. In the paroxysmal stage, coughing spells increase in severity. The paroxysmal cough is distinctive: a long series of coughs between which there is little or no inspiratory effort. The child may gag, develop cyanosis, and appear to be struggling for breath. Then the whoop, or noise made by the forced inspiratory effort that follows the coughing attack. The last stage is the convalescent stage wherein the cough subsides over several weeks to months. Infants older than four months of age and children with suspected pertussis should be treated. Laboratory confirmation should not delay the initiation of treatment.
A 47-year-old woman, with no past medical history and no hospitalizations, presents with cough, green sputum, and fever. Her vitals are T 100.7°F, HR 94, BP 123/76, RR 18, oxygen saturation 97%. She is well appearing and her blood work (CBC and BMP) is unremarkable. A chest X-ray shows a left lower lobe infiltrate. Which of the following represents the best management for this patient? Discharge home with oral antibiotics and follow up Draw blood cultures and discharge home on oral antibiotics Order a chest CT scan Start IV antibiotics, draw blood cultures, and admit
Correct Answer ( A ) Explanation: This otherwise healthy, well appearing patient presents with community-acquired pneumonia (CAP), which affects 2-4 million US patients every year. This patient can be managed as an outpatient with oral antibiotics and follow up. There are a number of prognostic tools for pneumonia to help guide disposition. Among these are the Pneumonia Severity Index (PSI) and CURB-65 both of which would classify this patient as low risk.
13-year-old boy presents to your ED with worsening cough and exercise intolerance. The triage note says he has seen his primary care physician twice over the last month and was given albuterol which has not improved his symptoms. His vital signs show a temperature of 37.2°C, heart rate of 110 beats per minute, respiratory rate of 28 breaths per minute, blood pressure of 110/82 mm Hg, and pulse oximetry of 93% on room air. A chest X-ray obtained from triage is shown above. Which of the following findings do you expect on your physical examination? Coarse crackles Pleural friction rub Rhonchi Stridor
Correct Answer ( A ) Explanation: This patient has evidence of pulmonary venous congestion and cardiomegaly on X-ray, consistent with congestive heart failure (CHF) due to dilated cardiomyopathy. Dilated cardiomyopathy is the most common form of cardiomyopathy in children and the cause is unknown in about half of the cases. It causes dilation of all four cardiac chambers and significantly weakens systolic contraction leading to symptoms of congestive heart failure. These patients can present with both symptoms of left sided heart failure such as pulmonary coarse crackles, venous congestion, fatigue, or tachypnea and dyspnea on exertion, and right-sided congestive heart failure with hepatomegaly and peripheral edema. While peripheral edema may present as jugular venous distention and ankle edema in adults, this is less common in younger children who usually present with periorbital edema. Patients with new pulmonary venous congestion require a thorough cardiac work-up and admission to an intensive care unit. On auscultation, pulmonary edema is associated with coarse crackles, usually discontinuous and start early in inspiration and extend into expiration. Stridor (D) describes a high-pitched, monophonic sound made when breathing that is best heard over the anterior neck. It is associated with many conditions such as epiglottitis, croup, tracheitis, and foreign body obstruction.
Which of the following is the most common sign seen in patients presenting with a pulmonary embolism? Hemoptysis Hypoxia Swollen calf Tachypnea
Correct Answer ( D ) Explanation: Tachypnea is the most common vital sign abnormality seen in acute pulmonary embolism (PE). PE is a common, life-threatening disease caused by the occlusion of pulmonary arteries by blood clots. The majority of these clots originate in the deep venous system (more commonly in the legs than in the upper body). Patients can present with various symptoms including weakness, shortness of breath, malaise, syncope, dizziness or chest pain. In addition, PE can cause changes to all of the major vital signs. Sinus tachycardia is the most common abnormality found on ECG.
Which of the following characteristics of transfusion-related acute lung injury (TRALI) differentiates it from transfusion-associated circulatory overload (TACO)? High fever Hypertension Hypoxemia Pulmonary edema
Correct Answer ( A ) Explanation: Transfusion-related acute lung injury (TRALI) complicates transfusions in 1 of every 5,000 units of PRBCs. Although rare, it is associated with high mortality. The pathophysiology is thought to be due to an acute neutrophilic response that leads to endothelial damage and massive capillary leak in the pulmonary vasculature. The diagnosis is clinical with patients presenting with signs and symptoms of acute lung injury or acute respiratory distress syndrome. TRALI is characterized by hypoxemia (oxygen saturation <90% on room air), and bilateral pulmonary infiltrates presenting within 6 hours of blood transfusion. There should not be any evidence of pulmonary artery hypertension or generalized fluid overload. Hypotension, tachycardia, and fever are common. Fever is not common in transfusion-associated circulatory overload (TACO). Treatment of TRALI is supportive, with low tidal volumes for mechanical ventilation and maintenance of euvolemia. Despite the initial appearance of fluid overload, these patients are normovolemic or intravascularly depleted secondary to the increase in vascular permeability and extravasation. Diuresis may lead to further deterioration and shock.
A 22-year-old man presents with an asthma exacerbation. Which of the following is true regarding treatment modalities? Anticholinergic agents promote bronchodilation by decreasing cyclic adenosine monophosphate Beta-2-agonists promote bronchodilation by increasing cyclic adenosine monophosphate The onset of action for beta-2-agonists is 10 minutes The peak effect of corticosteroids occurs within 3-5 hours
Correct Answer ( B ) Explanation: Asthma is responsible for nearly two million Emergency Department visits per year. The mainstay of treatment consists of inhaled beta-2-agonists, inhaled anticholinergics, and corticosteroids. The combination of these drugs results in decreased rates of hospitalization and improvements in pulmonary function in the first 90 minutes. Beta-2-agonists promote bronchodilation by increasing cyclic adenosine monophosphate (cAMP). Their primary effect is on small peripheral airways with onset of action in less than five minutes. Studies have shown that administration of beta-2-agonists via metered-dose inhaler with a spacer provides similar bronchodilation when compared with nebulization. While use of the MDI may be more economical and take less time, it does require more supervision to ensure adequate use. Anticholinergic agents promote bronchodilation by decreasing cyclic guanosine monophosphate (cGMP), not cyclic adenosine monophosphate (A). Their primary effect is on the larger airways. Onset of action is also longer than with beta-2-agonists, taking up to one to two hours for peak effect. Corticosteroids work by inhibiting recruitment of inflammatory cells. They are indicated in all cases of moderate to severe asthma and in those who demonstrated limited response to beta-2-agonists. Onset of action is one to two hours with peak effect in about 24 hours, not three to five hours (C). Many studies have shown that oral and intravenous steroids provide equal benefit. Onset of action for beta-2-agonists is less than five minutes, not 10 minutes (D).
56-year-old man comes to the clinic complaining of a 6-day history of a mucopurulent cough and shortness of breath. His temperature is 37.6°C (99.8°F). Auscultation of the lungs reveals rhonchi and wheezing. Chest X-ray shows thickening of the bronchial walls in both lower lobes. Laboratory studies show a slightly elevated white blood count. Which of the following medications is most appropriate to administer for this patients cough? Aspirin Dextromethorphan Penicillin Prednisone
Correct Answer ( B ) Explanation: Based on the constellation of findings, this patient most likely has acute bronchitis. Acute bronchitis is a self-limited inflammation of the bronchi and typically presents with mucopurulent cough for more than 5 days. Chest X-ray will show thickening of the bronchial walls in the lower lobes and auscultation of the lungs usually reveals wheezing and rhonchi. Acute bronchitis is most commonly caused by viruses (eg, respiratory syncytial virus, adenovirus, rhinovirus, influenza). Most cases of acute bronchitis are self-limited and resolve on their own within a few weeks. For the patient's cough, over-the-counter medications, such as dextromethorphan or guaifenesin can be offered. Although the benefits of these medications for symptom improvement in patients with acute bronchitis are uncertain, multiple clinical practice guidelines suggest that offering medications for symptom relief may help reduce requests for antibiotics.
A 47-year-old woman presents with several months of a persistent cough that recently resulted in hemoptysis. A review of system also confirms episodes of diarrhea, rash, and flushing. Physical exam is normal with the exception of focal wheezing over the right upper lobe. Which of the following is the most likely diagnosis? Alpha-1-antitrypsin deficiency Carcinoid tumor Pulmonary hypertension Pulmonary tuberculosis
Correct Answer ( B ) Explanation: Bronchial carcinoid tumors commonly cause persistent coughing with hemoptysis and focal wheezing. As these tumors begin to secrete hormonal mediators, they may cause carcinoid syndrome during which symptoms can include diarrhea, flushing, head and neck edema, bronchospasm, or hives. Specific manifestations depend on the type of hormone released, as this can vary. Carcinoid tumors should also be suspected in patients with recurrent pneumonia. Most diagnosed carcinoid tumors grow in the central bronchi. Peripherally-located bronchial carcinoid tumors are rare and usually do not produce symptoms. Suspicion of a bronchial carcinoid tumor should prompt work-up with imaging. A CT scan may help localize the lesion and assess growth over time. Octreotide scintigraphy can also aid in localization. Fiberoptic bronchoscopy usually allows for biopsy in centrally-located tumors. However, biopsy is often complicated by bleeding, as these tumors are generally very well vascularized. Though bronchial carcinoid tumors rarely metastasize, they commonly compromise respiratory function through bleeding and airway obstruction. These tumors are often resistant to radiation and chemotherapy. Fortunately, surgical excision generally gives a favorable prognosis. Alpha-1-antitypsin deficiency (A) may cause respiratory symptoms seen in this patient (persistent coughing, wheezing, and shortness of breath), however it does not classically cause the symptoms of carcinoid syndrome. Pulmonary hypertension (C) is less likely to cause wheezing or hemoptysis and more likely to present with dyspnea on exertion, anginal pain, or even syncope. Physical exam findings may not include wheezing, but rather jugular venous distension, edema, and an accentuated pulmonic valve component on auscultation. Pulmonary tuberculosis (D) may cause a persistent cough and hemoptysis, but is not usually associated with diarrhea, rashes, or afebrile flushing.
What is the most common symptom seen in acute bronchitis? Chest pain Cough Fever Sputum
Correct Answer ( B ) Explanation: Bronchitis is defined as inflammation of the lower airways and is typically caused by viral pathogens. The most common presenting complaint is cough in these patients. In healthy adult patients, acute bronchitis is typically caused by a viral pathogen. As such, it is not amenable to antibiotic therapy. In patients with chronic bronchitis or chronic obstructive pulmonary disease (COPD), exacerbations of symptoms may be the result of bacterial pathogens causing acute bronchitis on top of their chronic bronchitis. Treatment with antibiotics should be considered in this subset of patients with bronchitis. Chest pain (A) often occurs as a result of coughing in patients with bronchitis. Fever (C) may or may not be present and does not indicate a higher likelihood of bacterial infection. Sputum (D) is present in about half of patients with acute bronchitis.
You evaluate a 65-year-old patient for shortness of breath and note on exam decreased breath sounds at the left lung base. You are suspicious of a small pleural effusion. In which of the following views on the chest radiograph is the small pleural effusion most likely to be detected? Lateral Lateral decubitus left side down Lateral decubitus right side down Posterior-anterior (PA)
Correct Answer ( B ) Explanation: Classic physical signs of a pleural effusion include diminished breath sounds, dullness to percussion, decreased tactile fremitus, and occasionally a localized pleural friction rub. Chest radiograph confirms the suspicion of pleural effusion. The classic radiographic appearance of a pleural effusion is blunting of the costophrenic angle on the upright chest radiograph Pleural Effusion Transudate: CHF (most common) Exudate: infection > malignancy, PE ↓ Breath sounds + dull percussion + ↓ tactile fremitus CXR: blunting of the costophrenic angle
Which of the following best describes cor pulmonale? Altered structure and function of the left ventricle Altered structure and function of the right ventricle Congenital heart disease Right-sided heart disease due to left-sided heart disease
Correct Answer ( B ) Explanation: Cor pulmonale is the result of pulmonary hypertension associated with diseases of the lung, upper airway, pulmonary vasculature or chest wall. The disease presents as altered structure and function of the right ventricle. Altered structure in cor pulmonale includes both hypertrophy and dilation. Patients present with dyspnea, angina or syncope on exertion, lethargy, and fatigue. Physical exam findings are related to both right-sided heart disease and pulmonary hypertension. They include peripheral edema, elevated jugular venous pressure with prominent v-wave, increased or narrowly split second heart sound and left holosystolic heart murmur at the left lower sternal border. Diagnostic testing such as ECG, echocardiogram and chest X-ray may all be abnormal in patients with cor pulmonale. Treatment is aimed at the underlying cause.
A 16-month-old boy and his adoptive parents present to your office with a complaint of cough and concern about the patient's weight. The parents indicate that they don't have much information about the patient's birth parents and since adopting him four months ago they have noticed daily shortness of breath, cough and wheezing. He is also a fussy eater and is in the second percentile of weight on the growth chart. Which lab finding is most consistent with the diagnosis? Elevated exhaled nitric oxide testing Elevated sweat chloride Elevated white blood cell count Positive skin allergy test
Correct Answer ( B ) Explanation: Cystic fibrosis (CF) is an autosomal recessive disease that occurs predominantly in Caucasians. Multiple organ systems are affected because of a mutation in the cystic fibrosis transmembrane conductance regulator protein, a regulatory protein found in exocrine tissues. Of the patients who survive the neonatal period, 90% have pulmonary complications. Historically patients were diagnosed with cystic fibrosis after presenting with symptoms. Now newborn screening allows for detection of the disease before patients become symptomatic. Clinical presentation includes meconium ileus, failure to thrive and respiratory symptoms. Patients with CF have a shortened lifespan and end-stage lung disease is generally the cause of death. Clinical symptoms, a positive newborn screening or a sibling with the disease are reasons to test for CF. Initial evaluation is with sweat chloride testing and an elevated result should be referred for DNA testing to detect CF-related mutations.
A mother who is at 31 weeks gestation is at high risk of preterm delivery. Which of the following is given prenatally to prevent the risk of infant respiratory distress syndrome? Albumin Betamethasone Phosphatidylglycerol Sphingomyelin
Correct Answer ( B ) Explanation: Glucocorticoids, most commonly betamethasone, are administered to help prevent infant respiratory distress syndrome (hyaline membrane disease) in mothers with premature deliveries. The action of steroids increase the production of surfactant, the key substance which is lacking in newborns who suffer from this condition. Several organizations recommend treatment in women at risk of delivering before 34 weeks gestation. Lung maturity may be tested in women >30 weeks gestation. It is accomplished through the use of amniocentesis. Fluid samples are then sent for different laboratory studies, including the lecithin-to-sphingomyelin ratio, the surfactant-to-albumin ratio and phosphatidylglycerol levels. Albumin (A), phosphatidylglycerol (C) and sphingomyelin (D) are laboratory markers used in the detection of poor lung maturity, not in the prevention of infant respiratory distress syndrome.
A 4-year-old is brought to the emergency department by his parents after they found him in the backyard shed choking and gagging. Before this event, he was otherwise healthy. His temperature is 101°F, heart rate is 95, blood pressure is 100/60, and respiratory rate 40. On exam, he appears sleepy but continues to cough. Pulmonary findings include moderate retractions and diffuse wheezes. A chest X-ray shows patchy infiltrates. What is the most likely diagnosis? Foreign body aspiration Hydrocarbon ingestion Organophosphate ingestion Status asthmaticus
Correct Answer ( B ) Explanation: Hydrocarbons (e.g. lamp oil, gasoline, lighter fluid, turpentine, benzene, kerosene) can be aspirated easily when ingested and may cause pneumonitis with volumes as low as < 1 mL. The lower viscosity compounds distribute across a larger lung surface. Symptoms of aspiration are non-specific and include grunting, gagging, choking, tachypnea, fever, retractions and persistent coughing. These findings may be delayed, so asymptomatic children should be observed for several hours. The chest X-ray typically shows diffuse bilateral infiltrates. Airway support is important because symptoms can evolve into respiratory failure. Hydrocarbons cause acute respiratory distress syndrome (ARDS) by inactivation of type II pneumocytes, which leads to surfactant deficiency. Foreign body aspiration (A) can cause similar respiratory symptoms, but the chest X-ray will either be normal or show a foreign body if it is radio-opaque. Organophosphate ingestion (C) can cause bronchospasm, but symptoms are not limited to the respiratory tract and include diarrhea, urination, miosis, emesis, lacrimation and salivation. An individual with status asthmaticus can have similar respiratory symptoms, but the fever and chest X-ray findings are inconsistent with the disease process. This patient has no history of wheezing, and status asthmaticus (D) generally has a more insidious onset. During an asthma exacerbation, the chest X-ray may show hyperinflation of the lungs, not patchy infiltrates.
What finding on high resolution computed tomographic imaging of the chest is most consistent with idiopathic pulmonary fibrosis? Ground glass patches Honeycombing Non-caseating granulomas Solitary nodule
Correct Answer ( B ) Explanation: Idiopathic pulmonary fibrosis (IPF) is a type of fibrosing interstitial pneumonia of unknown cause. It is characterised by a chronic nonproductive cough and gradual exertional dyspnea that usually develops over several months. IPF is most common in men, aged 60-70 years, with a history of cigarette smoking. The pathogenesis of IPF is likely related to epithelial cell damage and improper repair leading to chronic and progressive symptoms. Medical treatment for IPF has proven difficult, making for a poor prognosis, although nintedanib and pirfenidone seem to delay disease progression. This treatment coupled with supportive care, such as supplemental oxygen, can help maintain a normal level of activity for patients. Since treatment is often delayed by misdiagnosis, it is vital that providers perform a thorough physical exam which often reveals bibasilar crackles and finger clubbing as the disease progresses. Additionally, imaging with high resolution CT reveals structural changes to the lung parenchyma, such as traction bronchiectasis or honeycombing, which refers to the appearance of clusters of cystic airspaces in the lungs. These findings are characteristic of IPF, and in the setting of the appropriate clinical picture, are enough to confirm a diagnosis.
A 76-year-old man with a 30-year pack history of smoking tobacco presents with a chief complaint of increasing dyspnea on exertion over the past 7 months. What aspect of his history and physical examination would suggest idiopathic pulmonary fibrosis, rather than another diagnosis, as the source of his dyspnea? Chest pain Digital clubbing Joint pain Productive coughing
Correct Answer ( B ) Explanation: Idiopathic pulmonary fibrosis (IPF) is a type of fibrosing interstitial pneumonia that most commonly occurs in men over 50 years of age. Although it is linked to cigarette smoking and has been shown to occur in families, its exact cause is unknown. The pathogenesis of IPF is likely related to epithelial cell damage and improper repair leading to chronic and progressive symptoms. The most characteristic symptoms of IPF are a chronic nonproductive cough and gradual exertional dyspnea that usually develops over several months. Medical treatment for IPF has proven difficult, making for a poor prognosis, although nintedanib and pirfenidone seem to delay disease progression. This treatment coupled with supportive care, such as supplemental oxygen, can help maintain a normal level of activity for patients. Since treatment is often delayed by misdiagnosis, it is vital that providers perform a thorough physical exam and order appropriate images. Imaging on high resolution CT reveals structural changes to the lung parenchyma, such as traction bronchiectasis or honeycombing, which refers to the appearance of clusters of cystic airspaces in the lungs. While physical exam may be unremarkable during early disease, it often reveals bibasilar crackles and digital clubbing as the disease progresses, and this clinical presentation along with typical findings on high resolution CT is enough to establish diagnosis.
Which of the following oxygen delivery methods provides the highest fraction of inspired oxygen? Nasal cannula Non-rebreather mask Simple face mask Venturi mask
Correct Answer ( B ) Explanation: Many emergency department patients require supplemental oxygen. Several methods of oxygen delivery are available which provide varying concentrations of oxygen. The choice of method depends on on the patient's clinical condition and oxygen requirement. Nasal cannulae deliver oxygen via prongs inserted into the patient's nostrils, and are capable of oxygen flow rates of 2-5 L per minute, resulting in a FiO2 of 20-50%. A simple face mask can deliver oxygen at flow rates of 6-10 L/min for an FiO2 of 40-60%, and is useful in patients who find nasal prongs irritating, have epistaxis, or other reasons nasal prongs are problematic. Flow rates are determined by the patient's respiratory rate and tidal volume. A venturi mask mixes oxygen with room air at a settable concentration, and can deliver accurate and constant flow rates up to 40% FiO2. A venturi mask is helpful in situations where it is desirable to avoid excessive oxygen administration, such as in a patient with chronic obstructive pulmonary disease. Finally, a non-rebreather mask uses an oxygen reservoir and is capable of delivering the highest concentration of oxygen. Oxygen flow rates approach 10-15 L/minute, delivering an FiO2 of up to 100%. A non-rebreather mask must be secured tightly to the face, and can be uncomfortable for patients. Patients requiring intubation are typically preoxygenated using a nonrebreather mask in order to optimize oxygenation. Noninvasive positive pressure ventilation via continuous positive airway pressure (CPAP) and bilevel positive airway pressure (BiPAP) are also useful in optimizing oxygenation prior to intubation, especially if adequate oxygen saturations cannot be achieved using conventional means.
What type of cancer is classically associated with asbestos exposure? Large cell carcinoma Mesothelioma Small cell carcinoma Squamous cell carcinoma
Correct Answer ( B ) Explanation: Mesothelioma is a rare form of cancer that develops from cells of the mesothelium. Mesothelioma is most commonly caused by exposure to asbestos. The most common anatomical site for mesothelioma is the pleura, but it can also arise in the peritoneum, the pericardium, or the tunica vaginalis. Most people who develop mesothelioma have worked in jobs where they inhaled or ingested asbestos fibers, or were exposed to airborne asbestos dust and fibers in other ways. Concomitant cigarette smoking has been shown to accelerate the progression of asbestosis and increase cancer risk further. Early changes associated with asbestosis on chest radiograph include bilateral, irregular, linear opacities at the lung periphery, especially pronounced in the lower lung fields. The middle and upper lobes are typically involved later in the course of the disease. Mesothelioma commonly metastasizes, but death in these patients is usually due to local invasion.
A 45-year-old patient with newly diagnosed diabetes mellitus type 2 presents to your office for her annual exam. She has had her hepatitis B vaccination, but wants to know if she needs any additional vaccinations because of her new diagnosis. Which of the following is the most appropriate next step in her management? Administer annual influenza vaccine only Administer pneumococcus and annual influenza vaccines Administer pneumonia prophylaxis with trimethoprim-sulfamethoxazole The patient does not need any additional vaccines since she is up to date
Correct Answer ( B ) Explanation: Patients with diabetes mellitus require regular monitoring and health maintenance to prevent diabetes-related complications. Health maintenance for these patients includes three vaccinations: annual influenza, pneumococcus (repeated at age 65 if given prior to that age) and the hepatitis B three dose series. Patients with diabetes mellitus require annual foot, dental and dilated eye examinations, blood pressure monitoring, and smoking cessation counseling. Upon diagnosis, a serum creatinine should be drawn. Annual fasting serum lipids and urinary albumin-to-creatinine ratios should be monitored. Hemoglobin A1C should be obtained every 3-6 months with a goal of <7%.
Which of the following is the most common cause of a pleural effusion in developed countries? Bacterial pneumonia Heart failure Malignancy Tuberculosis
Correct Answer ( B ) Explanation: Pleural effusions occur when fluid accumulates between the parietal and visceral pleurae. In a healthy patient, fluid is continually produced by the parietal pleura which reduces friction allowing for smooth lung expansion. If there is overproduction or decreased reabsorption, fluid can accumulate in the potential space. Depending on the volume present, pleural effusions can be clinically silent or cause symptoms, such as dyspnea or chest pain. On physical exam, there will be decreased breath sounds and dullness to percussion. Pleural effusions can be seen on upright chest X-ray when there is more than 150-200 cc of fluid present. Smaller effusions may only be seen on left lateral decubitus films or CT scan. Pleural effusions can be classified as transudates or exudates based on Light criteria. Fluid is an exudate if one or more of the following are present: (1) pleural fluid/serum protein ratio > 0.5, (2) pleural fluid/serum LDH > 0.6, (3) pleural fluid LDH is > 2/3 of the upper limit for serum LDH. Common causes of transudates include congestive heart failure, cirrhosis, and nephrotic syndrome. Common causes of exudates include malignancy, infection, and pulmonary embolism. Overall, the most common cause of a pleural effusion in the Western world is heart failure.
In a patient with suspected HIV infection and shortness of breath, which of the following findings is most suggestive of Pneumocystis jiroveci pneumonia? Bradycardia despite relative volume depletion Elevated serum lactate dehydrogenase Hyponatremia Unilateral lobar consolidation on chest radiograph
Correct Answer ( B ) Explanation: Pneumocystis pneumonia is one of the most common opportunistic infections in patients with HIV. Patients typically present with an insidious nonproductive cough, dyspnea, unexplained fever for longer than two weeks, chest pain, and fatigue. The greater the elevation of lactate dehydrogenase (LDH), the worse the prognosis. Relative bradycardia (A) is commonly associated with typhoid fever (Salmonella typhi), Legionnaire's disease (Legionella pneumophila), infectious mononucleosis, and pneumonia caused by Chlamydia species. The syndrome of inappropriate antidiuretic hormone secretion (SIADH) is one of several causes of hyponatremia (C). SIADH may be seen in patients with certain pulmonary infections, including Legionnaire's disease and tuberculosis
A 36-year-old veterinarian presents with myalgias, dry cough, and severe headache. His vital signs include blood pressure 138/74 mm Hg, heart rate 82 beats/minute, temperature 39°C, and oxygen saturation 94% on room air. He has hepatosplenomegaly on abdominal exam. His chest X-ray shows patchy perihilar infiltrates. What of the following is the most appropriate antibiotic for this patient? Amoxicillin-clavulanate Doxycycline Levofloxacin Trimethoprim-sulfamethoxazole
Correct Answer ( B ) Explanation: Psittacosis is caused by Chlamydia psittaci, an obligate intracellular gram-negative organism. It is harbored in avian species making bird owners, veterinarians, and pet-shop employees particularly susceptible to infection. Patients present with high fevers, severe headache, myalgias, nonproductive cough, and hepatosplenomegaly. Chest X-rays show patchy perihilar or lower lobe infiltrates. Patients may have proteinuria and elevated liver transaminases. Diagnosis should be considered in patients with community acquired pneumonia and exposure to birds. The treatment of choice is a 14-21 day course of doxycycline. Complications are uncommon in patients treated with appropriate antibiotics.
Which of the following is the first line treatment for latent TB infection? Doxycycline Isoniazid Lamivudine/Zidovudine Penicillin
Correct Answer ( B ) Explanation: Standard therapy for latent TB infection (LTBI) is isoniazid (INH). In the United States, latent tuberculosis infection is the most prevalent form of tuberculosis. LTBI is the term given to patients with a positive purified protein derivative (PPD) skin test without evidence of active TB. PPD has been used for more than 100 years and relies on delayed-type hypersensitivity (DTH) to M. tuberculosis cellular proteins. Because PPD relies on DTH, any factor that reduces the DTH affects the host response to PPD. The most common clinical example is use of corticosteroids, which blunt the DTH response and can complicate PPD interpretation. Therefore, PPD testing should not be performed while a patient is taking corticosteroids. Also, TB testing should be targeted to those with higher risk of infection and should not routinely be done in those with low risk. Patients at increased risk for progression to active TB include those who have been recently infected (recent PPD converters); patients who are HIV seropositive; patients who have silicosis, diabetes, or chronic renal failure (including those receiving hemodialysis); solid-organ transplant recipients; patients with gastrectomy or jejunoileal bypass or head and neck cancer; injection drug users; patients with chest radiograph evidence of prior TB; and patients who weigh at least 5% less than ideal body weight. Patients taking chronic corticosteroid therapy and those who are to receive tumor necrosis factor alpha (TNF-alpha) blockers (e.g., infliximab) are also at risk.
A 12-year-old girl presents to the clinic with one day of cough, fever to 102°F, and extreme fatigue. She is awake and alert but appears tired. Her oxygen saturation is 95 percent, and respiratory rate is 15 breaths per minute. Lung auscultation reveals fine crackles in the left upper lobe. Which of the following is treatment of choice? AAmpicillin BAzithromycinCorrect Answer CCefotaxime DClindamycin
Correct Answer ( B ) Explanation: The above patient's constellation of fever, cough, and focal lung findings are concerning for community-acquired pneumonia. The onset of symptoms was acute and severe, and the lung findings are focal, which is suggestive of a typical bacterial etiology. S. pneumoniae is the most frequent cause of "typical" bacterial pneumonia in children of all ages. However, in otherwise healthy children five years and older with CAP who are not ill enough to require hospitalization, M. pneumoniae and C. pneumoniae are the most likely pathogens. A macrolide is the empirical treatment of choice for typical bacterial pneumonia in otherwise healthy children > 5 years old being treated in outpatient setting. Among the macrolide antibiotics, clarithromycin and azithromycin have a more convenient dosing schedule and fewer side effects than erythromycin, but erythromycin is less expensive. Macrolide antibiotics may provide coverage for S. pneumoniae, which is the most frequent typical bacterial pathogen for all age groups. However, approximately 40 to 50 percent of S. pneumoniae isolates are resistant to macrolides. Failure to respond to macrolide therapy may indicate the development of a complication, a macrolide-resistant pathogen, or the need to alter therapy to provide better pneumococcal coverage. Given the significant resistance of S. pneumoniae to macrolides, fluoroquinolones (e.g., levofloxacin, moxifloxacin) are another reasonable alternative for the outpatient treatment of CAP. In addition to their excellent Gram-negative spectrum, the fluoroquinolones are active against a number of the pathogens responsible for CAP, including beta-lactam-susceptible and non-susceptible S. pneumoniae, M. pneumoniae, and C. pneumoniae. However, S. pneumoniae resistant to levofloxacin has been identified
A seven-year-old boy is brought by his mother to the clinic because of coughing. For the past week, he has had a nonproductive cough. On physical examination, vital signs are normal, with erythematous posterior pharynx, and clear breath sounds. Complete blood count is normal. Chest radiograph reveals perihilar infiltrates. Polymerase chain reaction from the boy's nasopharyngeal specimen comes back positive for Mycoplasma pneumoniae. Which of the following is the most appropriate therapy? Amoxicillin Clarithromycin Levofloxacin Oseltamivir
Correct Answer ( B ) Explanation: The boy has a lower respiratory tract infection due to Mycoplasma pneumoniae. The cough caused by M. pneumoniae infection ranges from nonproductive to mildly productive. Wheezing and dyspnea also may occur, although dyspnea is not a common complaint. Chills are common, but rigors are very rare. Additional respiratory symptoms include pharyngitis, rhinorrhea, and ear pain. Compared to those with pyogenic pneumonia, patients with mycoplasma pneumonia tend to have a more gradual onset of symptoms, less respiratory distress, and usually a normal white blood cell count. The Infectious Diseases Society of America (IDSA) suggests serology or polymerase chain reaction (PCR) tests for the laboratory diagnosis of M. pneumoniae. When available, PCR from a nasopharyngeal specimen can be done rapidly, has a high specificity, and is the diagnostic test of choice. Empiric treatment for M. pneumoniae pneumonia often is initiated based on clinical suspicion given the difficulty with definitive diagnosis. Suspected or documented M. pneumoniae lower respiratory tract infections are treated with a macrolide or tetracycline antibiotic, which is consistent with those in the Pediatric Infectious Diseases Society (PIDS) and the Infectious Diseases Society of America (IDSA) guidelines. Among the macrolide antibiotics, clarithromycin and azithromycin have a more convenient dosing schedule and fewer side effects than erythromycin, but erythromycin is less expensive.
A 29-year-old man with a history of HIV presents with shortness of breath and fever. He has a productive cough but denies hemoptysis. You obtain the chest radiograph seen above. Which of the following is true regarding the patient's diagnosis? Elevated LDH is common Spread is by the hematogenous route Steroids should be administered prior to antibiotics if the PaO2 is <80 Trimethoprim-sulfamethoxazole is the treatment
Correct Answer ( B ) Explanation: The chest radiograph demonstrates miliary tuberculosis (TB), or acute disseminated tuberculosis. The term miliary was first used to describe the pathologic lesions seen on radiography that appeared as small millet seeds. Miliary TB occurs when the host is unable to contain a recently acquired or a dormant TB infection. The condition was mostly seen in young children after primary infection but now is more common in the elderly and in persons infected with HIV. Spread of the mycobacteria occurs through the hematogenous route, which leads to the multisystem nature of miliary TB. Clinically, patients develop many of the similar signs and symptoms of active pulmonary TB—fever, weight loss, anorexia, and weakness. Hemoptysis is uncommon. The classic miliary pattern seen in the radiograph is present in approximately 50% of cases. Hyponatremia is sometimes seen from the development of SIADH. Mortality rates are higher than for other forms of TB, which is likely due to a delay in treatment. Elevated LDH (A) commonly occurs in Pneumocystis jiroveci pneumonia. Also with Pneumocystis jiroveci pneumonia, steroids (C) should be administered prior to antibiotics when the PaO2 is <70 or the A-a gradient is >35. Trimethoprim-sulfamethoxazole (D) is the treatment for Pneumocystis jiroveci pneumonia. Treatment of miliary TB is the same as pulmonary TB.
72-year-old man with a 40 pack-year smoking history presents to the emergency department complaining of shortness of breath and cough for two days. He reports difficulty breathing when walking short distances in his home. His cough is nonproductive. His oxygen saturation is 87% on room air. He is alert but has difficulty speaking in complete sentences. On auscultation, there is diffuse expiratory wheezing. Chest radiograph shows hyperexpansion and flattened diaphragms. Which of the following is the most appropriate therapy? Administration of supplemental oxygen, intravenous methylprednisolone, theophylline Administration of supplemental oxygen, nebulized albuterol and ipratropium, oral prednisone Administration of supplemental oxygen, nebulized albuterol, intravenous antibiotics Administration of supplemental oxygen, oral prednisone, terbutaline
Correct Answer ( B ) Explanation: The most appropriate therapy in a patient with a chronic obstructive pulmonary disease (COPD) exacerbation is administration of supplemental oxygen, nebulized albuterol and ipratropium, and oral prednisone. Antibiotics are necessary in patients with increased dyspnea, increased sputum production, and increased sputum purulence. Administration of supplemental oxygen should target a pulse oximetry between 88 and 92 percent to avoid worsened hypercapnia. Goal arterial oxygen tension is 60 to 70 mm Hg. A nasal cannula can effectively provide oxygen at flow rates up to 6 liters per minute and is much more convenient and comfortable for the patient, especially when eating. Venturi masks, another method of oxygen delivery, are advantageous in that they deliver precise oxygen content but are not as well tolerated by patients. Endotracheal intubation must be considered if the patient has altered mental status, profound acidemia, or cardiac dysrhythmia. Inhaled short-acting beta agonists such as albuterol are the mainstay in treatment of COPD exacerbations. These medications can be given via nebulizer or metered-dose inhaler with a spacer, though in the setting of an acute exacerbation nebulized therapy is preferred. Short-acting beta agonists are frequently administered with a short-acting anticholinergic such as ipratropium, even though evidence regarding this is conflicted. A short course (less than fourteen days) of an oral glucocorticoid is recommended in patients with a COPD exacerbation if they are tolerating oral intake, are not in shock, and are having a mild to moderate exacerbation. The reason for this is that oral glucocorticoids are more rapidly absorbed, achieving peak levels within one hour of ingestion, and are almost completely bioavailable. In addition to the exceptions noted above, intravenous glucocorticoids may be administered if the patient already failed treatment with oral glucocorticoids. There is debate about the optimal duration of treatment with glucocorticoids. Global Initiative for Chronic Obstructive Lung Disease (GOLD) guidelines recommend the equivalent of forty milligrams of prednisone daily for five days. Other societies suggest a course of less than fourteen days. The duration of treatment is determined by the severity of the exacerbation and the patient's response to treatment. At the end of the glucocorticoid treatment course, the medication can be discontinued rather than tapered. Treatment courses in COPD are generally too short to cause adrenal suppression, so this should not be a concern when stopping the patient's medication. In addition to the above, smoking cessation should be addressed with the patient.
Which of the following treatments is most appropriate for a patient with acutely symptomatic sarcoidosis? Lung transplantation Prednisone Theophylline Tiotropium
Correct Answer ( B ) Explanation: The treatment of pulmonary sarcoidosis is based on the underlying pathology. Essentially, sarcoidosis is the result of an excessive immune system reaction. As such, anti-inflammatory medications are logical modalities. In up to 75% of all patients with pulmonary sarcoidosis, non-steroidal anti-inflammatory medications are all that is required to obtain symptom relief. However, 15% of patient with pulmonary sarcoidosis exhibit more severe symptoms and pulmonary decline. The cornerstone of therapy for these patients is oral or inhaled corticosteroids, usually given in a tapered fashion over a 6-month period. Acute flares can usually be treated successfully with 3 week tapering doses. Interestingly, there is not much concrete data to support the benefit of corticosteroid treatment even though it is widely used.
A 47-year-old woman with diabetes mellitus is found to have latent tuberculosis during a routine employee health purified protein derivative (PPD) test. She should receive supplementation with pyridoxine (vitamin B6) if her drug regimen contains which of the following medications? Ethambutol Isoniazid Pyrazinamide Rifampin
Correct Answer ( B ) Explanation: The use of isoniazid requires supplementation with pyridoxine in high risk patients (eg. those with diabetes, HIV, chronic alcoholism, renal disease or pregnant or breast-feeding women) to prevent the development of peripheral neuropathies during treatment of active or latent tuberculosis. Ethambutol (A), a bacteriostatic anti-tuberculosis drug, is more likely to cause optic neuritis. This can be distinguished with a visual acuity screening and red-green discrimination test, and is usually reversible with discontinuation of the drug. Pyrazinamide (C), a bactericidal anti-tuberculosis drug, is known to cause hyperuricemia, hepatotoxicity, and joint pain. Side effects should be monitored during treatment with uric acid and liver function tests. Rifampin (D), a bactericidal anti-tuberculosis drug, may also cause hepatitis, as well as flu-like symptoms and poor clotting. Monitoring parameters should include liver function tests and complete blood counts. Patients should also be cautioned that this drug can color bodily secretions orange and stain contact lenses.
An 18-year-old woman with a history of asthma presents to her primary care provider for follow-up. Currently she experiences symptoms of coughing, wheezing and shortness of breath 3 days a week and uses her rescue inhaler 3 days a week. She awakens at night with similar symptoms 3 times a month. She reports some minor interference with her daily routine. Which of the following options is an appropriate management plan at this time? AHigh dose inhaled corticosteroid and short-acting beta-agonist as needed BLow dose inhaled corticosteroid and short-acting beta-agonist as needed COral corticosteroid and short-acting beta-agonist as needed DShort acting beta-agonist as needed
Correct Answer ( B ) Explanation: This patient can be classified as having mild persistent asthma. The recommended regimen for her includes a low dose inhaled corticosteroid (ICS) and short-acting beta-agonist (rescue inhaler). Asthma is a type of obstructive lung disease marked by chronic inflammation of the lower airways. Classic symptoms include recurrent episodes of cough, shortness of breath and wheezing. A pulmonary function test is initially obtained to confirm diagnosis. Test results demonstrate a decreased FEV1 (forced expiratory volume in 1 second) to FVC (forced vital capacity). Administration of a short acting beta agonist shows some reversibility. A step-wise approach is used in treating symptoms of asthma based on the severity. Inhaled corticosteroids are useful in reducing airway inflammation and bronchial hyper-responsiveness. Peak flow monitoring is an effective way to assess asthma control at home. In addition, it is important to reduce airway irritants and triggers to avoid exacerbation of symptoms.
A 55-year-old man, smoker, presents to the ED with hemoptysis and dyspnea for four weeks. His vital signs are T 37°C, BP 146/76 mm Hg, HR 85 bpm, RR 20 per minute, and oxygen saturation 96% on RA. His lung exam reveals distant breath sounds on the left side. His chest X-ray is shown above. What is the most likely cause of his hemoptysis? ABronchitis BLung cancerCorrect Answer CPneumonia DPulmonary embolism
Correct Answer ( B ) Explanation: This patient has a lung mass that has eroded into a pulmonary vessel. The pulmonary blood supply is under low pressure, so bleeding can present as blood-tinged sputum or gross hemoptysis. Massive hemoptysis is caused by hemorrhage from a bronchial vessel in the majority of cases because bronchial vessels are under systemic blood pressure. Some 80%-90% of patients with cancer as the cause of hemoptysis will have abnormalities on the chest X-ray. Those with normal chest x-rays with a high suspicion for an underlying neoplasm should undergo a chest CT scan and have an outpatient fiber-optic bronchoscopy.
A 68-year-old man presents for a wellness visit. He is a former smoker, having quit 3 years ago. He has a 35 pack-year history of smoking. He has no other comorbidities and is feeling well at the time of his visit. What screening needs to be performed with regard to lung cancer? Chest X-ray Low dose chest CT scan No screening is indicated Pulmonary function tests
Correct Answer ( B ) Explanation: This patient needs annual low dose chest CT scans until he has been a non-smoker for 15 years consecutively. United States Preventive Services Task Force (USPSTF) guidelines state patients aged 55-80 years old who are either smokers or former smokers who quit within the past 15 years and have a total of 30 pack-years or more need annual low dose chest CTs until they have been non-smokers for 15 or more years. Lung cancer is the third most common cancer in the USA and the top cause of cancer related deaths. Smoking is the most common risk factor for lung cancer and is associated with 85% of diagnosed lung cancer in the USA. Incidence increases with age and cumulative smoke exposure. Low dose CT scanning has a sensitivity of 93.8% and a specificity of 73.4% when screening for lung cancer.
A 30-year old woman presents with shortness of breath. She has a history of asthma since childhood. She says that she uses her rescue inhaler 3 days each week. On average, she wakes up in the middle of the night with symptoms approximately three times a month. Her vital signs are BP 120/76, HR 112, RR 26, and oxygen saturation 92% on room air. The patient appears to be in mild distress with intercostal retractions present, and respiratory exam is remarkable for diffuse wheezing. This patient is diagnosed with an acute asthma exacerbation. Which of the following best classifies this patient's asthma? Intermittent Mild persistent Moderate persistent Severe persistent
Correct Answer ( B ) Explanation: This patient's asthma is classified as mild persistent. Symptoms of asthma most frequently include shortness of breath and wheezing, but patients may also complain of cough and chest tightness. Mild persistent asthma is classified as symptoms more than twice per week but not daily, nocturnal symptoms three or four times each month, exacerbations causing minor limitations in activity and sleep, and the use of an inhaled short-acting beta-2-agonist more than twice a week but not daily. These patients are generally treated with a low-dose inhaled corticosteroid with a short acting beta-agonist as needed.
A 28-year-old man presents to the Emergency Department with dyspnea. On an anterior-posterior chest X-ray, you notice a visceral pleural line with a radiolucent area devoid of vascular and pulmonary markings on the right side only. What is the most likely diagnosis? Adenocarcinoma Pneumonia Pneumothorax Pulmonary embolism
Correct Answer ( C ) Explanation: A pneumothorax is characterized by unilateral findings of a visceral pleural line with no vascular pulmonary markings in between this line in the chest wall on chest X-ray. A pneumothorax is a collection of air in the pleural space between the lung and the chest wall. The cause may be spontaneous, iatrogenic, or traumatic. The highest frequency of spontaneous pneumothorax occurs in tall, thin males aged 10 to 30 years. The usual patient presentation is with symptoms of dyspnea and chest pain. Physical exam findings are decreased or absent breath sounds unilaterally, as well as unilateral chest expansion and hyperresonance. Chest X-ray confirms the presence of pleural air, with a visceral pleural line evident. A small pneumothorax may resolve spontaneously, but most require placement of a chest tube. A tension pneumothorax is a medical emergency. This is often due to a chest wound or laceration that allows inspiration of air but not expiration. This causes mediastinal shift to the contralateral side and cardiovascular compromise. A large-bore needle is inserted into the chest to allow expiration of air, before placement of a chest tube.
A 22-year-old woman presents with dyspnea. She has a history of asthma and noted increased difficulty breathing starting yesterday. She says she has been using her albuterol inhaler every 15 minutes for the last four hours without relief. What laboratory abnormality is likely to be found in this patient? Hypocalcemia Hypoglycemia Hypokalemia Hyponatremia
Correct Answer ( C ) Explanation: Albuterol, a beta-2-adrenergic agonist, is a widely used medication in the treatment of asthma and chronic obstructive pulmonary disease. Beta-2-adrenergic agonists work selectively on the beta-2 receptors on bronchial smooth muscle resulting in bronchodilation. The most common side effects include tremor and tachycardia. Overuse of these medications can also result in hypokalemia due to increased activity of Na-K-ATPase pumps in skeletal muscle which drive potassium intracellularly. Serum potassium levels can be decreased by 0.5 to 1.5 mEq/L. While this is unlikely to result in clinically significant hypokalemia, it can be useful in patients who present with acute hyperkalemia. While not recommended as monotherapy in the treatment of hyperkalemia, it has been shown to have additive effects when used in conjunction with insulin and glucose in transiently lowering potassium levels.
A 43-year-old man with asthma presents with wheezing. After 6 inhaled albuterol treatments he feels better and his lungs are clear. Which of the following is true regarding further management? Intravenous corticosteroid is indicated Intravenous magnesium sulfate is indicated Oral corticosteroid is indicated Oral respiratory antibiotic is indicated
Correct Answer ( C ) Explanation: The patient presents with a moderate asthma exacerbation that has resolved with beta-agonists and should have oral corticosteroids added to his treatment. Corticosteroids inhibit the release of inflammatory mediators and cytokines and decrease recruitment of inflammatory cells in asthma. This results in decreased airway inflammation and secondarily limits induced bronchoconstriction. The effect of steroids begins within hours in an acute asthma exacerbation and reduces both the rate of relapse and rate of admission in severe attacks. In general, patients with moderate to severe reactions should have short-course corticosteroids added to their treatment regimens in the Emergency Department. Oral prednisone or prednisolone are appropriate interventions.
An unimmunized 7-year-old boy without any previous medical history presents with fever and cough. The parents report that he has been coughing for 2 weeks. The patient has sudden fits of coughing followed sometimes by post-tussive emesis. Which of the following is an appropriate plan? Albuterol nebulizer Corticosteroid therapy Macrolide antibiotic Respiratory viral panel
Correct Answer ( C ) Explanation: Although a vaccine for pertussis was developed in the 1940s, it remains a cause of significant respiratory illness in the US each year. Decreasing immunity in adults has contributed to persistent annual cases of this disease. The disease occurs most commonly in children under the age of one who have not received the entire vaccine series and nonimmunized children. Pertussis is caused by Bordatella spp. which specifically adheres to the ciliated epithelial cells of the respiratory tract. There are three phases to the illness. The catarrhal phase lasts 1-2 weeks after an incubation period of 7-10 days. Patients are most infectious during this phase, the symptoms of which are indistinguishable from an average upper respiratory infection. Phase two is the paroxysmal phase characterized by the resolution of fever and increased cough. For 2-4 weeks, patients have paroxysms of the classic "staccato" cough. Patients cough in fits followed sometimes by the "whoop" when they forcefully inhale. Post-tussive emesis, syncope and brief apneic episodes may occur. The convalescent third phase lasts for weeks to months and is characterized by a residual cough. Antibiotic therapy with a macrolide is indicated to decrease the infectivity of patients. The use of antibiotics does not decrease the duration of illness or severity of illness. The primary treatment is supportive care. Erythromycin is contraindicated for infants < 1 month due to increased risk for pyloric stenosis. Infants < 1 month old should receive azithromycin.
A 65-year-old man presents with cough and progressive dyspnea on exertion. He has no significant cardiac disease. He worked as an asbestos remover for thirty years. Examination reveals crackles but no wheezing. Pulmonary imaging reveals multiple basilar round opacities and pleural plaques. This patient is most at risk for developing which of the following diseases? ALoffler's syndrome BLofgren's syndrome CMesothelioma DMultiple myeloma
Correct Answer ( C ) Explanation: Asbestosis is one of the occupational lung diseases known as pneumoconiosis. It is a chronic inflammatory condition of the pulmonary interstitium, resulting in fibrosis and restrictive lung disease. It occurs in those chronically exposed to asbestos, namely manufacturers of asbestos-containing products or those involved in the removal of asbestos-containing materials. Symptoms usually only manifest after long exposure occurs. They include a progressive and sometimes severe dyspnea, especially on exertion, and dry cough. Bibasilar crackles are typical. In addition to the interstitial fibrosis, pleural thickening and plaques, as well as pleural effusion, occurs. Respiratory failure may be the end result of this condition. Pulmonary function testing reveals restrictive ventilatory defects. Diagnosis is mostly based on the classic occupational history and the key imaging findings of pleural plaques and small, basilar opacities. Although not necessary in most cases, biopsy results will show asbestos bodies. A rare form of cancer, malignant mesothelioma, is most commonly caused by chronic asbestos exposure. Although this cancer mainly occurs in the mesothelial cells of the pleura, it can also develop in the pericardium, peritoneum and tunica vaginalis.
Which of the following statements is true concerning aspiration pneumonitis? Antibiotics should be administered early in the course Corticosteroid administration decreases the incidence of associated lung injury It is caused by an inflammatory chemical injury The ABG is diagnostic
Correct Answer ( C ) Explanation: Aspiration pneumonitis is an inflammatory chemical injury of the tracheobronchial tree and pulmonary parenchyma produced from the inhalation of regurgitated gastric contents. Aspiration pneumonitis can lead to aspiration pneumonia due to the breakdown of the pulmonary defense mechanisms caused by the chemical irritation. Aspiration pneumonitis is usually associated with depressed level of consciousness that allows for regurgitation of gastric contents and inhibits the protective upper airway reflexes from preventing aspiration. The classic patient has a depressed mental status due to recreational or therapeutic drugs and regurgitates a large volume of gastric contents. The elderly and those patients with a history of stroke are also at increased risk for aspiration pneumonitis. Clinically, patients may have minor symptoms (nonproductive cough and tachypnea). With larger aspirations, patients may develop tracheobronchitis with bronchospasm, bloody or frothy sputum, and respiratory distress. The chest radiograph usually shows unilateral focal or patchy consolidations in the dependent lung segments.
A 37-year-old man presents with cough and shortness of breath. Vital signs are T 102°F, BP 110/76, HR 108, RR 20, and oxygen saturation of 92% on room air. His chest X-ray is shown above. Which of the following helps determine the causative organism? Exposure to white powder History of smoking Recent influenza infection Residence in Connecticut
Correct Answer ( C ) Explanation: Cavitary lesions of the lung have multiple causes, including both infectious and non-infectious etiologies. These include bacterial pneumonia, fungal disease, tuberculosis, malignancies and some pulmonary vascular disease. The bacteria most commonly associated with cavitations are anaerobes, aerobic gram-negative bacilli and Staph aureus. After a recent influenza infection, patients may develop a Staph aureus pneumonia. Of particular concern is community-associated methicillin-resistant Staph aureus (CA-MRSA) after influenza especially in a rapidly progressive pneumonia in younger, healthy patients. Staph pneumonias often have necrotizing features creating the cavitation and may also lead to the development of pneumatoceles
A 60-year-old man with a past medical history of tobacco use presents to clinic with a chief complaint of cough. The cough has been present for five months, and he now has difficulty breathing when walking to his mailbox. He also had a "coughing fit" similar to this last year lasting three to four months. What is the most likely diagnosis? Acute bronchitis Asthma Chronic bronchitis Emphysema
Correct Answer ( C ) Explanation: Chronic bronchitis is a form of chronic obstructive pulmonary disease that is defined as a chronic productive cough for at least three months in at least two successive years. In chronic bronchitis, the bronchi become irreversibly inflamed causing mucus hypersecretion and coughing. It is described as the "blue bloater" as patients are commonly overweight and cyanotic. Symptoms include chronic cough, purulent sputum, and dyspnea, particularly on exertion. Physical examination in a patient with chronic bronchitis reveals decreased breath sounds, increased resonance upon percussion of the lung fields, and use of accessory muscles to breathe. Treatment includes smoking cessation, lifestyle changes with pulmonary rehabilitation, inhaled bronchodilators, and inhaled steroids
You are seeing an elderly man with new onset of peripheral edema, head fullness and neck venous engorgement. Initial testing shows normal left heart function. You suspect cor pulmonale. Which of the following tests is the most accurate in confirming this diagnosis? Cardiac magnetic resonance imaging Electrocardiography Right heart catheterization Ultrafast ECG-gated computed tomography
Correct Answer ( C ) Explanation: Cor pulmonale is defined as an alteration in the structure and function of the right ventricle caused by a primary disorder of the respiratory system. Although the most common cause of right heart failure is left-sided heart disease, cor pulmonale is right heart dysfunction due to a lung, and not heart, problem. Just like systemic hypertension causes changes in left ventricular function, pulmonary hypertension causes changes in right ventricular function. This major underlying pathology is largely due to some kind of pulmonary vascular bed compromise, which can be primary pulmonary hypertension or thromboembolic disease, but more commonly anatomic compromise (COPD, interstitial lung disease and rheumatologic or connective tissue/collagen vascular disorders) or vasoconstrictive compromise (chronic hypoxic states and acidemia). When evaluating a patient with right heart failure, cor pulmonale is considered if pulmonary pathology is causative. However, if the etiologic evaluation is void of a pulmonary source, then the diagnosis of cor pulmonale cannot be made. In this situation, the clinician then tries to pinpoint a cardiac or blood disorder. Even when a pulmonary source is found to be a cause of right side heart failure, it is equally important to determine if there is such a coexisting non-pulmonary cause, such as increased blood viscosity, atrial and ventricular defects, congenital heart disease, cardiomyopathies and constrictive pericarditis. The general approach to evaluating a patient with suspected cor pulmonale begins with routine lab testing, chest radiography and electrocardiography (ECG). Further investigation of underlying pulmonary pathology is then accomplished via pulmonary function testing, ventilation/perfusion (V/Q) scanning and chest computed tomography. Right heart catheterization is the most accurate but invasive test to confirm the diagnosis of cor pulmonale.
A 58-year-old man presents with shortness of breath for 2 days. He complains of a six-month history of a dry cough, unintentional weight loss, and night sweats. He has no past medical history, but he has a 40 pack-year smoking history. In the ED, his vital signs are BP 132/76, HR 72, RR 16, oxygen saturation 96% on room air, and temperature 98.8°F. An ECG reveals no acute abnormality, and a chest X-ray shows a right middle lobe irregular mass and a right-sided pleural effusion. A thoracentesis is performed. What findings would be expected on pleural fluid analysis? Fluid:Blood LDH ratio < 0.6 Fluid:Blood Protein ratio < 0.5 Glucose < 60 mg/dL LDH < 200
Correct Answer ( C ) Explanation: Given his extensive smoking history, irregular mass noted on chest X-ray, and his recent unintentional weight loss, night sweats, and chronic cough, his pleural effusion is most likely secondary to an underlying lung malignancy. A pleural effusion is defined as more than 15 cc of fluid between the parietal and visceral pleuras. The most common causes are congestive heart failure (CHF), pneumonia, and malignancy. An effusion can be classified as transudative or exudative. A transudative effusion occurs secondary to increased hydrostatic pressure or decreased oncotic pressure. Common causes of transudative effusions include CHF, hypoalbuminemia, cirrhosis, and nephrotic syndrome. An exudative effusion is caused by increased permeability of pleural surfaces or decreased lymphatic flow. Common causes of exudative effusions include infection, such as pneumonia or tuberculosis, neoplasm, connective tissue disease, pulmonary embolism, uremia, pancreatitis, or esophageal rupture. A thoracentesis can be performed to analyze the fluid and determine a cause. Light's Criteria defines a transudative effusion by a LDH <200 (or less than 2/3 of the upper limit of the normal serum LDH level), a fluid:blood LDH ratio <0.6, and a fluid:blood protein ratio <0.5. In an exudative effusion, the glucose level is <60 mg/dL. In addition, if an exudative effusion is suspected, other tests such as fluid amylase, cell count, gram stain, culture, and cytology should be obtained. Fluid:blood LDH ratio <0.6 (A), fluid:blood protein ratio <0.5 (B), and LDH<200 (D) are all representative of a transudative effusion.
A 27-gestational-week-old infant is delivered in the emergency department. The neonate is cyanotic, tachypneic and shows chest wall retractions. An emergent chest radiograph reveals a diffuse ground-glass appearance. Meconium aspiration is not suspected. Which of the following is the most likely diagnosis? Erythroblastosis fetalis Hydrops fetalis Infant respiratory distress syndrome Persistent pulmonary hypertension of the newborn
Correct Answer ( C ) Explanation: Infant (neonatal) respiratory distress syndrome is the new term for hyaline membrane disease. It is also known as respiratory distress syndrome of the newborn and surfactant deficiency disorder. It occurs in premature infants whose pulmonary system has not completely matured, namely those born before 34 weeks (especially between 26 and 28 weeks) gestational age, affecting about 1% of all newborns. It is due to a deficiency of surfactant, a lipid-protein substance produced by type II pneumocytes which decreases surface tension in, and prevents collapse of, alveoli. Symptoms include tachypnea, tachycardia, cyanosis, nasal flaring, expiratory grunting and chest wall retractions. Diagnosis is made clinically and radiographically. The typical radiograph shows decreased lung volume, air-bronchograms and a ground-glass appearance. Treatment includes oxygen, continuous positive airway pressure, exogenous surfactant and in severe cases, intubation and extracoporeal membrane oxygenation. Hemolytic disease of the newborn, also known as Rh disease, or Rh incompatibility, occurs in subsequent pregnancies of a Rh negative woman and RH positive fetus. Mild cases present as neonatal anemia and reticulocytosis, while severe causes result in erythroblastosis (A), hyrops fetalis or stillbirth. Hydrops fetalis (B) is an abnormal accumulation of edema in at least two fetal compartments, due to several maternofetal immune and nonimmune conditions. Its common manifestation is prenatal heart failure, not respiratory failure. Persistent pulmonary hypertension of the newborn (PPHN) (D) is due to a failure of the circulatory transition which occurs during birth, mainly due to a right-to-left intracardiac shunt. It occurs mainly in term or near-term neonates, and only very infrequently in preterm neonates. Lung fields are mostly clear in radiographs of those neonates diagnosed with PPHN. It is commonly caused by meconium aspiration.
Which of the following complications can be prevented by simultaneously administering pyridoxine and isoniazid in a patient with tuberculosis exposure? Color blindness Hepatitis Peripheral neuropathy Renal failure
Correct Answer ( C ) Explanation: Isoniazid (INH) inhibits the enzyme responsible for the conversion of pyridoxine (vitamin B6) to one of its active metabolites, pyridoxal phosphate (PLP). This depletion of vitamin B6 may lead to complications such as peripheral neuropathy and seizures. Therefore, vitamin B6 should be administered concomitantly to patients taking isoniazid. PLP is also a coenzyme required for the synthesis of gamma-aminobutyric acid (GABA), an inhibitory neurotransmitter. Decreased GABA formation in the setting of vitamin B6 deficiency may also contribute to seizures. Color blindness (A) is not a complication of INH. However, another commonly used drug in TB, ethambutol, is associated with retrobulbar neuritis and red-green color blindness. INH is metabolized by the liver and gets converted to an ammonium molecule that can lead to hepatotoxicity (B). However, this is not affected by vitamin B6 supplementation. Renal failure (D) is a complication of pyridoxine overdose.
A 65-year-old smoker with no documented cardiac disease presents with several months of worsening cough and exertional dyspnea. He denies exposure to inhalation toxins or dusts. Examination reveals bibasilar inspiratory crackles and fingernail clubbing. Spirometry reveals a restrictive lung pattern. A chest computed tomography scan is read as "honeycombing in the periphery and bases." A bronchoalveolar lavage report is only significant for lymphocytosis and a histological classification of usual interstitial pneumonia. Which of the following is the most likely diagnosis? Chronic obstructive pulmonary disease Community-acquired pneumonia Idiopathic pulmonary fibrosis Pneumoconiosis
Correct Answer ( C ) Explanation: Nomenclature of restrictive pulmonary disease can be confusing. It can be simplified if one first dissects the lung into airspace (mostly obstructive diseases), parenchyma (the lung "meat" which doesn't contain air, aka interstitium) and chest wall (the mechanism that runs the airspaces and parenchyma). Restrictive disease encompasses abnormalities of the parenchyma and chest wall. Idiopathic pulmonary fibrosis (IPF) represents a main subclassification of the more than 200 interstitial lung diseases. IPF is a chronic progressive, and often fatal, interstitial lung disease which affects adults over 50 years of age (men > women and smokers > non-smokers). Although linked to tobacco, occupational toxin exposure, gastroesophageal reflux and genetic predisposition, the exact etiology remains unknown, hence the term idiopathic. It is felt the pathology is based on abnormal wound healing, oxidant-antioxidant imbalance and excessive deposition of collagen. Patients experience a progressive dry cough, exertional dyspnea, inspiratory "velcro-like" crackles and fingernail clubbing. Radiography is nonspecific, but high resolution computed tomography reveals the diagnostic findings of fibrotic changes referred to as "honeycombing" (small cystic spaces enveloped by thick, fibrotic, well-defined walls, mainly occurring in the periphery and bases). Bronchoalveolar lavage (BAL) is an important diagnostic test, however, it is mainly used to rule-out other pulmonary disease such as malignancy and infections. In IPF, the tissue obtained from lung washings will show a histological appearance of usual interstitial pneumonia, the classic pathological description of IPF.
Which of the following physiologic responses would occur after application of noninvasive positive pressure ventilation in a patient presenting with an acute exacerbation of chronic obstructive pulmonary disease? Increased afterload Increased alveolar dead space Increased tidal volumes Increased venous return
Correct Answer ( C ) Explanation: Noninvasive positive pressure ventilation applies a consistently positive airway pressure to increase laminar flow. This leads to airway stenting, elimination of dead space through alveolar recruitment, and an increase in tidal volumes and minute ventilation. The beneficial effects of positive pressure ventilation are not only realized in the pulmonary system but also in the cardiovascular system. Patients with pulmonary edema from decompensated heart failure benefit from the increased intrathoracic pressure which decreases venous return and increases left heart output and thus decreases afterload. Increased afterload (A) is incorrect because the increased intrathoracic pressure increases left heart output which decreases afterload. Increased alveolar dead space (B) is incorrect because airway stenting recruits alveoli and decreases the alveolar dead space. Increased venous return (D) is incorrect because the increased intrathoracic pressure actually decreases the venous return.
A transudative pleural effusion is identified after thoracentesis. Which of the following clinical scenarios is most consistent with this type of effusion? 27-year-old woman with a lupus flare 47-year-old woman alcoholic with an elevated lipase 65-year-old man with an ejection fraction of 15% and pulmonary edema 72-year-old man recently diagnosed with lung cancer
Correct Answer ( C ) Explanation: Pleural effusions are broken into two main classes: transudative and exudative. Transudative effusions develop when there is increased hydrostatic pressure or decreased oncotic pressure causing the translocation of fluid. Approximately 90% of transudative effusions are caused by congestive heart failure as in the clinical scenario described Exudative effusions result from an increase in membrane permeability and can be seen in pancreatitis (B), lupus serositis (A) and malignancy (D). The most common cause of exudative effusion is pneumonia when the adjacent pleural tissue becomes inflamed. Malignant effusions are the second most common cause.
You are treating a 50-year-old coal-miner's hypertension with lisinopril. He has been complaining of 3-months of progressive dyspnea. You order a chest radiograph which shows bilateral upper lobe honeycombing. A high resolution computed tomogram shows multiple small, round opacities only in the upper lobes. The lower lobes appear normal. Which of the following is the most likely diagnosis? Goodpasture's syndrome Granulomatosis with polyangiitis (GPA) Pneumoconiosis Scleroderma
Correct Answer ( C ) Explanation: Pneumoconiosis is an occupational respiratory disease due to inhalation of inorganic dusts. It is common in miners. In 2010, it resulted in 125,000 deaths in the US. History is key in determining the probable agent. A common form is Coalworker's pneumoconiosis, also known as miner's lung or black lung, and which is due to chronic inhalation of coal and carbon particles. Machinists, especially grinders, and pottery makers, can develop the silicon particle form called silicosis. Construction workers, especially boat builders, can suffer from asbestosis. Clinically, the diagnosis is made in a patient with dyspnea and the classic occupational exposures as above. Radiography may reveal small cystic radiolucencies described as honeycombing. High-resolution computed tomography offers better evaluation, and typically shows small, round opacities which denote inflammatory areas of dust-laden macrophages and fibrosis, as is quite common in the carbon deposits of coalworker's pneumoconiosis. Interestingly, the upper lobes are more affected in miner's lung and silicosis, while the lower lobes are more affected in asbestosis. Progressive pulmonary fibrosis, with resultant restrictive lung disease pathology and pulmonary function test results, typically occurs. Other than removing oneself from the offending agent and dusty environment, treatment is mainly supportive Goodpasture's syndrome (A) is an interstitial lung disease characterized by alveolar hemorrhage and glomerulonephritis. Scleroderma (D) is a collagen vascular disease whose pulmonary findings include lower lobe fibrosis and pulmonary hypertension. Upper lobe pathology is uncommon. Granulomatosis with polyangiitis (GPA) (B) is a vasculitic interstitial lung disease whose imaging reveals necrotizing granulomas
A 33-year-old man presents with five days of gradual onset nonproductive cough, fatigue, and fever. He also notes a 15 pound weight loss over the last month. He is tachypneic with a heart rate of 105 beats/minute, temperature of 38.2oC, and an oxygen saturation of 89% on room air. On examination, white plaques are noted on his tongue and his lungs are clear on auscultation. His chest X-ray is shown above. Which of the following is the most likely causative agent? Influenza A Klebsiella pneumoniae Pneumocystis jiroveci Streptococcus pneumoniae
Correct Answer ( C ) Explanation: Pneumocystis jiroveci (formerly carinii) pneumonia (PCP) is an opportunistic fungal pathogen which primarily affects immunocompromised patients including those with HIV/AIDS, as well as cancer and organ transplant patients on immunosuppressants. PCP is one of the most common AIDS-defining opportunistic infections. Patients present with gradual onset of nonproductive cough, fever, dyspnea, and decreased exercise tolerance. Patients with previously undiagnosed HIV/AIDS may also present with symptoms, such as weight loss and oral candidiasis. Lung auscultation is often normal although rales or rhonchi may be present. Chest X-ray shows bilateral diffuse interstitial perihilar infiltrates that extend in a "bat-wing" pattern, although atypical findings, such as a normal chest X-ray or apical infiltrates can be seen. Spontaneous pneumothorax is also seen in about 10% of patients. Arterial blood gases are often abnormal with decreased pO2 and respiratory alkalosis. LDH is increased, with the degree of elevation used as a marker for a worse prognosis. In HIV patients, CD4 counts are typically < 200 cells/mm3. Intravenous trimethoprim-sulfamethoxazole is the antibiotic of choice. Steroids are indicated in patients with moderate to severe disease as indicated by pO2 < 70 mmHg. They have been found to decrease mortality and accelerate recovery.
A 64-year-old man presents with a cough and shortness of breath for 2 weeks. His chest X-ray is shown above. Which of the following tests would indicate an infectious process as the cause of the above finding? Pleural fluid to serum LDH ratio < 0.6 Pleural fluid to serum protein ratio < 0.1 Pleural fluid to serum protein ratio > 0.5 Pleural fluid WBC 5,000 cells/mm3
Correct Answer ( C ) Explanation: The patient presents with a large pleural effusion that may be secondary to a pneumonia and a pleural fluid:serum protein ratio > 0.5 indicates the presence of an exudative effusion consistent with an infectious cause. Pleural effusions can result from a number of different pathologies. This includes CHF, malignancy, bacterial pneumonia, pulmonary embolism and pancreatitis. In developing countries, tuberculosis infection is the most common cause of pleural effusions. Pleural effusions can either be transudative (CHF, cirrhosis, pulmonary embolism) or exudative (malignancy, pneumonia, pulmonary embolism, pancreatitis). Patients will often present with dyspnea and pleuritic chest pain and dullness to percussion over the effusion. It is often difficult clinically to determine the cause of the effusion but testing of a sample of the fluid can differentiate exudative from transudative processes. Light's criteria (see table) is typically used for this purpose. If a single criterion is consistent with an exudative process, the fluid is exudative in origin.
Which of the following is the most common opportunistic respiratory infection in patients with acquired immunodeficiency syndrome? Cytomegalovirus Mycobacterium tuberculosis Pneumocystis jiroveci Streptococcus pneumoniae
Correct Answer ( C ) Explanation: Pneumocystis jiroveci is the most common opportunistic infection in patients with acquired immunodeficiency syndrome (AIDS). Pneumocystis jiroveci pneumonia (PCP) typically occurs in human immunodeficiency virus (HIV)-positive patients with a CD4 < 200 cells/microL who are not on potent antiretroviral therapy or PCP prophylaxis. PCP can also be seen in patients with cancer, stem cell or solid organ transplantation, primary immunodeficiencies, or severe malnutrition. P. jiroveci is primarily transmitted via the airborne route. The clinical manifestations of PCP include fever, nonproductive cough, and dyspnea. Fever and tachypnea are the most common physical exam findings. Rhonchi and crackles may be heard on chest exam. The most common abnormalities seen on chest radiograph are diffuse, bilateral, interstitial, or alveolar infiltrates. The gold standard for diagnosing PCP is immunofluorescent staining using fluorescein-labeled monoclonal antibodies . Empiric therapy should be initiated in patients who are acutely ill and in whom there is a high clinical suspicion. Trimethoprim-sulfamethoxazole (TMP-SMX) is the mainstay treatment for PCP. Patients with mild to moderate disease, based on respiratory status, can be managed with oral TMP-SMX. Patients with severe disease should receive intravenous therapy. Trimethoprim-dapsone is an alternative treatment for patients with a sulfa allergy. Adjunctive corticosteroids are recommended in patients with moderate-to-severe disease. TMP-SMX is also the most effective agent for PCP prophylaxis.
Which of the following diagnoses should be considered when an unexplained isolated pleural effusion is found on chest radiograph? Bacterial pneumonia Congestive heart failure Pulmonary embolism Viral pleuritis
Correct Answer ( C ) Explanation: Pulmonary embolism is the most commonly overlooked condition in the workup of pleural effusion. Any patient with an unexplained pleural effusion should be evaluated for possible pulmonary embolism using CT or V/Q scanning. Pleural effusion resulting from pulmonary embolism usually occupies less than 33% of the hemithorax but is accompanied by a greater degree of dyspnea than would be expected. The classic radiographic appearance of a pleural effusion is blunting of the costophrenic angle on the upright chest radiograph. On the AP or PA projection, a volume of 250-500 mL of fluid is required before the effusion can be appreciated. A lesser amount may be visible on the lateral projection in the posterior costophrenic gutter
A young woman with dyspnea on exertion presents for evaluation. She is subsequently diagnosed with sarcoidosis. Which of the following would you expect to appear in her lymph node biopsy report? Acid-fast organisms Congo red stain birefringence Noncaseating granulomas Reed-Sternberg cells
Correct Answer ( C ) Explanation: Sarcoidosis is an idiopathic widespread inflammatory condition. It is distinguished by accumulation of T cells and mononuclear phagocytes in affected organs, along with noncaseating granulomas and disturbance in normal tissue architecture. The lung is the most commonly involved organ, with skin, eye, liver, and lymph node involvement also common. The disorder can be acute, subacute and self-limiting, or a chronic waxing and waning disease occurring over many years. In the U.S.A., African-Americans are affected 10x more than Caucasians. Patients may be asymptomatic or complain of fatigue, malaise, weight loss, and fever. Commonly, patients present with symptoms based on the involved organ. For the lungs, this may include exertional dyspnea; dry, nonproductive cough; and vague chest pain. Exam findings are usually normal but may sometimes include crackles. Lymphadenopathy may be present. Chest X-ray is classic for bilateral hilar lymphadenopathy. Diagnosis is made by fiberoptic bronchoscopy with transbronchial biopsy showing a sarcoid granuloma, which is a well-formed noncaseating epithelioid cell granuloma surrounded by a rim of fibroblasts and lymphocytes. Management includes systemic corticosteroids as first-line therapy with methotrexate as an alternative. Acid-fast organisms (A), such as Mycobacterium species, are present in pulmonary tissues of patients with tuberculosis, not pulmonary sarcoidosis. In the presence of amyloid fibrils, congo red stain will appear as green birefringence (B) under polarized light. This is a diagnostic test for amyloidosis, not sarcoidosis. Reed-Sternberg cells (D) are found in Hodgkin lymphoma, not sarcoidosis.
Which of the following is the most common electrocardiogram finding in an acute pulmonary embolism? Right bundle branch block S wave in lead I, Q wave in III, inverted T wave in III (S1Q3T3) Sinus tachycardia T wave inversions in leads V1-V4
Correct Answer ( C ) Explanation: Sinus tachycardia is the most common ECG abnormality seen in a pulmonary embolism. It is seen in almost half of all cases. A pulmonary embolism most commonly occurs when a deep venous thrombosis (DVT) embolizes from the lower extremities and travels to the pulmonary vasculature. This leads to a ventilation-perfusion mismatch in the lungs resulting in hypoxia. Patients with a pulmonary embolism present with dyspnea, pleuritic chest pain, and tachycardia. Risk factors include malignancy, immobilization, recent surgery, and genetic hypercoagulable states. In cases with a large clot burden, acute pulmonary hypertension occurs and a right heart strain pattern will be seen on ECG. A massive pulmonary embolism is defined as a large clot burden causing obstructive shock. Treatment includes systemic thrombolytics, catheter-directed thrombolytics, and rarely embolectomy.
Which of the following is the most common location of aspirated foreign bodies in children? Left lower bronchus Left main bronchus Right main bronchus Trachea
Correct Answer ( C ) Explanation: The most common location of aspirated foreign bodies in children is the right main bronchus, followed by the left main bronchus, and the trachea. Foreign body aspiration (FBA) into the trachea or bronchi is a potentially life-threatening event because it can obstruct the airway and prevent oxygenation and ventilation. In children younger than two years old, FBA is a frequent cause of mortality and morbidity. The peak incidence of FBA is between 1-2 years of age. Peanuts, seeds, popcorn, and pieces of toys are some of the most commonly aspirated objects by infants and toddlers. Older children are more likely to aspirate non-food items. Round, solid objects (e.g. grapes and hotdogs) are the most dangerous aspirated objects because they can completely occlude the airway. Signs and symptoms depend on the degree of foreign body obstruction. Children who present with severe respiratory distress, stridor, aphonia, and loss of consciousness represent a true medical emergency and require urgent recognition. However, less emergent situation are more common and present with wheezing, cough, and diminished breath sounds. In children or infants who present with complete airway obstruction, dislodgement using back blows or the Heimlich maneuver should be attempted. Oxygen administration and other life support should be provided until rigid bronchoscopy can be performed. Blind sweeps of the mouth should be avoided due to the risk of pushing an object further down the throat. In less emergent situations, a plain chest radiograph may be useful depending on if the object is opaque and the degree of airway obstruction. Normal findings on radiography cannot be used to rule out FBA. All cases of suspected FBA should undergo examination of the tracheobronchial tree with a rigid bronchoscopy. Retained aspirated foreign bodies may lead to bronchiectasis.
What is the most common cause of croup? Adenovirus Haemophilus influenzae type b Parainfluenza virus Streptococcus spp.
Correct Answer ( C ) Explanation: The parainfluenza virus is the most common cause of croup. Croup is classically associated with a barking, seal-like cough and inspiratory stridor. Radiographs may show subglottic narrowing ("steeple sign") caused by edema. The typical age group is 6 months to 3 years, but the condition can be seen in children up to 5 years. The infection and inflammation are usually self-limiting, and conservative management is recommended. Evidence supports the routine use of corticosteroids in most children with croup. Intervention at an earlier phase of the illness reduces the severity of symptoms and the rates of return to a health care practitioner for additional medical attention, ED visits, and hospital admissions. Many children respond to a single, oral dose of dexamethasone. For those who do not tolerate the oral preparation, nebulized budesonide or intramuscular dexamethasone are reasonable alternatives. Mild disease can be treated with humidified oxygen. Moderate to severe disease should be treated with steroids and nebulized racemic epinephrine Adenovirus (A) is a common cause of URIs, but less common than parainfluenza virus as a cause for croup. Haemophilus influenzae type b (B) is implicated in various infections such as otitis media and epiglottitis, but less common as a cause of croup. Streptococcus spp. (D) infection is not a common cause of croup.
A 35-year-old woman comes to the clinic complaining of difficulty seeing, blurred vision, eye pain, and cough. She describes the cough as being dry and nonproductive. She has no past medical history and takes no medications. Ophthalmologic examination shows uveitis. Chest X-ray reveals bilateral hilar adenopathy. Which of the following laboratory findings would also most likely be associated with this patient's condition? Elevated C-reactive protein Elevated ESR Elevated serum angiotensin converting enzyme levels Increased sweat chlorid
Correct Answer ( C ) Explanation: Uveitis, bilateral hilar adenopathy, and a dry cough most likely indicate sarcoidosis. Sarcoidosis is granulomatous disease that can form nodules in multiple organs. Patients with sarcoidosis typically present with fatigue, weight loss, arthritis, dry eyes, blurry vision, and respiratory symptoms (eg, cough, dyspnea). Management usually involves the use of corticosteroids (eg, prednisone). Elevated levels of serum angiotensin converting enzyme (ACE) are typically elevated in about 75% of patients with sarcoidosis. However, elevated levels of ACE can also be seen in other conditions, such as, asbestosis, diabetes, and lung cancer. Therefore, it is not routinely used as a diagnostic test.
A 33-year-old woman comes to the clinic complaining of blurred vision, cough, and fatigue. Her blood pressure is 135/90 mm Hg and temperature is 38.6°C (101.5°F). She has no past medical history. Physical examination shows several subcutaneous nodules on both lower extremities. Ophthalmologic examination shows uveitis. Chest X-ray shows bilateral hilar adenopathy and reticular opacities. Laboratory studies show leukopenia, eosinophilia, and an elevated erythrocyte sedimentation rate. Which of the following is the most likely diagnosis? Asbestosis Cystic fibrosis Sarcoidosis Tuberculosis
Correct Answer ( C ) Explanation: Uveitis, bilateral hilar adenopathy, dry cough, subcutaneous nodules (eg, erythema nodosum), and an elevated erythrocyte sedimentation rate (ESR) most likely suggest sarcoidosis. Sarcoidosis is granulomatous disease that can form nodules in multiple organs (eg, skin, lungs). Patients typically present with fatigue, weight loss, arthritis, dry eyes, blurry vision, and respiratory symptoms (eg, cough, dyspnea). Management usually involves the use of corticosteroids (eg, prednisone). Bilateral hilar adenopathy, seen on chest x-ray, is a classic finding associated with sarcoidosis.
A 32-year-old woman presents to your office with a complaint of productive cough that started 6 days ago. She reports occasional wheezing, chest wall tenderness, and has been afebrile since the onset of symptoms. Her husband had similar symptoms and was prescribed azithromycin by his primary care provider. Which of the following is the most appropriate next step in management? Order a chest X-ray Prescribe a course of azithromycin Send a sputum sample for culture Symptomatic treatment
Correct Answer ( D ) Explanation: Acute bronchitis is an inflammation of the bronchi caused by upper respiratory infection. It is a self-limiting condition that is most commonly of viral etiology. Patients with acute bronchitis generally have few systemic symptoms. The most common clinical presentation is a cough lasting up to 10-20 days. The cough may be productive and patients often mistakenly attribute purulent sputum in acute bronchitis with a bacterial infection. The symptoms of acute bronchitis are less severe than those of influenza or pneumonia and help determine diagnosis and course of treatment. Symptomatic treatment with acetaminophen or NSAIDs and other supportive care measures are recommended as initial management of patients with acute bronchitis.
Which of the following organisms is most associated with pneumonia and bullous myringitis? Bordetella pertussis Haemophilus influenzae Streptococcus bovis Streptococcus pneumoniae
Correct Answer ( D ) Explanation: Although bullous myringitis is described as a classic finding in M. pneumoniae infections, it is not specific for mycoplasmal infection and is present in only a few cases. S. pneumoniae infection is most likely the cause of bullous myringitis in the setting of pneumonia. S. pneumoniae is a common cause of community acquired pneumonia. Bullous myringitis is an inflammation of the eardrum in which painful, fluid filled vesicles form. Patients with S. pneumoniae will present with typical signs of pneumonia. The presence of otalgia should prompt evaluation for bullous myringitis. Bullous myringitis will resolve with antibiotics directed at the S. pneumoniae pneumonia. Bullous Myringitis S. pneumoniae Fluid filled vesicles on TM Pneumonia + otalgia Bullous myringitis is not primarily associated with mycoplasma pneumoniae: Bullous myringitis is a presentation of acute otitis media in which bullae are seen on the tympanic membrane. Despite earlier belief that this condition was associated with mycoplasmal infection, more recent evidence indicates that the prevalence of viral, bacterial, or mycoplasmal infection is the same in bullous myringitis as in nonbullous otitis media.
A new nursing home employee is being screened for tuberculosis and develops a 6 mm induration on his right volar forearm after injection of a purified protein derivative (PPD). This reading indicates prior tuberculosis exposure if he has had which of the following life circumstances? He is a former guard at a correctional facility He is a HIV-negative injection drug user He is a recent immigrant from Latin America He is an organ recipient on daily immunosuppressives
Correct Answer ( D ) Explanation: An organ recipient on daily immunosuppression therapy is among the group of people who would be considered to have a positive purified protein derivative tuberculin skin test, or PPD, at a reading of 5 mm or greater of induration. Other patients considered to have a positive PPD at 5 mm would include those with recent contact to people with active tuberculosis, those with chest radiographs suggestive of infection, and any HIV-positive patients. Patients who recently emigrated from countries with a high tuberculosis prevalence, are injection drug users, or are residents or employees of high-congregate areas are considered to have a positive PPD reading at 10 mm. Many immunocompromising conditions also allow for a positive reading at 10 mm. Otherwise healthy adults without significant risk of prior exposure would have a positive PPD read at 15 mm of induration. The tuberculin skin test, in which purified protein derivative is placed under the skin of the volar forearm, is the most common method used in screening at-risk people for tuberculosis exposure. This test has high sensitivity and specificity. However, patients who were previously vaccinated with bacillus Calmette-Géurin may falsely test positive. In these patients, an interferon gamma release assay should replace the PPD for tuberculosis screening. All suspected cases of tuberculosis should be reported to government health authorities. A sputum culture should be ordered to determine drug sensitivities of the isolate. Beyond routine screenings, clinicians should be alert for signs and symptoms of pulmonary tuberculosis in patients. These include persistent cough, anorexia, weight loss, fever, night sweats, and blood-streaked sputum production. If tuberculosis is diagnosed, patients must complete treatment with a 6-9 month course of an appropriate anti-tuberculosis drug regimen to prevent the spread of illness. A guard at a correctional facility (A), an HIV-negative injection drug user (B), and a recent immigrant from Latin America (C) would all need a PPD reading of 10 mm of induration or greater for there to be concern of a previous exposure to tuberculosis.
16-year-old boy is taken to his doctor for snoring. His mother reports that his snoring keeps others in the house awake and that sometimes his breathing pauses during sleep with gasping or choking. His teachers report that he falls asleep frequently at school. On exam, he has a body mass index of 31 kg/m² and has enlarged tonsils. His symptoms are concerning for obstructive sleep apnea so the pediatrician refers him for an overnight polysomnography. Which of the following can be a longterm complication of obstructive sleep apnea? Cerebrovascular accident Lung scarring Nasal polyps Pulmonary hypertension
Correct Answer ( D ) Explanation: Children with obstructive sleep apnea (OSA) often are obese, have enlarged tonsils or have significant allergies. OSA can lead to abnormal growth and development, bedwetting, behavioral and learning problems, daytime sleepiness, and hyperactivity. Long-standing repetitive oxygen desaturations and hypercapnia episodes during sleep can lead to vascular remodeling and pulmonary hypertension which can lead to cor pulmonale (right ventricular hypertrophy). Treatment may include weight loss, managing allergic rhinitis, and removal of the adenoids and tonsils. Nonsurgical approaches to treatment include weight loss, CPAP, and bite guards that bring the lower jaw forward. Surgical treatments include uvulopalatopharyngoplasty, jaw surgery, or removal of the tonsils and adenoids.
A 58-year-old man presents to your office with complaints of chronic cough and difficulty breathing. He has smoked approximately 2 packs of cigarettes per day since he was 14 years old and has not seen a doctor in many years. You order spirometry testing, which shows airflow limitation (predicted FEV1 45%). Which of the following is the most appropriate therapy? Azithromycin Guaifenesin Prednisone Tiotropium
Correct Answer ( D ) Explanation: Chronic obstructive pulmonary disease (COPD) is a progressive condition characterized by airflow limitation that causes an enhanced inflammatory response in the airways and lungs. Historically COPD was divided into three subtypes: chronic bronchitis, emphysema and asthma. Asthma is no longer grouped with COPD and current definitions of COPD do not differentiate between emphysema and chronic bronchitis. Patients generally have a lengthy smoking history and are in their fifth decade of life. They often have a chronic cough with sputum production. Diagnosis is by pulmonary function tests including spirometry. Treatment is based on the severity of the disease. Long-acting anticholinergics such as tiotropium are the mainstay of therapy.
A 4-year-old patient is brought to the emergency department by his parents after a choking incident that occurred one hour ago. The parents tell you that their son was eating steak, started to laugh and then began choking. Since the incident, he has been coughing and wheezing. Physical exam reveals unilateral diminished breath sounds with auscultation of the lungs. Which of the following is the most appropriate next step in management? Begin a course of antibiotics and steroids Chest X-ray Flexible bronchoscopy Rigid bronchoscopy
Correct Answer ( D ) Explanation: Foreign body aspiration (FBA) can be a life-threatening event and is a common cause of morbidity and mortality in children. The majority of pediatric FBA incidents occur in children less than 3 years of age. Types of foreign bodies that are commonly aspirated by children include peanuts, popcorn, other nuts and seeds, food particles and pieces of toys. FBA is a true medical emergency if there is a complete airway obstruction. Children with cyanosis, altered mental status and severe respiratory distress require immediate medical intervention including life support and rigid bronchoscopy to remove the foreign body. All cases of suspected FBA require that the tracheobronchial tree be examined. Rigid bronchoscopy is used for this examination and is both a diagnostic and management tool for partial and complete obstructions caused by FBA. Chest X-ray (B) may detect FBA if the aspirated object is radioopaque or depending on the severity of the obstruction. Normal findings on a chest X-ray do not rule out FBA since foods are not radioopaque and are commonly aspirated by pediatric patients. Flexible bronchoscopy (C) is often used in children with pneumonia or other respiratory symptoms. Use of flexible bronchoscopy for diagnosis of FBA may dislodge the object and cause a partial obstruction to become complete.
A 68-year-old woman presents with abrupt onset of fever, malaise, body aches, headache, and sore throat. Rapid strep test is negative and nasal smear test is positive for influenza B. Which of the following is the most appropriate therapy? Acyclovir Amantadine Azithromycin Oseltamivir
Correct Answer ( D ) Explanation: Oseltamivir is a neuraminidase inhibitor that is effective against influenza A and B. Influenza is an acute respiratory viral illness, which occurs mainly during the winter in outbreaks and pandemics worldwide. Although it is acutely debilitating, influenza is usually a self-limiting disease. However, it has increased morbidity and mortality in older and very young populations, as well as those with respiratory complications. Transmission of the virus is through respiratory secretions (coughing and sneezing). Symptoms generally include rapid onset of fever and myalgias, often accompanied by cough, sore throat, chills, and headache. Early use of antiviral medication (less than 2 days after onset) is recommended by the CDC for five days in high-risk patients, which results in reduced mortality and morbidity. Acyclovir (A) is used to treat viral infections due to certain herpes viruses, including common cold sores and shingles. Amantadine (B) is effective against influenza A, but ineffective against influenza B.
A 72-year-old man is sent to the interventional radiology department by his primary care doctor to undergo a thoracentesis after a chest radiograph revealed a moderate left-sided pleural effusion. Analysis reveals a pleural fluid/serum protein ratio > 0.9 and a pleural fluid/serum LDH ratio > 0.8. Based on these findings, what is the most likely diagnosis? Cirrhosis with ascites Congestive heart failure Hypoalbuminemia Malignancy Nephrotic syndrome
Correct Answer ( D ) Explanation: Pleural effusions result from the accumulation of fluid between the visceral and parietal pleura. They form as a result of two different processes. Transudative effusions contain little to no protein and result from an increase in hydrostatic pressure or a decrease in oncotic pressure of the pulmonary microvasculature. In contrast, exudative effusions are protein rich. They occur due to increased permeability of the pleura or impaired lymphatic drainage. Light's criteria are used to distinguish transudative from exudative effusions. Out of the answer choices above, malignancy is the only one associated with an exudative effusion. Congestive heart failure (B) is the most common cause of transudative effusions and results from increased hydrostatic pressure. Cirrhosis of the liver (A) and nephrotic syndrome (E) result in transudates due to combined increases in hydrostatic pressure and decreases in oncotic pressure. Hypoalbuminemia (C) can also lead to a transudative effusion due to a significant decrease in oncotic pressure.
A patient with significant dyspnea presents for evaluation. You order spirometric testing and obtain results consistent with restrictive pulmonary disease. Which of the following findings is most consistent with restrictive pulmonary disease? Alveolar destruction Bronchoconstriction Excess mucus production Parenchymal abnormalities
Correct Answer ( D ) Explanation: Pulmonary diseases can be classified as obstructive and restrictive. The obstructive conditions, asthma, emphysema, chronic bronchitis, bronchiolitis and bronchiectasis, exist in a state of increased airway resistance, increased compliance and decreased parenchymal recoil. These conditions result in a normal or slightly increased forced vital capacity (FVC), and a decreased forced expiratory volume in the first second of expiration (FEV1) to FVC ratio. On the other hand, restrictive conditions result in decreased airway compliance and increased recoil. This leads to a decrease in the FVC and FEV1 but a near normal FEV1/FVC ratio. The restrictive pulmonary conditions include the interstitial lung diseases (ILD) (parenchymal abnormalities) and the chest wall disorders. Interstitial lung disease includes idiopathic interstitial pneumonia (mostly idiopathic pulmonary fibrosis), drug and radiation induced (iatrogenic) pulmonary fibrosis, hypersensitivity pneumonitis, collagen vascular disease, pneumoconiosis and sarcoidosis. Chest wall restrictive lung disease includes pleural disease, severe obesity and some neuromuscular disorders, such as Guillain-Barre syndrome
Which of the following is a risk factor for pulmonary embolism? Chronic obstructive pulmonary disease Cirrhosis Low body mass index Recent abdominal surgery
Correct Answer ( D ) Explanation: Pulmonary embolism (PE) is the obstruction of pulmonary venous vasculature by material that originated elsewhere in the body, most commonly a thrombus. Other source material for pulmonary embolism includes air, fat from long bone fractures, and tumor material from malignancy. The most common source of thromboembolic disease to the lungs is the proximal deep veins of the lower extremities (eg. iliac, femoral, and popliteal veins). Pulmonary embolism has a large spectrum of clinical presentations, ranging from asymptomatic to hemodynamic instability with overt obstructive shock and death. Symptoms of pulmonary embolism may present acutely, subacutely, or even chronically and most commonly include dyspnea with pleuritic chest pain, cough, and hemoptysis. Patients may also have symptoms of deep venous thrombosis, including unilateral leg swelling, tenderness, and palpable cords. Risk factors for PE are similar to those for venous thromboembolic disease and include recent surgical or accidental trauma, immobilization, malignant neoplasm, pregnancy or use of estrogen products, clotting disorders, and central venous catheter or pacemaker placement. Patients with low risk for PE based on risk factors may be screened with a D-dimer level, which is extremely sensitive but nonspecific for pulmonary embolism. Hemodynamically stable patients with higher risk should undergo definitive imaging with computed tomographic pulmonary angiography or, if unable to tolerate contrast dye due to poor renal function, ventilation perfusion scanning. Management depends on the hemodynamic stability of the patient. The cornerstone of therapy in stable patients is immediate and long term anticoagulation. Hemodynamically unstable patients require fibrinolysis or embolectomy.
An obese 34-year-old woman is brought to the Emergency Department with respiratory distress. Two months ago she was in the hospital for knee surgery. Paramedics report an acute onset of dyspnea and pleuritic chest pain. She also complains of a tender thigh on the same side of her knee surgery. She is tachycardic and tachypneic, and mildly hypotensive. Examination reveals decreased breath sounds but no hyperresonance. An emergent chest radiograph is relatively normal except for some mild atelectasis. Which of the following is the most likely diagnosis? Acute bronchitis Pleural effusion Pneumothorax Pulmonary embolism
Correct Answer ( D ) Explanation: Pulmonary embolism refers to the obstruction of a pulmonary artery by thrombus, tumor, air or fat that originated elsewhere in the body, mostly from the deep veins of the lower extremities. It is classified as acute or chronic, and massive or submassive. It is often a fatal disease, leading to a mortality rate of 30% without treatment. Risk factors include immobilization, surgery or central venous instrumentation within the last three months, stroke/paresis/paralysis, cancer, chronic cardiac disease, autoimmune disease, obesity, >1 pack per day tobacco use, hypertension and a history of deep vein thrombosis. Most patients experience dyspnea with or without wheezing, cough, pleuritic chest pain, orthopnea, lower extremity pain or swelling, tachypnea, tachycardia, jugular venous distension, decreased breath sounds and an accentuated pulmonic component of S2. These signs and symptoms are variable and nonspecific. Furthermore, up to 32% of patients present asymptomatically. As such, the diagnosis can be difficult. The mainstay of treatment is anticoagulation.
A 38-year-old man presents with fever, fatigue, cough, and increasing dyspnea. The patient denies chills or night sweats. On physical exam, lungs are clear to auscultation bilaterally. Hepatosplenomegaly is noted on abdominal exam. Chest X-ray findings include bilateral hilar adenopathy and diffuse reticular infiltrates. Labs reveal leukocytopenia, hypercalcemia, and elevated erythrocyte sedimentation rate. What is the most likely diagnosis in this patient? Histoplasmosis Mycoplasma pneumoniae Pulmonary tuberculosis Sarcoidosis
Correct Answer ( D ) Explanation: Sarcoidosis is a multi-organ disease, usually of idiopathic origin. It is more common in the African American population and the European caucasian population. Patients commonly present with respiratory symptoms, including cough, dyspnea, and chest discomfort. Patients may also have malaise, fever, and multiple non-pulmonary symptoms. Sarcoidosis is characterized by noncaseating granulomatous inflammation in multiple affected organs. Histoplasmosis (A) patients are often asymptomatic. Chest X-ray shows a chronic fibrocavitary pneumonia. Mycoplasma pneumoniae (B) is associated with a gradual increase in constitutional symptoms and non-productive cough. Chest X-ray shows patchy infiltrates. Pulmonary tuberculosis (C) is associated with cough, night sweats and chills. Chest X-ray shows apical cavitation.
A 39-year-old stonemason presents with dyspnea that has been present for several months. A chest X-ray shows an eggshell calcification of the hilar lymph nodes and multiple small 1-2 mm nodules throughout the lung. Which of the following is the most likely diagnosis? Asbestosis Byssinosis Siderosis Silicosis
Correct Answer ( D ) Explanation: Silicosis is one of many forms of pneumoconiosis, a chronic fibrotic lung disease caused by the inhalation of inorganic materials. Silicosis is due to the prolonged inhalation of silica (silicon dioxide) particles. It is most likely in sandblasting, rock mining, and stone cutting. Silicosis is usually asymptomatic, but prolonged-exposure cases result in dyspnea and obstructive, restrictive pulmonary dysfunction. Small, round nodules are formed through the lung. Chest radiography showing calcification of the hilar lymph nodes ("eggshell" calcification) is highly suggestive of silicosis. Treatment is supportive and smoking cessation is essential. Corticosteroids can be used to relieve any chronic alveolitis. Patients with silicosis are at increased risk of pulmonary tuberculosis. If active or inactive tuberculosis is suspected, multi-drug treatment should be initiated. Byssinosis (B) is an asthma-like disorder caused by the inhalation of cotton dust in textile workers. Siderosis (C) is a form of pneumoconiosis caused by inhalation of metallic iron or iron oxide, most likely in the mining, welding, and foundry occupations.
Which of the following can cause an exudative pleural effusion? Cirrhosis Congestive heart failure Nephrotic syndrome Systemic lupus erythematosus
Correct Answer ( D ) Explanation: Systemic lupus erythematosus can cause an exudative pleural effusion. A pleural effusion is an accumulation of fluid in the pleural space and can either be exudative or transudative. Exudates result primarily from pleural and lung inflammation which result in increased capillary and pleural membrane permeability. It can also be caused by impaired lymphatic drainage of the pleural space. Disease in virtually any organ can cause exudative pleural effusions by a variety of mechanisms, including infection, malignancy, immunologic responses, lymphatic abnormalities, noninfectious inflammation, iatrogenic causes, and movement of fluid from below the diaphragm. Some examples include pneumonia, tuberculosis, pancreatitis, connective tissue disorders such as lupus or rheumatoid arthritis, and malignancy, more commonly breast, lung, lymphoma or leukemia.
Which of the following is most appropriate in the treatment plan for a patient with idiopathic pulmonary fibrosis? Albuterol Azathioprine Bosentan Pulmonary rehabilitation
Correct Answer ( D ) Explanation: The goal in the treatment of idiopathic pulmonary fibrosis is to reduce symptoms, prevent acute exacerbations, vaccinate, slow progression and increase survival. There are several treatment options, however, most medication trials have shown no benefit. Sufficient clinical evidence that any treatment improves survival or quality of life for patients with idiopathic pulmonary fibrosis (IPF) is lacking. Supportive care, supplemental oxygen and pulmonary rehabilitation appear to be the most beneficial options. Lung transplantation may be an option in those with disease progression and minimal comorbidities. IPF prognosis is poor. Only 20-30% of patients survive five years after diagnosis.
A 25-year-old man presents for evaluation of fever and cough. He reports last week that he was diagnosed with influenza. In the last 2 days he developed a worsening cough productive of large amounts of sputum. Vital signs are T 101°F, HR 98, BP 120/60, RR 18, and 95% oxygen saturation on room air. His chest X-ray demonstrates a lobar infiltrate in the left lower lobe. Which of the following would you most likely expect to see on the patient's Gram stain? Gram negative bacilli Gram negative diplococci Gram positive bacilli Gram positive cocci in clusters
Correct Answer ( D ) Explanation: The patient had a recent influenza infection and now presents with a lobar infiltrate. Staphylococcus aureus pneumonia is classically associated with causing post-influenza bacterial pneumonia. On Gram stain this is seen as Gram positive cocci in clusters
A 67-year-old woman presents with shortness of breath and a cough for 3 days. The patient admits to recently having a fever, chills, rhinorrhea, and myalgias for 10 days. She had one episode of diarrhea and has been nauseated. Her past medical history is positive for COPD. She does not smoke but she drinks two cans of beer daily. In the ED, her vital signs are BP 120/76, HR 108, RR 20, oxygen saturation 97% on room air, and temperature 101.2°F. A chest X-ray shows a left lower lobe consolidation. It is suspected that this patient has pneumonia caused by Staphylococcus aureus. Which aspect of this patient's history supports this suspicion? Alcohol consumption Gastrointestinal symptoms History of COPD Recent viral symptoms
Correct Answer ( D ) Explanation: The patient's recent history of fever, chills, rhinorrhea, and myalgias raises suspicion for influenza. Staphylococcus aureus is a common pathogen causing pneumonia in those currently or recently infected with influenza virus. Patients with pneumonia often present with tachypnea, tachycardia, dyspnea, chest pain, cough, fevers, and rales. S. aureus, a common nosocomial pathogen in adults, can lead to a necrotizing pneumonia. In those with gastrointestinal (B) or neurologic symptoms, Legionella pneumophilia should be considered. In those with excessive alcohol consumption (A), a common pathogen is Klebsiella pneumoniae. A history of COPD (C) raises suspicion for Haemophilius influenzae and Pseudomonas aeruginosa.
A 2-month-old boy presents with a fever and cough. Which of the following is suggestive of Chlamydia pneumonia in this infant? Bullous myringitis Diarrhea Rusty-colored sputum Staccato cough
Correct Answer ( D ) Explanation: There are multiple causes of bacterial pneumonia and key characteristics of the patient's clinical picture may help to identify the causative organism. Chlamydia trachomatis pneumonia is associated with the characteristic staccato cough. Pneumonia due to C. trachomatis is recognized in most affected infants between 3 and 16 weeks of age, although essentially all are symptomatic before 8 weeks. Initial therapy for chlamydial pneumonia should be initiated on a presumptive diagnosis, based on clinical and radiographic findings, until diagnostic test results are available.Treatment is with a macrolide antibiotic. The American Academy of Pediatrics (AAP) Committee on Infectious Disease and the Centers for Disease Control and Prevention (CDC) recommend oral azithromycin (10 mg/kg on day one and then 5 mg/kg on day two-four) for either chlamydial pneumonia. For chlamydial conjunctivitis, erythromycin should be used. Both erythromycin and azithromycin are associated with increased risk of infantile hypertrophic pyloric stenosis, particularly in infants younger than two weeks.The infant's mother and her sexual partners should be evaluated and treated for C. trachomatis infection. They also should be evaluated for other sexually transmitted diseases. Bullous myringitis (A) are blisters (bullae) that are seen on the tympanic membrane during inspection of the middle ear. Bullous myringitis was previously linked to Mycoplasma pneumoniae but it appears, based on middle ear aspirate culture results, that typical acute otitis media pathogens (S. pneumoniae) are the most likely pathogens. Diarrhea (B) and gastrointestinal complaints are classically associated with Legionella pneumophila. Rusty-colored (C) or bloody sputum is associated with Streptococcus pneumoniae infection. Patients with Streptococcal infection are often acutely ill appearing and classically describe the onset of a shaking chill followed by fever and cough.
A full term male infant is delivered by cesarean section because of dystocia due to macrosomia. Apgar scores are 8 and 10. An hour after delivery he begins to have tachypnea without hypoxemia. A chest radiograph shows diffuse parenchymal infiltrates and fluid in the pulmonary fissures. The symptoms resolve without treatment within 24 hours. What is the most likely diagnosis? Laryngomalacia Meconium aspiration syndrome Respiratory distress syndrome Transient tachypnea of the newborn
Correct Answer ( D ) Explanation: This child had transient tachypnea of the newborn, the most common cause of neonatal respiratory distress. It is a benign condition due to residual pulmonary fluid remaining in the lungs after delivery. Signs of respiratory distress such as tachypnea, nasal flaring, grunting, retractions, hypoxia, and increased oxygen requirement become evident shortly after birth. The disorder is transient, with symptoms usually resolving within 72 hours after birth. Risk factors include cesarean delivery, macrosomia, male gender, and maternal asthma and diabetes mellitus. Chest radiography is the diagnostic standard for transient tachypnea of the newborn. The characteristic findings include prominent perihilar streaking, which correlates with the engorgement of the lymphatic system with retained lung fluid, and fluid in the fissures. Small pleural effusions may be seen. Patchy infiltrates have also been described. Medical care of transient tachypnea of the newborn is supportive. As the retained lung fluid is absorbed by the infant's lymphatic system, the pulmonary status improves.
A 42-year-old woman who spent two days hospitalized after she underwent an appendectomy three weeks ago presents with cough, green sputum and fever. Her vitals are T 100.7°F, HR 94, BP 123/76, RR 18, and oxygen saturation 97%. She is well appearing and her blood work (CBC and BMP) is unremarkable. A chest X-ray shows a left lower lobe infiltrate. Which of the following represents the best management for this patient? Discharge home with oral antibiotics and follow up Draw blood cultures and discharge home on oral antibiotics Order a chest CT scan Start IV antibiotics and admit
Correct Answer ( D ) Explanation: This patient has a health-care associated pneumonia (HCAP) requiring IV antibiotics and admission. HCAP is defined as infection occurring within 90 days of a 2-day or longer hospitalization; in a nursing home or long-term care residence; within 30 days of receiving intravenous antibacterial therapy, chemotherapy, or wound care or after a hospital or hemodialysis clinic visit. HCAP requires IV broad spectrum antibiotics because it may involve both the typical pathogens involved in community-acquired pneumonia (CAP) (Mycoplasma pneumonia, Haemophilus influenzae, Streptococcus pneumoniae and Chlamydia pneumoniae) as well as more resistant organisms (Acinetobacter species, Pseudomonas species, Staphylococcus aureus (including MRSA) Enterobacter species, Escherichia coli, Proteus species, Klebsiella species etc.). Treatment should be as follows (one antibiotic from each category):
A 29-year-old woman presents to the ED complaining of pain on inspiration. Over the previous 3 days, she has experienced a low-grade fever, sore throat, and body aches. Auscultation of her lungs reveals normal breath sounds. Her upright chest radiograph is seen above. Which of the following is the most appropriate next step in management? D-dimer Doxycycline Inspiratory and expiratory radiographic views Supportive care
Correct Answer ( D ) Explanation: This patient has pleuritic chest pain in the setting of a recent upper respiratory infection (URI). Viral pleuritis and pulmonary infarction are commonly associated with pleuritic chest pain. The patient's history often helps to establish the diagnosis of pleural inflammation. The pain of viral pleuritis usually is preceded by several days of a typical viral prodrome, with low-grade fever, sore throat, and other upper respiratory or constitutional symptoms. Management of viral pleuritis includes supportive care with NSAIDs. In the absence of these prodromal symptoms, an alternative etiology for pleuritis should be sought. A patient history of congestive heart failure, liver disease, uremia, or malignancy should direct further evaluation.
A 26-year-old man presents with a 2-week history of fever and a cough. He was diagnosed with HIV four months ago and is not on any antiretroviral medications. His vital signs are BP 122/76, HR 78, RR 16, oxygen saturation 92% on room air, and temperature 99.2°F. Chest X-ray demonstrates diffuse interstitial infiltrates bilaterally. Given this presentation, it is suspected that he has pneumonia secondary to Pneumocystitis jiroveci. Which of the following antibiotics is used to treat suspected pneumonia caused by Pneumocystitis jiroveci? Azithromycin Clindamycin Penicillin Trimethoprim-sulfamethoxazole
Correct Answer ( D ) Explanation: This patient has pneumonia secondary to Pneumocystitis jiroveci (PCP pneumonia). PCP generally occurs in immunocompromised patients, including HIV patients with a CD4 count less than 200 cells/mL. Once a patient's CD4 count is less than 200 cells/mL, daily prophylaxis should be initiated with trimethoprim-sulfamethoxazole to prevent PCP. Patients with PCP often present with a cough, unexplained fever, shortness of breath, and hypoxia out of proportion to the clinical picture. Chest X-ray classically shows diffuse interstitial infiltrates, but a negative chest X-ray can be seen in up to 20% of patients. Since the organism cannot be grown in the laboratory, diagnosis depends on other means. Most often, diagnosis is confirmed through bronchoscopy with a bronchoalvelolar lavage. A classic finding in patients with PCP is an elevated lactate dehydrogenase. Treatment, which should be initiated upon early suspicion and before diagnosis, is with trimethoprim-sulfamethoxazole. In addition, patients who are hypoxic with a PaO2 of less than 70 mm Hg (equivalent to an oxygen saturation of <~93%) or have an elevated alveolar-arterial gradient of greater than 35 mm Hg, should receive steroids prior to initiation of antibiotics. Azithromycin (A) is a macrolide antibiotic commonly prescribed for community-acquired pneumonia. Clindamycin (B) is generally given for aspiration pneumonia. Penicillin (C) is infrequently prescribed due to high resistance, but it is remains the first line treatment for syphilis.
A 35-year-old woman comes to the urgent care clinic complaining of a 3-week history of difficulty seeing, blurred vision, eye pain, and cough. She describes the cough as being dry and nonproductive. She has no past medical history and takes no medications. Her blood pressure is 140/85 mm Hg and her temperature is 38.1°C (100.5°F). Ophthalmologic examination shows uveitis. Chest X-ray shows bilateral hilar adenopathy. Which of the following additional findings would also most likely be found in this patient? Ferruginous bodies Increased sweat chloride Positive PPD skin test Subcutaneous nodules
Correct Answer ( D ) Explanation: Uveitis, bilateral hilar adenopathy, and a dry cough most likely indicate sarcoidosis. Sarcoidosis is granulomatous disease that can form nodules in multiple organs. When these nodules form in the skin, they are referred to as subcutaneous nodules (ie, erythema nodosum). Patients with sarcoidosis typically present with fatigue, weight loss, arthritis, dry eyes, blurry vision, and respiratory symptoms (eg, cough, dyspnea). Management usually involves the use of corticosteroids (eg, prednisone).
A 50-year-old man with a history of hypertension, diabetes and stage III chronic kidney disease with a GFR of 45, presents to the emergency department complaining of shortness of breath and stabbing chest pain for the past hour. Vital signs are BP 145/70, RR 36, HR 115, and pulse oximetry 89% on room air. An ECG reveals sinus tachycardia. Two days ago he returned to Los Angeles on a business trip from China. Which of the following is the most appropriate diagnostic test for this patient? AChest CT angiography BChest radiography CPulmonary angiography DVentilation/perfusion scan
Correct Answer ( D ) Explanation: Ventilation/perfusion scan is the most appropriate diagnostic imaging modality for this patient who likely has a pulmonary embolism. For most patients with suspected pulmonary embolism, chest CT angiography is the first-choice diagnostic imaging modality because it is sensitive and specific for the diagnosis. However, this patient has stage III chronic kidney disease and chest CT angiography is contraindicated. Ventilation/perfusion scanning is reserved for those with suspected pulmonary embolism in whom chest CT angiography is contraindicated such as renal insufficiency with an estimated GFR < 60, contrast allergy, pregnancy or morbid obesity. In general, ventilation/perfusion scanning is a sensitive test for the diagnosis of pulmonary embolism, but is poorly specific due to the high number of false-positive test results
*PNA* *Fungal* Opportunistic infection in AIDS. Dx/Tx?
Cryptococcosis; IV Amphotericin B + Oral flucytosine
What type of COPD is associated with hyperventilation flat diaphragm on CXR and a normal Hgb/HCT?
Emphysema
*COPD* Which type might have respiratory alkalosis and why?
Emphysema (pink puffers): blow off CO2
Question: What is the antibiotic of choice for psittacosis in children and pregnant women?
Erythromycin. Psittacosis Patient with a history of exposure to birds Complaining of high fevers, severe headache, myalgias, nonproductive cough PE will show hepatosplenomegaly CXR will show patchy perihilar or lower lobe infiltrates Most commonly caused by Chlamydia psittaci Treatment is doxycycline
A 46-year-old man presents with right eye pain and a change in his vision. He was recently diagnosed with tuberculosis and started on a 4-drug regimen. Based on his history and physical exam findings, you are concerned for optic neuritis. Which of the following medications is likely responsible for his symptoms? AEthambutolCorrect Answer BIsoniazid CPyrazinamide DRifampin
Ethambutol can cause optic neuritis resulting in decreased visual acuity and possible blindness. The medication should be discontinued immediately at the first sign of visual loss. Because of the difficulty in monitoring vision in children, ethambutol is not recommended in this population.
Health Maintenance/Pulmonology A 62 year-old female is admitted to a nursing home during an outbreak of influenza. In review of her records, you note that she did not receive the flu vaccine this year. Which of the following is the most appropriate drug of choice for influenza prophylaxis in this patient? Answers A. Ciprofloxin (Cipro) B. Zanamivir (Relenza) C. Clarithromycin (Biaxin) D. Alpha-2b interferon (Avonex)
Explanations (u) A. Ciprofloxin is indicated for postexposure prophylaxis of anthrax. (c) B. Either zanamivir or oseltamivir are indicated for prophylactic use against influenza A or B. (u) C. Clarithromycin is indicated for prophylaxis against disseminated Mycobacterium avium complex. (u) D. Alpha-2b interferon is indicated for treatment of several disorders, such as chronic hepatitis B & C, but has no role in prophylactic treatment of any condition.
27-year-old man with a history of asthma presents to your office for his annual exam. He reports that in the past month he has experienced wheezing and shortness of breath about once per week which resulted in using his rescue inhaler. He woke up once because of coughing. He has a peak flow meter and home readings have been 85-90% of his personal best. Which of the following best describes the classification of his asthma? Intermittent Mild persistent Moderate persistent Severe persistent
Explanation: Asthma severity is determined by monitoring patient symptoms over the past 2-4 weeks, determination of the patient's current lung function and number of asthma exacerbations that require oral glucocorticoids each year. A step-wise approach is used to classify asthma severity and determine approach to treatment. Intermittent asthma is defined as having two or fewer daytime symptoms per week, two or fewer nighttime awakenings due to symptoms in the past month, use of short-term beta agonist rescue inhaler less than two times per week, no impairment in activities of daily living (ADLs) between exacerbations, home readings of lung function within the normal range (greater than or equal to 80% of normal), and zero or one exacerbation requiring oral glucocorticoids in the past year.
Diagnostic Studies/Pulmonology An adult patient who is HIV positive receives a PPD. He develops an area of induration that measures 8 mm after 48 hours. Which of the following is the most appropriate interpretation of this test result? Answers A. positive B. negative C. active infection D. falsely negative
Explanations (c) A. A reaction size of greater than or equal to 5 mm in a HIV positive patient is considered a positive tuberculin skin test reaction. (u) B. See A for explanation. (u) C. A positive PPD identifies patients that have been infected with Mycobacterium tuberculosis, but does not indicate whether the disease is currently active or inactive. (u) D. See A for explanation.
Scientific Concepts/Pulmonology Which of the following is the major pathogenetic mechanism that causes asthma? A. Airway inflammation B. Increased pulmonary secretions C. Presence of Ghon complexes D. Irreversible fibrosis
Explanations (c) A. Airway inflammation is the major pathogenetic mechanism that leads to the development of asthma. (u) B. Increased pulmonary secretions are the mechanism in chronic bronchitis. (u) C. The presence of Ghon complexes is noted in pulmonary tuberculosis. (u) D. Irreversible fibrosis of the lung parenchyma is associated with interstitial lung diseases.
Health Maintenance/Pulmonology A 17 year-old male who is trying out for the track team notes excessive coughing with chest tightness when running. Which of the following is the most appropriate preventive agent for this patient? Answers A. Albuterol inhaler (Proventil) B. Inhaled corticosteroids C. Aminophylline (Theo-Dur) D. Ipratropium (Atrovent)
Explanations (c) A. Albuterol is a beta-2 agonist that results in bronchodilation that makes this a useful agent in a patient with exercise-induced asthma when used just prior to exercise. (u) B. Inhaled corticosteroids are effective in exercise-induced asthma but are not acute acting and due to the side effect profile, they are not first line agents. (u) C. Aminophylline is not used as a first-line agent as a bronchodilator for patients with exercise-induced asthma. (u) D. Ipratropium main use is with suppression of mucous secretions and this is not a component of the exercise-induced asthma patient.
Clinical Intervention/Pulmonology A 4 year-old boy is sent home from day care for a severe cough following one week of cold symptoms, including sneezing, conjunctivitis, and nocturnal cough. He presents with paroxysms of cough followed by a deep inspiration, and occasional post-tussive emesis. During severe paroxysms, he exhibits transient cyanosis. What is the most appropriate treatment for exposed contacts at his day care center? A. Macrolide prophylaxis B. Isolation C. Observation and treatment only if symptomatic D. Supportive care only
Explanations (c) A. All close contacts of a patient with pertussis should be treated with macrolide prophylaxis, regardless of age, immunization history, or symptoms. (u) B. Isolation of contacts is impractical and unnecessary. (u) C. Pertussis is rarely diagnosed before the paroxysmal stage, by which time exposure of contacts to the pathogen is assured. (u) D. While supportive care is essential in those contacts with symptoms, macrolide prophylaxis is mandatory in all contacts to prevent further spread of the illness.
History & Physical/Pulmonology A foreign body lodged in the trachea that is causing partial obstruction will most likely produce what physical examination finding? A. stridor B. aphonia C. inability to cough D. progressive cyanosis
Explanations (c) A. An inspiratory wheeze is called stridor, which indicates a partial obstruction of the trachea or larynx. (u) B. Aphonia, inability to cough and progressive cyanosis are seen with complete obstruction of the trachea, not partial obstruction. (u) C. See B for explanation. (u) D. See B for explanation.
Clinical Intervention/Pulmonology A 59 year-old otherwise healthy female develops acute dyspnea and chest pain one week post total abdominal hysterectomy. Echocardiogram demonstrates normal heart size with normal right and left ventricular function. Lung scan demonstrates two segmental perfusion defects. Which of the following is the next step in the management of this patient? A. Anticoagulation B. Embolectomy C. Thrombolysis D. Inferior vena cava filter
Explanations (c) A. Anticoagulation is the treatment of choice in patients with pulmonary embolism with normal ventricular function and no absolute contraindications. (u) B. Embolectomy is not indicated as initial treatment of a pulmonary embolism in patients with normal ventricular function. (h) C. Thrombolysis is contraindicated in patients within 10 days of having major surgery. (u) D. An inferior vena cava filter is considered in patients with contraindications to anticoagulation therapy or failed anticoagulation therapy.
Diagnosis/Pulmonology A 4 year-old patient presents with episodic wheezing and a non-productive cough for the last 4 weeks. His symptoms are worse at night. Past medical history reveals a history of atopic dermatitis. Physical examination at this time is unremarkable. Which of the following is the most likely diagnosis? A. Asthma B. Bronchiolitis C. Croup D. Cystic fibrosis
Explanations (c) A. Asthma is a chronic inflammatory disorder of the airways. It is characterized by episodic or chronic symptoms of airflow obstruction, breathlessness, cough, wheezing, and chest tightness. The strongest identifiable predisposing factor for the development of asthma is atopy. (u) B. Bronchiolitis is common in infants and young children presenting with acute onset of cough, rhinorrhea, tachypnea, and expiratory wheezes. (u) C. Croup usually presents with a prodrome of upper respiratory tract symptoms followed by onset of a barking cough and stridor. (u) D. Cystic fibrosis is an autosomal recessive disease and is characterized by a chronic cough, sputum production, dyspnea, and wheezing. Steatorrhea, diarrhea, and abdominal pain are also common.
Clinical Therapeutics/Pulmonology A 4 month-old infant is brought to the clinic by his mother with complaints of a cough for the past 3 weeks. Initially, symptoms included running nose, sneezing and an irritating cough. Over the past week the cough has changed to persistent staccato, paroxysmal forceful coughs ending with a loud inspiration. WBC is 20,0000/mcl with 72% lymphocytes. Which of the following is the drug of choice for managing this patient? A. Azithromycin (Zithromax) B. Ceftriaxone (Rocephin) C. Ampicillin (Unasyn) D. Gentamicin (Garamycin)
Explanations (c) A. Azithromycin terminates respiratory tract carriage of Bordetella pertussis. (u) B. Ceftriaxone does not eradicate Bordetella pertussis. (a) C. Ampicillin may be used for macrolide-intolerant patients however it is not the drug of choice. (u) D. Gentamicin does not eradicate Bordetella pertussis.
iagnostic Studies/Pulmonology Which of the following is the most common radiographic presentation of lung abscess? A. cavitation B. pleural thickening C. hilar mass D. hyperinflation
Explanations (c) A. Cavitation is seen with lung abscess or progressive primary tuberculosis. (u) B. Pleural thickening is noted in mesothelioma. (u) C. Hilar and mediastinal abnormalities are common on chest radiography in patients with lung cancer. (u) D. Hyperinflation is the main clinical feature in emphysema.
Clinical Intervention/Pulmonology Which of the following is a major contraindication to curative surgical resection of a lung tumor? Answers A. Liver metastases B. Vagus nerve involvement C. Non-malignant pleural effusion D. Chest wall invasion of the tumor
Explanations (c) A. Distant metastases, except for solitary brain and adrenal metastases are an absolute contraindication for pulmonary resection. Other absolute contraindications include MI within past 3 months, SVC syndrome due to metastatic tumor, bilateral endobronchial tumor, contralateral lymph node metastases and malignant pleural effusion. (u) B. See A for explanation. (h) C. See A for explanation. (u) D. See A for explanation.
Clinical Therapeutics/Pulmonology A 55 year-old man with a history of chronic bronchitis presents with two days of increased dyspnea and cough with worsening purulent sputum production. He is currently using inhaled albuterol as needed. In addition to systemic corticosteroids, what pharmacologic agent is warranted at this time for treatment of this patient? A. Antibiotic B. Inhaled corticosteroid C. Long acting beta-agonist D. Theophylline
Explanations (c) A. Empiric antibiotic treatment is indicated in the treatment of acute exacerbations of COPD if there are sputum changes suggestive of bacterial infection, such as increased quantity and purulence. (u) B. Inhaled corticosteroids are not indicated in the management of acute exacerbations of COPD. (u) C. Long acting beta-agonists are not indicated in the management of acute exacerbations of COPD. (u) D. Theophylline is rarely used in the management of COPD and has no place in the management of acute exacerbations of COPD.
Diagnostic Studies/Pulmonology A patient should be tested for tuberculosis prior to being treated with A. etanercept (Enbrel). B. cyclosporine (Neoral). C. methotrexate (Rheumatrex). D. prednisone (Deltasone).
Explanations (c) A. Etanercept is an anti-cytokine agent used in the treatment of rheumatoid arthritis and has as a side effect the potential for serious infections. One of these side effects includes reactivation of dormant tuberculosis. (u) B. Cyclosporine, methotrexate, and prednisone do not have the requirement to check for tuberculosis prior to initiating treatment. (u) C. See B for explanation. (u) D. See B for explanation.
History & Physical/Pulmonology A 19 year-old male presents to the ED complaining of a sudden onset of dyspnea and left sided chest pain. He denies fever, chills, cough or sore throat. General survey shows that he is 6 feet 2 inches tall and weighs 135 lbs. Vital signs are BP 86/60 mmHg, HR 130 bpm, RR 28, temp. 98.6 degrees F. Which of the following would you likely find on examination of his thorax? A. Left-sided hyperresonance B. Increased tactile fremitus of left base C. Scattered rales throughout D. Increased anterior/posterior diameter
Explanations (c) A. Hyperresonance is present with accumulation of air in the pleural space. (u) B. Increased tactile fremitus is present with lung consolidation. (u) C. Mycoplasma pneumonia, common in young adults, presents with scattered rales audible throughout. (u) D. Increased A/P diameter is common in older patients with history of COPD.
Clinical Intervention/Pulmonology A solitary pulmonary nodule is found on a pre-employment screening chest x-ray in a 34 year-old nonsmoking male. There are no old chest x-rays to compare. Which of the following is the most appropriate next step in the evaluation? A. CT scan of the chest B. Needle biopsy of the lesion C. Positron emission tomography of the chest D. Fiberoptic bronchoscopy
Explanations (c) A. In the absence of old x-rays in a nonsmoking individual less than 35 years old, CT scan of the chest is the next step in the evaluation of a solitary pulmonary nodule. (u) B. A needle biopsy would be indicated for a person greater than 35 years old and/or with a history of smoking to evaluate a solitary pulmonary nodule. (u) C. Positron emission tomography (PET scan) would be indicated if the CT scan was nonconclusive. (u) D. Fiberoptic bronchoscopy would be indicated only in the presence of a history of tobacco use or if the lesion was suggestive of malignancy.
History & Physical/Pulmonology Kussmaul breathing is characterized by A. rapid, deep labored breathing. B. irregular and varying depth of breathing. C. frequently interspersed deeper breaths. D. periods of deep breathing alternate with periods of apnea.
Explanations (c) A. Kussmaul breathing is characterized by rapid deep labored breathing. (u) B. This characterizes ataxic breathing. (u) C. This characterizes sighing. (u) D. Periods of deep breathing alternate with periods of apnea is Cheyne-Stokes breathing.
History & Physical/Pulmonology A 55 year-old female presents to the emergency department with complaints of dyspnea, chest pain and coughing with hemoptysis. Past medical history includes breast cancer 5 years ago, currently in remission. Vital signs are Temp. 98.6 degrees F, BP 150/90 mmHg, P 110 bpm, RR 20. Physical examination shows her right leg swollen with pain on palpation of deep veins. Which of the patient's history or examination findings is most suggestive of a pulmonary embolus (PE)? A. Leg swelling and pain with palpation of deep veins B. Heart rate > 100 C. Hemoptysis D. Past history of cancer
Explanations (c) A. Leg swelling and pain with palpation of the deep veins are consistent with a DVT and increase the likelihood of a PE. (u) B. While a tachycardic rate may be present in a patient with a PE, it is not specific for a PE. (u) C. While hemoptysis may be present in a PE, it is not a specific finding. (u) D. History of cancer may place a patient at increased risk for PE, however is not the most suggestive finding in this patient.
Scientific Concepts/Pulmonology Which of the following causes of pneumonia is most likely to be complicated by diarrhea? A. Legionella B. Chlamydophila C. Mycoplasma D. Pneumococcal
Explanations (c) A. Legionella is a water-borne pathogen that can cause diarrhea as a complication of its infection. (u) B. Chlamydophila pneumonia is associated with dry cough, low-grade fever, and hoarseness. (u) C. Mycoplasma pneumonia is most likely complicated by bullous myringitis, hoarseness, rash, and chronic cough. (u) D. Pneumococcal pneumonia presents with a single rigor, rust-colored sputum, and productive cough. There is no diarrhea as part of its infection.
History & Physical/Pulmonology Which of the following is the most likely to develop into a persistent cough in the adult patient? A. Pertussis B. Allergic rhinitis C. Pharyngitis D. Heart failure
Explanations (c) A. Pertussis is suspected in patients with persistent cough that lasts longer than 2-3 weeks. Allergic rhinitis, pharyngitis and heart failure are all potential causes of acute cough. (u) B. See A for explanation. (u) C. See A for explanation. (u) D. See A for explanation.
Health Maintenance/Pulmonology What is the most effective prophylaxis against respiratory syncytial virus (RSV) infection in the general pediatric population? A. Proper hand-washing techniques B. A monoclonal antibody C. H. Influenzae B vaccine D. Oseltamivir (Tamiflu)
Explanations (c) A. Proper hand washing and reduction in exposure is most effective in general population to prevent RSV. (u) B. Prophylaxis with a monoclonal antibody has proven effective in high risk infants but is not indicated in the general pediatric population. (u) C. Prophylaxis with H. Influenzae B vaccine has reduced incidence of epiglottis, not RSV. (u) D. Oseltamivir is not indicated in the prophylaxis of RSV.
Diagnosis/Pulmonology An infant born at 30 weeks' gestation begins to have respiratory difficulty shortly after birth. Examination reveals rapid, shallow respirations at 80 per minute with associated intercostal retractions, nasal flaring and progressive cyanosis. Chest x-ray reveals the presence of air bronchograms and diffuse bilateral atelectasis. Which of the following is the most likely diagnosis? A. Respiratory distress syndrome B. Spontaneous pneumothorax C. Transient tachypnea syndrome D. Meconium aspiration syndrome
Explanations (c) A. Respiratory distress syndrome (hyaline membrane disease) is the most common cause of respiratory distress in a premature infant. This diagnosis is supported by the chest x-ray findings of air bronchograms and diffuse bilateral atelectasis, causing a ground-glass appearance. (u) B. Although spontaneous pneumothorax will present with respiratory distress at birth, the chest x-ray would reveal findings of lung collapse. (u) C. While transient tachypnea syndrome also may present at birth with respiratory distress, the chest x-ray would reveal findings of increased pulmonary vasculature markings, perihilar streaking and fluid in the interlobular fissures. (u) D. Meconium aspiration syndrome usually occurs in term or post-term infants. Typical chest x-ray findings include patchy infiltrates, coarse streaking of both lung fields, increased anteroposterior diameter and flattening of the diaphragm.
Scientific Concepts/Pulmonology Which of the following describes the pathophysiological changes of pulmonary sarcoidosis? A. granulomas and inflammation of alveoli, small bronchi and small blood vessels B. inflammation and destruction of the structural components of the bronchial wall C. increased permeability of the alveolar-capillary membrane and diffuse alveolar damage D. edema of the airways with eosinophils, neutrophils, and lymphocytes
Explanations (c) A. Sarcoidosis is characterized by granulomas and inflammation of alveoli, small bronchi and small blood vessels. (u) B. Bronchiectasis is characterized by inflammation and destruction of the structural components of the bronchial wall. (u) C. Acute respiratory distress syndrome is characterized by increased permeability of the alveolar-capillary membrane and diffuse alveolar damage. (u) D. Asthma is characterized by edema of the airways with eosinophils, neutrophils, and lymphocytes.
Diagnosis/Pulmonology A 36 year-old African American female comes to the clinic for an insurance physical which requires a chest x-ray. She denies any respiratory symptoms. Examination of her chest is negative. X-ray results show marked lymphadenopathy in the right paratracheal region. Angiotensin-converting enzyme (ACE) levels are elevated. Which of the following is the most likely diagnosis? A. Sarcoidosis B. Tuberculosis C. Pulmonary fibrosis D. Lymphoma
Explanations (c) A. Sarcoidosis is characterized by paratracheal lymphadenopathy and elevated ACE levels. It is more common in African American patients and may be asymptomatic. (u) B. See A for explanation. (u) C. See A for explanation. (u) D. See A for explanation.
Clinical Intervention/Pulmonology A 60 year-old patient with COPD characteristic of emphysema presents with a cough and increased sputum production. The following information is noted: Temperature 100°F (37.8°C); Respiratory rate 20/min; Heart rate 88 beats/min; pH 7.44; PaO2 75 mmHg; PaCO2 40 mmHg; O2 saturation 92%. Physical examination is remarkable for increased AP diameter, diminished breath sounds without wheezes, rhonchi, or other signs of respiratory distress. Which of the following would be an appropriate treatment for this patient? A. Broad-spectrum antibiotic B. Admission to the hospital C. Oxygen at 6 L/min by nasal cannula D. Brief course of oral theophylline
Explanations (c) A. Sputum production is extremely variable from patient to patient, but any increase in sputum with a history of COPD reported by a patient must be regarded as potentially infectious and treated promptly. (u) B. Admission is only warranted if the patient's respiratory status requires ventilatory assistance. This patient's blood gases are unremarkable for a patient with COPD and the patient is not in respiratory distress. (u) C. Oxygen therapy should only be used for severe hypoxemia and should only be given at a low concentration, such as 2 L/min. Higher dose oxygen may stop the hypoxemic ventilatory drive. (u) D. Oral theophylline is considered a secondary bronchodilator. The use of a metered-dose inhaler would be a preferable first-line treatment if this method of treatment were chosen.
Clinical Therapeutics/Pulmonology A 47 year-old HIV positive female presents with a complaint of a nonproductive cough. She is febrile, tachypneic and tachycardic. Lung exam reveals bilateral rales. Chest x-ray shows diffuse interstitial infiltrates. What is the recommended treatment in this patient? A. trimethoprim-Sulfamethoxazole (Bactrim) B. tetracycline (Sumycin) C. amantadine (Symmetrel) D. ticarcillin (Ticar)
Explanations (c) A. TMP-SMX is the drug of choice for all forms of pneumocystis. (u) B. Tetracycline, amantadine and ticarcillin are not effective against PCP. (u) C. See B for explanation. (u) D. See B for explanation.
Clinical Intervention/Pulmonology A 40 year-old male nonsmoker in good health undergoes a routine chest x-ray for an insurance physical. Results show an isolated, well-defined, coin lesion 1 cm in size. Which of the following is the next step in the evaluation of this problem? A. Review old radiographs B. Order chest CT C. Schedule lung biopsy D. Prepare for surgical lung resection
Explanations (c) A. The first and most important step in the radiographic evaluation is to review old radiographs to estimate doubling time, an important marker for malignancy. (u) B. See A for explanation. (u) C. See A for explanation. (u) D. See A for explanation.
Clinical Therapeutics/Pulmonology A patient presents with occasional wheezing and chest tightness that occurs approximately once a week and at night only about once a month. Peak expiratory flow is 85% of predicted. Which of the following is the most appropriate initial treatment? A. Albuterol (Proventil) inhaler B. Montelukast (Singular) C. Salmeterol (Serevent) inhaler D. Sustained release theophylline
Explanations (c) A. This patient has mild intermittent asthma which is initially treated with inhaled beta 2-agonists as needed. No long-term control medications are indicated. (u) B. Leukotriene modifiers, such as montelukast, may be added to the treatment of uncontrolled asthma as a long- term controller after the initiation of inhaled corticosteroids. (u) C. Long-acting beta 2-agonists, such as salmeterol, are indicated for long-term control of asthma that is categorized as moderate persistent to severe persistent. (u) D. Sustained release theophylline is an alternative treatment for asthma that is at least categorized as mild persistent; however its narrow therapeutic window and side effects limit its use.
History & Physical/Pulmonology A 24 year-old male presents in respiratory distress and appears quite ill. A Gram stain and culture of the sputum reveals gram-positive cocci in clumps and a chest x-ray reveals multiple patchy infiltrates with some cavitations. Which of the following is most likely to also be found in his medical history? A. IV drug abuse B. Alcohol abuse C. Poor dental hygiene D. HIV positive patient
Explanations (c) A. This patient has pneumonia caused by Staphylococcus aureus which is commonly associated with a history of intravenous drug use, influenza epidemics and the hospital setting. (u) B. A history of alcohol abuse is commonly seen with pneumonia caused by Klebsiella pneumoniae. (u) C. Poor dental hygiene is associated with pneumonia caused by anaerobes. (u) D. HIV positive patients are most at risk for development of pneumonia caused by Pneumocystis jiroveci.
iagnosis/Pulmonology A 69 year-old male with a history of chronic lymphocytic leukemia presents to the clinic complaining of cough, dyspnea and production of copious amounts of foul smelling sputum. Physical examination reveals crackles at the lung bases. Chest x-ray shows dilated and thickened bronchi that appear as ring-like markings. Which of the following is the most likely diagnosis? A. Bronchiectasis B. Tuberculosis C. Adenocarcinoma D. Pulmonary fibrosis
Explanations (c) A. This patient has signs and symptoms consistent with bronchiectasis including CXR findings of dilated and thickened bronchi that may appear as tram-tracks or as ring-like markings. (u) B. TB would present with CXR findings in the apical or posterior segments of the upper lobes. (u) C. Radiographic findings of adenocarcinoma include enlarged nodule or mass; persistent opacity, atelectasis or pleural effusion. The sputum would not likely be foul smelling. (u) D. Pulmonary fibrosis does not present with dilated bronchi or ring-like markings on CXR
History & Physical/Pulmonology A 45 year-old male presents with sudden onset of pleuritic chest pain, productive cough and fever for 1 day. He relates having symptoms of a "cold" for the past week that suddenly became worse yesterday. Which of the following findings will most likely be seen on physical examination of this patient? Answers A. spoken "ee" heard as "ay" B. hyperresonant percussion note C. wheezes over the involved area D. vesicular breath sounds over involved area
Explanations (c) A. This patient most likely has a bacterial pneumonia with consolidation, which would produce egophony, where a spoken "ee" is heard as "ay." (u) B. Consolidation from bacterial pneumonia causes findings of dullness to percussion, late inspiratory crackles and bronchial breath sounds over the involved area. (u) C. See explanation B. (u) D. See explanation B.
Diagnosis/Pulmonology A 24 year-old male presents complaining of a 9 month history of increasing shortness of breath, dyspnea on exertion, and a cough productive of white sputum, mostly in the mornings. He denies orthopnea, PND, peripheral edema, fever, chills, night sweats, recent changes in weight, palpitations, chest pain, food intolerances, or other complaints. Patient has a history of recurrent lung infections. He states that his father had chronic pulmonary problems and died at age 42 from unknown lung disease. The patient denies smoking, alcohol or illicit drug use. On physical examination, the respiratory rate is 22 per minute, pulse of 98 bpm, temperature of 98.7 degrees. Pulmonary exam reveals end-expiratory wheezes bilaterally and hyperresonance to percussion. His cardiac exam is normal. Chest x- ray shows decreased lung markings. ECG is normal. Pulmonary function tests show an FEV1 63% of expected and residual capacity is 123% of expected. Which of the following is the most likely diagnosis? A. Emphysema B. Pulmonary fibrosis C. Ventricular septal defect D. Congestive heart failure
Explanations (c) A. This person has an obstructive lung disease based on PFTs. Emphysema is the most likely diagnosis, and may be related to alpha-1 antitrypsin deficiency based on family history and lack of smoking history and young age. (u) B. The PFTs from a person with pulmonary fibrosis would be consistent with a restrictive pattern. This patient has an obstructive pattern of lung disease. (u) C. Ventricular septal defect will have a systolic murmur associated with it. (u) D. Congestive heart failure might explain some of the symptoms of this patient (increasing shortness of breath and DOE), he denies other common symptoms, such as orthopnea and peripheral edema. CHF should not result in changes in the PFTs.
Health Maintenance/Pulmonology A 3 year-old girl is diagnosed with atopic dermatitis. Which of the following disorders is this child at risk for in the future? A. Asthma B. Tinea pedis C. Squamous carcinoma D. Systemic lupus erythematosus (SLE)
Explanations (c) A. Up to 50% of patients with atopic dermatitis develop asthma and/or allergic rhinitis in the future. (u) B. Patients with atopic dermatitis are more likely to get superimposed viral or bacterial infections such as herpes simplex or staphylococcal, but they are not more at risk for fungal infections. (u) C. Patients with atopic dermatitis are at no greater risk for any skin cancer. (u) D. Lupus is a connective tissue disorder of the immune system, but unrelated to atopic dermatitis.
Clinical Therapeutics/Pulmonology A 70 year-old male is admitted to the ICU with fever, leukocytosis and purulent sputum. Sputum culture shows methicillin-resistant gram-positive cocci in clusters. Which of the following medications should be ordered for this patient? A. Vancomycin (Vancocin) B. Clindamycin (Cleocin) C. Azithromycin (Zithromax) D. Astreonam (Azactam)
Explanations (c) A. Vancomycin is the drug of choice for methicillin-resistant S. aureus. (u) B. Clindamycin is used if the patient is at risk for Legionella infection. (u) C. Azithromycin is more commonly used for H. influenza coverage. (u) D. Astreonam covers gram positive organisms but not methicillin-resistant S. aureus.
Diagnostic Studies/Pulmonology A 14 year-old male presents to the ED experiencing a severe asthma attack. His respiratory effort is shallow and he is using accessory muscles to breathe. Auscultation of his chest reveals no audible wheezing. Vital signs include BP 90/60 mmHg, P 160 bpm, RR 52. An arterial blood gas (ABG) is ordered. Normal ABG values at your institution are pH 7.35-7.45, CO2 35-45, pO2 80-95. Which of the following ABG findings suggests the poorest prognosis? A. pH = 7.27 pCO2 = 46 pO2 = 56 B. pH = 7.60, pCO2 = 18 pO2 = 80 C. pH = 7.44, pCO2 = 38 pO2 = 90 D. pH = 7.52, pCO2 = 28, pO2 = 80
Explanations (c) A. pH = 7.27 pCO2 = 46 pO2 = 56 is associated with the poorest prognosis in this patient. (u) B. See A for explanation. (u) C. See A for explanation. (u) D. See A for explanation.
Clinical Intervention/Pulmonology A patient with severe COPD presents to the Emergency Department with a 3 day history of increasing shortness of breath with exertion and cough productive of purulent sputum. An arterial blood gas reveals a pH of 7.25, PaCO2 of 70 mmHg and PaO2 of 50 mmHg. He is started on albuterol nebulizer, nasal oxygen at 2 liters per minute, and an IV is started. After one hour of treatment, his arterial blood gas now reveals a pH of 7.15, PaCO2 100 mmHg and PaO2 of 70 mmHg. Which of the following is the most appropriate next step in his treatment? A. Decrease the oxygen flow rate. B. Administer oral corticosteroids. C. Intubate the patient. D. Administer salmeterol (Serevent)
Explanations (h) A. Decreasing the oxygen flow rate would be harmful as it would decrease the amount of oxygen delivered to the patient. (u) B. Administration of steroids is an important treatment modality but this patient is in respiratory failure and needs more immediate therapy. (c) C. This person has increasing respiratory failure as indicated by the raising PaCO2 levels. Intubation is required at this time. (h) D. Long-acting beta agonist therapy such as salmeterol is not utilized for rescue therapy.
Diagnostic Studies/Pulmonology A 23 year-old female with history of asthma for the past 5 years presents with complaints of increasing shortness of breath for 2 days. Her asthma has been well controlled until 2 days ago and since yesterday she has been using her albuterol inhaler every 4-6 hours. She is normally very active, however yesterday she did not complete her 30 minutes exercise routine due to increasing dyspnea. She denies any cough, fever, recent surgeries or use of oral contraceptives. On examination, you note the presence of prolonged expiration and diffuse wheezing. The remainder of the exam is unremarkable. Which of the following is the most appropriate initial diagnostic evaluation prior to initiation of treatment? Answers A. chest x-ray B. sputum gram stain C. peak flow D. ventilation - perfusion scan
Explanations (u) A. A chest x-ray should be ordered in an asthmatic patient only if you are concerned about the presence of pneumonia or pneumothorax, neither of which is supported by the H&P findings noted above. (u) B. A sputum gram stain is performed in patients who you suspect have an infectious process, such as pneumonia. (c) C. A peak flow reading will help you to gauge her current extent of airflow obstruction and is helpful in monitoring the effectiveness of any treatment interventions. (u) D. A ventilation-perfusion scan (V/Q scan) is indicated in cases of suspected pulmonary embolism. The patient above does not have any risk factors that would lead you to suspect such a diagnosis.
Diagnosis/Pulmonology A 28 year-old man presents to the emergency department complaining of sudden onset of shortness of breath associated with sharp right-sided chest pain increased with breathing. On physical examination, respirations are 20 per minute and blood pressure is 120/76 mm Hg. Auscultation of the chest reveals absent breath sounds over the right apex with normal heart sounds. Percussion of the right apex is noted to be hyperresonant. Which of the following is the most likely diagnosis? A. Hemothorax B. Pneumothorax C. Pulmonary embolus D. Foreign body aspiration
Explanations (u) A. A hemothorax usually results from trauma. While breath sounds would be absent over the involved area, the percussion note would be dull, not hyperresonant. (c) B. This patient most likely has a spontaneous pneumothorax which is supported by the presenting symptoms of sudden onset of dyspnea and pleuritic chest pain as well as the physical exam findings of absent breath sounds and hyperresonance to percussion. (u) C. While a pulmonary embolism most commonly presents with dyspnea and pain on inspiration, the physical examination is often unremarkable and would not reveal the findings of absent breath sounds and hyperresonance to percussion. (u) D. Foreign body aspiration is most common in children and the elderly. Aspiration of a food bolus is the most common cause in adults which leads to a history of a choking episode followed by dysphagia, wheezing and coughing. Physical examination findings are dependent on the location of the obstruction.
Health Maintenance/Pulmonology An asymptomatic 60 year-old female with a 30 pack year history of smoking presents to the clinic requesting a chest x-ray to check for lung cancer. Which of the following do you recommend? A. A low-dose helical computed tomography (CT) B. Carcinoembryonic antigen (CEA) C. Serial chest radiographs to identify early stage malignancy D. No testing and referral for smoking cessation
Explanations (u) A. A low-dose helical computed tomography is very sensitive but is expensive, has increase false-positive tests and increases over-diagnosis. Mortality benefit remains to be proved. (u) B. CEA lacks clinical validation as a screening for lung cancer. (u) C. No major advisory organization recommends serial radiograph screening for lung cancer. (c) D. No major advisory organization recommends screening for lung cancer.
Clinical Therapeutics/Pulmonology A 6 year-old boy is brought to the pediatric clinic by his mother for an evaluation of his asthma. He coughs about 3 days out of the week with at least 2-3 nights of coughing. Which of the following would be the most appropriate treatment for this patient? A. Mast cell stabilizer B. Long acting beta agonist C. Leukotriene receptor antagonist D. Low dose inhaled corticosteroid
Explanations (u) A. A mast cell stabilizer is an alternative treatment but not the preferred treatment. (u) B. Long acting beta agonist can be used as adjunctive therapy with an anti-inflammatory. (u) C. Leukotriene receptor antagonists are an alternative treatment but not the preferred treatment. (c) D. Low dose inhaled corticosteroids are the preferred treatment for mild persistent asthma.
Clinical Intervention/Pulmonology Which of the following is an indication for hospitalization in a patient who has acute bronchiolitis? A. pulse oximetry of 94% on room air B. children between 4-6 months of age C. moderate tachypnea with feeding difficulties D. hyperinflation and interstitial infiltrates on chest x-ray
Explanations (u) A. A pulse oximetry reading of 94% on room air is equivalent to a PaO2 of approximately 80 mm Hg which indicates the child is not in severe respiratory distress. (u) B. Children less than 2 months of age require hospitalization. (c) C. Indications for hospitalization include moderate tachypnea with feeding difficulties. (u) D. Hyperinflation and interstitial infiltrates on chest x-ray are frequently seen with acute bronchiolitis and by themselves are not an indication for hospitalization.
History & Physical/Pulmonology A 27 year-old woman presents with one week of worsening productive cough, dyspnea, fever and malaise. Which of the following physical examination findings would support the diagnosis of lobar pneumonia? A. Decreased tactile fremitus B. Egophony C. Hyperresonance to percussion D. Wheeze
Explanations (u) A. Consolidation would increase the transmission of vocal vibrations and manifest as increased tactile fremitus. (c) B. Transmission of vocal sounds through consolidation leads to the changes heard with egophony. (u) C. Dullness, not hyperresonance, would be expected with consolidation. (u) D. Wheezing is associated with narrowing of the small airways as seen in asthma.
Clinical Intervention/Pulmonology A 42 year-old male is brought to the emergency department with a stab wound to his right lateral chest wall. On physical examination, the patient is stable with decreased breath sounds on the right with dullness to percussion. An upright chest x-ray reveals the presence of a moderate pleural effusion. Subsequent diagnostic thoracentesis contains bloody aspirate. Which of the following is the next most appropriate intervention? A. Thoracotomy B. Needle aspiration C. Close observation D. Tube thoracostomy
Explanations (u) A. A thoracotomy is indicated in a small percentage of patients that do not respond to IV administration of fluids and evacuation of the hemothorax via tube thoracostomy. (u) B. Needle aspiration as treatment for a hemothorax is not recommended as it fails to adequately drain the bloody fluid. (u) C. Close observation is only indicated for small effusions in a patient that remains hemodynamically stable. (c) D. This patient has a hemothorax. Drainage of a hemothorax is best obtained through insertion of a chest tube (tube thoracostomy).
Clinical Therapeutics/Pulmonology Which of the following classes of medications is most likely to cause a persistent cough? A. Tricyclic antidepressants B. Calcium channel blockers C. Beta-adrenoceptor blocking agents D. Angiotensin converting enzyme inhibitors
Explanations (u) A. Adverse effects of tricyclic antidepressants are due to blockage of acetylcholine receptors causing such symptoms as blurred vision, dry mouth, urinary retention and constipation. (u) B. Side effects are uncommon with calcium channel blockers, but include constipation, dizziness, headache and a feeling of fatigue. (u) C. Common adverse effects of beta-adrenoceptor blocking agents include bradycardia and central nervous system symptoms, such as fatigue or insomnia. (c) D. Common side effects of angiotensin converting enzyme inhibitors (ACE inhibitors) include a dry cough.
Scientific Concepts/Pulmonology Patients with long-term exposure to silica, coal dust, and asbestos may develop which of the following as complications? A. Airwayhyperreactivity B. Epithelial hyperplasia C. Pulmonary fibrosis D. Upper airway obstruction
Explanations (u) A. Airway hyperreactivity is classically seen with asthma and while acute exposure to occupational mineral dusts may cause airway hyperreactivity, this does not persist in long-term exposures. (u) B. Epithelial hyperplasia is one of the mechanisms involved in the pathogenesis of chronic bronchitis and is not involved in the pneumoconioses. (c) C. The principal processes in the pathogenesis of this set of diseases is inflammation and subsequent fibrosis. (u) D. The upper airway is not involved in this disease process.
Clinical Therapeutics/Pulmonology An 85 year-old nursing home resident presents with abrupt onset of cough, sore throat, headache, myalgias, and malaise. On examination the patient's temperature is 102 degrees F; the rest of the exam is unremarkable. Nasal smear is positive for Influenza B. Which of the following is the treatment of choice in this patient? A. Amantadine (Symmetrel) B. Oseltamivir (Tamiflu) C. Acyclovir (Zovirax) D. Nevirapine (Viramune)
Explanations (u) A. Amantadine is only used to treat influenza A. (c) B. Oseltamivir is used to treat both influenza A and B. (u) C. Acyclovir is used to treat viral infections due to certain herpes viruses. (u) D. Nevirapine is used to treat infection due to HIV.
Clinical Intervention/Pulmonology A 3 month-old male presents with a hoarse cough and thick purulent rhinorrhea for the past 2 days. The mother noted that yesterday he appeared to get worse and seemed to have increasing problems breathing and trouble feeding. Examination reveals a temperature of 100.2 degrees F and respiratory rate of 80/minute with nasal flaring and retractions. Lung examination reveals a prolonged expiratory phase with inspiratory rales. He is tachycardic. Pulse oximetry reveals oxygen saturation of 89%. Chest x-ray reveals hyperinflation with diffuse interstitial infiltrates. Which of the following is the most appropriate intervention? Answers A. antibiotics B. hospitalization C. inhaled corticosteroids D. Racemic epinephrine
Explanations (u) A. Antibiotics are utilized to treat bacterial, not viral, illnesses. (c) B. This infant most likely has bronchiolitis. While most cases are mild and can be treated at home, hospitalization is recommended for infants with hypoxia on room air, moderate tachypnea with feeding difficulties and marked respiratory distress with retractions. Additionally hospitalization is recommended for infants less than 2-3 months of age, a history of apnea or an underlying chronic cardiopulmonary disease. (u) C. The use of corticosteroids in children with bronchiolitis has not been studied and does not appear to be helpful. (u) D. Racemic epinephrine is not indicated in the treatment of bronchiolitis.
Clinical Therapeutics/Pulmonology A 42 year-old male with unremarkable past medical history is admitted to the general medical ward with community-acquired pneumonia. He has a 20 pack-year history of cigarette smoking. He is empirically started on ceftriaxone (Rocephin). Which of the following antibiotics would be most appropriate to add to his empiric treatment regimen? Answers A. Piperacillin B. Vancomycin C. Clindamycin D. Azithromycin
Explanations (u) A. Antipseudomonal penicillins, such as piperacillin, added to an aminoglycoside or fluoroquinolone are indicated for empiric treatment of nosocomial pneumonia. (u) B. Use of vancomycin should be restricted to cases where there is a high index of suspicion of involvement of methicillin resistant Staphylococcus aureus (MRSA). (u) C. Clindamycin is indicated in cases of suspected anaerobe involvement. (c) D. Patients with community-acquired pneumonia who require hospitalization on the general medical ward are treated with an extended-spectrum beta-lactam antibiotic, such as ceftriaxone, with a macrolide, such as azithromycin. Addition of a macrolide is also recommended due to the patient's smoking history and possible involvement of Hemophilus influenzae.
History & Physical/Pulmonology A patient presents with respiratory complaints. Chest x-ray reveals calcification of the hilar nodes with an eggshell pattern. Which of the following occupations is most consistent with these chest x-ray findings? A. building demolitioners B. coal miners C. sandblasters D. farmers
Explanations (u) A. Asbestosis is the most likely occupational risk and has an indistinct heart border appearance on CXR described as looking like "ground glass". (u) B. The CXR of a coal miner consists of irregular opacities ranging from a reticular pattern to a nodular pattern. (c) C. Silicosis can occur in sandblasters and produce a CXR appearance of calcification of the hilar nodes with an "eggshell" pattern with long term exposure (u) D. Farmers lung results from spores and produces a hypersensitivity pneumonitis. The CXR would show a patchy fibrosis.
Diagnosis/Pulmonology A 40 year-old woman presents with 3 months of dry cough and intermittent low-grade fever. She is a non-smoker and has no significant family history or past medical history. A purified protein derivative (PPD) test was recently performed at work and was negative. On physical examination she is afebrile with stable vital signs. Lung auscultation reveals crackles in bilateral upper lobes. Chest x-ray shows hilar and mediastinal adenopathy, mild interstitial disease in the upper lung zones, and several small granulomas in both lungs. What is the most likely diagnosis? A. Asbestosis B. Cryptococcosis C. Sarcoidosis D. Tuberculosis
Explanations (u) A. Asbestosis typically presents as interstitial disease in the lower lungs and this patient has no known exposure to asbestos. (u) B. Cryptococcosis typically shows pleural-based nodules on x-ray and this patient has no known risk factors (HIV disease, COPD, chronic steroid use). (c) C. Sarcoidosis classically presents as a vague systemic illness with radiographic evidence of any or all of the following: granulomas, hilar and mediastinal adenopathy and interstitial infiltrate. (u) D. The patient's recent negative PPD makes tuberculosis unlikely.
History & Physical/Pulmonology On physical examination you note diminished breath sounds over the right lower lobe with decreased tactile fremitus and dullness to percussion. Which of the following is the most likely cause? Answers A. asthma B. consolidation C. pneumothorax D. pleural effusion
Explanations (u) A. Asthma is characterized by decreased tactile fremitus, but would have resonant to hyperresonant percussion, not dullness. (u) B. Consolidation from pneumonia is characterized by dullness to percussion, but would have an increased, not decreased, tactile fremitus. (u) C. A pneumothorax is characterized by decreased to absent tactile fremitus, but would have a hyperresonant percussion note, not dullness. (c) D. A decreased tactile fremitus and dullness to percussion would be found in a pleural effusion.
Clinical Intervention/Pulmonology A 45 year-old male presents to the clinic complaining of morning sluggishness, daytime fatigue, headaches. He admits to drinking two cocktails each evening. His bed partner reports his loud cyclical snoring, breath cessation and thrashing movements of his extremities during sleep. BMI is 40. Heart examination reveals regular rate and rhythm without S3, S4, or murmur and lungs are clear to auscultation. Polysomnography shows apneic episodes lasting as long as 60 seconds. Which of the following clinical interventions would most likely provide for the acute cessation of apneic episodes? A. Avoidance of alcohol B. Weight loss C. Nasal continuous positive airway pressure D. Supplemental oxygen
Explanations (u) A. Avoidance of alcohol is also a necessary step in managing sleep apnea however does not provide immediate relief of apneic episodes. (u) B. Weight loss is also a necessary step in managing sleep apnea however does not provide immediate relief of apneic episodes. (c) C. Nasal CPAP is curative in many patients with sleep apnea. (u) D. Oxygen lessens the severity of nocturnal desaturation but may lengthen the apnea episodes.
Diagnosis/Pulmonology A 5 year-old male presents with a history of recurrent episodes of acute bronchitis, characterized by fever and productive cough. He has no known significant past medical history. His pulmonary examination reveals crackles in the bilateral lower lobes. The remainder of his physical examination is normal. Chest x-ray demonstrates platelike atelectasis and dilated, thickened airways in the middle and lower lungs. Which of the following is the most likely diagnosis? A. Acute bronchitis B. Bronchiectasis C. Pneumonia D. Tuberculosis
Explanations (u) A. Barring underlying pulmonary pathology,the chest x-ray in acute bronchitis should be normal. (c) B. Bronchiectasis typically presents as recurrent episodes of acute bronchitis. Platelike atelectasis and dilated and thickened airways, sometimes described as tram lines, are common radiographic findings. (u) C. While the history may suggest pneumonia, the radiographic findings do not support this diagnosis. (u) D. Tuberculosis would present with cavitating granuloma formation more commonly at the apices.
Diagnostic Studies/Pulmonology A 42 year-old male smoker presents for further evaluation of a 4 cm solitary pulmonary nodule discovered on a recent chest x-ray. Which of the following diagnostic tests is indicated next? A. Bone scan B. Thoractotomy C. Mediastinoscopy D. CT scan of chest
Explanations (u) A. Bone scanning is indicated for evaluation of bone metastases, most commonly secondary to cancer of the breast or prostate. (u) B. Diagnostic thoractotomy is indicated for biopsy of the lesion should the CT scan of the chest indicate a suspicious malignant lesion that is inaccessible to thoracoscopy. (u) C. Mediastinoscopy can be utilized to further evaluate any enlarged mediastinal lymph nodes that may be found on the CT scan of the chest, but is not indicated prior to the CT scan. (c) D. A CT scan of the chest is needed to further evaluate the characteristics of the solitary pulmonary nodule and to determine lymph node involvement or presence of multiple lesions.
Health Maintenance/Pulmonology A mother of a newborn infant presents to the office concerned about reducing the risk of sudden infant death syndrome (SIDS). The infant was delivered at 39 weeks gestation weighing 7 pounds 9 ounces. There is no family history of SIDS and this is her first child. Which of the following is appropriate advice to reduce the risk of SIDS? A. Bottle feeding with soy formula B. Offer a pacifier at nap and bedtimes C. Have the infant sleep in the prone position D. Infant should sleep with the parents to allow close observation
Explanations (u) A. Bottle feeding with soy formula does not reduce the risk of SIDS. (c) B. Use of a pacifier during sleeping is a current recommendation to decrease the risk of SIDS. (u) C. Sleeping prone has been consistently shown to increase, not decrease, the risk of SIDS. (u) D. Bed sharing with parents has been shown to increase, not decrease, the risk of SIDS. This risk is increased in infants less than 4 months old and when older children are also present in the bed.
Clinical Therapeutics/Pulmonology Which of the following is the greatest risk factor for the development of drug resistance in the treatment of tuberculosis? Answers A. A child with cystic fibrosis B. An elderly patient in a nursing home C. Non-adherence to prescribed drug regimen D. Patients with a history of diabetes mellitus
Explanations (u) A. Children with cystic fibrosis are at an increased risk for various lung infections, but not drug resistance. (u) B. While institutionalized patients, such as nursing home residents, are at increased risk for infection with Mycobacterium tuberculosis, the patient is not at increased risk for drug resistance. (c) C. Non-adherence to prescribed drug regimens is a risk factor for the development of drug resistance as well as being a major cause of treatment failure and continued transmission of tuberculosis. Patient education about the importance of drug therapy is important in an attempt to avoid this. (u) D. Patients with a history of diabetes mellitus are at increased risk for active disease, not drug resistance.
Diagnosis/Pulmonology A patient who appears very anxious enters the office complaining of dizziness with perioral and extremity paresthesias. She vaguely describes some chest discomfort. Physical examination is unremarkable, except for moderate tachypnea with obvious sighing respiration. This clinical picture is most consistent with A. bronchial asthma. B. hyperventilation syndrome. C. spontaneous pneumothorax. D. emphysema.
Explanations (u) A. Bronchial asthma attacks are associated with increased dyspnea and prolonged expiration. Patients may use accessory muscles of respiration as part of this acute condition. (c) B. Anxiety may result in hyperventilation that can result in perioral numbness and paresthesias of the extremities. These paresthesias are due to decreased CO2 in the blood stream that results from the hyperventilation. Anxious patients also will have nondescript chest pain as part of this condition and may also complain of dizziness. (u) C. Spontaneous pneumothorax patients will primarily complain of significant chest pain along with their dyspnea. These patients will not have perioral or extremity paresthesias. (u) D. Emphysema alone will not result in hyperventilation or the production of perioral or extremity paresthesias and is a chronic progressive rather than an acute onset condition.
Health Maintenance/Pulmonology In patients with COPD, which of the following has been shown to decrease rate of malignancy and cardiovascular disease and improve survival? A. bronchodilator therapy B. pulmonary rehabilitation C. oral glucocorticosteroids D. smoking cessation
Explanations (u) A. Bronchodilator therapy is used for symptomatic treatment in patients with COPD. (u) B. Pulmonary rehabilitation improves quality of life, dyspnea and exercise capacity. It also has been shown to reduce the rate of hospitalization. (u) C. Chronic use of oral glucocorticosteroids is not recommended because of an unfavorable benefit/risk ratio. (c) D. Smoking cessation has been shown to provide significant improvement in decreasing the rate of decline in pulmonary function.
Diagnosis/Pulmonology A 74 year-old male with a history of coronary artery disease and atrial fibrillation presents to the clinic for follow-up of his shortness of breath. Patient's medications include amiodarone (Cordarone) and metoprolol (Lopressor). His chest x-ray reveals patchy ground-glass infiltrates. Which of the following is the most likely diagnosis? A. COPD B. Tuberculosis C. Bronchiectasis D. Pulmonary fibrosis
Explanations (u) A. COPD appears as hyperinflation with flattening of the diaphragm on chest radiograph. (u) B. Tuberculosis presents with pulmonary infiltrates on chest radiograph most often apical; cavitations may be seen with progressive primary tuberculosis. (u) C. Chest x-ray in bronchiectasis shows dilated and thickened bronchi that appear as ring-like markings. (c) D. Pulmonary fibrosis presents with ground-glass infiltrates on CXR and is often associated with certain medication use.
Health Maintenance/Pulmonology Which of the following is the most effective way for patients with persistent asthma to monitor the severity of their symptoms? A. call the health care provider regularly B. keep a diary of symptoms C. monitor peak flow D. ask a family member to monitor symptoms
Explanations (u) A. Calling the health care provider regularly or asking a family member to monitor symptoms is not effective in patients understanding how to manage their asthma symptoms. (u) B. Keeping a diary may be effective for patients to understand their cause of symptoms, but would not be effective in helping to manage their symptoms. (c) C. Monitoring peak flow is the most effective way for the patients and health care providers to manage symptoms and guide treatment. (u) D. See A for explanation.
A 2 month-old infant has been diagnosed with pneumonia due to Chlamydia trachomatis. Which of the following is the treatment of choice? A. Ceftriaxone (Rocephin) B. Doxycycline C. Levofloxacin (Levaquin) D. Erythromycin
Explanations (u) A. Ceftriaxone is a third-generation cephalosporin that may be safely used in children, however is not indicated for the treatment of Chlamydial pneumonia. (h) B. Doxycycline is a tetracycline and is contraindicated in children under eight years of age secondary to damaging effects on bone and teeth enamel. (h) C. Levofloxacin is a fluoroquinolone and is contraindicated for use in children under 18 years of age secondary to damaging effects that may occur with growing cartilage. (c) D. Erythromycin or sulfisoxazole is the treatment of choice for an infant with Chlamydial pneumonia.
Diagnostic Studies/Pulmonology A patient presents with a history of progressive worsening of dyspnea over the past several years. He gives a history of having worked as a ship builder for over 50 years. He denies any alcohol or tobacco use. On examination you note clubbing and inspiratory crackles. Which of the following chest x-ray findings support your suspected diagnosis? Answers A. hyperinflation and flat diaphragms B. interstitial fibrosis and pleural thickening C. cavitary lesions involving the upper lobes D. "eggshell" calcification of hilar lymph nodes
Explanations (u) A. Chest x-ray findings of hyperinflation and flat diaphragms suggest long-standing chronic obstructive lung disease. (c) B. This patient most likely has asbestosis, which is supported by his occupation as a ship builder and clinical presentation as noted above. Chest x-ray findings include interstitial fibrosis, pleural thickening and calcified pleural plaques on the diaphragm or lateral chest wall. (u) C. Chest x-ray findings of cavitary lesions involving the upper lobes suggest pulmonary tuberculosis. (u) D. Chest x-ray findings of "eggshell" calcification of hilar lymph nodes strongly supports the diagnosis of silicosis.
Diagnostic Studies/Pulmonology A 62 year-old homeless patient presents complaining of fever, weight loss, anorexia, night sweats and a chronic cough that recently became productive of purulent sputum that is blood streaked. On physical examination, the patient appears chronically ill and malnourished. Which of the following chest x-ray findings supports your suspected diagnosis? A. Hyperinflation and flat diaphragms B. Interstitial fibrosis and pleural thickening C. Cavitary lesions involving the upper lobes D. "Eggshell" calcification of hilar lymph nodes
Explanations (u) A. Chest x-ray findings of hyperinflation and flat diaphragms suggest long-standing chronic obstructive lung disease. (u) B. Interstitial fibrosis and pleural thickening on a chest x-ray are found in cases of interstitial lung disease. (c) C. This patient most likely has tuberculosis. A chest x-ray finding of cavitary lesions involving the upper lobes would support this suspected diagnosis. (u) D. Chest x-ray findings of "eggshell" calcification of hilar lymph nodes strongly supports a diagnosis of silicosis.
Clinical Therapeutics/Pulmonology A previously healthy 8 month-old boy is hospitalized for acute bronchiolitis. He has no known significant past medical or family history. On admission, he exhibits nasal flaring and retractions with a respiratory rate of 68, axillary temperature of 102.0 degrees F and O2 saturation of 86%. Which of the following medications is indicated? A. Prednisolone B. Oxygen C. Ceftriaxone (Rocephin) D. Palivizumab (Synagis)
Explanations (u) A. Corticosteroids are not indicated for the treatment of previously healthy infants with bronchiolitis. (c) B. Oxygen is an important supportive therapy for hypoxemic infants with bronchiolitis. Bronchodilators would also be initiated in this patient. (u) C. Antibiotics are not indicated in the treatment of bronchiolitis unless there is a secondary bacterial infection. (u) D. Palivizumab is used only for prevention of RSV infection.
Diagnostic Studies/Pulmonology Which of the following is essential to make a diagnosis of cystic fibrosis? Answers A. Positive family history B. Elevated sweat chloride C. Recurrent respiratory infections D. Elevated trypsinogen levels
Explanations (u) A. Cystic fibrosis is a genetic disease, but a positive family history in and of itself is not enough to diagnose the condition. (c) B. The diagnosis of cystic fibrosis is made only after an elevated sweat chloride test or demonstration of a genotype consistent with cystic fibrosis. (u) C. While recurrent respiratory infections are a classic presentation of cystic fibrosis, the diagnosis relies on confirmation, as noted in explanation B. (u) D. Trypsinogen levels are used as a neonatal screening test and if elevated should be followed by more definitive testing to confirm the diagnosis.
Scientific Concepts/Pulmonology Which of the following pathophysiological processes is associated with chronic bronchitis? A. Destruction of the lung parenchyma B. Mucous gland enlargement and goblet cell hyperplasia C. Smooth muscle hypertrophy in the large airways D. Increased mucus adhesion secondary to reduction in the salt and water content of the mucus
Explanations (u) A. Destruction of the gas-exchanging structures in the lung is characteristic of emphysema. (c) B. Chronic bronchitis results from the enlargement of mucous glands and goblet cell hypertrophy in the large airways. (u) C. There may be smooth muscle hypertrophy in chronic bronchitis but it is not to the extent as found in asthma and is not an underlying factor in the pathology of chronic bronchitis. (u) D. Abnormal absorption of sodium and a reduced rate of chloride secretion in cystic fibrosis leads to thickening of the mucus and increase in adhesion of the mucus.
Clinical Therapeutics/Pulmonology A 36 year-old male developed a sore throat and was treated with IM penicillin. Within 20 minutes, he felt faint, became dyspneic, and had diarrhea. Upon entry to the emergency department, he was pale and apprehensive. He had a thready pulse, and systolic blood pressure was 40 mmHg. Which of the following is the most appropriate initial agent to use? Answers A. Dopamine B. Epinephrine C. Hydrocortisone D. Diphenhydramine
Explanations (u) A. Dopamine is not indicated in the treatment of allergic reactions. (c) B. Epinephrine is the drug of first choice for emergency use and should be given as soon as anaphylactic shock is suspected or diagnosed. (u) C. Hydrocortisone should be given as an adjunct to epinephrine, but it is not the drug of first choice. (u) D. Diphenhydramine should be given as an adjunct to treatment, but it is not the drug of first choice.
History & Physical/Pulmonology Which of the following is a common symptom associated with laryngotracheobronchitis (viral croup)? Answers A. drooling B. high fever C. "hot potato" voice D. barking cough
Explanations (u) A. Drooling and a "hot potato" voice are seen with epiglottitis, not viral croup. (u) B. Fever is usually absent or low grade in patients with viral croup. (u) C. See A for explanation. (c) D. Viral croup is characterized by history of an upper respiratory tract symptoms followed by onset of a barking cough and stridor.
History & Physical/Pulmonology Which of the following clinical manifestations is most commonly seen in viral croup? A. drooling B. wheezing C. sputum production D. inspiratory stridor
Explanations (u) A. Drooling is common in epiglottitis not viral croup. (u) B. Wheezing is noted in asthma. (u) C. Sputum production is noted in bacterial infections. (c) D. Viral croup typically presents with a barking cough and stridor.
History & Physical/Pulmonology The finding of egophony is most consistent with A. emphysema. B. atelectasis. C. pneumothorax. D. lobar pneumonia.
Explanations (u) A. Emphysema presents with diminished or absent breath sounds and hyperresonance to percussion without egophony. (u) B. Atelectasis most commonly has decreased breath sounds and dullness to percussion without egophony. (u) C. Pneumothorax presents with absent breath sounds, tactile fremitus, and resonance to percussion without egophony. (c) D. Egophony occurs with consolidation caused by lobar pneumonia.
Health Maintenance/Pulmonology A 33 year-old HIV-positive woman develops an 8mm area of induration following the administration of a purified protein derivative (PPD) test. Her chest radiograph shows no evidence of active tuberculosis (TB) infection. Which of the following is the most appropriate clinical intervention? A. Four-drug regimen for 4 months B. Isoniazid with Rifampin C. Observation only D. Repeat PPD and chest radiograph in 3 months
Explanations (u) A. Greater than 5 mm of induration is positive in an HIV-infected patient. A positive PPD and negative chest film is considered latent TB infection and, while requiring treatment, does not require the full four-drug regimen. (c) B. Isoniazid with Rifampin is recommended in HIV positive patients with a positive PPD and a negative chest x- ray. (u) C. Latent TB infection is associated with a risk of progression to tuberculosis and observation alone is inadequate. (u) D. Repeat screening is not helpful since the diagnosis of latent TB infection has already been established.
Scientific Concepts/Pulmonology Which of the following conditions will produce a transudative pleural effusion? A. Kaposi's sarcoma B. Pneumonia C. Cirrhosis D. Mesothelioma
Explanations (u) A. Kaposi's sarcoma, pneumonia, or mesothelioma will produce a transudative pleural effusion. (u) B. See A for explanation. (c) C. Transudative pleural effusions result from alteration in the formation of pleural fluid, the absorption of pleural fluid, or both, by systemic factors. Local factors affecting pleural fluid absorption and/or formation produce exudative pleural effusions. (u) D. See A for explanation.
Clinical Intervention/Pulmonology A 62 year-old male presents with a history of dyspnea on exertion and chronic cough worse with arising in the mornings. He has a 40-year-pack history of cigarette use. On examination there is increased AP diameter and decreased breath sounds with a prolonged expiratory phase. Pulse oximetry reveals an oxygen saturation of 93% on room air. In addition to smoking cessation, which of the following is an appropriate intervention at this time? Answers A. Home oxygen therapy B. Maintenance oral steroids C. Prophylactic antibiotic therapy D. Recommend influenza and pneumococcal vaccines
Explanations (u) A. Home oxygen therapy is indicated in COPD patients with an oxygen saturation < or equal to 88% or a pO2 < or equal to 55 mm Hg taken at rest breathing room air. (u) B. While oral steroids may be utilized in treatment of COPD, they are usually reserved for end stage disease due to the multiple systemic side effects of prolonged use. In addition only about 10% of patients show any increase in FEV1 and there use should be reserved for patients who show a 20% or greater improvement in FEV1. (u) C. Use of antibiotics should be reserved for treatment of acute exacerbations of COPD, acute bronchitis or documented bacterial infections, not prophylaxis. (c) D. In addition to smoking cessation, patients may benefit from vaccination against both influenza and pneumococcal disease.
History & Physical/Pulmonology A 30 year-old male presents with sudden onset of chills, fever, chest pain and a cough productive of greenish-brown sputum. On examination his temperature is 102 degrees F. He appears acutely ill and his respirations are shallow. Chest x-ray demonstrates left lower lobe consolidation. Which of the following findings would most likely be present on examination of his left lower lung? A. Hyperresonance B. Vesicular breath sounds C. Increased tactile fremitus D. Wheezing
Explanations (u) A. Hyperresonance is an abnormal sound as a result of air trapping as in COPD. (u) B. Vesicular breath sounds are the description of normal lung sounds. (c) C. Increased tactile fremitus occurs in the presence of fluid or a lung consolidation such as lobar pneumonia. (u) D. Wheezing is a musical noise sounding like a squeak caused by high velocity air flow through a narrow or obstructed airway.
Diagnosis/Pulmonology Which of the following physical examination findings would be consistent with a pleural effusion? A. Hyperresonance to percussion B. Increased tactile fremitus C. Unilateral lag on chest expansion D. Egophony
Explanations (u) A. Hyperresonance to percussion would be suggestive of emphysema or pneumothorax. (u) B. Increased tactile fremitus would be consistent with a consolidation. (c) C. A lag on chest expansion may be seen in the presence of a pleural effusion. (u) D. The presence of egophony would be consistent with a consolidation.
Clinical Therapeutics/Pulmonology A 40 year-old alcoholic male presents with sudden onset of severe chills, fever, dyspnea and cough productive of red mucoid sticky sputum. He appears ill looking with cyanosis. Examination reveals vital signs: Temp - 102 degrees F; Pulse - 120 /minute and regular; 89 RR - 28/min; BP 90/62 mm Hg. Lungs reveal minimal rales and dullness in the right upper lobe with decreased breath sounds. Chest x-ray reveals right upper lobe consolidation with a bulging fissure. Gram stain reveals many white blood cells and many gram- negative rods. Which of the following is the most appropriate drug of choice? Answers A. Ticarcillin B. Cefotaxime C. Doxycycline D. Clindamycin
Explanations (u) A. Infections caused by Klebsiella organisms are resistant to both ticarcillin and ampicillin. (c) B. This patient most likely has pneumonia caused by Klebsiella. A third generation cephalosporin, such as cefotaxime, is the preferred antimicrobial therapy against Klebsiella pneumoniae. Alternative antibiotic choices may include a carbapenem, beta-lactam/beta- lactamase inhibitor or a fluoroquinolone. (u) C. See B for explanation. (u) D. See B for explanation.
Clinical Intervention/Pulmonology A 67 year-old man presents complaining of gradually worsening fatigue and shortness of breath. He is a previous smoker with an 80 pack-year smoking history. He denies chest pain, night sweats, or hemoptysis. On physical examination, you note a very thin male who appears older than his stated age. Lung and heart sounds are barely audible to auscultation. Which of the following interventions is likely to alter the disease course? A. Inhaled bronchodilator therapy B. Inhaled steroid therapy C. Home oxygen D. Theophylline
Explanations (u) A. Inhaled bronchodilators afford symptomatic relief for some patients with COPD but do not alter the disease course. (u) B. Inhaled steroid therapy may reduce the number and severity of COPD exacerbations but has not been shown to alter the disease course. (c) C. Home oxygen therapy has been shown to prolong life in patients with COPD and alter the natural history of the disease. (u) D. Theophylline is a third-line agent for treating COPD and will not alter the natural history of the disease
Clinical Intervention/Pulmonology An elderly patient with poorly-controlled Type 2 diabetes and renal disease develops a fever of 102°F orally, productive cough, and dyspnea. Physical examination demonstrates a respiratory rate of 32/min, labored breathing, and rales at the left base. Pulse oximetry is 90%. Which of the following is the next appropriate step in the management of this patient? A. Administer nebulized corticosteroids B. Admit to the hospital C. Oral antimicrobial therapy D. Endotracheal intubation
Explanations (u) A. Inhaled corticosteroids are not utilized in the management of community-acquired pneumonia. (c) B. Community acquired pneumonia is the most deadly infectious disease in the U.S. Important risk factors for increased morbidity and mortality include advanced age, alcoholism, comorbid medical conditions, altered mental status, respiratory rate greater than 30 breaths/min, hypotension, and a BUN greater than 30. (u) C. Due to the age of the patient, comorbid diseases, and current signs of respiratory distress, intravenous not oral antimicrobial therapy is indicated. (u) D. Endotracheal intubation is indicated for respiratory failure unresponsive to conservative management.
Scientific Concepts/Pulmonology Which of the following best describes the pathophysiology of emphysema? Answers A. Interstitial inflammation and fibrosis B. Alveoli enlargement and loss of septa C. Mucosal edema and inflammatory response D. Excessive mucus secretion and chronic cough
Explanations (u) A. Interstitial inflammation and fibrosis are seen with restrictive causes of lung disease, such as asbestosis. (c) B. Emphysema results from alveoli enlargement with loss of septal wall integrity without any evidence of fibrosis. (u) C. Mucosal edema and inflammatory response are seen with asthma. (u) D. Excessive mucus secretion and chronic cough are characteristic of chronic bronchitis.
Clinical Therapeutics/Pulmonology A 2 year-old presents to the emergency department in acute respiratory distress. The parents relate a history of a recent upper respiratory illness that was followed by a sudden onset of barking cough during the night, but this morning they noted increased difficulty breathing. The child is noted to have stridor at rest, but has no evidence of cyanosis. Which of the following is the most appropriate initial intervention? A. Intravenous antibiotics B. Endotracheal intubation C. Inhaled mucolytic agent D. Nebulized racemic epinephrine
Explanations (u) A. Laryngotracheobronchitis is caused by viruses, not bacteria, and therefore antibiotic therapy is not indicated. (u) B. If patients fail to respond to initial treatment and progress to impending respiratory failure, endotracheal intubation is then indicated. (u) C. Inhaled mucolytic agents are not indicated in the treatment of laryngotracheobronchitis. (c) D. This patient most likely has laryngotracheobronchitis (viral croup). Treatment with nebulized racemic epinephrine and glucocorticosteroids is indicated for patients with stridor at rest.
History & Physical/Pulmonology Which of the following is a common presenting clinical manifestation of a patient with interstitial lung disease? A. Early inspiratory crackles B. Progressive dyspnea on exertion C. Productive cough with copious sputum D. Decreased breath sounds with hyperresonant percussion
Explanations (u) A. Late, not early, inspiratory crackles are associated with interstitial lung disease. (c) B. Patients with interstitial lung disease commonly present with progressive dyspnea on exertion and a cough with minimal sputum production. (u) C. A productive cough of copious amounts of sputum is most typical of a patient with chronic bronchitis. (u) D. Physical examination findings of decreased breath sounds with hyperresonant percussion is consistent with a diagnosis of chronic obstructive lung disease.
Diagnostic Studies/Pulmonology Which of the following chest x-ray abnormalities would most likely be seen in a patient with hypersensitivity pneumonitis? A. Lobar consolidation B. Apical infiltration C. Granulomatous inflammation D. Diffuse nodular densities
Explanations (u) A. Lobar consolidation is seen in community-acquired pneumonia. (u) B. Apical infiltration is seen in tuberculosis. (u) C. Granulomatous inflammation is seen in sarcoidosis. (c) D. Diffuse nodular densities are seen in hypersensitivity pneumonitis.
Diagnosis/Pulmonology A 56 year-old female with a 35 pack year smoking history presents to the clinic with shortness of breath and cough. On examination, she is thin with no recent weight loss. She appears uncomfortable, breath sounds are diminished without adventitious sounds. Pulmonary function tests show a marked increase in total lung capacity (TLC) and a decreased FEV1. What is the most likely diagnosis for this patient? A. Persistent asthma B. Chronic obstructive pulmonary disease C. Idiopathic fibrosing interstitial pneumonia D. Sarcoidosis
Explanations (u) A. Lung function in asthma is evaluated by FEV1/FVC ratio with reduction noted with airflow obstruction. (c) B. Lung volume measurements in COPD reveal a marked increase in residual volume indicative of air trapping. (u) C. Pulmonary function testing in idiopathic fibrosing interstitial pneumonia shows a loss of lung volume with normal to increased airflow rates in interstitial lung disease. (u) D. Restrictive changes with decreased lung volumes and diffusing capacity are common in sarcoidosis.
Scientific Concepts/Pulmonology The most common pathogen identified in community acquired pneumonia (CAP) is A. Mycoplasma pneumoniae. B. Staphylococcus aureus. C. Legionella pneumophila. D. Streptococcus pneumoniae.
Explanations (u) A. M. pneumoniae, S. aureus, Legionella pneumophila are common pathogens for CAP, but do not account for a majority of the cases. (u) B. See A for explanation. (u) C. See A for explanation. (c) D. S. pneumoniae accounts for a majority of cases of community acquired pneumonia.
Diagnosis/Pulmonology A 79 year-old female presents with productive cough for 2 days. She has associated fever, chills and shortness of breath. On physical exam, RR 30, BP 90/60, T 101.3. There is no JVD. Lungs reveal crackles at the left lower lobe and decreased breath sounds with dullness to percussion. Heart exam reveals RRR with no S3 or S4. No edema is noted. On chest x-ray the patient has a left sided pleural effusion. Examination of the pleural fluid reveals a decreased glucose and an elevated pleural fluid LDH. Pleural fluid cytology reveals squamous epithelial cells. What is the most likely cause of the patient's effusion? A. malignancy B. bacterial pneumonia C. heart failure D. pulmonary embolus
Explanations (u) A. Malignancy is also a leading cause of an exudative pleural effusion, second to bacterial pneumonia. Cytology in this case was normal. (c) B. Bacterial pneumonia is the leading cause of an exudative pleural effusion. (u) C. Pleural effusions in heart failure are transudative, not exudative pleural effusions. (u) D. PE can be both an exudative and transudative classification of effusion, however, dyspnea is usually the most common symptom.
Clinical Intervention/Pulmonology A 36 year-old male who is hospitalized because of severe injuries from a motor vehicle accident develops rapid onset of profound dyspnea. Initial chest x-ray shows a normal heart size with diffuse bilateral infiltrates. Follow-up chest x- ray shows confluent bilateral infiltrates that spare the costophrenic angles. Which of the following is the best clinical intervention for this patient? A. Provide supplemental oxygen B. Insert chest tube C. Tracheal intubation D. Administer corticosteroids
Explanations (u) A. Marked hypoxemia is refractory to treatment with supplemental oxygen in ARDS. (u) B. Chest tube insertion is not indicated in a patient with ARDS. (c) C. Treatment of hypoxemia in acute respiratory distress syndrome (ARDS) usually requires tracheal intubation. (u) D. Routine use of corticosteroids is not recommended in the management of ARDS.
Clinical Therapeutics/Pulmonology A 25 year-old male with a history of asthma presents complaining of increasing episodes of evening and daytime symptoms. He is on a short acting inhaled beta agonist prn. He is presently using his short acting beta agonist on a daily basis. Which of the following is the most appropriate addition to this patient's regimen? A. methylxanthine oxidase inhibitor B. long acting beta agonist inhaler C. leukotriene inhibitor D. inhaled corticosteroid
Explanations (u) A. Methylxanthine oxidase inhibitor preparations may have beneficial effects in some patients, but their value is limited due to a narrow therapeutic window and modest efficacy. (u) B. Long acting beta agonist inhalers should not be used in place of anti-inflammatory therapy. (u) C. Leukotriene inhibitors are less desirable alternatives to inhaled corticosteroids. (c) D. According to the stepwise approach for managing asthma by the National Asthma Education and Prevention Program, inhaled corticosteroids are indicated for mild to moderate persistent asthma.
*Influenza* Oseltmavir S/E
GI - especially nausea
Diagnosis/Pulmonology A 15 year-old male was seen last week with complaints of sore throat, headache, and mild cough. A diagnosis of URI was made and supportive treatment was initiated. He returns today with complaints of worsening cough and increasing fatigue. At this time, chest x-ray reveals bilateral hilar infiltrates. A WBC count is normal and a cold hemagglutinin titer is elevated. The most likely diagnosis is A. tuberculosis. B. mycoplasma pneumonia. C. pneumococcal pneumonia. D. staphylococcal pneumonia.
Explanations (u) A. Most children with pulmonary tuberculosis are asymptomatic with few physical examination findings. The results of the diagnostic studies do not support tuberculosis as the most likely diagnosis. (c) B. The insidious onset of symptoms, the interstitial infiltrate on chest x-ray, and elevated cold hemagglutinin titer make this diagnosis the most likely. (u) C. The clinical presentation of bacterial pneumonias in children is variable, but usually involves fever of acute onset. The WBC count is also usually elevated, making this a less likely diagnosis. (u) D. See C for explanation.
Diagnosis/Pulmonology A 32 year-old African American female presents with complaints of a gradual worsening of exertional dyspnea associated with a mild dry cough. She has tried various cough preparations on her own without any significant relief. Her examination is essentially unremarkable. A chest x-ray reveals the presence of bilateral hilar adenopathy. Which of the following is the most likely diagnosis? Answers A. silicosis B. sarcoidosis C. tuberculosis D. mycoplasma pneumonia
Explanations (u) A. Most patients with silicosis are asymptomatic, but in late stages it may present with dyspnea. A chest x-ray finding highly suggestive of silicosis is the calcification of the periphery of the hilar lymph nodes ("eggshell" calcification). (c) B. Patients with sarcoidosis present with an insidious onset of dyspnea that may be associated with malaise and fever. Incidence is the highest in the African American population and females are affected more frequently than males. Typical chest x-ray findings include bilateral hilar and right paratracheal lymphadenopathy. (u) C. The most common pulmonary complaint of tuberculosis is chronic cough associated with fatigue, weight loss, fever and night sweats. While dyspnea may be present it is a sign of extensive disease. In addition to hilar lymphadenopathy on chest x-ray, primary tuberculosis would also reveal small homogeneous infiltrates and segmental atelectasis. (u) D. While development of mycoplasma pneumonia is gradual, symptoms commonly include not only cough and dyspnea, but also fever, headache and sore throat. On exam most patients will also have rales and wheezes. A chest x-ray reveals diffuse interstitial infiltrates.
Clinical Therapeutics/Pulmonology An elderly nursing home resident is admitted with methicillin-resistant Staphylococcus aureus pneumonia. Which of the following is the most appropriate treatment to initiate? A. Nafcillin B. Vancomycin C. Clindamycin D. Doxycycline
Explanations (u) A. Nafcillin is classified as a penicillinase-resistant penicillin, however would not be effective against methicillin- resistant strains of Staphylococcus aureus. (c) B. Vancomycin with or without the addition of gentamicin or rifampin and linezolid is the treatment of choice for methicillin-resistant Staphylococcus aureus. (u) C. Clindamycin is primarily utilized in treatment of severe anaerobic infections, but is not indicated for the treatment of methicillin-resistant Staphylococcus aureus. (u) D. Doxycycline is a tetracycline and is only utilized to treat respiratory infections that are susceptible to tetracycline, such as Mycoplasma or Chlamydial pneumonias.
Clinical Therapeutics/Pulmonology A patient taking bleomycin (Blenoxane) should be monitored for which of the following side effects? A.Optic neuritis B. Hyperuricemia C. Encephalopathy D. Pulmonary fibrosis
Explanations (u) A. Optic neuritis is a potential side effect of ethambutol, used in the treatment of tuberculosis. (u) B. Hyperuricemia or encephalopathy are not known side effects of bleomycin. (u) C. See B for explanation. (c) D. Pulmonary fibrosis and pulmonary infiltrates are known side effects of bleomycin.
Clinical Therapeutics/Pulmonology A 22 year-old female with a history of asthma presents with complaints of increasing "asthma" attacks. The patient states she has been well controlled on albuterol inhaler until one month ago. Since that time she notices that she has had to use her inhaler 3-4 times a week and also has had increasing nighttime use averaging about three episodes in the past month. Spirometry reveals > 85% predicted value. Which of the following is the most appropriate intervention at this time? Answers A. Oral prednisone B. Oral theophylline C. Salmeterol inhaler D. Beclomethasone inhaler
Explanations (u) A. Oral corticosteroids, such as prednisone, are added to therapy in severe persistent asthma. While a course of oral corticosteroids may be needed for mild exacerbations of asthma, they are not added until inhaled corticosteroids have failed to control the symptoms. (u) B. Due to its safety profile, oral theophylline is now considered a third or fourth line treatment option for asthma. (u) C. Long acting inhaled beta2-agonists, such as salmeterol, are not added to the treatment regimen until the symptoms indicate a moderate persistent asthma. Long acting inhaled beta2 - agonists should also not be used in place of inhaled steroids. (c) D. This patient has progressed to mild persistent asthma. In addition to her inhaled beta2- agonist (albuterol), she should be started on an anti-inflammatory agent. Inhaled corticosteroids, such as beclomethasone, are preferred for long-term control. Other options may include cromolyn or nedocromil.
History & Physical/Pulmonology A 69 year-old male presents with complaint of increasing dyspnea over the past 6-8 months. The patient denies cough, chest pain or smoking history. Physical examination reveals inspiratory crackles at the bases and clubbing of the nails. Chest x-ray reveals interstitial fibrosis of the lower lungs, thickened pleura and calcified pleural plaques of the lateral chest wall. Pulmonary function testing shows a restrictive pattern with a decreased diffusing capacity. What information is most likely noted in this patient's history? A. Coal mining B. Silica exposure C. Textile work D. Asbestos exposure
Explanations (u) A. Patients with coal miners pneumoconiosis are typically asymptomatic with unremarkable pulmonary function tests. CXR shows small opacities in the upper lungs. (u) B. Patients with a history of silica exposure are also asymptomaticandhaveunaffectedpulmonaryfunction tests. CXR shows small rounded opacities throughout the lung and calcified hilar lymph nodes. (u) C. Textile workers present with an asthma-like disorder with chest tightness, cough and dyspnea that is worse on the first day back to work and improves as the week goes on. (c) D. Asbestos exposure often presents years later with increasing dyspnea and interstitial fibrosis of the lower lungs, thickened pleura and calcified pleura plaques. They will have a restrictive pattern on PFT.
Health Maintenance/Pulmonology Which of the following is an indication for a pediatric patient to receive the 23-valent polysaccharide vaccine (Pneumovax)? A. Children at any age with a history of asthma B. All children at 2,4,6 and 12-18 months of age C. All children at 12-23 months of age in a two dose series D. Children age 24-59 months at high risk for invasive pneumococcal disease
Explanations (u) A. Pediatric patients with cystic fibrosis, not asthma, are included in the indications for vaccination with Pneumovax, however they must be at least 2 years old. (u) B. The 7-valent pneumococcal conjugate vaccine (Prevnar) is currently recommended to be given to children under the age of two on the schedule outlined. (u) C. While a two dose series is recommended for appropriate pediatric patients that receive Pneumovax, the recommended timing between doses is 3-5 years. (c) D. Pneumovax is licensed for use in children over the age of 23 months and is indicated for all pediatric patients at increased risk for pneumococcal disease.
Scientific Concepts/Pulmonology Which of the following mechanisms leads to a primary pneumothorax? Answers A. Penetrating or blunt trauma forces B. Underlying lung cancer C. Pressure of air in the pleural space exceeds room air pressure D. Rupture of subpleural apical blebs due to high negative intrapleural pressures
Explanations (u) A. Penetrating or blunt trauma force is responsible for a traumatic pneumothorax. (u) B. A pneumothorax that results from an underlying lung disease is classified as a secondary pneumothorax. (u) C. When pressure of air in the pleural space exceeds room air pressure, it leads to a tension pneumothorax. (c) D. A primary spontaneous pneumothorax is thought to result from a rupture of subpleural apical blebs secondary to high negative intrapleural pressures.
History & Physical/Pulmonology Which of the following physical exam findings is consistent with moderate emphysema? A. Increased tactile fremitus B. Dullness to percussion C. Distant heart sounds D. Deviated trachea
Explanations (u) A. Physical examination findings in emphysema include a midline trachea, diffuse hyperresonant to percussion, and decreased tactile fremitus. (u) B. See A for explanation. (c) C. Distant heart sounds are common in emphysema patients due to hyperinflation of the lungs. (u) D. See A for explanation.
History & Physical/Pulmonology Which of the following is a physical examination finding that is consistent with a diagnosis of lobar pneumonia? A. Resonant to percussion B. Late inspiratory crackles C. Decreased tactile fremitus D. Tracheal shift toward uninvolved side
Explanations (u) A. Physical examination findings that are consistent with a diagnosis of lobar pneumonia include dullness to percussion, increased tactile fremitus and a trachea that is midline. (c) B. Late inspiratory crackles are a physical examination finding that is consistent with lobar pneumonia. (u) C. See A for explanation. (u) D. See A for explanation.
Diagnostic Studies/Pulmonology A 64 year-old female with a 50 pack year smoking history, presents with worsening dyspnea on exertion, a persistent cough, and increasing oxygen requirement from 2 to 3 liters. She denies any cardiac history. What is the most likely chest x-ray finding in this patient? A. pulmonary vascular congestion B. left lower lobe infiltrate C. apical infiltrates D. hyperinflation with bullae
Explanations (u) A. Pulmonary vascular congestion represents congestive heart failure not COPD. (u) B. Left lower lobe infiltrate represent an infectious process, such as pneumonia. (u) C. Apical infiltrates represent an infectious process, such as tuberculosis. (c) D. Hyperinflation with bullae is a consistent finding in patients with emphysema, such as this patient.
Scientific Concepts/Pulmonology A 20 year-old male presents with 3 weeks of constitutional and upper respiratory symptoms, including malaise, sore throat, dry cough, and fever. Lung auscultation demonstrates diffuse crackles bilaterally. What is the most likely infectious agent involved? A. Respiratory syncytial virus B. Influenza virus C. Mycoplasma pneumoniae D. Streptococcus pneumoniae
Explanations (u) A. RSV is characterized by wheezing on auscultation and the course is typically 3-7 days. (u) B. Influenza pneumonia is characterized by a more precipitous onset and fulminant course. (c) C. The indolent course suggests an atypical pneumonia and Mycoplasma is the most common atypical agent. (u) D. Pneumococcal pneumonia is typically characterized by a more severe illness and more fulminant course.
Diagnostic Studies/Pulmonology What is the diagnostic modality of choice to diagnose cystic fibrosis (CF)? A. Chest radiograph B. Clinical features C. Sweat chloride concentration testing D. Genotyping
Explanations (u) A. Radiographic findings may suggest the diagnosis but are not specific. (u) B. While clinical features may suggest the need for testing they are not useful in confirming the diagnosis. (c) C. The standard for diagnosis is two positive sweat chloride concentration tests obtained on separate days or identification of CF mutations or an abnormal nasal potential difference measurement. (u) D. Genotyping screens for only a fraction of the known CF mutations.
Diagnostic Studies/Pulmonology A 56 year-old male with a 40 pack-year smoking history presents complaining of progressive shortness of breath. Spirometry reveals an FEV1 of 2 L (40% of predicted), an FVC of 4 L (80% of predicted) and an FEV1/FVC of 50%. These findings are most consistent with A. sarcoidosis. B. chronic bronchitis. C. interstitial lung disease. D. congestive heart failure.
Explanations (u) A. Sarcoidosis, interstitial lung disease and congestive heart failure most commonly produce a restrictive pattern on spirometry with a reduction in forced expiratory volume in one second (FEV1) and forced vital capacity (FVC) secondary to the decrease in total lung capacity (TLC), but the FEV1/FVC ratio is usually normal or increased, not decreased. (c) B. The reduced FEV1 and FEV1/FVC is characteristic of an obstructive pattern that is seen in chronic obstructive pulmonary diseases, such as chronic bronchitis. (u) C. See A for explanation. (u) D. See A for explanation.
Health Maintenance/Pulmonology You are seeing 62 year-old African American male for health maintenance. He is a former cigarette smoker with a 40 pack-year history. He quit smoking 10 years ago. He denies cough, hemoptysis, shortness of breath, chest pain, weight loss, or night sweats. What method of screening for lung cancer is appropriate in this patient? A. Chest radiograph B. Spiral CT of the chest C. Sputum cytology D. No screening is recommended
Explanations (u) A. Screening chest radiographs have not been shown to improve outcomes when used to screen asymptomatic patients. (u) B. While CT may yet prove valuable for screening, it is cost-prohibitive and has not yet been validated as a screening modality in asymptomatic patients. (u) C. Sputum cytology is not an effective screening tool. (c) D. No routine screening for lung cancer is recommended for asymptomatic smokers or former smokers.
Diagnostic Studies/Pulmonology You are evaluating a patient whom you suspect has asthma. You perform spirometry before and after administration of an inhaled short-acting bronchodilator. After administration of the bronchodilator, which of the following spirometry results would suggest reversibility? A. DecreaseinFEV1 B. IncreaseinFEV1 C. Decrease in FVC D. Increase in FVC
Explanations (u) A. See B for explanation. (c) B. In asthma, the airway obstruction should be at least partially relieved be a short-acting bronchodilator. This would be reflected in an increased forced expiratory volume in 1 second (FEV1). (u) C. The forced vital capacity (FVC) is not a function of obstruction and is generally normal in early mild asthma or lower than expected in severe or long-standing asthma. Either way, it is not expected to change with administration of a short-acting bronchodilator. (u) D. See C for explanation.
Diagnostic Studies/Pulmonology A 57 year-old man is being evaluated for shortness of breath. The following spirometric data are obtained: VC 4.90 L (predicted), 5.15 L (observed) 105% predicted FRC 3.99 L (predicted), 4.37 L (observed) 110% predicted RV 2.47 L (predicted), 3.17 L (observed) 128% predicted FEV1 3.50 L (predicted), 2.35 L (observed) 67% predicted These findings are consistent with which of the following? A. No demonstratable abnormality B. Restrictive lung disease C. Obstructive lung disease D. A ventilation/perfusion mismatch
Explanations (u) A. See C for explanation. (u) B. Restrictive lung disease would show decreased total lung capacity, vital capacity, and normal to increased FEV1. (c) C. Spirometry findings in obstructive lung disease typically show normal or increased total lung capacity, decreased vital capacity, prolonged FEV1, and increased residual volume. (u) D. A ventilation/perfusion scan would be abnormal with a pulmonary embolism.
Health Maintenance/Pulmonology A 65 year-old with COPD receiving their first pneumococcal conjugate vaccination should be revaccinated in A. 1 year. B. 3 years. C. 5 years. D. 10 years.
Explanations (u) A. See C for explanation. (u) B. See C for explanation. (c) C. A single revaccination for a person over the age of 65 is recommended if it has been more than 5 years since they received their first vaccination. (u) D. See C for explanation.
Clinical Therapeutics/Pulmonology Which of the following will result in decreased serum theophylline levels in a patient with COPD? A. Cimetidine B. Congestive heart failure C. Cigarette smoking D. Ciprofloxacin
Explanations (u) A. See C for explanation. (u) B. See C for explanation. (c) C. Cigarette smoking will increase the hepatic clearance of theophylline, resulting in decreased levels in the system. The use of cimetidine or ciprofloxacin or the presence of congestive heart failure will reduce hepatic clearance and causing an increase in theophylline serum levels. (u) D. See C for explanation.
Scientific Concepts/Pulmonology A 3 year-old male with cystic fibrosis develops pneumonia. Which of the following is the most likely etiology of the pneumonia? A. Escherichia coli B. Staphylococcus epidermidis C. Pseudomonas aueroginosa D. Streptococcus pneumoniae
Explanations (u) A. See C for explanation. (u) B. See C for explanation. (c) C. Initially in the first few months of life, respiratory infection is common with Staphylococcus aureus and Haemophilus influenzae, but after that Pseudomonas aueroginosa becomes the major causative organism for infections. (u) D. See C for explanation.
Clinical Therapeutics/Pulmonology What is the mechanism of action of salmeterol (Serevent) in the treatment of asthma? A. Anti-inflammatory B. Immunotherapy for specific allergens C. Relaxing of bronchial smooth muscle D. Reduction of leukotriene production
Explanations (u) A. See C for explanation. (u) B. See C for explanation. (c) C. The mechanism of action for salmeterol is the relaxation of bronchial smooth muscle. (u) D. See C for explanation.
Diagnostic Studies/Pulmonology A 2 year-old presents with sudden onset of cough and stridor. On examination the child is afebrile and appears non- toxic with a respiratory rate of 42 breaths per minute. What is the next step in the evaluation of this patient? A. Lateral soft tissue x-ray of the neck B. Indirect laryngoscopy C. Finger sweep D. Chest x-ray
Explanations (u) A. See D for explanation. (u) B. See D for explanation. (u) C. See D for explanation. (c) D. Chest x-ray should be done first when foreign body aspiration is suspected.
Scientific Concepts/Pulmonology An 18 month-old male presents with his parents who report symptoms of a barking cough and intermittent stridor that has worsened over the past 12 hours. They note improvement in symptoms when he was taken outdoors to the cool night air. Which of the following is the most likely organism causing this patient's symptoms? A. Rubeola virus B. Adenovirus C. Influenza virus D. Para-influenza virus
Explanations (u) A. See D for explanation. (u) B. See D for explanation. (u) C. See D for explanation. (c) D. Croup is most often caused by parainfluenza virus.
Clinical Therapeutics/Pulmonology Which of the following is the most appropriate therapeutic agent for acute influenza? A. azithromycin (Zithromax) B. acyclovir (Zovirax) C. tetracycline (Sumycin) D. zanamivir (Relenza)
Explanations (u) A. See D for explanation. (u) B. See D for explanation. (u) C. See D for explanation. (c) D. Zanamivir is an anti-viral agent that is active against the influenza virus.
Health Maintenance/Pulmonology Which of the following forms of lung cancer is associated with the poorest prognosis? A. squamous cell B. adenocarcinoma C. large cell D. small cell
Explanations (u) A. See D for explanation. (u) B. See D for explanation. (u) C. See d for explanation. (c) D. Small cell lung cancer is the most common type of lung cancer that is metastatic at the time of discovery, and therefore has the poorest prognosis.
Diagnostic Studies/Pulmonology An O2 saturation of 90% corresponds to what PO2 value? A. 90 mmHg B. 80 mmHg C. 70 mmHg D. 60 mmHg
Explanations (u) A. See D for explanation. (u) B. See D for explanation. (u) C. See D for explanation. (c) D. O2 sat values above 90% correspond with a PO2 >70 mmHg and values less than 94% represent hypoxemia. Less than 90% O2 sat warrants measurement of arterial blood gasses.
Diagnosis/Pulmonology A 32 week preterm infant has an APGAR score of 9 at 5 minutes. Thirty minutes after delivery, tachypnea, retractions, and expiratory grunting are noted. Cyanosis and dyspnea appear with little response to oxygen. Physical examination reveals poor air movement bilaterally. A chest x-ray reveals air bronchograms and a fine reticular granular pattern. Which of the following conditions should be suspected? A. Atelectasis B. Diaphragmatic hernia C. Respiratory distress syndrome D. Pneumothorax
Explanations (u) A. Small areas of atelectasis usually are asymptomatic. While larger areas may present with similar clinical findings, the chest x-ray findings are not consistent with atelectasis. (u) B. Chest x-ray in a patient with a diaphragmatic hernia would not show a fine reticular granular pattern. (c) C. Clinical findings of increasing cyanosis unresponsive to oxygen therapy and the characteristic x-ray findings are most consistent with respiratory distress syndrome. (u) D. Chest x-ray in a patient with a pneumothorax would not show a fine reticular granular pattern.
Diagnosis/Pulmonology A 53 year-old female status post abdominal hysterectomy 3 days ago suddenly develops pleuritic chest pain and dyspnea. On exam she is tachycardic and tachypneic with rales in the left lower lobe. A chest x-ray is unremarkable and an EKG reveals tachycardia. Which of the following is the most likely diagnosis? Answers A. atelectasis B. pneumothorax C. pulmonary embolism D. myocardial infarction
Explanations (u) A. Small atelectasis is commonly asymptomatic, while large atelectasis may produce signs of dyspnea and cough. Exam reveals absence of breath sounds in the area involved and dullness to percussion. A chest x-ray would reveal various findings dependent on the location of the atelectasis, but would not be normal. (u) B. While a pneumothorax commonly presents with pleuritic chest pain and dyspnea, exam would reveal the presence of diminished breath sounds and hyperresonance on the involved side. A chest x-ray would reveal presence of a pleural line on the expiratory chest x-ray. (c) C. Risk factors for pulmonary embolism include advanced age, surgery and prolonged bedrest. While the diagnosis of pulmonary embolism is difficult due to nonspecific clinical findings, the most common symptoms include pleuritic chest pain and dyspnea associated with tachypnea. Chest x-ray and EKG are usually normal. (u) D. While a myocardial infarction usually presents with dyspnea, the chest pain is not usually pleuritic in nature. An EKG would commonly reveal ST segment changes, which would be consistent with ischemia or infarct.
Diagnosis/Pulmonology A 53 year-old female status post abdominal hysterectomy 3 days ago suddenly develops pleuritic chest pain and dyspnea. On exam, she is tachycardic and tachypneic with rales in the left lower lobe. A chest x-ray is unremarkable and an EKG reveals sinus tachycardia. Which of the following is the most likely diagnosis? A. Atelectasis B. Pneumothorax C. Pulmonary embolism D. Myocardial infarction
Explanations (u) A. Small atelectasis is commonly asymptomatic, while large atelectasis may produce signs of dyspnea and cough. Exam reveals absence of breath sounds in the area involved and dullness to percussion. A chest x-ray would reveal various findings dependent on the location of the atelectasis, but would not be normal. (u) B. While a pneumothorax commonly presents with pleuritic chest pain and dyspnea, exam would reveal the presence of diminished breath sounds and hyperresonance on the involved side. A chest x-ray would reveal presence of a pleural line on the expiratory chest x-ray. (c) C. This patient's risk factors for pulmonary embolism include advanced age, surgery, and prolonged bedrest. While the diagnosis of pulmonary embolism is difficult to make due to nonspecific clinical findings, the most common symptoms include pleuritic chest pain and dyspnea associated with tachypnea. Chest x-ray and EKG are usually normal. (u) D. While a myocardial infarction usually presents with dyspnea, the chest pain is not usually pleuritic in nature. An EKG would commonly reveal ST segment changes which would be consistent with ischemia or infarct.
Clinical Intervention/Pulmonology A 62 year-old male presents with a right hilar mass. Needle-biopsy of the mass reveals the presence of small-cell carcinoma and a bone scan reveals the presence of scattered hot spots throughout the skeleton. Which of the following is the most appropriate treatment? A. Lobectomy B. Pneumonectomy C. Thoracic radiation therapy D. Combination chemotherapy
Explanations (u) A. Small-cell carcinoma of the lung is rarely treatable with surgical resection. Surgery may be indicated as part of the treatment protocol for small peripheral lesions without any evidence of metastasis. (u) B. See A for explanation. (u) C. While thoracic radiation therapy has been shown to be beneficial for patients with limited small-cell lung cancer, no benefit has been observed for patients with extensive disease defined as the presence of metastatic disease. (c) D. Combination chemotherapy is the treatment of choice for a patient with small-cell carcinoma of the lung.
Health Maintenance/Pulmonology A recent Haitian immigrant presents to the clinic for an employment physical examination before starting work at a local hospital. The patient has a history of receiving bacilli Calmette-Guerin (BCG) vaccination. Screening for tuberculosis for this employee should include which of the following tests? A. Sputum induction B. PPD skin test C. Chest x-ray D. No screening needed
Explanations (u) A. Sputum induction should not be used as a screening test for tuberculosis. (u) B. False-positive tuberculin skin test reactions can occur in persons previously vaccinated against M. tuberculosis with BCG. PPD should be avoided as a screening test in these patients. (c) C. Chest x-ray is the test of choice in patients where the PPD test is not indicated or in high-risk individuals. (u) D. This patient has recently emigrated from a possible endemic region and should be screened for tuberculosis.
Scientific Concepts/Pulmonology A 62 year-old male smoker presents to the clinic with the complaint of a chronic cough, hemoptysis, and weight loss. Chest CT shows a mass obstructing the bronchus with hilar and mediastinal lymph node abnormalities. Bronchoscopy with biopsy is performed. On reviewing pathology results you explain to the patient that his type of lung cancer is prone to early hematogenous spread, is rarely amenable to surgical resection and has a very aggressive course. What type of lung cancer is most likely in this patient? A. Squamous cell B. Small cell C. Large cell D. Adenocarcinoma
Explanations (u) A. Squamous cell carcinoma, large cell carcinoma, and adenocarcinoma spread more slowly and have the possibility of cure in early stages following resection and chemotherapy. (c) B. Small cell lung cancer is very aggressive with a median survival (untreated) of 6-18 weeks. -=-80-0 (u) C. See A for explanation. (u) D. See A for explanation.
Clinical Intervention/Pulmonology A 75 year-old man with a long history of COPD presents with acute onset of worsening dyspnea, increased productive cough, and marked agitation. While in the emergency department he becomes lethargic and obtunded. His ABG's reveal a PaO2 40 mmHg, PaCO2 65 mmHg, and arterial pH 7.25. Which of the following is the most appropriate management at this point? A. oxygen supplementation with a 100% non-rebreather mask B. noninvasive positive pressure ventilation (NIPPV) C. endotracheal intubation and mechanical ventilation D. emergency tracheostomy
Explanations (u) A. Supplemental oxygen and positive pressure ventilation are inadequate for patients with overt respiratory failure. (u) B. See A for explanation. (c) C. This patient is in severe respiratory arrest with markedly impaired mental status; conventional mechanical ventilation is required. (h) D. Tracheostomy is indicated for an obstructed airway.
Clinical Intervention/Pulmonology A 68 year-old male with history of COPD is brought to the emergency department following a motor vehicle collision. On physical examination there is evidence of head trauma. The left side of the chest wall appears to move inward with inspiration and outward with expiration. A chest x-ray reveals multiple rib fractures on the left. Which of the following is the most appropriate intervention? A. Surgical fixation of the fractured ribs B. Application of elastic binders and adhesive tape C. Endotracheal intubation and mechanical ventilation D. Chest physiotherapy that encourages frequent coughing
Explanations (u) A. Surgical fixation of flail chest is less reliable than positive-pressure ventilation and is performed only rarely in the United States. (u) B. While application of elastic binders and adhesive tape was historically utilized to stabile the chest, this intervention has been found to decrease chest expansion and worsen lung atelectasis. (c) C. Indications for early endotracheal intubation and mechanical ventilation in treatment of flail chest include patients that are over the age of 65, have comorbid lung disease and associated severe head trauma. Other indications include shock, three or more associated injuries and fracture of eight or more ribs. (u) D. Conservative treatment for mild to moderate flail chest includes analgesic relief of pain, chest physiotherapy that encourages frequent coughing and restriction of fluids to prevent fluid overload, however this patient needs early ventilatory support.
Clinical Therapeutics/Pulmonology A 26 year-old man is stung by a bee, and shortly thereafter, a wheal develops at the site of the sting. He soon feels flushed and develops hives, rhinorrhea, and tightness in the chest. He is seen in the urgent care center. Immediate therapy should be to A. transfer him to a local hospital emergency department. B. apply a cold compress to site of the sting. C. administer subcutaneous epinephrine. D. administer oral albuterol.
Explanations (u) A. Systemic (anaphylactic) reactions can rapidly become life-threatening. Delay in treatment may cause death. (u) B. This is only supportive local therapy and does not address the need to treat the systemic reaction present. (c) C. Epinephrine hydrochloride 1:1000, 0.2 to 0.5 mL subcutaneously is indicated for the initial treatment of this systemic reaction. Additional injections may be given every 20 to 30 minutes if needed. (u) D. Albuterol is indicated in the presence of bronchospasm (suggested by the presence of chest tightness), but would be delivered by an aerosol, not an oral, route.
Clinical Therapeutics/Pulmonology An immunocompromised patient presents with signs and symptoms consistent with Legionella pneumophila who has not responded to initial antibiotic therapy with a macrolide. Which of the following should be added? A. Clarithromycin (Biaxin) B. Rifampin (Rifadin) C. Levofloxacin (Levaquin) D. Amoxicillin-clavulanate (Augmentin)
Explanations (u) A. The macrolides (Clarithromycin) and fluoroquinolones (Levofloxacin) should be used for initial treatment, but not for adding to failed treatments when a macrolide was already used. (c) B. Rifampin should be used as an adjunct in patients with either a macrolide or quinolone antibiotic, who have failed therapy, are immunocompromised or have severe illness. (u) C. See A for explanation. (u) D. Legionella pneumophila does not respond to Beta-lactam antibiotics.
Diagnostic Studies/Pulmonology Which of the following findings confirm the adequacy of a sputum specimen for Gram stain and culture? A. Decreased red blood cells B. Decreased bronchial epithelial cells C. Increased Gram positive cocci D. Increased polymorphonuclear leukocytes
Explanations (u) A. The presence or absence of red blood cells is not a criterion for determining the adequacy of a sputum sample. (u) B. The presence of bronchial epithelial cells confirms the sample came from the lower respiratory tract. (u) C. The presence of Gram positive cocci has no bearing on the adequacy of the sputum sample. (c) D. The presence of increased polymorphonuclear leukocytes and the absence of squamous epithelial cells are the criteria utilized to evaluate the adequacy of a sputum sample.
Health Maintenance/Pulmonology Which of the following is an independent risk factor for development of a mesothelioma? A. Cigarette smoking B. Asbestos exposure C. Radon gas exposure D. Chronic obstructive lung disease
Explanations (u) A. There has not been any evidence of association between cigarette smoking and the development of mesothelioma. (c) B. Studies confirm the association of asbestos exposure to the development of mesothelioma. (u) C. After cigarette smoking, radon gas is the second most common risk factor for development of bronchogenic lung cancer, not mesothelioma. (u) D. Chronic obstructive lung disease is associated with an increased risk of bronchogenic lung cancer, not mesothelioma.
Health Maintenance/Pulmonology Which of the following is an independent risk factor for development of a mesothelioma? Answers A. Cigarette smoking B. Asbestos exposure C. Radon gas exposure D. Chronic obstructive lung disease
Explanations (u) A. There has not been any evidence of association between cigarette smoking and the development of mesothelioma. (c) B. Studies confirm the association of asbestos exposure to the development of mesothelioma. (u) C. After cigarette smoking, radon gas is the second most common risk factor for development of bronchogenic lung cancer, not mesothelioma. (u) D. Chronic obstructive lung disease is associated with an increased risk of bronchogenic lung cancer, not mesothelioma.
Diagnostic Studies/Pulmonology A 26 year-old man presents to the emergency room complaining of shortness of breath, palpitations, and tingling of the lips and fingers. He appears anxious and describes a sensation of impending doom. His ECG and plain chest radiograph are normal. Which of the following arterial blood gas findings would you expect in this patient? A. pH 7.32 pCO2 49 mm Hg bicarbonate 24 mEq/L B. pH 7.40 pCO2 40 mm Hg bicarbonate 25 mEq/L C. pH 7.50 pCO2 23 mm Hg bicarbonate 21 mEq/L D. pH 7.52 pCO2 40 mm Hg bicarbonate 44 mEq/L
Explanations (u) A. These values are consistent with a respiratory acidosis. (u) B. These values fall within the normal range. (c) C. This patient is suffering an acute anxiety attack and resultant hyperventilation and respiratory alkalosis. Labs show alkaline pH, depressed pCO2 and low to normal bicarbonate. (u) D. These values suggest metabolic alkalosis.
Diagnostic Studies/Pulmonology A 73 year-old obese female with a 20 pack year smoking history presents complaining of chronic productive cough. She states that it has been occurring over the past 3 years more frequently November through February. Which of the following pulmonary function test values would you expect to find decreased? A. tidal volume B. forced expiratory volume in 1 second/forced vital capacity C. residual volume D. total lung capacity
Explanations (u) A. Tidal volume is usually unchanged, residual volume and total lung capacity are decreased with a restrictive disease pattern. This scenario presents with bronchitis which is an obstructive disease. (c) B. Forced expiratory volume in 1 second/forced vital capacity is decreased in obstructive lung diseases such as bronchitis. (u) C. See A for explanation. (u) D. See A for explanation.
Diagnostic Studies/Pulmonology A post-op patient has signs and symptoms highly suggestive of a pulmonary embolism. The results of the CT scan of the lung is nondiagnostic. What is the most appropriate next step in the evaluation? A. Ventilation perfusion (V/Q) scan B. Ultrasound of the legs C. Echocardiography D. D-dimer
Explanations (u) A. Ventilation perfusion scans are performed prior to the CT scan of the chest and would not likely add additional information to this clinical scenario. (c) B. In a patient with a high likelihood of pulmonary embolism or an inpatient, as in this case, ultrasound of the legs would be the next diagnostic step after a nondiagnostic CT. (u) C. Although echocardiography may show right ventricular free wall hypokinesis with normal motion of the apex suggestive of pulmonary embolism, more than 50% of patients with a pulmonary embolism will have normal echocardiography. Echocardiography is not used in the diagnosis on inpatients.; (u) D. In a post-op patient, a d-dimer will be positive regardless of the presence or absence of a pulmonary embolism.
Clinical Therapeutics/Pulmonology A 22 month-old male infant presents with one day of barking cough preceded by three days of cold symptoms. On physical examination, his axillary temperature is 100.4°F and he has no stridor at rest. Inspiratory stridor is evident when he becomes agitated during the examination. There are no signs of respiratory distress or cyanosis. Which of the following is the most appropriate treatment for this patient? A. Nebulized albuterol B. Nebulized epinephrine C. Oral amoxicillin D. Oral dexamethasone
Explanations (u) A. Viral croup is an upper airway disease and there is no role for bronchodilator therapy. (u) B. Nebulized racemic epinephrine is only indicated in the treatment of croup in cases of moderate to severe rest stridor, respiratory distress, or hypoxia. (u) C. Croup is almost always a viral illness and antibiotics have no role in the treatment of this condition. (c) D. Corticosteroids are beneficial in the treatment of croup. Intramuscular administration has shown no benefit over oral administration.
History & Physical/Pulmonology Which of the following is the most likely presentation of an acute pulmonary embolism (PE) in a patient without preexisting cardiac or pulmonary disease? A. Anginal chest pain B. Cough C. T achypnea D. Palpitations
Explanations (u) A. While it is quite common for PE to present with pleuritic chest pain, angina-like pain is only rarely reported. (u) B. Cough is reported in roughly one-third of patients with PE. But is not the most common presentation. (c) C. Tachypnea is the most common symptom in acute PE. (u) D. Palpitations are uncommonly reported in acute PE.
cientific Concepts/Pulmonology Which of the following has been found to be an important reservoir in the transmission of pertussis? Answers A. Mosquitoes B. Adult humans C. Dogs and cats D. White-tailed deer
Explanations (u) A. While mosquitoes have been implemented in the spread of several infectious diseases, pertussis is not one of them. (c) B. Since neither the vaccine nor the disease itself provides lasting immunity against pertussis, adult humans are an important reservoir in the transmission of pertussis. (u) C. Contact with dogs and cats has lead to the development of upper and lower respiratory infections that are caused by Bordetella bronchiseptica, but not pertussis. (u) D. White-tailed deer are part of the transmission cycle for Lyme disease, not pertussis.
Health Maintenance/Pulmonology A pediatric patient presents with a history of multiple recurrent respiratory infections associated with failure to thrive. A sweat chloride test is elevated. Which of the following is a common cause of death in patients with this condition? A. Diabetic ketoacidosis B. Pulmonary infection C. Intestinal obstruction D. Acute respiratory failure
Explanations (u) A. While patients with cystic fibrosis most likely will eventually develop insulin-dependent diabetes mellitus, diabetic ketoacidosis is not a common cause of death. (c) B. This patient has cystic fibrosis. The most common causes of death include pulmonary complications, such as infections, and terminal chronic respiratory failure associated with cor pulmonale. (u) C. While intestinal obstruction may occur in patients with cystic fibrosis, it is not a common cause of death. (u) D. See B for explanation.
Diagnosis/Pulmonology You are called to the nursery to see a male infant, born by uncomplicated vaginal delivery. He weighs 2,600 grams and has one deep crease on the anterior third of each foot. Respirations are 88 breaths/minute with expiratory grunting and intercostals retractions. He is cyanotic on room air and becomes pink when placed in 60% oxygen. Chest x-ray shows atelectasis with air bronchograms. Which of the following is the most likely diagnosis? Answers A. neonatal pneumonia B. congenital heart disease C. hyaline membrane disease D. chronic lung disease of prematurity
Explanations (u) A. While tachypnea, grunting, retractions and cyanosis may be signs of neonatal pneumonia, they are primarily late findings of progressive respiratory distress and would not be seen immediately at the time of delivery. A chest x-ray in pneumonia would also most commonly reveal an infiltrate or effusion. (u) B. While congenital heart disease may present with cyanosis, the chest x-ray will reveal a cardiac abnormality, such as cardiomegaly. (c) C. Hyaline membrane disease is the most common cause of respiratory distress in the premature infant. The infant typically presents with tachypnea, cyanosis and expiratory grunting. A chest x-ray reveals hypoexpansion and air bronchograms. (u) D. Chronic lung disease of prematurity is a complication in about 20% of infants with hyaline membrane disease. It is defined as respiratory symptoms, oxygen requirement and chest x-ray abnormalities at 1 month of age so it cannot be diagnosed at this time in this newborn.
Diagnosis/Pulmonology A 45 year-old male presents with complaints of a chronic cough productive of mucopurulent sputum. The cough has been present for the past 3 years, but he attributed it to a "smoker's cough". He has been coughing up a lot of sputum lasting all winter long for the past 2 years. He denies any hemoptysis, weight loss or chest pain. Physical examination reveals a moderately obese male in no acute respiratory distress. Lung fields reveal presence of scattered rhonchi and wheezes. There is 1+ peripheral edema. Which of the following is the most likely diagnosis? A. Lung cancer B. Bronchiectasis C. Chronic bronchitis D. Interstitial lung disease
Explanations (u) A. While the respiratory complaints of lung cancer are associated with the location and type of primary tumor, anorexia and weight loss is seen in the majority of patients. Patients will also usually have a new cough or a change in a chronic cough and may complain of hemoptysis and nonspecific chest pain. (u) B. While bronchiectasis presents with a chronic cough productive of copious amounts of purulent sputum, these patients most commonly also have associated complaints of hemoptysis, weight loss and pleuritic chest pain. Examination of the lungs reveals persistent crackles at the bases. (c) C. This patient most likely has chronic bronchitis which is defined as sputum production and cough for at least 3 months of the year for 2 consecutive years which is primarily caused by cigarette smoking. (u) D. Interstitial lung disease is characterized by progressive exertional dyspnea and cough, however sputum production is minimal and the examination of the lungs reveals fine, late inspiratory crackles at the bases in the majority of patients.
Diagnosis/Pulmonology A 60 year-old patient returned from the recovery room to the floor following a subtotal gastrectomy. At 3 AM the next morning, the patient's temperature is 102° F (39° C) and pulse is 112/min. Which of the following is the most likely cause? A. wound infection B. atelectasis C. phlebitis D. shock
Explanations (u) A. Wound infection does not present this early. (c) B. Atelectasis is the most common pulmonary complication, affecting 25% of patients with abdominal surgery. It is more common in elderly and overweight patients and occurs within the first 12 to 24 hours postoperatively. (u) C. Phlebitis occurs more commonly after the second postoperative day. (u) D. In shock, the pulse is usually thready and the temperature is not elevated.
MC site of carcinoid tumors? 2nd MC?
GI tract; lung
*PNA* *Viral* Treat CMV with
Ganciclovir Valganciclovir
*PNA* Pneumocystis jirovecii (PCP) CT scan results
Ground glass appearance
*Bronchiectasis* MCC of bronchiectasis if NOT secondary to CF?
H. influenza
PNA w/ Smokers, COPD
H. influenza
*PNA* 2nd MCC of CAP
H. influenzae
*PNA* Clinical S/Sx: pharyngitis, bullous myrinigitis
H. influenzae
Question: What is the most common cause of pleural effusion in the U.S.?
Heart failure. Pleural Effusion Transudate: CHF (most common) Exudate: infection > malignancy, PE ↓ Breath sounds + dull percussion + ↓ tactile fremitus CXR: blunting of the costophrenic angle
One Step Further Question: Is the lactate dehydrogenase level high or low in an exudate pleural fluid?
High.
Chest X-ray shows RUL abscess. Most likely organism?
Klebsiella (often associated with aspiration)
What is the Dx? Previously healthy patient with abrupt onset of fever headache malaise occurring in the winter months.
Influenza
*PNA* Severe illness in ETOHics, debilitated, chronic illness, aspirators. Associated w cavitary lesions.
Klebsiella pneumoniae
Treatment for immunocompromised patient with fungal pneumonia?
Itraconazole or fluconazole
What pneumonia is associated with air conditioning vents and spas?
Legionella
An older patient presents with pneumonia and diarrhea. What is the most likely diagnosis?
Legionella pneumonia
Organism causing pneumonia spread by contaminated water?
Legionella pneumonia
*PNA* Bug also causes GI Sx like N/V/D, increased LFTs and hyponatremia
Legionella pneumophilia
Question: The differential diagnosis of interstitial lung disease (ILD) includes which three major conditions?
Lung malignancy, lung infection and congestive heart failure. Idiopathic Pulmonary Fibrosis Patient will be a male With a history of smoking Complaining of chronic dry cough and dyspnea Labs will show restrictive characteristics - decrease FVC and FEV1 but a near normal FEV1/FVC ratio CXR will show "honeycombing" Treatment is O2, pulmonary rehabilitation
*PNA* Common bug in *immunocompromised; HIV*, neutropenic, s/p transplant. Also common in those with structural abnormalities like CF and bronchietasis
Pseudomonas aeruginosa *Hospital acquired
Pneumonia often related to post intubation, ventilator or hospital?
Pseudomonas pneumonia
*Bronchiolitis* Single best predictor of disease in children
Pulse ox; under 96% admit
*CF* PFTs
Obstructive, irreversible
*ARDS* Criteria? (3)
Pa/FIO2 ratio <300 (or 200?) not responsive to O2 Bilateral pulmonary infiltrates PCWP ≤ 18 mmHg
One Step Further Question: What should be co-administered with isoniazid?
Pyridoxine (vitamin B6) to prevent peripheral neuropathy. Latent Tuberculosis Positive PPD criteria: 15 mm: no ↑ risk 10 mm: high-risk, homeless, health-care workers, IVDU, foreign born 5 mm: immunosuppressed, recent TB contact, abnormal CXR, steroid use Rx: INH for 9 months
*Asthma* Best and most objective way to assess asthma exacerbation severity and patient response in the ED.
Peak expiratory flow rate >15% from initial attempt = responsive to Tx
Rapid Review Pneumocystis Pneumonia (PCP)
Pneumocystis Pneumonia (PCP) Patient with a history of HIV Complaining of gradual onset of non-productive cough Labs will show CD4 < 200, increased LDH CXR will show bilateral infiltrates (bat wing pattern) Most commonly caused by Pneumocystis jirovecii Treatment is TMP-SMX
An HIV patient with ground glass appearance on CXR should make you think of what diagnosis?
Pneumocystis jiroveci pneumonia (PJP used to be known as PCP)
*PNA* Fungal pneumonia that occurs in a *compromised host* (HIV). Symptoms include fatigue, dry cough, DOE, and *oxygen desaturation with ambulation*. Labs show *increased LDH*.
Pneumocystis jirovecii (PCP)
Fever, cough, sputum. Crackles, decreased breath sounds, dullness to percussion, +egophony, pectoriloquy. CXR - infiltrates or consolidation
Pneumonia
Productive cough, pleuritic chest pain, fever. Lobar consolidation or bilateral interstitial infiltrates.
Pneumonia
Productive cough, pleuritic chest pain, fever. lobar consolidation = bacteria. bilateral interstitial infiltrates = viral/pneumocystis
Pneumonia
Question: What qualifies as a complicated pneumonia?
Pneumonia involving a parapneumonic effusion or empyema, necrotizing pneumonia, lung abscess, or pneumatocele. Community-acquired Pneumonia (CAP) Patient will be complaining of sudden onset of cough, fatigue, and fever PE will show rales with auscultation of lung fields Most commonly caused by Streptococcus pneumoniae Treatment: macrolides 1st line for children > 5 yrs
If you hear crackles in the lung fields, list three pulmonary issues that should be in your differential.
Pneumonia, fibrosis, CHF, Bronchitis
Non caseating granulomas, erythema nodosum, bilateral hilar adenopathy, elevated ACE
Sarcoidosis
A 54-year-old male with moderate COPD presents to the emergency department with a 4-day history of progressive shortness of breath. He is noted to be lethargic and in respiratory distress with accessory muscle use and a respiratory rate of 36. He has poor air movement without any wheezing. The patient is intubated and placed on mechanical ventilator support with the following settings: Mode: Assist control ventilator rate (set): 14 Patient spontaneous rate: 22 Tidal volume: 450 mL F i O 2 : 100% PEEP: 5 cm An arterial blood gas is obtained: pH: 7.22 pCO 2 : 64 P a O 2 : 240 HCO 3 : 22 Which of the following would help improve the pH? a) Decrease the F i O 2 to 50%. b) Deeper sedation to decrease respiratory rate to 14. c) Increase the tidal volume to 500 mL. d) Increase the set rate on the ventilator to 22 breaths per minute. e) Start continuous infusion of bicarbonate mixed in dextrose 5%.
Pulmonary Medicine Answer C. Increase the tidal volume to 500 mL. Explanation This patient has acute hypercarbic respiratory failure with respiratory acidosis, which necessitated mechanical ventilatory support. In order to increase the pH, the respiratory acidosis must be improved. This is accomplished by increasing the overall minute ventilation (MV). MV is the product of the respiratory rate and the tidal volume. The patient is already breathing at a rate of 22. Increasing the rate on the ventilator is unlikely to have any overall change in the minute ventilation, but increasing the tidal volume while maintaining the same respiratory rate will increase the MV. The patient is indeed hyperoxygenated and should have the F i O 2 reduced to avoid oxygen toxicity, but this will do little to improve the respiratory acidosis. Deeper sedation may actually reduce the respiratory rate and thus the MV, causing a reduction in the pH. Bicarbonate infusion has no role in a respiratory acidosis but may be used in cases of severe metabolic acidosis.
A 25-year-old complains of "shortness of breath" that began at a social gathering. He complains of paresthesias of his hands and feet and developed left-sided pleuritic chest pain. PMH: Major depression On exam he is tachypneic. ABG: pH 7.47, pCO 2 32, pO 2 92 Which of the following is the most likely diagnosis? a) Anaphylactoid reaction b) Asthma exacerbation c) Panic attack d) Hypersensitivity pneumonitis e) Pulmonary embolism
Pulmonary Medicine Answer C. Panic attack Explanation This healthy patient develops sudden dyspnea associated with paresthesias suggesting a respiratory alkalosis. He is not hypoxic, however. The best way to determine whether severe disease, such as a pulmonary embolism, is occurring in the alveolar-capillary unit is to calculate the A-a gradient. Recall this calculation: P A O 2 - P a O 2 , when P A O 2 = 150 - (PCO 2 x 1.25) Normal = 5-20 mmHg Therefore, in this case: P A O 2 = 150 - (32 x 1.25) = 150 - 40 = 110 A-a gradient = 110 - 92 = 18 = NORMAL! This normal gradient indicates no problem at the alveolar-capillary unit (such as asthma or pulmonary embolism). The most likely explanation is supratentorial hyperventilation. Anaphylaxis has a very pronounced clinical presentation that is not described in this case.
A 67-year-old man with a 100-pack-year history of smoking (2 ppd for 50 years) is essentially dragged in by his wife, who says that he has withered away to nothing and has been acting very confused lately. He says he is fine but keeps calling you his grandchild. You don't get much more information out of him. PAST MEDICAL HISTORY: Prostatectomy 5 years ago HTN for 20 years on an ACE inhibitor SOCIAL HISTORY: Retired used car salesman Lives with his wife of 50 years Doesn't drink FAMILY HISTORY: Father died at age 75 of lung cancer Mother died at age 74 of lung cancer Brother died at age 74 of lung cancer Sister died at age 74 of lung cancer REVIEW OF SYSTEMS: No fever or chills Has had night sweats on occasion 30-lb weight loss in last 6 months No appetite Coughed up blood once last week (about a teaspoon, according to wife) PHYSICAL EXAMINATION: Oriented only to person, place; thinks the year is 1978 BP 110/70, P 92, RR 14, Temp 99° F, Ht 6'1", Wt 170 HEENT: PERRLA, EOMI TMs clear Throat clear Neck: Supple, no masses Heart: RRR with II/VI systolic murmur (heard for 10 years now) Lungs: Coarse scattered crackles; no focal findings Abdomen: Bowel sounds present; no hepatosplenomegaly Extremities: No cyanosis, clubbing, or edema LABORATORY: CXR: Central/hilar mass with area of cavitation Calcium: 11.5 mg/dL Based on your findings, which of the following types of lung cancer does this man most likely have? a) Bronchoalveolar carcinoma b) Adenocarcinoma c) Small cell d) Large cell e) Squamous cell carcinoma
Pulmonary Medicine Answer E. Squamous cell carcinoma Explanation Squamous cell more commonly involves the central/hilar area with local extension, and it is the most likely cancer to cavitate. Also, it is most commonly associated with hypercalcemia (large cell is next). Adenocarcinoma and large cell are usually peripheral. Bronchoalveolar carcinoma is just a subtype of adenocarcinoma. Squamous and small cell are usually central (remember the S-S sounds like Sentral). With small cell, cavitation never occurs. So if you put together central lesions, this leaves squamous and small cell; then put in cavitation and that leaves only squamous. Plus, for an added bonus, squamous is the one most commonly to have hypercalcemia.
Hampton's hump on CXR should make you think of what diagnosis?
Pulmonary embolism
Patients with Virchow's triad have an increased risk for what pulmonary disorder?
Pulmonary embolism
What is the most likely Dx? 1 week post partum has syncopal episode and is now tachycardic and short of breath.
Pulmonary embolism
What is the Dx? Young healthy female smoker on oral contraception with acute chest pain and SOB?
Pulmonary embolus
Where are most aspirated foreign bodies located?
Right main bronchus. Foreign Body Aspiration Patient will be a child Complaining of a sudden episode of coughing, wheezing, or stridor Comments: most common is the right main bronchus
*PNA* Bug most likely to form a lung abscess
S. aureus
What other bacterial pneumonia are patients who have pseudomonas pneumonia prone to get?
S. aureus pneumonia shares similar risk factors specifically nosocomial
Question: What is the most common bacterial cause of pneumonia in hospitalized patients?
S. pneumoniae.
*Asthma* 1st line Tx for acute exacerbation
SABA (a bronchodilator): - Albuterol - Levalbuterol - Terbutaline - Epi
Lung CA that has the slowest rate of growth; this type of tumor has the lowest propensity for metastasis for all types of lung CA
SCC
This type of lung CA is *bronchial* in origin; centrally located (This allows it to be picked up on cytology).
SCC
Which type of lung CA may be picked up on cytology and why?
SCC because it grows centrally
Type of lung CA that has cells of pulmonary neuroendocrine origin; making it associated w paraneoplastic syndromes
SCLC
*Symptoms throughout the day, nighttime awakenings often 7×/week*, short-acting β-agonist use several times a day, extremely limited normal activity
Severe persistent asthma
*Asthma Classification* Use of SABA severe times per day. Nightly awakenings. Extremely limited activity. Tx?
Severe. SABA PRN + high dose ICS + LABA. +/- Omalizumab.
What is the most common patient complaint with emphysema?
Shortness of breath
Sandblasting, mines, egg shell calcification
Silicosis
Most aggressive lung cancer?
Small cell
SIADH/Hyponatremia with what type of lung CA?
Small cell
What two lung cancers are typically found centrally on CXR?
Small cell and squamous cell
SVC syndrome is MC with what type of lung CA?
Small cell.
*COPD* 1st-line COPD treatment
Smoking cessation
A single abnormality in the lung that is smaller than 3 cm in diameter. Surrounded by normal lung tissue; not associated with any other abnormality in the lung or nearby LNs. Patient will be *Asx*.
Solitary pulmonary nodule
Question: What pathogen causes pneumonia that is associated with bullous myringitis?
Streptococcus pneumoniae. Bullous myringitis was previously linked to Mycoplasma pneumoniae but it appears, based on middle ear aspirate culture results, that typical acute otitis media pathogens are the true cause. Post-viral Pneumonia Patient with a history of influenza Complaining of fever, productive cough with bloody sputum and shortness of breath CXR will show multiple cavitary lesions Most commonly caused by Staphylococcus aureus
Question: A pleural effusion is most difficult to detect in which radiographic position?
Supine.
Treatment for acute respiratory distress syndrome (ARDS)?
Supportive care/ventilatory support - Find and treat underlying cause
Question: What is the treatment of choice for respiratory distress syndrome?
Surfactant. Transient Tachypnea of the Newborn Patient will be a child shortly after birth Complaining of sudden onset of difficulty breathing PE will show tachypnea, nasal flaring, retractions and hypoxia CXR will show parenchymal infiltrates and fluid in the pulmonary fissures Most commonly caused by residual pulmonary fluid Treatment is supportive, self-resolving within 72 hours Comments: Most common cause of neonatal respiratory distress
Then only curative therapy for NSCLC's
Surgery, up to stage III. Lobectomy is considered superior to wedge resection. +/- adjuvant chemo
Tx of non-small cell lung CA
Surgical resection especially if localized to the chest
What is the gold standard for the diagnosis of cystic fibrosis?
Sweat chloride test
*Epiglottitis* Treatment if allergic to beta-lactams (2)
TMP-SMX Clindamycin
*PNA* Tx of Hospital Acquired - add what (2 options) if PCP suspected
TMP-SMX +/- corticosteroids (Bactrim > Clindamycin)
*TB* What is recommended before initiating treatment with Ethambutol, and why?
Testing vision, including color vision, is suggested prior to initiating and throughout treatment with ethambutol, which can cause irreversible cumulative toxicity to the optic nerve.
An 18-year-old man presents for evaluation and relates that he has a herd of cattle. About 3 weeks ago, he was helping a cow deliver, and he had to assist the cow by manually removing the calf and the placenta. The cow was not ill before the delivery. He reports that he became ill about 2 days ago with a high fever, night sweats, and cough. He has noted that he also has a left upper quadrant tenderness in his belly. PAST MEDICAL HISTORY: Negative; healthy farm boy SOCIAL HISTORY: Lives with his mother, a widow Has 3 cats, 2 dogs, and a pet iguana Chews tobacco Doesn't drink alcohol FAMILY HISTORY: Dad died at the age of 35 in a bull-riding accident Mother healthy, 40 Has 2 younger sisters REVIEW OF SYSTEMS: Complains of joint aches and pains with the fever Headache Weakness PHYSICAL EXAMINATION: BP 110/80, P 110, RR 20, Temp 103° F, Ht 6' 1", Wt 210 lbs Well-developed, very muscular man in some distress HEENT: PERRLA, EOMI TMs clear Throat: mild erythema Neck: Supple, no meningismus Heart: RRR without murmurs, rubs, or gallops Lungs: Coarse breath sounds with defined crackles at the right base Abdomen: Bowel sounds present; liver edge palpated 5 cm below right costal margin; spleen tip palpated 4 cm below left costal margin Extremities: No cyanosis, clubbing, or edema Skin: No rashes LABORATORY: WBC: 18,000 with 75% polys, 20% bands Hgb: 16.0 mg/dL Platelets: 150,000 Electrolytes: Normal AST: 100 ALT: 120 CXR: Right lower lobe pneumonia Which of the following is the likely etiology of his pneumonia? a) Coxiella burnetii b) Streptococcus pneumoniae c) Yersinia pestis d) Staphylococcus aureus e) Francisella tularensis
Topic Pulmonary Medicine Answer A. Coxiella burnetii Explanation In this case, the fact that he worked with animal placentas should make you think about Coxiella burnetii. Remember, if you see some type of animal "placenta" or birthing event, consider Coxiella burnetii. Recent literature reported outbreaks with cats—so be suspicious if Tabby is around delivering kittens and people are getting pneumonia. The other clue that this might be Coxiella is the marked hepatosplenomegaly. This is one of those "pneumonia with splenomegaly" organisms.
A 16-year old boy with a history of asthma presents to the ED with severe shortness of breath and audible wheezing. He uses an inhaled corticosteroid and a long acting beta-2-agonist at home daily. However, he has had to use his short acting beta-2-agonist roughly every hour for the past day. In the ED, his vital signs are BP 114/72, HR 106, RR 28, oxygen saturation 94% on room air, and temperature 99.0°F. Diffuse wheezing is appreciated and intercostal retractions are observed. After oxygen is initiated, what is the next step in management for this patient? Begin nebulized albuterol Give epinephrine Give systemic corticosteroids Prepare for intubation
The first step in treatment of an acute asthma exacerbation includes oxygen and beta-2-agonist nebulizers, such as albuterol. Ipratropium is often added to the first three doses of albuterol in those with a severe exacerbation. Response to treatment is monitored by clinical appearance, physical examination, and peak-flow measurements. A mnemonic for the ED treatment of asthma is: BIOMES - Beta-agonists, ipratropium, oxygen, magnesium sulfate, epinephrine, and steroids.
What is the main treatment for a pleural effusion?
Thoracentesis
What is the gold standard for diagnosis of a pleural effusion?
Thoracocentesis
A 30-year-old female with asthma presents to your office for routine followup. She is currently using her albuterol inhaler 2-3 times/week and is waking from sleep about once/week needing a treatment. She is not on any long-term therapy. She has never been hospitalized and usually does well using albuterol until she gets a cold, at which time she has moderate to severe exacerbations. On physical examination today she is stable with no wheezes. Her peak flow is in the green zone. How would you change her daily therapy? a) Add daily inhaled corticosteroids. b) Do not change anything. c) Add inhaled steroids only during a cold. d) Continue current regimen but give oral steroids every time she gets a cold. e) Change the albuterol to a long-acting beta-agonist.
Topic Pulmonary Medicine Answer A. Add daily inhaled corticosteroids. Explanation This patient has mild persistent asthma and therefore requires a long-term control medication. Inhaled glucocorticoids are now considered the first-line agent and cornerstone therapy for mild persistent asthma. They are safe when used at the appropriate doses. There is a delayed onset of action, and therefore they must be given daily. The goal will be to limit major exacerbations and the need for oral steroids.
A 58-year-old woman has had pain and tenderness of the long bones of the lower extremities, knees, and ankles for 2 months. She has smoked 1 pack of cigarettes daily for 30 years. She has had rheumatoid arthritis for 15 years. Examination of the lower extremities shows tenderness to palpation of the femur and tibia bilaterally and slight warmth and tenderness of the knees and ankles. X-ray films show periosteal new bone formation of the femurs. Her symptoms are most likely a complication of which of the following? a) Adenocarcinoma of the lung b) Osteoporosis c) Rheumatoid arthritis d) Small cell lung cancer e) Squamous cell carcinoma
Topic Pulmonary Medicine Answer A. Adenocarcinoma of the lung Explanation Hypertrophic pulmonary osteoarthropathy (HPOA) consists of clubbing, painful periosteal hypertrophy of long bones, and symmetrical arthralgia of the large joints. The features can appear separately or in combination. Radiographs of the long bones show thickening and hypertrophy of the periosteum. HPOA occurs most commonly as a paraneoplastic syndrome associated with adenocarcinoma of the lung. Resection of the primary tumor may relieve the symptoms of HPOA. RA doesn't present with long-bone pain. Osteoporosis is asymptomatic unless a fracture develops. HPOA is not associated with squamous cell carcinoma and is not typically associated with small cell lung cancer.
A 50-year-old man with a history of pneumonia diagnosed 2 days ago presents for follow-up. He was seen as an outpatient and sent home on oral levofloxacin. He says he took 1 pill and it made his stomach hurt, so he stopped the medication. He said he meant to call and let you know, but he was too busy and thought he would get better without the medicine. Now he complains of right-sided chest pain that is pleuritic in character. He says it really hurts to take a deep breath. PAST MEDICAL HISTORY: Essentially negative; few office visits for sildenafil prescriptions SOCIAL HISTORY: Works as an attorney; prosecutes medical malpractice cases Lives alone FAMILY HISTORY: Mother alive and healthy Father died at age 70 of myocardial infarction Brother 49, healthy, mechanic REVIEW OF SYSTEMS: Fever has been persistent and unremitting since early this morning Chills prominent Sputum production has increased markedly since yesterday Chest pain as described above Minor sore throat Generalized body aches and pains No arthritis No vision changes No rash PHYSICAL EXAMINATION: BP 120/70, P 90, RR 20 (splinting), Temp 103.5° F HEENT: PERRLA, EOMI TMs clear Throat clear Neck: Supple; no masses Heart: RRR without murmurs, rubs, or gallops Lungs: Upper lung fields clear Left lower lung has scattered crackles Right lower lung has the following localized findings: absent breath sounds, dullness to percussion, vocal fremitus is absent Abdomen: Bowel sounds are present; no hepatosplenomegaly Extremities: No cyanosis, clubbing, or edema Skin: No rashes noted now LABORATORY: WBC: 15,000 cells/mm3 with 80% polys and 10% bands Hemoglobin/Hematocrit: 15.5 mg/dL; 52% Platelets: 350,000 CXR: Marked consolidation of the right LL with pleural effusion noted bilaterally; right much greater than left Pleural fluid: WBC 70,000 with 90% polys pH 7.02 Gram stain: Few lancet-shaped gram-positive diplococci Based on your findings, which of the following is the most appropriate next step? a) Admit to the hospital, place a chest tube, and start intravenous ceftriaxone and azithromycin. b) Admit to the hospital, start intravenous ceftriaxone plus azithromycin, and observe on therapy 24 hours before placing a chest tube. c) Give IM shot of ceftriaxone and oral azithromycin; observe in waiting room and discharge home if doing better in 4 hours. d) Admit to the hospital, start intravenous ceftriaxone plus azithromycin; get pulmonary consult to decide if he needs a chest tube. e) Admit to the hospital, place a chest tube, and start intravenous vancomycin and gentamicin (for synergy).
Topic Pulmonary Medicine Answer A. Admit to the hospital, place a chest tube, and start intravenous ceftriaxone and azithromycin. Explanation He began with a community-acquired pneumonia and now has an empyema by definition: He has organisms seen on Gram stain of his pleural fluid! The organism is likely pneumococcus. Because he has an empyema, you must place the chest tube as soon as possible; he will not get better without drainage of the pus. Intravenous antibiotics alone will not work. Waiting 24 hours or consulting a pulmonologist will not change the basic fact that he needs "drano"! If you chose to send this patient home, you get 20 lashes with a wet noodle. Remember: Organisms on Gram stain of pleural fluid = needs chest tube. Finally, the choice of vancomycin and gentamicin for synergy is not correct. This choice would be useful if you thought he had community-acquired MRSA, for which he has no risk factors and his Gram stain is not consistent (you would expect gram-positive cocci in clusters for Staphylococcus).
An otherwise healthy 50-year-old woman presents after being discharged from the hospital with a recent diagnosis of lung cancer with a 2-cm isolated tumor. She was found to have squamous cell carcinoma, stage 1A (T1, N0, M0). She is here to find out what further therapy is indicated. Which of the following is the next therapy for her? a) After surgical resection, no further therapy is indicated. b) After surgical resection, radiation therapy is indicated. c) After surgical resection, chemotherapy and radiation therapy are indicated. d) After surgical resection, chemotherapy is indicated. e) After surgical resection, follow-up resection is indicated.
Topic Pulmonary Medicine Answer A. After surgical resection, no further therapy is indicated. Explanation She has a very localized lesion to the lung and a small tumor < 3 cm. She does not have any nodal involvement and has no metastases. Also, she has a non-small cell type of cancer, so surgical resection should be sufficient. She does not need further therapy at this point, and studies have shown that adjuvant chemotherapy for this stage is actually deleterious. If her tumor was > 3 cm and she was Stage 1B (no nodes, no metastases), some trials have indicated that adjuvant chemotherapy may be useful and could be considered because of the size of the tumor.
A 25-year-old woman with past history of mild persistent asthma presents with exacerbation of asthma. This is the fourth exacerbation in 3 months. Prior to 3 months ago, she had been well controlled with last exacerbation being at age 6. On presentation today, she had increased cough productive of brown sputum (described as plugs) and wheezing. With each exacerbation, a CXR was obtained showing infiltrates in different areas, usually upper lobes. Lab evaluation showed elevated peripheral blood eosinophil count. Which of the following is her most likely etiology? a) Allergic bronchopulmonary aspergillosis b) Aspiration pneumonia c) GE reflux d) Chronic sinusitis
Topic Pulmonary Medicine Answer A. Allergic bronchopulmonary aspergillosis Explanation This is an allergic reaction (not invasive infection) to Aspergillus with Type I and Type III reactions. It is usually associated with very high IgE (> 1,000). As in the case above, suspect this in asthmatics with fleeting infiltrates (usually upper lobes), eosinophilia, clinical worsening of asthma, and cough productive of brown plugs. CXR shows "fleeting infiltrates." Treat with corticosteroids and itraconazole.
A 33-year-old woman presents with facial pain and nasal congestion. She reports she had the onset of sore throat and rhinorrhea 5 days ago. Over the past 48 hours she has had facial pressure over the maxillary sinus and yellow nasal discharge. On exam, T 99.2° F, P 90, BP 110/70; nose: swollen turbinates; neck: no adenopathy; chest: clear Which of the following treatments would you recommend? a) Decongestants/nasal irrigation b)Decongestants/nasal irrigation + Metronidazole c) Decongestants/nasal irrigation + Amoxicillin d) Decongestants/nasal irrigation + TMP/Sulfa e) Decongestants/nasal irrigation + Amoxicillin-clavulanate
Topic Pulmonary Medicine Answer A. Decongestants/nasal irrigation Explanation This patient presents with a 5-day history of rhinorrhea, sore throat, and facial pain. She is afebrile with swollen nasal turbinates on exam. Her symptom complex is most consistent with the common cold. The CDC campaign to limit antibiotic use advises to wait 7-10 days before considering treatment with antibiotics for "sinusitis" for facial pain/persistent congestion. The yellow discharge does not increase the likelihood of a bacterial infection. The best treatment would be to begin saline nasal irrigation and topical decongestants for several days without antibiotic treatment.
A 27-year-old complains of "chest pain" and fever. He is diagnosed with pneumonia and treated with levofloxacin as an outpatient 10 days ago. At the time he had rust-colored sputum with lancet-shaped, gram-positive diplococci. Initially he improved, but now on day 4 after starting therapy, he has developed new fever and right-sided, pleuritic chest pain. A CT scan of the chest is done, and the results are presented here: What is the most appropriate next step in management? a) Attempt tube thoracostomy with CT guidance with possible need for more invasive therapy depending on response. b) Send acid-fast smears and cultures of sputum and place a TB skin test. c) Schedule a CT-pulmonary angiogram. d) Begin prednisone 60 mg daily with 10-day wean. e) Add oral amoxicillin-clavulanic acid for anaerobe coverage.
Topic Pulmonary Medicine Answer A. Attempt tube thoracostomy with CT guidance with possible need for more invasive therapy depending on response. Explanation The CT scan shows pleural effusion with multiple loculations. The only effective therapy for this is adequate drainage. Commonly this can be attempted by CT or ultrasound guided tube thoracostomy, which can be done with multiple tubes if multiple loculations are present. If this fails, then more invasive therapy such as video-assisted thoracoscopic surgery (VATS) with debridement may be necessary. Antibiotic therapy alone would be recommended if this was an uncomplicated pleural effusion (free flowing without loculations, pleural fluid pH > 7.2, and negative Gram stain on the pleural fluid). Adding anaerobic coverage is an excellent idea, but oral penicillins are not going to achieve adequate tissue levels for this level of severe disease. Tuberculosis is not very likely in the face of a bacterial infection without risk factors. This is an infection and not a pulmonary embolism, so CT-pulmonary angiogram is not indicated. Prednisone is not going to do anything to treat the infection. For an uncomplicated pleural effusion with a single loculation, antibiotics alone may be effective.
A 56-year-old male is brought to your office because his wife complains of his constant snoring. She reports that he seems to stop breathing sometimes at night. He does report unrefreshed sleep, headaches upon awakening, and takes a nap nearly every day in his office after lunch. You suspect obstructive sleep apnea and order a polysomnogram. The report indicates that the patient has an AHI (apnea/hypopnea index) of 18 events/hour associated with oxygen desaturations as low as 79%. Which of the following is true about obstructive sleep apnea (OSA)? a) CPAP (continuous positive airway pressure) can be effective in eliminating all respiratory related events due to OSA. b) Patients with treated OSA have a higher incidence of hypertension. c) Treatment with CPAP has not been shown to reduce subsequent cardiovascular events in patients with coronary artery disease. d) Neck circumference < 16 inches is a risk factor for OSA. e) Nocturnal oxygen alone has been shown to be an effective therapy.
Topic Pulmonary Medicine Answer A. CPAP (continuous positive airway pressure) can be effective in eliminating all respiratory related events due to OSA. Explanation Obstructive Sleep Apnea (OSA) is diagnosed via polysomnography. Apneas are complete cessation of airflow during sleep, and hypopneas are a reduction in flow during sleep. The total of apneas plus hypopneas is divided by the total sleep time to provide the AHI (apnea/hypopnea index). Mild OSA is present with an AHI of 5-15 events/hour, moderate at 15-35, and severe OSA at > 35 events/hour. OSA has been shown to be associated with a higher risk of hypertension, coronary artery disease, and stroke. OSA treated with CPAP has been shown to reduce incidence of hypertension and future cardiovascular events. CPAP therapy for OSA is the treatment of choice and has been shown to reduce the incidence of hypertension and reduce cardiovascular mortality in patients with coronary artery disease. Risk factors for developing OSA include elevated BMI, increased neck circumference (> 17"), male gender, post-menopausal women, and advancing age. CPAP treats OSA by causing a splinting open of the upper airways to provide adequate flow. Oxygen alone doesn't provide the same benefit and is not an appropriate solo therapy for OSA.
A 56-year-old man is brought to the emergency department 2 hours after the onset of severe shortness of breath that began during a social gathering. He thinks perfume from a guest triggered his shortness of breath. PMH: Major depressive disorder, hypertension, tobacco abuse (1½ ppd cigarettes x 30 years) ROS: Stable early a.m. cough x 5 years and shortness of breath with exertion of 1 flight of stairs. Meds: HCTZ 25 mg qd, zoloft 50mg qd PE: Moderate distress with RR 32, BP 138/86, afebrile Normal PE except for tachypnea ABG: pH 7.47, pCO 2 32 mmHg, pO 2 85 mmHg CXR: blunting at right costophrenic angle Which of the following is the most appropriate next step in patient care? a) CTA Albuterol and ipratropium inhalation by metered-dose inhaler with a spacer plus oral prednisone 60 mg. b) Change sertraline to paroxetine and recommend follow-up with his primary care provider in 72 hours. c) Albuterol inhalation by metered-dose inhaler with a spacer. d) Intramuscular injection of epinephrine for anaphylactoid reaction.
Topic Pulmonary Medicine Answer A. CTA Explanation If you did this question sequentially, you can see that I changed this question slightly from the first time you saw it (if you didn't do this sequentially, then never mind and forget that I said this ☺). I made the patient a little older and gave him a smoking history as a risk factor for occult malignancy. These two changes make his risk for DVT and PE much higher than in the first setting, where the patient was young and healthy. Immediately, your differential diagnosis should include PE at the top of the list because of these changes in epidemiology. On first inspection of the blood gas, you can appreciate the respiratory alkalosis. This is more supportive evidence of a PE, but you could consider that he's hyperventilating because of a panic attack. If you calculate the A-a gradient, you'll see that even though a PO 2 of 85 doesn't look so bad, the gradient is abnormal: P A O 2 = 150 - (PCO 2 x 1.25) = 150 - (32 x 1.25) = 110 A-a gradient = 110 - 85 = 25 (normal 5-20), elevated! Something's going on at the alveolar-capillary unit. Of the choices listed, the only appropriate next step to evaluate for a pulmonary embolism is to perform a CTA, which is now considered first-line imaging in patients in whom you suspect PE. He does not have anaphylaxis, so epinephrine is inappropriate. He does not have asthma or COPD, so inhalation medications are a waste of time. Changing his antidepressant might be appropriate if this were a true panic attack, but the abnormal A-a gradient mandates a workup for pulmonary embolism.
A 19-year-old male presents to the local college infirmary with complaints of fever, cough, and chest pain. A review of his chart indicates the patient was seen for sore throat approximately 2 weeks prior to onset of current illness. Which of the following is his most likely diagnosis? a) Chlamydophila (formerly known as Chlamydia) pneumoniae b) Streptococcus pyogenes c) Klebsiella pneumoniae d) Legionella
Topic Pulmonary Medicine Answer A. Chlamydophila (formerly known as Chlamydia) pneumoniae Explanation This is the TWAR pathogen. These patients present with fever, malaise, headache, sore throat, and nonproductive cough. The sore throat is commonly seen 2-3 weeks prior to onset of pneumonia. Treatment is with a macrolide or doxycycline. Streptococcus pyogenes presents with acute onset of fever, chills, chest pain, and dyspnea similar to pneumococcus. Klebsiella in children typically occurs in those with underlying immunosuppression or prolonged endotracheal intubation. With Legionella, expect to hear symptoms involving the gastrointestinal tract such as vomiting, diarrhea and abdominal pain, neurologic manifestations, high fever, and possibly hyponatremia on lab evaluation.
A 45-year-old man presents with a history of stable asthma on the following regimen of medications for 20 years: Cromolyn sodium, theophylline, and inhaled steroids. He is doing well; however, last weekend he developed a sore throat and a cough. He went to a local doctor and was prescribed an unknown antibiotic, which he has continued to take. You are concerned that he was put on an antibiotic for a viral infection. Meanwhile, the patient throws up and says he has been really nauseated for 2 days and hasn't been able to keep anything down. Physical exam findings are within normal limits except that his mucous membranes are a little dry. His vital signs are normal and he does not have any signs of dehydration at this point. Which of the following antibiotics could be causing his troubles? a) Ciprofloxacin b) Amoxicillin c) Cefixime d) Sulfamethoxazole
Topic Pulmonary Medicine Answer A. Ciprofloxacin Explanation The problem is that he is on theophylline and adding ciprofloxacin has caused his theophylline clearance to decrease, thus resulting in increased serum levels of theophylline. The result of increased theophylline levels in the toxic range usually begins with nausea and vomiting. The other antibiotics listed do not induce changes in theophylline clearance.
For which of the following is a 5 mm PPD considered significant? a) Close contact with a documented case b) Health-care workers c) IV drug abusers d) Homeless persons e) Prisoners
Topic Pulmonary Medicine Answer A. Close contact with a documented case Explanation Note that health-care workers, IV drug abusers, prisoners, and homeless persons are considered positive at 10 mm. Outside of close contact with a documented case, for 5 mm to be considered positive, you have to have HIV or other major cell-mediated dysfunction, fibrotic changes on CXR consistent with prior TB, an organ transplant, or be significantly immunosuppressed (generally considered so if taking the equivalent of 15 mg/day or more of prednisone for at least 1 month).
You are asked to evaluate a patient due to their shortness of breath. He is 74 years old with a history of coronary artery disease with CABG eight years prior. He reports exertional dyspnea, difficulty breathing when lying flat, and increased lower extremity edema. A chest radiograph is done and reveals a moderately sized, right-sided pleural effusion. On your examination, he had diminished breath sounds along the lower right portion of the right lung with dullness to percussion along the lower right back. A thoracentesis is performed and 900 mL of clear, straw-colored fluid is removed. Pleural Fluid: Serum: Protein: 1.9 Protein: 4.2 LDH: 120 LDH 315 pH: 7.30 WBC: 200; RBC: 0 Gram stain: no organisms Two days later a CXR is repeated, and the effusion is still present, although slightly smaller. The patient is comfortable and not in any distress. What should be done next? a) Echocardiogram b) Placement of a large bore chest tube c) Repeat thoracentesis d) Placement of a small bore chest tube e) Cardiac catheterization
Topic Pulmonary Medicine Answer A. Echocardiogram Explanation This patient has a moderately sized pleural effusion that you can determine is transudative in nature because it does not meet the criteria of an exudative effusion according to Light's criteria. These criteria are: Pleural fluid protein/serum protein ratio greater than 0.5 Pleural fluid LDH/serum LDH ratio greater than 0.6 Pleural fluid LDH greater than 2/3 the upper limits of normal of the serum LDH The most common etiologies for a transudative effusion are congestive heart failure, hepatic hydrothorax from cirrhosis, nephrotic syndrome, and severe hypoalbuminemia. This patient has a history of coronary artery disease and other stigmata to suggest congestive heart failure, including orthopnea and edema. In addition, pleural effusions in CHF are typically bilateral and symmetric or right-sided if unilateral. Since this is a transudative effusion, the underlying cause should be sought, and an echocardiogram would be the next most reasonable step to gauge cardiac and valvular function. Proceeding with cardiac catheterization would be premature at this point, as there is not enough evidence to suggest ischemia as an etiology. Repeat drainage, either by chest tube or thoracentesis, would likely result in reaccumulation because the underlying cause has not been clarified. This would not be recommended at present, especially if the patient is stable and comfortable.
A 52-year-old man presents to your office with a 2-month history of exertional dyspnea and nonproductive cough. There is no history of wheezing or prior asthma. He has a 10-pack-year smoking history but quit 10 years ago. There is no history of chest pain, and he has a copy of an ECG done for a recent insurance physical that was normal. He works in an office with no known occupational exposure, and he is on no prescription medications. On exam, he is a well-developed male in no apparent distress. He has no adenopathy. The cardiovascular exam is normal. On examination of the chest, you note late inspiratory crackles at the bases. He has no clubbing, cyanosis, or edema. LABORATORY: Hematocrit 45%; hemoglobin 15 g/dL CXR—PA and Lateral: Normal ABG: pH 7.45; pCO2 36 mmHg; pO2 70 on room air Office spirometry: Reduced FVC, reduced FEV1, but FEV1/FVC normal Which of the following should be the next step in this patient's workup? a) High-resolution CT of the chest b) Pulmonary function tests at a PFT lab, which would include a determination of his total lung capacity (TLC) and diffusing capacity (DLCO) c) Open lung biopsy d) Transbronchial biopsy
Topic Pulmonary Medicine Answer A. High-resolution CT of the chest Explanation Nearly 10% of patients presenting with interstitial lung disease (ILD) will have a normal CXR. High-resolution chest CT is the test of choice to evaluate for the presence of an interstitial lung disease. HRCT will be helpful diagnostically in assessing the distribution of the disease as well as providing guidance for a possible lung biopsy. In addition, HRCT may provide a clue to the staging of the disease (ground-glass appearing airspace disease suggests an active cellular component vs. burnt out fibrosis). This case is a little different from a similar case you saw elsewhere; here, the CXR is normal and thus you would want more information/data (HRCT) before proceeding directly to bronchoalveolar lavage and/or transbronchial biopsy. Full PFTs would merely provide confirmation of what you already suspect: This patient has an interstitial lung disease with probably restricted lung volumes and a reduced diffusing capacity. Flexible bronchoscopy with transbronchial biopsy could be entertained if we felt a strong suspicion for sarcoid or an infectious etiology. The yield on transbronchial biopsy is very good for these etiologies. However, in order to provide an adequate tissue sample for examination in most ILDs, an open lung biopsy (traditional or thorascopic) is required—but again, we would wait on the HRCT first.
A 45-year-old female presents to the Emergency Department with a 7-day history of progressive shortness of breath. She was seen earlier this week by her primary care provider, who ordered a chest radiograph. This revealed a hazy area of infiltration in her right lower lobe, and he started a quinolone antibiotic. Upon presentation to the ED 4 days later, she was noted to be in severe respiratory distress with a respiratory rate of 45, HR of 130, and oxygen saturations of 78% on 4 L oxygen. She was promptly intubated and placed on mechanical ventilation. Chest radiograph revealed bilateral alveolar infiltrates throughout the entire lung fields. Her BNP level was normal, and an echocardiogram revealed normal LV function and moderately elevated pulmonary artery pressures. Currently she is on FiO2 of 100%. Her ABG reveals a PaO2 of 90 mmHg. Blood cultures grew Streptococcus pneumoniae. She has now been admitted to the intensive care unit. Which of the following interventions has been shown to reduce mortality in patients with this disease process? a) Initiation of mechanical ventilation with tidal volumes set at 4-6 mL/kg of ideal body weight b) Initiation of mechanical ventilation with tidal volumes set at 8-10 mL/kg of actual body weight c) Use of prone positioning d) Use of continuous narcotic infusion to improve ventilator synchrony e) Use of pulmonary artery catheter to measure pressures to guide further management
Topic Pulmonary Medicine Answer A. Initiation of mechanical ventilation with tidal volumes set at 4-6 mL/kg of ideal body weight Explanation This patient presents with pneumonia with development of ARDS (Acute Respiratory Distress Syndrome). She has a P/f (PaO2 divided by fraction of inspired oxygen) ratio of 90, with ratios ≤ 350 consistent with acute lung injury and ≤ 250 consistent with ARDS. Furthermore, she has bilateral infiltrates, and her echocardiogram doesn't reveal any evidence of left heart disease to implicate cardiogenic pulmonary edema. The ARDSNET study group compared groups to lower versus higher tidal volume groups, with a significant reduction in mortality in the patients with tidal volumes set at 4-6 mL/kg of ideal body weight. Prone positioning has only been shown to temporarily improve oxygenation. The use of sedatives and narcotics are most often required to allow for patient comfort and synchrony but have not been shown to affect mortality. Pulmonary artery catheters have been specifically studied in patients with ARDS, with no overall outcome improvement in those patients that had these invasive devices placed.
A 68-year-old man with a history of smoking for 50 years presents to the emergency department with acute onset of shortness of breath and right-sided chest pain, which is sharp and worse with inspiration. His wife says he has had a chronic daily cough for 30 years, but today it was worse—and then he developed this acute shortness of breath. PAST MEDICAL HISTORY: Negative for hospitalizations Hypertension for 30 years, on various agents in the past Diabetes insipidus diagnosed 5 years ago SOCIAL HISTORY: Former police officer, retired 6 years Lives with wife of 50 years Also lives with his 45-year-old unemployed son, which causes a lot of tension in the household according to the patient's wife You diagnose a pneumothorax in the emergency department. His CXR also shows a "honeycomb appearance" with interstitial changes and small cystic spaces in the upper lung fields. Putting together the pneumothorax with his CXR results, diabetes insipidus, and smoking history, which of the following are you most likely to find at lung biopsy? a) Langerhans cells b) Plasma cells c) Fungal elements d) Acid-fast bacilli e) Chylous material
Topic Pulmonary Medicine Answer A. Langerhans cells Explanation This patient most likely has eosinophilic granuloma. Langerhans cells are the predominant cell form. It occurs in smokers and men much more often than in women. About 10% of patients will present with pneumothorax. The diabetes insipidus is seen when the granuloma involve the posterior pituitary. Treatment is to stop smoking. If he had lytic bone lesions (particularly skull), diabetes insipidus, and exophthalmus, then he would have Hand-Schüller-Christian syndrome.
A 45-year-old non-smoker presents with dyspnea on exertion for 2 months. He has bibasilar crackles and has restrictive disease on pulmonary function testing. He is sent for transbronchial biopsy. On further questioning, you learn he works with beryllium. Which of the following would help differentiate between sarcoid and berylliosis? a) Lymphocyte transformation test (LTT) on BAL lymphocytes and/or blood b) CD4 to CD8 ratios c) Kveim skin test d) Non-caseating granulomas on biopsy e) ACE level
Topic Pulmonary Medicine Answer A. Lymphocyte transformation test (LTT) on BAL lymphocytes and/or blood Explanation These 2 diagnoses are very similar, and the only test listed that will differentiate between the two is this one, also called the beryllium lymphocyte proliferation test. A positive LTT indicates berylliosis. The Kveim test is no longer used. ACE levels are non-specific. CD4 to CD8 ratios are also non-specific. Both sarcoid and berylliosis present with non-caseating granulomas.
A 35-year-old female presents to the emergency department with complaints of shortness of breath for 3 days. She has had a dry cough, fever, and chills. Her symptoms of shortness of breath have progressed very rapidly such that she cannot walk from room to room without difficulty. She has no significant past medical history and works as a waitress in a diner. She is a nonsmoker and denies any illicit drug use. On presentation, she is in marked respiratory distress. P: 125, BP: 90/45, Oxygen sats: 79% room air, T: 39.4° C, RR: 38 She has marked accessory muscle use. Lung exam reveals diffuse bilateral crackles and coarse rhonchi. She has a tachycardic rhythm and no peripheral edema. CXR reveals bilateral alveolar infiltrates without cardiomegaly or pleural effusions. Echocardiogram reveals normal LVEF, mild TR, and moderate elevation of pulmonary artery systolic pressures. Her WBC is elevated to 18,000. Her renal function is normal. Blood cultures are now growing Streptococcus pneumoniae . The patient is intubated and placed on mechanical ventilation. Her initial ABG on 100% FiO2 is: pH: 7.29, pCO2: 58, pO2: 74, HCO3: 21. Which of the following is associated with a reduction in mortality in this condition? a) Mechanical ventilation with tidal volumes set at 6-8 mL/kg of ideal body weight b) Use of nitric oxide during ventilation c) Use of neuromuscular blockade during ventilation d) Placement of the patient in the prone position e) Initiation of high-dose corticosteroids
Topic Pulmonary Medicine Answer A. Mechanical ventilation with tidal volumes set at 6-8 mL/kg of ideal body weight Explanation This patient presents with hypoxemic respiratory failure associated with bilateral alveolar infiltrates, high FiO2 requirement, and no evidence of congestive heart failure. This is most consistent with the acute respiratory distress syndrome (ARDS). The leading differential diagnosis for ARDS is congestive heart failure, but her echocardiogram is not supportive of significant cardiac dysfunction. The most common etiologies for ARDS are sepsis, pneumonia, aspiration, and transfusion-related acute lung injury. ARDS requires support by mechanical ventilation in a great majority of patients. Studies have shown a reduction in mortality when lung protective strategies are used (6-8 mL/kg) as compared to traditional strategies (10-12 mL/kg). Unfortunately, all other interventions for ARDS have not shown a mortality benefit. The use of neuromuscular blockade may be beneficial in the first 24 hours of ARDS to improve oxygenation. Prone positioning, which involves placing the patient on their front, has also been shown to improve oxygenation for brief periods of time. Use of high-dose steroids have not been shown to be beneficial in ARDS, although medium-dose steroids may help with lung injury scores if started early in the course of ARDS. Nitric oxide is a selective pulmonary vasodilator that will temporarily improve oxygenation. However, studies have not revealed any benefit and, in fact, may be harmful in patients with ARDS.
An obese 60-year-old man with diabetes mellitus complains that lately he will fall asleep during the middle of conversations and that he spends half his day asleep in front of the television set. His favorite show, he says, is the "Andy Griffith Show," and he has all of the episodes taped. He tries to talk about the show, but he falls asleep mid-sentence. His wife says this happens all the time. You send him for formal sleep testing, and he is found to have moderate obstructive sleep apnea-hypopnea (OSAH). Besides weight loss and avoidance of alcohol and sedatives, which of the following have been proven to be effective in treatment of OSAH? a) Nasal continuous positive airway pressure (CPAP) b) Watching "Leave It to Beaver" instead of "Andy Griffith" reruns c) Sleeping supine d) Using hypnotics to induce sleep at night e) Sleeping under a fan
Topic Pulmonary Medicine Answer A. Nasal continuous positive airway pressure (CPAP) Explanation The continuous pressure "splints" the nasopharynx open at night. Unfortunately, it is uncomfortable for many patients. Sleeping supine and using hypnotics actually would worsen obstructive sleep apnea-hypopnea (OSAH). Sleeping under a fan would not have any effect. The type of television show shouldn't affect this either—although I know of no scientific study to prove this wrong.
Which of the following is not considered an indication to place a patient on treatment for latent tuberculosis infection (assume all CXRs are normal)? a) PPD reading at 72 hours of 7 mm in an asthmatic patient on 5 mg/day of prednisone b) PPD reading at 72 hours of 11 mm in a diabetic c) PPD reading at 48 hours of 6 mm in a patient who lives with a person who has active tuberculosis d) PPD reading at 48 hours of 16 mm in a healthy 20-year-old e) PPD reading at 72 hours of 11 mm in a prisoner
Topic Pulmonary Medicine Answer A. PPD reading at 72 hours of 7 mm in an asthmatic patient on 5 mg/day of prednisone Explanation According to the guidelines, the cut-off is 10 mm for this patient. All of the other patients require treatment; generally, you would use INH for 9 months in each of these other patients. If this patient had been on 15 mg or more of daily prednisone, then they would have met the criteria for therapy.
Which of the following is most consistent with an effusion due to rheumatoid arthritis? a) Pleural fluid glucose < 30 mg/dL b) Pleural fluid glucose > 80 mg/dL c) Pleural fluid pH < 7.0 d) Pleural fluid white cell count of 15,000 e) Pleural fluid triglyceride > 115 mg/dL
Topic Pulmonary Medicine Answer A. Pleural fluid glucose < 30 mg/dL Explanation Low glucose values with normal protein and WBC counts are classic for rheumatoid arthritis pleural effusions. Pleural effusions with large numbers of WBCs or a low pH are consistent with severe bacterial infection or empyema. High triglycerides in a pleural effusion are consistent with a chylous effusion.
A 42-year-old man presents to your office complaining of cough, wheezing, and shortness of breath that has been progressive for the past 3 months. PMH is negative, and he takes no medications. He works in private practice as an attorney and has been in a monogamous marriage for 17 years. He drinks 1 glass of wine nightly and denies the use of tobacco or drugs. PE: Normal vital signs Normal exam except for diffuse fine crackles auscultated in the lung bases CXR: Diffuse interstitial infiltrates Which of the following is the most appropriate next step in patient care? a) Schedule a high-resolution CT scan of the chest. b) Prescribe albuterol by MDI. c) Begin prednisone 60 mg daily. d) Refer to the pulmonologist for bronchoscopy. e) Order a sweat test.
Topic Pulmonary Medicine Answer A. Schedule a high-resolution CT scan of the chest. Explanation This clinical scenario describes an otherwise healthy man in his 40s who develops dyspnea. Radiograph clarifies his disease as interstitial. History and physical exam give you no further clues as to etiology. The question specifically queries whether you know that biopsies are no longer used in the diagnosis of interstitial lung disease except to exclude cancer and infections. High-resolution CT scans are the procedure of choice to evaluate possible interstitial lung disease because trained radiologists can often supply a diagnosis simply by evaluating a pattern. Sweat tests are useful to diagnose cystic fibrosis, which is not an interstitial lung disease. Treatment with a beta-agonist or prednisone should not be initiated until a diagnosis is made.
A 65-year-old man presents to his physician's office with a 9-month history of dyspnea on exertion associated with a mild cough. He has daily sputum expectoration, most pronounced in the morning. He has a past medical history notable for coronary artery disease for which he is on aspirin, simvastatin, and atenolol. He is a 50 pack-year smoker and works in an office-based setting. On physical exam, he has no evidence of resting or exertional hypoxemia. His breath sounds are moderately-to-severely diminished without any adventitial sounds. Pulmonary function tests reveal: forced expiratory volume in 1 second (FEV1) is 1.5 L (60% of predicted), forced vital capacity (FVC) is 2.3 L, and the FEV1/FVC ratio is 0.65. TLC is 5.25 L (125% predicted) and DLCO is 25 mmHg (56% predicted). Which of the following will be most effective for improving this patient's long-term survival? a) Smoking cessation b) Oxygen therapy c) Pulmonary rehabilitation d) Inhaled tiotropium e) Inhaled corticosteroids combined with long-acting beta-agonists
Topic Pulmonary Medicine Answer A. Smoking cessation Explanation Smoking is the single most preventable cause of lung disease, which accelerates the rate of lung function decline as compared to non-smokers. For those patients who are successful with smoking cessation, lung function decline as defined by FEV1 will approach the same rate as non-smokers after approximately one year. FEV1 severity is an independent predictor of mortality in patients with COPD, hence, smoking cessation is the best answer. Pharmacologic therapies have been shown to improve dyspnea scores, quality of life, and reduce exacerbations. However, they have not consistently shown an improvement in lung function or mortality. Pulmonary rehabilitation has been shown to have similar favorable outcomes as medications as well as improved exercise tolerance and skeletal muscle strength. Finally, oxygen has been shown to improve mortality in those patients who demonstrate resting, exertional, or nocturnal hypoxemia. This patient does not manifest any of these.
A 45-year-old woman who was diagnosed with influenza A last week presents today with a much worse cough and return of her fever. She was feeling better near the end of the week, but now, in the last 24 hours, has become acutely ill again. She says that she has pain in her left lower chest when she takes a deep breath. Before this influenza diagnosis, she has been healthy. PAST MEDICAL HISTORY: Negative Took amantadine for 5 days; finished 3 days ago SOCIAL HISTORY: Works as a bartender in local pub Doesn't drink; Doesn't smoke but exposed to second-hand all night long, 5 days a week FAMILY HISTORY: Noncontributory REVIEW OF SYSTEMS: Fevers to 103° F Chills Sore throat now resolved Body achiness is severe again PHYSICAL EXAMINATION: BP 110/70, P 80, RR 24, Temp 102° F Ill-appearing woman in moderate distress HEENT: PERRLA, EOMI TMs clear Throat slightly hyperemic Neck: Supple, no masses Heart: RRR without murmurs, rubs, or gallops Lungs: Diffuse crackles fairly localized to the left base Abdomen: Bowel sounds present; no hepatosplenomegaly Extremities: No cyanosis, clubbing, or edema LABORATORY: Pending Besides Streptococcus pneumoniae, which of the following organisms should you also consider in this patient? a) Staphylococcus aureus b) Haemophilus influenzae c) Mycoplasma pneumoniae d) Staphylococcus epidermidis e) Streptococcus pyogenes
Topic Pulmonary Medicine Answer A. Staphylococcus aureus Explanation She recently had influenza, a classic setting where staphylococcal pneumonia will occur. Streptococcus pneumoniae is still very common in these patients also. Influenza virus is known to both increase respiratory colonization by S. aureus and impair ciliary function (and therefore clearance of staphylococci). Influenza does not predispose a patient to any of the other answer options nearly as much as to S. aureus.
You are seeing a 50-year-old nurse who works at a local hospital. He has had annual tuberculin skin testing for 30 years. His last PPD a year ago was 7 mm. Today, he presents at 72 hours for reading of his PPD placed earlier in the week. He is healthy and denies any health problems, particularly no fevers, sweats, or weight loss. This is an employee health check, and therefore no physical exam or other information is obtainable. LABORATORY: PPD at 72 hours: 17 mm CXR: Normal Based on the data presented, which of the following is the most appropriate next step? a) Start INH 300 mg daily for 9 months. b) Repeat PPD in 2 weeks; if still positive then start therapy. c) No treatment because he is older than 35 years old. d) 4-drug therapy because he is high-risk by being in a hospital environment. e) Start INH 300 mg daily for 12 months.
Topic Pulmonary Medicine Answer A. Start INH 300 mg daily for 9 months. Explanation This nurse is a recent converter. He has had a 10-mm increase in 1 year's time. By definition, he should receive prophylaxis. His CXR is normal, and he has no signs or symptoms of tuberculosis; therefore, he does not need to be treated for active tuberculosis. 12 months of INH is no longer recommended for anyone, including HIV-infected patients! It is not reasonable to repeat the PPD in 2 weeks; he has a positive result, and it will not change with repeated testing.
A 70-year-old man with a history of working in a brickyard for 50 years presents for evaluation at his granddaughter's request. He has been retired for 10 years. He is still active and plays bingo at the local church every day. He usually wins about once a week. Members of his bingo group are around his age, and recently one of his contemporaries at the bingo hall was diagnosed with tuberculosis. Your patient has been healthy and has no complaints. He denies weight loss, cough, fevers, or night sweats. PAST MEDICAL HISTORY: Prostatic hypertrophy diagnosed 5 years ago; doing well currently HTN for 40 years MEDICATIONS: Propranolol 20 mg q day ECASA q day SOCIAL HISTORY: Widowed for 20 years Lives alone; still drives without difficulty Volunteers at local nursing home on occasion Never smoked; Doesn't drink alcohol FAMILY HISTORY: Mother died at age 80 of "old age" Father died at age 75 of stroke Brother alive, 68, healthy except HTN Sister died at age 50 of stroke REVIEW OF SYSTEMS: No sore throat No vision changes No chest pains No headaches Minor arthritis-type pain in knees in the early morning; better with movement GU symptoms much improved; no difficulty initiating urine stream PHYSICAL EXAMINATION: BP 130/69, P 66, RR 15, Temp 98.8° F Ht 6' 1", 190 lbs HEENT: PERRLA, EOMI TMs clear Throat clear Neck: Supple, no masses Heart: RRR without murmurs, rubs, or gallops Lungs: Coarse breath sounds but clear Abdomen: Bowel sounds present in all 4 quadrants, no hepatosplenomegaly, nontender Extremities: No cyanosis, clubbing, or edema GU: Normal male genitalia, no masses LABORATORY: CXR: Small nodules located in upper lobes; calcified hilar lymph nodes with "hilar eggshell calcification" PPD: 20 mm at 72 hours 3 induced sputum samples for AFB: All negative smears and cultures Based on these findings, which of the following is the most appropriate next step? a) Start INH prophylaxis. b) Initiate workup for asbestos-related disease process. c) Ignore +PPD in this 70-year-old man. Sputum tests are negative; therefore, it is unlikely he needs prophylaxis. d) Initiate 4-drug therapy for tuberculosis. e) Start treatment for silicosis.
Topic Pulmonary Medicine Answer A. Start INH prophylaxis. Explanation This patient has silicosis based on his history of working in a brickyard and the classic CXR findings of small nodules in the upper lobes that are calcified and the "hilar eggshell calcifications." There is no specific treatment for uncomplicated silicosis. The problem with silicosis, however, is that alveolar macrophages are made ineffective by the ingestion of silica; therefore, patients with silicosis are at increased risk of tuberculosis. Therefore, anytime a patient with known silicosis has a positive PPD, no matter what age or duration, he should be "prophylaxed" with INH. He has no evidence of active disease, and therefore treatment with 4-drug therapy is not indicated. Asbestosis more commonly involves the lower lobes, and his presentation does not warrant a big workup for this disease.
A 68-year-old woman is admitted to a nursing home, and one year later she develops a cough. Workup is implemented and she has a positive PPD. A CXR is done and shows a left apical infiltrate, and sputums are positive for acid-fast organisms. She is placed in immediate respiratory isolation. Which of the following do you recommend for your patient? a) Start INH, rifampin, PZA, and ethambutol. b) Further TB workup is necessary to determine what should be done. c) Start INH and rifampin. d) Await sensitivities to begin therapy. e) Start INH only.
Topic Pulmonary Medicine Answer A. Start INH, rifampin, PZA, and ethambutol. Explanation She has active tuberculosis and must receive 4-drug therapy. With CXR and positive sputum she does not need further workup. You will begin 4-drug therapy and await sensitivities. If the Mycobacterium tuberculosis is sensitive, then you can place her on 2-drug therapy.
A 25-year-old female presents to your office for initial evaluation of her asthma. She reports she was diagnosed with asthma 3 years ago. Her symptoms include daily cough that is nonproductive and seems to be worse at night. She has occasional wheezing and some dyspnea on exertion when exercising. She has been prescribed a short-acting beta-agonist that she uses 4-5 times daily with relief that lasts about 30-60 minutes. She has no other significant past medical history. Her sister also has asthma. She denies any smoking history. She has 2 dogs at home and has never had allergy testing. She has no symptoms of reflux. Which of the following is the next best step in management for this patient? a) Start a low-dose inhaled corticosteroid. b) Start a long-acting beta-agonist. c) Start once-daily oral leukotriene receptor antagonist. d) Continue current treatment regimen with short-acting beta-agonist. e) Start a proton pump inhibitor every night.
Topic Pulmonary Medicine Answer A. Start a low-dose inhaled corticosteroid. Explanation This patient has asthma that would be categorized as moderate persistent asthma, given the frequency of her symptoms, and needs further titration of her care. All patients with any category of persistent asthma should be started on maintenance therapy with the foundation of an inhaled corticosteroid. In this case, this is the best answer. Additional therapies, such as leukotriene receptor antagonists, should be added sequentially if adequate controls of symptoms are not achieved with an inhaled corticosteroid alone. A long-acting beta-agonist has been associated with a higher risk of death in patients with asthma, particularly if not administered along with an inhaled steroid. They should not be prescribed alone in patients with asthma. She has no symptoms consistent with GERD. Studies have shown that asthma control is not improved with the addition of proton pump inhibitors in those without specific reflux symptoms.
A 25-year-old asthmatic patient has a history of frequent exacerbations. On CXR, she frequently has lung infiltrates that migrate and do not seem to respond to antibiotic therapy. She is usually afebrile during these episodes, but it really sets off her asthma and she has a significant exacerbation. Usually, she has to be admitted to the hospital and placed on systemic steroids, with aggressive pulmonary management. PAST MEDICAL HISTORY: Asthma since early childhood; has never required mechanical ventilation Has 1 child, age 2; no problems during pregnancy MEDICATIONS: Albuterol prn Cromolyn sodium daily Zafirlukast daily SOCIAL HISTORY: A 3rd year medical student (considering a career in Radiology) Married and lives with husband and 2-year-old son FAMILY HISTORY: Mother 50 and healthy Father 50 and has hypertension No siblings REVIEW OF SYSTEMS: Occasional headache Stressed by 3rd year rotations PHYSICAL EXAMINATION: Well-developed, well-nourished woman in moderate respiratory distress BP 110/60, P 90, RR 30, Temp 98° F HEENT: PERRLA, EOMI TMs clear Throat clear Neck: Supple; no masses Heart: RRR without murmurs, rubs, or gallops Lungs: Scattered wheezes especially in upper lung fields Poor airway movement Few scattered crackles Abdomen: Bowel sounds present; no hepatosplenomegaly Extremities: No cyanosis, clubbing, or edema Skin: No rashes LABORATORY: WBC: 10,000 with 50% polys, 20% lymphs, 30% eosinophils Hgb: 13.5 mg/dL Platelets: 340,000 CXR: Scattered infiltrates throughout all lung fields Sputum: Normal flora on bacterial stain KOH is shown in this image: Branching yeasts Based on your findings, which of the following is the best treatment for her condition? a) Systemic corticosteroids and itraconazole b) Amphotericin B 1 mg/kg IV q day c) Fluconazole 200 mg bid x 1 day then once daily thereafter d) Itraconazole 200 mg bid x 3 days, then daily thereafter e) Amphotericin gargles with 5 mg in a 200 cc suspension
Topic Pulmonary Medicine Answer A. Systemic corticosteroids and itraconazole Explanation The key here is that she has allergic bronchopulmonary aspergillosis (ABPA). The standard therapy used to be steroids alone; however, recent data have shown that adding itraconazole to the steroid therapy is beneficial. If you see something fungal that is branching, it is likely Aspergillus, especially if there is "right-angle" or 90-degree branching. Amphotericin IV or as a gargle is inappropriate in this instance, and fluconazole has limited activity against Aspergillus.
A 29-year-old woman is admitted to the hospital with a 1-week history of nonproductive cough, shortness of breath, and dyspnea on exertion. She is only able to walk a room's length before she needs to rest. Further symptoms include fever and chills. She has a known history of HIV disease and previous CD4+ cell count was 10 cells/µL ten months ago. She has otherwise been noncompliant with follow-up visits and has not been on any treatment for her HIV disease. On examination, the patient is in mild respiratory distress. Vital signs: T: 39.2° C, BP: 121/62, HR: 127, RR: 25, Pulse ox saturation: 88% on room air. She has a white exudate throughout the posterior oropharynx as well as on her tongue. Cardiovascular examination reveals tachycardia with regular rate and rhythm and no murmurs. Lung examination reveals diffuse crackles with poor air movement throughout. Labs reveal a normal WBC count and basic metabolic panel. Lactate dehydrogenase (LDH) is elevated by three times the normal range. Her P a O 2 is 60. Chest x-ray is below. Bronchoscopy with BAL is performed, and the results are pending. What is the best step in management for this patient? a) TMP/SMX and prednisone b) Vancomycin Amphotericin B c) Isoniazid, ethambutol, rifampin, and pyrazinamide d) Trimethoprim/sulfamethoxazole (TMP/SMX)
Topic Pulmonary Medicine Answer A. TMP/SMX and prednisone Explanation Based on the clinical presentation and findings on chest radiograph, this patient has P. jiroveci (formerly known as P. carinii ) pneumonia (PJP). Clinically, patients with HIV and PJP have a gradual onset of symptoms characterized by fever, cough, and progressive dyspnea. Other symptoms may include fatigue, chills, chest pain, and weight loss. Patients with PJP also tend to have diffusion abnormalities and hypoxemia, resulting in lower arterial oxygen saturations. The most common radiographic findings are diffuse bilateral alveolar or interstitial infiltrates. Occasionally findings will include pneumothorax, lobar or segmental infiltrates, cysts, nodules, or pleural effusions. PJP is an opportunistic infection in patients with AIDS when CD+ cell counts drop below 200 cells/µL. Additionally, LDH levels often are elevated but are nonspecific. TMP/SMX remains the initial drug of choice for treatment of PJP. Consensus guidelines recommend that HIV-infected patients with hypoxia be treated with adjunctive corticosteroids. Although the patient has signs suggestive of oral thrush, this is likely related to Candida , and amphotericin would not be indicated to treat this. Furthermore, fungal infections tend to be more nodular or cavitary in nature than the current patient. Although HIV-infected patients are at risk for Mycobacterium tuberculosis , for which four drug therapies would be indicated, this patient does not present with symptoms consistent with tuberculosis both radiographically and clinically. On the Board exam they will frequently give you scenarios in which a patient will present with shortness of breath and you have to discern the etiology. Some of their favorites include high-risk procedures/conditions for pulmonary embolism.
Which of the following is the treatment for acute methemoglobinemia? a) Carbon monoxide and methylene blue b) 100% oxygen and methylene blue c) Carbon dioxide and methylene blue d) Sodium pentathlon e) Ascorbic acid
Topic Pulmonary Medicine Answer B. 100% oxygen and methylene blue Explanation Most commonly this is seen with use of "poppers." Giving the methylene blue causes rapid reduction of methemoglobin back to hemoglobin. Ascorbic acid is useful for chronic hereditary methemoglobinemia. Note that the other agents would kill the patient.
An 18-year-old man comes to your office for follow-up examination. He has a known history of asthma since age 12. His PEF is 400 L/min (80% of personal best). He currently takes fluticasone 110 µg 2 puffs twice daily and uses an albuterol inhaler 3-4 times daily. He is compliant with his medications, but he still wakes up at night 3-4 times a week with coughing that is relieved with his albuterol inhaler. Which of the following is the best management option for this patient? a) No change in treatment and reassurance. b) Add a long-acting β-agonist such as salmeterol or formoterol to his fluticasone therapy. c) Prescribe prednisone 40 mg daily for 7 days. d) Start therapy with omalizumab. e) Prescribe azithromycin 500 mg daily for seven days.
Topic Pulmonary Medicine Answer B. Add a long-acting β-agonist such as salmeterol or formoterol to his fluticasone therapy. Explanation This patient has moderate, persistent asthma. Furthermore, the presence of nightly cough and frequent use of his rescue inhaler both support poorly controlled asthma. His maintenance regime should be augmented. Increasing the dose of his inhaled corticosteroid or adding a long-acting β-agonist are both acceptable treatment options. Systemic steroids such as prednisone are usually reserved for patients with acute exacerbations of asthma symptoms and PEFs below 50% of predicted. There is no evidence of an infection such as bacterial bronchitis, so the addition of azithromycin would not be helpful. Omalizumab, a monoclonal anti-IgE antibody, is currently the only approved anti-IgE therapy licensed in the United States for the prophylaxis of asthma exacerbations and control of symptoms in moderate to severe allergic asthma in patients ≥ 12 years of age. It is given as an add-on therapy to ICS in moderate to severe allergic asthma and has been shown to significantly reduce asthma exacerbations and allows doses of ICS to be reduced. It should be considered for those patients with poorly controlled disease despite the use of high-dose inhaled steroids and long-acting beta-agonists. This patient currently is only on a medium dose ICS.
A 40-year-old nurse presents with a PPD of 15 mm. A CXR reveals a cavitary lesion. Initial sputum reveals acid-fast organisms with cultures pending. She works and lives in rural Iowa where resistant tuberculosis has not been documented. Which of the following do you recommend for her? a) Because the risk of resistance is low, INH and RIF only are sufficient therapy. b) Begin 4-drug therapy with INH, RIF, PZA, and ethambutol. c) Begin 5-drug therapy with INH, RIF, PZA, ethambutol, and streptomycin. d) Do not start therapy until the cultures return with sensitivities. e) INH alone is sufficient therapy.
Topic Pulmonary Medicine Answer B. Begin 4-drug therapy with INH, RIF, PZA, and ethambutol. Explanation She has active tuberculosis and must be treated. In almost all areas of the U.S., 4-drug therapy is recommended for 2 months, followed by 2-drug therapy (INH, RIF) to finish 6 months of therapy. Much depends on the sensitivities as to whether more drugs are needed, but without a history of resistant organisms in the community, there is no need to add a 5th drug. INH alone is only used for treatment of latent tuberculosis, not active pulmonary disease. INH and RIF as initial therapy is incorrect based on current guidelines, which recommend 4-drug therapy for all until sensitivities return, even in areas of the U.S. with low risk of resistance to INH or RIF. You are presented a patient with asthma exacerbation. He is on beta-blocker medications after a recent myocardial infarction.
A 29-year-old female is 32-weeks pregnant with her first gestation. She presents to the emergency department with a 2-day history of acute shortness of breath and left-sided chest pain. She describes the pain as sharp, located in the mid axilla along the 6 th to 8 th rib location. She denies any recent trauma. She further denies any fevers, chills, syncope, cough, or sputum production. Her legs are swollen and have been progressively increasing in size over the last 4 weeks of her pregnancy, but they are equal in size and without erythema. Upon presentation her vitals: T: 99.1° F, P: 125 RR: 24 BP: 100/45 O 2 Sats: 92% on room air. She is without any respiratory distress. Her exam is notable for a regular tachycardia and a gravid uterus appropriate for stage of gestation. Lungs are clear to auscultation. She has 1+ lower extremity edema without any asymmetry. There is no sign of fetal distress. Chest radiograph is without any abnormalities, and EKG reveals a sinus tachycardia. Lower extremity venous Doppler ultrasound reveals an acute deep venous thrombosis in the left common femoral vein. Which of the following is the next best step in treatment? a) Stat C-section for emergent delivery. b) Begin unfractionated heparin by continuous infusion according to weight. c) Initiate aspirin therapy. d) Obtain a ventilation-perfusion scan. e) Initiate therapy with low molecular weight heparin at 1 mg/kg SQ q12 with concomitant warfarin therapy.
Topic Pulmonary Medicine Answer B. Begin unfractionated heparin by continuous infusion according to weight. Explanation This patient has developed a venous thrombolic event (VTE) related to her pregnancy. At the minimum, she has a deep venous thrombosis, and her signs and symptoms also suggest a pulmonary embolus. She will require treatment for her VTE with either unfractionated heparin or low-molecular-weight heparin. Although further imaging to exclude a pulmonary embolus would be helpful, this should not delay the initiation of anticoagulation therapy. Furthermore, the best study to evaluate for pulmonary embolus is chest CT angiogram, even in the setting of pregnancy. Aspirin therapy alone is not adequate treatment for VTE. Warfarin is contraindicated in pregnant patients due to its significant teratogenic properties. There are no signs of fetal distress, and the fetus is not yet at term; so emergent C-section would not be indicated.
A 50-year-old school teacher presents with concerns about asbestos exposure. He has been in a school that has recently been closed because of concerns of asbestos. Which of the following manifestations of asbestos-induced lung disease has the shortest latency period of occurrence? a) Diffuse pleural effusions b) Benign asbestos pleural effusions c) Asbestosis d) Localized pleural effusions e) Mesothelioma
Topic Pulmonary Medicine Answer B. Benign asbestos pleural effusions Explanation These appear to be immune mediated in character and have a relatively short latency period of 5-10 years. The other choices require much longer to show up, sometimes 20-30 years or more.
A 45-year-old man presents to your office with a 2-month history of exertional dyspnea and nonproductive cough. There is no history of wheezing or prior asthma. He smoked in his early 20s but now limits himself to an occasional cigar. There is no history of chest pain, and he has a copy of an ECG done for a recent insurance physical that was normal. He works in an office with no known occupational exposure, and he is on no prescription medications. On exam, he is a well-developed male in no apparent distress. He has no adenopathy. The cardiovascular exam is normal. On examination of the chest, you note late inspiratory crackles at the bases. He has no clubbing, cyanosis, or edema. LABORATORY STUDIES: Hematocrit 45%; hemoglobin 15 g/dL CXR-PA and lateral: Increased interstitial markings at the bases ABG: pH 7.45, pCO2 36 mmHg, pO2 70 mmHg on room air You send him for full PFT testing, and this is consistent with restrictive lung disease. Various serologic markers are sent for systemic disease and are negative. Which of the following is the appropriate next step in this patient's workup? a) Ventilation-perfusion scan. b) Bronchoalveolar lavage and/or transbronchial lung biopsy. c) CT angiography of the lung. d) Open-lung biopsy and resection of diseased lung. e) Follow serial pulmonary function tests at a PFT lab, which would include a determination of his total lung capacity (TLC) and diffusing capacity (DLCO).
Topic Pulmonary Medicine Answer B. Bronchoalveolar lavage and/or transbronchial lung biopsy. Explanation This patient has an interstitial lung disease with restricted lung volumes and a reduced diffusing capacity; and it may worsen without definitive diagnosis and treatment. Serial PFTs would merely provide confirmation of what you already suspect. You need to know which ILD the patient has and how to stage it in terms of activity to decide on treatment options. Some would begin with bronchoalveolar lavage with transbronchial lung biopsy in the right clinical scenario. If this is negative, then a high-resolution CT could be performed, or some would just proceed to a HRCT and the definitive open lung biopsy. The yield on transbronchial biopsy is very good for some etiologies (particularly sarcoid or infectious etiologies). However, in order to provide an adequate tissue sample for examination, an open-lung biopsy (traditional or thorascopic) is often required. This is not consistent with pulmonary embolism, and V/Q scanning or CT angiography are not indicated. Open-lung biopsy certainly could be considered, but you would not resect all of the areas of "diseased lung" without a known diagnosis or reason.
A 42-year-old woman with a history of end-stage renal disease on hemodialysis presents to the emergency department with fever of 103° F, shaking chills, hypotension 70/50, tachypneic, and tachycardic. Her dialysis graft site is erythematous, warm to the touch, and has purulent drainage at a prior access site. A diagnosis of septic shock is made, and the patient is given intravenous antibiotics, IV fluids, and arrangements are made to transfer the patient to the ICU with a vascular surgery consult to remove the hemodialysis graft. The intensivist meets the patient in the emergency department and immediately places a Swan-Ganz catheter out of concern of volume overloading this patient with end-stage renal disease and now a non-functioning dialysis catheter. The initial hemodynamic profile for this patient would most resemble which of the following? a) Cardiac Output L/min: Low; Systemic Vascular Resistance dynes-sec/cm5: High; Wedge Pressure mmHg: Normal/Low b) Cardiac Output L/min: High; Systemic Vascular Resistance dynes-sec/cm5: Low; Wedge Pressure mmHg: Low c) Cardiac output L/min: Low; Systemic Vascular Resistance dynes-sec/cm5: High; Wedge Pressure mmHg: Low d) Cardiac Output L/min: Low; Systemic Vascular Resistance dynes-sec/cm5: High; Wedge Pressure mmHg: High
Topic Pulmonary Medicine Answer B. Cardiac Output L/min: High; Systemic Vascular Resistance dynes-sec/cm5: Low; Wedge Pressure mmHg: Low Explanation Utilizing a Swan-Ganz catheter to generate a hemodynamic profile of shock, such as in this case, is a reasonable question for the exam. They may even throw in more data such as pulmonary artery pressure, mixed venous oxygen saturation, and even left ventricular stroke volume to foul you up. Stick to the parameters above and you should be able to narrow down the answer sufficiently. Remember: The hallmark of early, "warm" sepsis is a hyperdynamic heart (increased cardiac output) coupled with a very low systemic vascular resistance. Typically the patient in early septic shock is volume depleted and "third spacing" a considerable amount of the fluids given for resuscitation due to the low systemic vascular resistance. In all the other options listed, the systemic vascular resistance is high because it is the only thing holding the blood pressure together! Volume depletion due to hypovolemic shock (whether hemorrhagic or intravascular) is characterized by a low wedge pressure, a reduced cardiac output, and a high SVR. Cardiogenic shock is defined by a low cardiac output, high-filling pressures (think CHF), and a high SVR. Obstructive shock is typified by a massive PE or a tension pneumothorax. There is reduced cardiac filling, reduced cardiac output, and a high SVR. In pericardial tamponade, the filling pressures will be normal as a reduced volume is acted on by a decreased compliance of the ventricular wall.
A surgical colleague calls you one afternoon asking if you could see him in your office. You know him to be a healthy 50-year-old male who likes to play tennis and golf. When he arrives at the office, you notice that he appears thinner and less tan than usual. He is carrying a large manila envelope that you see contains x-rays. He reports that he has had a dry, nagging cough for the past 4 weeks. He self-medicated himself with a quinolone antibiotic that he had at the office and after 2 weeks, he switched to azithromycin. He grew concerned when he developed low-grade fever, the cough became productive of clear phlegm, and he noticed that he was winded early in a set of tennis. He had a chest x-ray at the hospital today and decided to bring it to you to look at. On physical exam you find no abnormalities, and his lungs are clear to auscultation. You walk him to the lab where a CBC is done, and you find a mild elevation in the WBC count. His chest x-ray shows diffuse bibasilar parenchymal fibrotic pattern and patchy airspace densities in the right lower lobe. Which of the following is the most likely diagnosis? a) Loeffler syndrome b) Cryptogenic organizing pneumonia (COP) c) Allergic bronchopulmonary aspergillosis d) Hypersensitivity pneumonitis e) Idiopathic pulmonary fibrosis
Topic Pulmonary Medicine Answer B. Cryptogenic organizing pneumonia (COP) Explanation While histologic confirmation is warranted, this is a classic presentation of COP. The flu-like illness that has not responded to antibiotics is a favorite of the ABIM. Histologic confirmation is needed because the patient will need a prolonged course (6-12 months) of corticosteroids and possible additional immunosuppressive agents. ABPA is unlikely because there is no prior history of asthma. Loeffler's is a self-limiting, benign eosinophilic pneumonia that is asymptomatic. IPF or other interstitial lung diseases are in the differential, but the time course is a little short and we have no occupational, environmental, or medication history. Hypersensitivity pneumonitis is not suggested by the history, and frequently the patient would appear more ill and have basilar rales on auscultation.
A 36-year-old woman has had increasing dyspnea for 8 years. She has no cough or increased sputum production. No smoking history. On physical exam, there is hyperresonance to percussion in the bases. A chest radiograph reveals increased lung volumes with flattening of the diaphragmatic leaves bilaterally. Which of the following laboratory findings is she most likely to have? a) Elevated serum iron stores b) Decreased alpha-1-antitrypsin c) Increased sweat chloride d) Decreased ceruloplasmin e) Positive methacholine challenge test
Topic Pulmonary Medicine Answer B. Decreased alpha-1-antitrypsin Explanation The illness script is consistent with obstructive pulmonary disease, specifically emphysema: dyspnea with hyperresonance in the bases and objective evidence of increased lung volumes and diaphragm flattening on radiograph. The most likely cause of emphysema in a young woman this age with no history of tobacco smoking is alpha-1-antitrypsin deficiency. The methacholine challenge test is used to diagnose patients with suspected asthma who have normal pulmonary function studies. This case does not describe asthma, because asthmatic symptoms are reversible, and this clinical presentation is one of chronic obstruction. Elevated serum iron stores are seen in hemochromatosis that does not directly affect the lungs. Decreased ceruloplasmin levels are associated with Wilson disease and have also been observed in research studies evaluating cigarette smokers and neonates with hyaline membrane disease. The sweat chloride test is used to diagnose cystic fibrosis that presents differently.
A 32-year-old female presents to your office complaining of wheezing for 2 weeks. At her last appointment 6 weeks ago, she was noted to have intentionally lost weight through an exercise program, and she requested to be taken off two of her medications, cimetidine and acyclovir. She was in a good state of health until developing cold symptoms two weeks ago, and since then, she has wheezed daily. She denies tobacco, drugs, or alcohol. She has not had any fever. The cold symptoms have since resolved. PMH: Asthma since childhood, previously stable x 5 years on her current regimen GERD Genital herpes Meds: theophylline, cimetidine (discontinued), acyclovir (discontinued), beclomethasone high-dose bid, salmeterol bid, montelukast qd, albuterol PRN At her last appointment, her theophylline level was 17 mg/dL (therapeutic range: 10-20 mg/dL) Which of the following is the most likely cause of her persistent wheezing? a) Antagonistic drug interaction between theophylline and montelukast b) Decreased serum theophylline concentration c) Undiagnosed Bordetella pertussis d) Tachyphylaxis to salmeterol e) Underlying interstitial lung disease
Topic Pulmonary Medicine Answer B. Decreased serum theophylline concentration Explanation This patient has severe asthma that is controlled on multiple modalities of therapy until such time as some of her ancillary meds are discontinued. Recall that both cimetidine and acyclovir increase theophylline levels; therefore, when the drugs are discontinued, the theophylline is metabolized faster, and levels in the serum decrease. The lower level may have precipitated an asthma attack, especially after the patient developed an upper respiratory infection. Tachyphylaxis to salmeterol does not occur, nor is there an antagonistic drug interaction between theophylline and montelukast. B. pertussis presents as a chronic cough lasting for weeks in adults after an initial upper respiratory presentation. Undiagnosed pertussis could be a factor here, but it is not the most likely explanation for her new wheezing. An undiagnosed interstitial lung process is also less likely given the recent changes in her medications and the resulting change in her theophylline levels.
A 35-year-old woman presents complaining of dyspnea with exertion since delivering her baby 6 months earlier. She is a nonsmoker. Her recent pregnancy and delivery were uncomplicated. She owns a parakeet. Her family history is significant for Factor V Leiden gene mutation and venous thromboembolic events in several family members. PE: Afebrile, BP 122/76, HR 82, RR 12, BMI 31 Well-nourished female in no respiratory distress Distant breath sounds and trace edema in both feet and otherwise normal exam ECG: Nonspecific T-wave changes, leads V4-V6 CXR: Normal ABG: pH 7.45, pCO 2 48, pO 2 76 D-dimer: Normal range Which of the following is the most likely diagnosis? a) Acute pulmonary embolism b) Dyspnea due to obesity c) Amniotic fluid embolus d) Hypersensitivity pneumonitis e) Primary pulmonary hypertension
Topic Pulmonary Medicine Answer B. Dyspnea due to obesity Explanation The patient in this case has two epidemiologic risk factors for pulmonary embolism (recent pregnancy, which can exacerbate an inherited thrombophilia and a familial Factor V Leiden mutation). You would be correct to worry that her dyspnea is related to a pulmonary embolism. But look at her BMI: It's 31. This is very important because sometimes the Boards will sneak obese patients by you by showing you only their BMI and not stating that they are "obese." Her blood gas is normal, as is her D-dimer. Her calculated A-a gradient is 14 mmHg (normal). 150 - (48 x 1.25) = 90 90 - 76 = 14 So all is well at her alveolar-capillary unit. The combination of her obesity, normal A-a gradient, and normal D-dimer suggests an alternative cause for her dyspnea. She has no physical exam findings to support primary pulmonary hypertension. A normal A-a gradient and a stable patient help to exclude an amniotic fluid embolus (and most amniotic fluid emboli occur during pregnancy or shortly after delivery). Hypersensitivity pneumonitis is associated with recurrent symptomatic pneumonias—conspicuously absent from her history, in spite of her bird ownership. (Don't select hypersensitivity pneumonitis just because she owns a bird! You need to carefully examine the situation to make sure that diagnosis fits.) Obesity is the only option that is supported by the clinical data.
A 55-year-old woman presents to your office for evaluation of dyspnea. She has no cough or chest pain and complains of shortness of breath with exertion only. No nocturnal symptoms. She has occasional episodes of near syncope but no palpitations. Chest radiograph doesn't reveal any abnormalities. Pulmonary function tests are ordered. Actual Predicted % FVC (L) 4.88 122 FEV1 (L) 3.09 126 FEV1/FVC .63 82 TLC (L) 6.17 107 RV (L) 1.41 81 DLCO 12 50 Based on her DLCO result, which of the following is unlikely? a) Pulmonary embolus b) Erythrocytosis c) Anemia d) Pulmonary arterial hypertension e) Emphysema
Topic Pulmonary Medicine Answer B. Erythrocytosis Explanation The diffusion capacity of carbon monoxide measures the ability of the lung to transfer and exchange gas. During testing, the patient fully exhales then inhales a gas mixture with CO. After a 10 s breath hold, rapid exhalation is done and the amount of gas is analyzed to determine the amount transferred. Disruption of the alveolar-capillary surface reduces the DLCO, which occurs in emphysema, interstitial inflammation and fibrosis, edema, and infections. In addition, reduced capillary lung volumes as occurs in pulmonary hypertension and pulmonary embolism also decrease the DLCO. Due to decreased hemoglobin, the DLCO will be low if not corrected for anemia. Elevation in hemoglobin will increase the carrying capacity of oxygen and carbon monoxide, thus increasing gas exchange and increasing the DLCO.
A 35-year-old Caucasian male with chronic lung disease of unknown etiology at this point presents for follow-up care. He has had episodes of sinusitis, bronchiectasis, and pancreatic insufficiency for at least 30 years. He has had recurrent pneumonias over the last 10 years. Usually, his pneumonias are associated with pseudomonal infections. PAST MEDICAL HISTORY: As above FAMILY HISTORY: Older brother with similar complaints Mother healthy Father healthy, of Scandinavian ancestry patient has 2 sons, both healthy SOCIAL HISTORY: Works as a used car salesman Doesn't smoke Doesn't drink REVIEW OF SYSTEMS: Frequent colds Frequent sinus infections Frequent bloody noses because of increased blowing of nose Chronic cough PHYSICAL EXAMINATION: BP 120/70, RR 18, Temp 98.5° F, P 90 HEENT: Nasal polyps; PERRLA, EOMI Throat: Normal tonsils Neck: Supple, no masses Heart: RRR without murmurs, rubs, or gallops Lungs: Coarse breath sounds; occasional expiratory wheeze anteriorly and posteriorly Abdomen: No HSM, nontender Extremities: Clubbing present, no cyanosis GU: Normal male genitalia; no masses Based on the history and physical exam, which of the following does not support the diagnosis of cystic fibrosis? a) Clubbing b) Family history c) Nasal polyps d) Recurrent sinusitis e) Recurrent pneumonias
Topic Pulmonary Medicine Answer B. Family history Explanation It would be exceedingly unusual for a male patient with cystic fibrosis (CF) to have children. The rest of the findings are consistent with CF. Remember that this gene is encoded on chromosome 7. It causes a defect in sodium and chloride transport channels in lungs particularly, but also in other organs.
A 72-year-old male who is a nursing home resident is brought to the emergency department due to fever to 39.1° C. Upon presentation, he is noted to have a heart rate of 125 and a blood pressure of 75/42. Urinalysis reveals many WBCs, and many bacteria and cultures are drawn. He appears to be in moderate distress and has moderate diffuse abdominal pain. He is fairly lethargic and barely arouses to verbal or painful stimuli. Initial labs drawn reveal a lactate level of 4.2, WBC of 23.5, and Hgb of 10.4. Which of the following would not be indicated in the treatment of this patient? a) Early initiation of empiric antibiotics Initiation of drotrecogin alfa (activated) by continuous IV infusion for 96 hours b) Admission to the intensive care unit c) Delivery of crystalloid fluid to target a central venous pressure of 8-12 mmHg d) Placement of a central venous catheter
Topic Pulmonary Medicine Answer B. Initiation of drotrecogin alfa (activated) by continuous IV infusion for 96 hours Explanation This is an example of a patient with septic shock with evidence of hypotension and organ hypoperfusion with an elevated lactate level. Early initiation of treatment is important with clear evidence of benefit for certain interventions. The early initiation of targeted empiric antibiotics has been shown to improve mortality. Maintenance of organ perfusion is vital to prevent end organ ischemia and failure. Placement of a central venous catheter can assist with the delivery of intravenous fluids, antibiotics, blood products, and vasopressors if indicated. Furthermore, estimates of intravascular filling pressures can be obtained from properly placed central venous catheters. Early goal-directed therapy in severe sepsis has been a protocol that has shown mortality benefit. This includes initial crystalloid infusion for goal CVP to 8-12 mmHg. If the patient remains hypotensive despite this, then vasopressors should be initiated. Admission to an intensive care unit would be appropriate given the evidence of septic shock. Drotrecogin alfa is a recombinant form of human activated protein C that has antithrombotic, antiinflammatory, and profibrinolytic properties. Initial studies suggested that there was some mortality benefit in patients with very severe sepsis. However, subsequent studies have not shown efficacy in the setting of the sepsis syndrome and the manufacturer has now withdrawn this medication from the market.
Which of the following agents would be useful in an acute exacerbation to improve bronchodilation quickly? a) Albuterol b) Ipratropium c) Epinephrine d) Cromolyn sodium e) Prednisone
Topic Pulmonary Medicine Answer B. Ipratropium Explanation Ipratropium is the only agent that will bronchodilate effectively in the face of chronic beta-blockade. Cromolyn sodium does not work acutely. Epinephrine and albuterol would have their effects blunted by the beta-blockade. Prednisone would also not be helpful acutely.
A 38-year-old man presents with an acute asthma attack. He has been feeling bad for a few days. This morning he awakened and could not breathe. When you see him in the emergency department, he is anxious and cannot talk because of his discomfort. You realize that you have to act quickly and note in his chart that he has had to be ventilated 3 times in the past for severe asthma exacerbations. PAST MEDICAL HISTORY: As above Most recent hospitalization was 1 year ago at another hospital Has been on an inhaled medium-dose steroid, an inhaled long-acting beta2-agonist, and zafirlukast SOCIAL HISTORY: Works as a puppet maker Lives with his friend FAMILY HISTORY: Mother with severe asthma Father with coronary artery disease REVIEW OF SYSTEMS: Deferred for the moment—he is about to crash! PHYSICAL EXAMINATION: BP 150/95, RR 40 with shallow breaths and marked accessory muscle use, P 120 HEENT: Cyanotic around his lips Heart: RRR without murmurs, rubs, or gallops Lungs: Faint squeaks is all you hear Abdomen: Benign LABORATORY: ABG: pH 7.06 pCO2 90 PaO2 55 Oxygen saturation 84% On 100% FiO2 You realize that you are not going to be able to ventilate him effectively without putting him on mechanical ventilation. Which of the following ventilator settings are appropriate for a severely ill asthmatic patient? a) Low rate, high tidal volume, high flows b) Low rate, small tidal volume, high flows c) Low rate, high tidal volume, low flows d) High rate, small tidal volume, high flows e) High rate, high tidal volume, low flows
Topic Pulmonary Medicine Answer B. Low rate, small tidal volume, high flows Explanation When ventilating an asthmatic patient, the idea of "permissive hypercapnia" is important to remember. Focus on getting the oxygen saturation up and don't worry as much about the pCO2. Each of the factors (low rate, small tidal volume, high flows) addresses the need for a prolonged expiratory phase. High flow on the inspiration allows for less time devoted to inspiration and more time to expiration. So, based on the idea of "permissive hypercapnia," it makes sense to have a lower rate and smaller tidal volume.
An 81-year-old female has severe emphysema with severe dyspnea on exertion. She is on 4 L oxygen continuously. She has difficulty with performing her activities of daily living (ADLs). She uses an inhaled long-acting anticholinergic, an inhaled long-acting beta-agonist/inhaled steroid combination, and oral theophylline. She has been to pulmonary rehabilitation. She asks your opinion about surgical options for her COPD. Which of the following is true? Lung volume reduction surgery has been shown to increase mortality in all patients who undergo the procedure. Lung volume reduction surgery improves outcomes in patients with predominantly upper-lobe disease and very low exercise capacity. This patient should be considered for lung transplantation. Lung volume reduction surgery has been shown to decrease mortality in all patients who undergo the procedure.
Topic Pulmonary Medicine Answer B. Lung volume reduction surgery improves outcomes in patients with predominantly upper-lobe disease and very low exercise capacity. Explanation Surgical options are available for patients with emphysema. However, outcomes are variable and depend on the patient's underlying condition. Transplantation is available for emphysema; however, most centers set the age limit at < 75 years of age, and this patient is unlikely to be eligible. Lung volume reduction surgery (LVRS) involves removal of diseased portions of emphysematous lungs, particularly large bullous regions. In a large multicenter randomized controlled trial, overall 90-day mortality was increased for those who underwent surgery as compared to those who did not. However, subset analysis did reveal improved functional outcomes and survival in those patients with predominantly upper-lobe disease and very low exercise capacity. At present, LVRS is only recommended for this patient population with emphysema.
A 65-year-old male comes to the emergency department with increasing respiratory distress over the past 2 days. He is a smoker and has known COPD, for which he has been hospitalized twice in the past year for exacerbations. The patient appears cyanotic, is in moderate distress utilizing accessory musculature, and is perched forward sitting on the stretcher. He is alert and reports feeling "better" since getting to the emergency department. Lung sounds are quite diminished in all lung fields, and an ABG on room air shows pH 7.30, pCO2 63 mmHg, and a pO2 of 44 mmHg. The respiratory therapist is awaiting your orders. Your orders for initial oxygen therapy should be which of the following? a) O2 100% by non-rebreather face mask with continuous pulse oximetry. b) Nasal oxygen, 1-2 L/min with follow-up ABG in 20 minutes. Increase oxygen flow based on the ABG to a pO2 of 55-60 mmHg. c) Institute BiPAP therapy because patient is already showing signs of inspiratory fatigue and respiratory acidosis. d) Nasal oxygen initially, 3-4 L/min with pulse oximetry to achieve a saturation of 92% or greater. e) Withhold oxygen therapy at this time since patient is stable and you do not want to induce further respiratory acidosis. f) Intubate the patient and place on mechanical ventilation with an initial FiO2 of 100%.
Topic Pulmonary Medicine Answer B. Nasal oxygen, 1-2 L/min with follow-up ABG in 20 minutes. Increase oxygen flow based on the ABG to a pO2 of 55-60 mmHg. Explanation Imperative treatment is to provide this patient with supplemental oxygen to achieve a hemoglobin saturation of 88-90%. This is to prevent further hypoxemia, which can cause cardiac arrhythmias and acute cor pulmonale. Oxygen supplementation may result in worsening respiratory acidosis. (HINT: not through reduction in respiratory drive—COPD patients are just like the rest of us in that our chemiotaxic centers for the control of ventilation are CO2 sensitive, not O2 driven!) Worsening respiratory acidosis can be detected only by ABGs, not pulse oximetry. If providing supplemental FiO2 to achieve an oxygen saturation of 88-90% results in severe respiratory acidosis, then positive pressure ventilation via noninvasive methods or conventional ventilation must be considered.
A 51-year-old female presents in an outpatient clinic with a new onset of a rash on her back and chest. This rash began about 2-3 months ago. Initially, she thought it was some form of fungal infection or allergic reaction and applied a 1% hydrocortisone cream without much benefit. The rash was not particularly itchy. At the same time, she had begun to develop some pain in her shoulders and hips but no early morning stiffness and no swelling of the joints of her fingers or toes. At age 16, she had presented with unexplained painful nodules on her lower extremities, but the cause of this was never fully understood and she has never had any recurrence. During her second normal pregnancy, she developed a weakness of one side of her face and was told that she had a Bell's palsy. Her facial expression returned to normal fairly soon after the birth of her child. Her internist had been concerned because he had found mild elevations of her calcium levels 3 years ago and again 6 months ago, but these have normalized on their own. There was no particular explanation found for this at that time. On examination, she was well appearing. Height was 5' 7", weight was 136, BP was 127/76, and pulse was 72. Cardiac and respiratory exams were benign. The abdomen was without hepatosplenomegaly. Musculoskeletal exam revealed no joint synovitis. Neurological exam was normal without any proximal myopathy. Cutaneous exam revealed several circumscribed, somewhat elevated, 2 x 2 coin-sized lesions on her chest and back. Her dermatologist suggested a biopsy. Which of the following is the most likely pathological finding that could be obtained from the skin biopsy of one of these coin lesions? a) Panniculitis b) Non-caseating granulomatous inflammatory changes c) Interface dermatitis d) Eosinophilic fasciitis
Topic Pulmonary Medicine Answer B. Non-caseating granulomatous inflammatory changes Explanation She has multi-organ involvement to suggest the diagnosis of sarcoidosis. The initial lesions on her legs were probably those of erythema nodosum. A biopsy would have found a probable panniculitis at that time. Hypercalcemia, facial nerve palsy, and cutaneous involvement are, again, typical. ACE levels may be elevated, but not always. A chest x-ray would probably be appropriate as part of her workup. Non-caseating granulomas are charactertistic of sarcoidosis.
A 42-year-old female complains of exertional dyspnea for about 2 years that seems to be gradually progressing. She now has difficulty doing simple household chores without dyspnea. She has no associated cough, sputum, fever, chills, or wheezing. She has occasional episodes of feeling lightheaded with exercise but no palpitations. She has no significant past medical history and takes no medications except for a multivitamin. She is a lifelong nonsmoker. Physical exam reveals normal vitals except a resting oxygen saturation of 89%. Lungs are without abnormal sounds. Cardiovascular exam reveals a 2/6 holosystolic murmur at the left midsternal border, an RV impulse at the left sternal border, and A2 and P2 sounds that are equal in intensity. Pulmonary function tests only reveal a reduced DLCO, which is 45% predicted. Chest radiograph is normal. Echocardiogram reveals normal left ventricular function but is notable for right atrial and ventricular dilatation, 3+ tricuspid regurgitation, and an estimated pulmonary artery systolic pressure of 84 mmHg. Right heart catheterization is performed, which estimated the mean pulmonary artery pressure to be elevated at 55 mmHg. Pulmonary capillary wedge pressure is normal. Chest angiogram is without pulmonary embolus. Serologic testing for connective tissue disease is all negative. Which of the following is not currently used for treatment in patients with this clinical entity? a) Subcutaneous prostacyclin therapy b) Oral cyclophosphamide daily c) Oral endothelin receptor antagonist d) Oxygen therapy e) Warfarin
Topic Pulmonary Medicine Answer B. Oral cyclophosphamide daily Explanation The patient in this scenario has a diagnosis of pulmonary arterial hypertension (PAH). Based on the information given, this is likely idiopathic primary pulmonary hypertension. Her exam findings are quite typical, and the echocardiogram is also indicative. Right heart catheterization, however, is the gold standard for making the diagnosis and assessing the degree of severity. Multiple studies have shown survival benefits in patients with PAH given warfarin. Oxygen improves exercise capacity and symptoms in patients with hypoxemia. Specific medications for PAH include calcium channel blockers (in patients with a positive vasodilator challenge), oral endothelin receptor antagonists, phosphodiesterase inhibitors, and prostacyclin analogues. This final category has been administered in inhaled, subcutaneous, and intravenous formulations. Cyclophosphamide has not been evaluated nor is it approved for use in PAH. Note that "not" questions will not usually appear on Boards; however, we occasionally use them for teaching purposes because essentially in 1 question we have provided you with 4 correct answers that you could be asked on the Boards about the treatment of PAH.
A 60-year-old male patient is found to have a 4 cm lung mass in his right upper lobe on CXR. CT scan confirms that it is suspicious for malignancy, but there are no other lesions or significant adenopathy. Bullous lung disease is noted in the apices, particularly on the right. A fine needle aspiration of the mass reveals non-small cell carcinoma. A thoracic surgery consult is requested, but the surgeon refers him back to you for pre-op clearance. The patient has a history of asthma and admits to at least a 30 pack/year history smoking, though he has cut back to "a few a day." He reports that he uses his beta-agonist inhaler only once or twice a week because it only makes him cough. On physical exam, he is well appearing except for a limp in his right leg. His respiratory rate is 16/minute. Examination of the chest is normal with only a few scattered end-expiratory wheezes. The remainder of the physical examination is unremarkable. Pulmonary function tests (with bronchodilators held prior to testing) reveal an obstructive pattern with FEV1 of 1.6 liters. There is minimal response to bronchodilators in the lab. ABG on room air show pH 7.41, PaCO2 38, and PaO2 of 87. In addition to initiating a smoking-cessation program, which of the following do you recommend? a) Refer him for a pulmonary consultation. b) Order a quantitative ventilation scan. c) Refer him to an oncologist for non-surgical treatment of his tumor. d) Start inhaled steroids and repeat PFT in 4 months.
Topic Pulmonary Medicine Answer B. Order a quantitative ventilation scan. Explanation The only cure for lung cancer is a surgical resection. As such, we strive to make any patient a surgical candidate. This patient's pre-op PFTs show an FEV1 less than 2.0 liters—the traditional cutoff for surgical candidates. Smoking cessation, inhaled steroids, and further intervention from a pulmonologist may improve his FEV1, but at the risk of delaying the curative procedure. Therefore, since there are asymmetrical bullae on the CT scan, it is likely that the ventilation scan will show less ventilation to the right lung, and a quantitative assessment would allow even a right pneumonectomy.
A 50-year-old male patient is brought to your office at his wife's insistence. You have seen him in the past for borderline hypertension, but when you last saw him 2 years ago, he was on no medications. His wife has insisted that he come today because she says that he is always sleeping. She adds that he fell asleep parking the new car in the driveway and crashed through the garage wall. She states that his snoring has gotten so bad that she has moved to another bedroom. The patient admits to being even sleepier during the daytime, though he thinks that he gets a good night's sleep. He knows he snores, but it doesn't bother him. He admits to waking up tired and often with morning headaches. He is concerned about his performance at work, where he sometimes operates heavy equipment. He denies that he is depressed and states that he doesn't use drugs or alcohol. On physical exam, you note that he has gained 20 lbs since his visit 2 years ago and that he now weighs 230 lbs. He is 5' 8". Blood pressure on repeated measurement is 170/105. On examination of his throat, you notice for the first time his large uvula. Cardiopulmonary as well as neurological exams are within normal limits. Which of the following would you recommend? a) Recommend an initial trial of acetazolamide and theophylline before more invasive testing is performed. b) Order polysomnography with trial of nasal continuous positive airway pressure (CPAP). c) Ear, nose, and throat consultation for uvulopalatopharyngoplasty. d) Order polysomnography. e) Advise weight loss, exercise regimen, and follow-up visit in 1 month.
Topic Pulmonary Medicine Answer B. Order polysomnography with trial of nasal continuous positive airway pressure (CPAP). Explanation The probability of this patient having serious obstructive sleep apnea is high. Therefore, a "split-study" polysomnography should be ordered such that the technician performing the study can analyze the results during the first part of the study and institute a trial of nasal CPAP if the preliminary result is unequivocally positive for serious obstructive sleep apnea (greater than 15 significant apnea/hypopnea events per hour). In this way, a patient can be started on nocturnal nasal CPAP as soon as possible. While weight loss, alcohol avoidance, and exercise are important instructions, it is unlikely that these measures alone will work in the short term in this patient. Uvulopalatopharyngoplasty is an alternative treatment for obstructive sleep apnea, but the results to date are inferior to CPAP. Theophylline and acetazolamide have not been shown in clinical trials to be beneficial.
A 16-year-old male presents to your office with complaints of wheezing on exertion. He has recently started cross-country track and notes wheezing at the height of his exercise that often remains for several hours after activity. He has tried his friend's asthma inhaler but can't tell if it helps. He denies smoking. PMH: Good health except for prolonged ICU stay for Guillain-Barré syndrome at age 12. Normal physical exam. Which of the following is the most appropriate course of action? a) Prescribe cromolyn sodium as prophylaxis for exercise-induced asthma. b) Perform PFTs with a flow-volume loop. c) Perform fluoroscopy of the diaphragm to document residual paralysis. d) Prescribe a short-acting beta-agonist before starting exercise.
Topic Pulmonary Medicine Answer B. Perform PFTs with a flow-volume loop. Explanation This young patient has an illness script consistent with asthma: reversible airway obstruction during exercise. The first step in diagnosis of asthma is to document reversibility of the obstruction with pulmonary function studies. Some form of pulmonary function testing should always be your first step in diagnosing asthma. The methacholine challenge is used to diagnose asthma in asymptomatic patients with normal spirometry but is not 100% sensitive; therefore, usually most recommend exercising the patient and then seeing if they have a fall in FEV 1 (this was not one of the choices here but would be a good alternative in someone with classic symptoms of EIB—which his symptoms are not classic). He could have exercise-induced bronchospasm (EIB), but he complains of wheezing that begins during exercise and is prolonged. EIB usually is associated with bronchodilation during exercise and bronchoconstriction after, such that patients complain of post-exercise coughing and/or wheezing. Usually, EIB symptoms will resolve within 15-60 minutes. He could have an upper airway fixed obstruction or some form of diaphragm paralysis as a result of Guillain-Barré, but most cases of Guillain-Barré recover completely. And this patient functions very well until the peak of exercise, suggesting his diaphragm and upper airways function normally. A flow-volume loop will also provide additional information regarding the upper airway that would help diagnose vocal cord dysfunction, benign adenomas, or fixed airway obstructions as alternate causes of wheezing. Patients who wheeze daily with exertion may have true asthma, not EIB, and they benefit from long-term control therapy as opposed to simply pre-exercise medications.
A 17-year-old woman presents to your office with fever for 3 days up to 105° F, complaints of severe sore throat, refusal to swallow, and is speaking with a "hot potato" voice. On exam, her oropharynx is erythematous with exudates, and the uvula is displaced laterally. Which of the following is the most likely etiology? a) Retropharyngeal abscess b) Peritonsillar abscess c) Epiglottitis d) Acute pharyngitis
Topic Pulmonary Medicine Answer B. Peritonsillar abscess Explanation Peritonsillar abscess usually occur in older children, adolescents, and young adults. It is almost always due to either group A strep or anaerobes. Patients present with high fever, severe throat pain, trismus, "hot potato" voice, and deviation of the uvula laterally away from the abscess. Treatment is with penicillin and incision and drainage. Retropharyngeal abscess usually occurs in infants and children < 5 years old. Symptoms are sore throat, neck stiffness, fever, dysphagia, and stridor. It can be difficult to see the swelling. The epiglottis is the flap that closes off the trachea when swallowing. Epiglottitis is a soft tissue infection of this flap. Patients have swelling, dysphagia, drooling, and are at risk for pharyngeal obstruction with life-threatening respiratory compromise. Physical examination my need to be done in the operating room.
A 52-year-old smoker presents for follow-up. He has stable dyspnea on exertion, but he notes new lower extremity edema. PMH: CHD, HTN, COPD PE: O2 sat 89% JVD, trace pedal edema Barrel-shaped chest Distant heart sounds, loud P2 Left sternal holosystolic murmur Wheezes Which of the following is the most appropriate next step in patient care? a) Increase the beclomethasone dose. b) Prescribe oxygen for continuous use. c) Prescribe oxygen for use during ambulation. d) Order dobutamine stress test with nuclear images. e) Change metoprolol to an ACE inhibitor.
Topic Pulmonary Medicine Answer B. Prescribe oxygen for continuous use. Explanation This smoking patient has COPD (FEV1/FVC < 0.70) with chronic hypoxia (oxygen saturation 89%) leading to pulmonary hypertension with cor pulmonale (split S2 with loud P2 component and murmur of tricuspid regurgitation, pedal edema). Continuous oxygen therapy in patients with COPD and hypoxemia improves survival. Review the indications for continuous oxygen prescription: Resting PaO2 ≤ 55 or O2 sat ≤ 88% PaO2 ≤ 59 mmHg or O2 sat ≤ 89% if cor pulmonale or erythrocytosis (Hct > 55%) The presence of cor pulmonale in this patient makes him eligible for continuous oxygen therapy. You need to memorize these oxygen parameters. Because his symptoms are stable, except for the new lower-extremity edema, change in his inhaled steroid dose is not warranted. Changing his beta-blocker to an ACE inhibitor will not change the edema. The development of pulmonary hypertension and cor pulmonale is expected from his degree of lung obstruction, so the clinical presentation should be attributed to his long-standing COPD. He has no new symptoms to suggest further cardiac decompensation, so repeating his stress test is not the correct answer.
A 35-year-old woman who had a stillbirth at 25-weeks gestation approximately 2 weeks previously presents with increasing dyspnea for 6 days, which has progressed to the point that she cannot perform daily activities without resting. Walking to the kitchen from her den causes severe dyspnea. She has no history of orthopnea or paroxysmal nocturnal dyspnea, chest pain, cough, or hemoptysis. She has no history of swelling or pain over her calves, and she has not had a history of bleeding disorder or thromboembolism. She has had 3 full-term normal vaginal deliveries 18 years, 6 years, and 3 years previously. PAST MEDICAL HISTORY: As above; otherwise negative SOCIAL HISTORY: Vice-President of Marketing for major computer company Lives with husband and 3 children Has never smoked Drank glass of wine/week before was pregnant; none during pregnancy REVIEW OF SYSTEMS: No fever No chills No productive cough 10-lb. weight loss after delivery PHYSICAL EXAMINATION: BP 130/90 mmHg, Pulse 125/min, RR 24/min, Temp 99.1° F HEENT: PERRLA, EOMI TMs clear Throat clear Neck: Supple; no masses Right jugular venous pressure was not raised Wave pattern showed prominent Y collapse Heart: Point of maximal impulse: 5th intercostal space in the midclavicular line with a parasternal heave S1 was loud; S2 with normal splitting with a loud pulmonary component S3 was present Holosystolic murmur over left lower sternal border; increasing with inspiration Lungs: Scattered rhonchi Abdomen: Bowel sounds present; no masses; no hepatosplenomegaly, nontender Extremities: No cyanosis, clubbing, or edema LABORATORY: ECG: Right axis deviation, an S-wave in Lead I, and a Q-wave and inverted T-wave in lead III (S1, Q3, T3). T-wave inversion in leads V2 and V3, and S wave persistence in leads V5 and V6. CXR: Normal CBC: WBC 7100/mm3; hemoglobin 11.5 gm/dL; platelets 375,000 ESR: 15 mm in 1 hour Glucose: 90 mg/dL BUN: 30 mg/dL Creatinine: 0.8 mg/dL Echocardiogram: Dilatation of the right ventricle and right atrium; moderate valvular insufficiency noted. Based on your findings, which of the following is the most likely diagnosis? a) Myocardial infarction b) Pulmonary embolism c) Sepsis d) Bacterial pneumonia e) Bacterial endocarditis
Topic Pulmonary Medicine Answer B. Pulmonary embolism Explanation There is an increased risk of thromboembolic disease during pregnancy with possible contributing factors including: Decreased vasomotor tone due to increased prostaglandins Venous compression by the gravid uterus Hypercoagulability due to increased levels of coagulation factors I, II, VIII, IX, and X Decreased plasma fibrinolytic activity This patient had her onset of pulmonary embolism approximately 1 week ago. Thrombolytic therapy is as effective if started in 6-14 days as compared to within 5 days of pulmonary embolism. Her ECG is classic. The S1, Q3, T3 pattern is what is classically described but usually not seen. Echocardiogram is consistent with PE. With treatment and resolution, her tricuspid regurgitation disappeared. Her right ventricle also returned to normal size as did her pulmonary artery systolic pressure.
A 50-year-old African-American woman with a history of hypertension and obesity presents with a 4-month history of worsening shortness of breath. She had been able to walk up a flight of stairs without any difficulty about 3 months ago. Now she complains of shortness of breath while walking around her home. Also, at night she has a new onset of orthopnea. She denies other symptoms at this point. PAST MEDICAL HISTORY: Hypertension for 10 years, treated with HCTZ 25 mg daily Delivered 5 healthy children in her 20s; no problems during pregnancies SOCIAL HISTORY: Works in a day care with 30 preschool children Says her job is very stressful but less stressful than her previous job, which was as a housemother for a fraternity house Never smoked Never drank alcohol No pets Widowed; lives alone FAMILY HISTORY: Father, 80; recent MI 1 year ago Mother, 79; recent admission to nursing home for Alzheimer's Sister, 55; healthy but obese Brother, 53; hypertension, on medication Sister, 48; with SLE REVIEW OF SYSTEMS: No headaches No chest pain No cough No fever No sweats No rashes No joint complaints PHYSICAL EXAMINATION: BP 120/85, P 90, RR 16, Temp 98.8° F Ht 5'2", Wt 260 lbs HEENT: PERRLA, EOMI, discs sharp Throat clear Neck: Supple, no masses Heart: RRR without murmurs, rubs, or gallops; loud pulmonic second sound Lungs: Clear to auscultation Extremities: Bilateral 2+ pitting edema; no cyanosis or clubbing noted Skin: No rashes Rectal: Heme-negative LABORATORY: Pulse oximetry on room air was 93% at rest; with walking dropped to 87% CXR: Echocardiogram: Ejection fraction of 80% and pulmonary hypertension with a PA systolic pressure of 61 mmHg. Right ventricular size and function were normal. Pulmonary function tests: FEV1 of 0.9 L (43% predicted) and an FVC of 1.6 L (59% predicted). High-resolution CT scan showed diffuse pulmonary nodules and hilar enlargement. Three sputum samples for acid-fast bacilli were negative. She underwent bronchoscopy with transbronchial biopsy for evaluation of her pulmonary nodules. The bronchoscope showed hyperemia with nodular irregularities and distal concentric narrowing in the main, segmental, and proximal subsegmental bronchi. The transbronchial biopsy is shown in the image below. Special stains for fungi and acid-fast bacilli were negative. Which of the following is the most likely diagnosis? a) Bacterial pneumonia b) Pulmonary sarcoidosis c) Asbestosis d) Tuberculosis e) Granulomatosis with polyangiitis (Wegener's)
Topic Pulmonary Medicine Answer B. Pulmonary sarcoidosis Explanation Her chest radiograph shows a diffuse miliary pattern, which is suspicious for tuberculous or metastatic carcinoma. Her biopsy shows 2 small, non-necrotizing granulomas. The differential diagnosis includes mainly infectious granulomatous disease, hypersensitivity pneumonia, and sarcoidosis. Granulomas may also be seen with aspiration pneumonia, in chronic beryllium disease, or exposure to titanium or aluminum dusts, as a response to certain drugs, and rarely with granulomatosis with polyangiitis (Wegener's), lymphocytic interstitial pneumonia, or eosinophilic pneumonia. Tuberculosis has been effectively ruled out at this point, and bacterial pneumonia does not fit this clinical picture or any of the results. Asbestosis would cause parenchymal fibrosis and involves lower lobes. The bronchoscopy findings are inconsistent with asbestosis. Granulomatosis with polyangiitis (Wegener's) is on the differential as mentioned above, but we have no evidence of renal disease, and it is much rarer than sarcoidosis in the African-American population.
On a routine x-ray film of the chest, a 62-year-old man has a solitary pulmonary nodule. He has smoked one pack of cigarettes daily for 40 years and has had early-morning cough and expectoration for 5 years. The lesion is 2.4 cm in diameter, peripheral, and contains no calcium deposits on chest CT. PMH is significant for rheumatoid arthritis for 10 years. Physical examination confirms bilateral metacarpal involvement with bilateral ulnar deviation of the digits. He has rheumatoid nodules over the extensor surface of his right forearm. Laboratory tests show a rheumatoid factor of 1:1280. His tuberculosis skin test was positive 2 years ago, and he completed 9 months of isoniazid. The patient feels well, except for his morning cough. Which of the following is the most appropriate management? a) Obtain sputum for mycobacterial culture and acid-fast smear. b) Refer for biopsy or surgical removal of the nodule. c) Repeat a low-dose helical CT scan in 3 months. d) Advise the patient to stop smoking; perform no additional studies. e) Start etanercept.
Topic Pulmonary Medicine Answer B. Refer for biopsy or surgical removal of the nodule. Explanation Although this patient has rheumatoid arthritis, a disease that is associated with the development of necrobiotic rheumatoid nodules in the lung, the burden of the correct diagnosis lies with the physician. The lesion cannot be assumed to be benign. Because patients with rheumatoid arthritis who smoke tobacco can develop lung carcinoma, tissue must be obtained and examined. Recognize also that the patient has all of the concerning features: advanced age, smoking history, no calcium deposition, and a larger-sized nodule. His previous positive TB skin test and treatment should not dissuade you from working up his solitary pulmonary nodule. This kind of question will be straightforward. The ABIM won't ask you about handling tiny nodules in low-risk patients, because different centers have different reasonable approaches. But the Boards may ask you to recognize the high-risk patient who definitely requires tissue resection.
Which of the following is associated with lower lobe interstitial infiltrates? a) Silicosis b) Rheumatoid fibrosis c) Ankylosing spondylitis d) Eosinophilic granuloma e) Pneumocystis jiroveci pneumonia in patients on inhaled pentamidine therapy
Topic Pulmonary Medicine Answer B. Rheumatoid fibrosis Explanation Rheumatoid fibrosis, asbestosis, and other pulmonary manifestations of connective tissue diseases, such as SLE, present with interstitial infiltrates of the lower lobes. The other conditions present with upper lobe disease.
A 32-year-old female complains of cough for 1 year duration. She also has exertional dyspnea as well as a purplish-colored rash that recurs intermittently across her cheeks and shins. Chest radiograph reveals bilateral hilar lymphadenopathy. She has no significant past medical history, no significant family history, and currently works in a supermarket. She denies smoking, alcohol, or drug use except for occasional marijuana. Mediastinoscopy is done to assess the enlarged lymph nodes. Biopsy indicates "the presence of well-formed non-caseating granulomas." Bacterial, AFB, and fungal cultures are all negative. Which of the following disorders is the most likely diagnosis? a) Lymphoma b) Sarcoidosis c) Tuberculosis d) Histoplasmosis e) Berylliosis
Topic Pulmonary Medicine Answer B. Sarcoidosis Explanation Sarcoidosis is a non-infectious multisystem disease of unknown cause, commonly affecting young and middle-age adults. The most common radiographic presentation is bilateral symmetrical hilar lymphadenopathy, although lung parenchymal changes are also quite common. Other extrapulmonary symptoms are often seen and can affect nearly every organ system. Generally occurs in the middle-age group, more common in women, and more common in African-Americans in the United States. The typical biopsy will reveal non-caseating granulomas as seen also in berylliosis, but no exposure history provided. Formation of granulomas is also seen in fungal and mycobacterial diseases, such as tuberculosis and histoplasmosis, but these tend to be caseating and associated with positive cultures. Although berylliosis and lymphoma can both present with lymphadenopathy, pathology characteristics are not those of granulomatous disease.
A 24-year-old male who lives in southwestern Ohio comes to the emergency department complaining of a rash on his legs. He reports that these "red bumps" erupted about 1 week ago, and they are becoming more painful. He has no significant past medical history and is a nonsmoker. He has no recent exposures to any new dusts, chemicals, detergents, etc. He also denies any new medications. He did eat shrimp about 10 days ago but does this often and has never had any reactions. He has never been allergy tested, but he has never had any food-related reactions. His rash appears below: On a previous emergency department visit, he had a chest radiograph done for cough about 2 months prior, which is also below: What is the most likely diagnosis? a) Berylliosis b) Sarcoidosis c) Histoplasmosis d) Poison ivy e) Allergic reaction to shellfish
Topic Pulmonary Medicine Answer B. Sarcoidosis Explanation This patient has classic features of Löfgren syndrome, which includes erythema nodosum and bilateral symmetrical hilar lymphadenopathy. Sarcoidosis is a noninfectious multisystem disease of unknown cause, commonly affecting young and middle-aged adults with pulmonary symptoms or findings being most common, specifically bilateral symmetrical hilar lymphadenopathy and pulmonary infiltrates. It can also cause skin lesions, eye lesions, and affect other organs including the liver, spleen, lymph nodes, heart, and central nervous system. The other diagnoses listed do not have the same constellation of skin and lung findings and thus are incorrect.
A 74-year-old woman with a 100-pack-year history of smoking (2 ppd for 50 years) is essentially dragged in by her husband, who says that she has withered away to nothing and has been acting very confused lately. She says she is fine but keeps calling you her grandchild. You don't get much more information out of her. PAST MEDICAL HISTORY: Hysterectomy 24 years ago HTN for 20 years; on an ACE inhibitor SOCIAL HISTORY: Retired used car salesperson Lives with her husband of 50 years Doesn't drink FAMILY HISTORY: Father died at age 75 of lung cancer Mother died at age 74 of lung cancer Brother died at age 74 of lung cancer Sister died at age 74 of lung cancer Brother recently diagnosed with lung cancer REVIEW OF SYSTEMS: No fever or chills Has had night sweats on occasion 30-lb weight loss in last 6 months No appetite Coughed up blood once last week (about a teaspoon, according to husband) PHYSICAL EXAMINATION: Oriented only to person, place; thinks the year is 1965 BP 110/70, P 92, RR 14, Temp 99° F, Ht 5' 2", Wt 140 HEENT: PERRLA, EOMI TMs clear Throat clear Neck: Supple, no masses Heart: RRR with II/VI systolic murmur (heard for 10 years now) Lungs: Coarse scattered crackles; no focal finding Abdomen: Bowel sounds present; no hepatosplenomegaly Extremities: No cyanosis, clubbing, or edema LABORATORY: CXR: Central mass seen in right hilum; no cavitation noted Serum sodium: 120 mg/dL Urine sodium: 60 mg/dL (normal should be less than 10 mg/dL with this serum sodium) Based on your findings, which of the following types of lung cancer does this woman most likely have? a) Adenocarcinoma b) Small cell carcinoma c) Large cell carcinoma d) Bronchoalveolar carcinoma e) Squamous cell carcinoma
Topic Pulmonary Medicine Answer B. Small cell carcinoma Explanation Let's put this together. It is central, which means it is either Small Cell or Squamous Cell (Remember S-S-Sentral). Now you presume from the information that the patient has SIADH, which generally is associated with small cell. Therefore, put together central carcinoma with SIADH and you get small cell as the answer. Adenocarcinoma, large cell, and bronchoalveolar (a type of adenocarcinoma) are usually peripheral.
A 23-year-old female has a history of asthma since age 16. She reports symptoms of dry cough and exertional dyspnea. Her cough is worse at night but also worsens with cold air. She has symptoms almost every night. She has an inhaler of albuterol that she uses nightly and as needed for increased shortness of breath. She denies smoking, has no pets, and currently is a full-time student. She has not noticed any seasonal variation or any worsening with different environments. She denies symptoms of reflux. What is the next best step in management? a) Refer for allergy testing. b) Start treatment with a medium dose inhaled corticosteroid along with albuterol as needed. c) Start treatment with a long-acting beta-agonist along with albuterol as needed. d) Initiate treatment with a proton pump inhibitor nightly.
Topic Pulmonary Medicine Answer B. Start treatment with a medium dose inhaled corticosteroid along with albuterol as needed. Explanation This patient with a history of chronic asthma has symptoms consistent with uncontrolled asthma. Her chronic dry cough, exertional dyspnea, and nocturnal cough are common symptoms found in asthma. Additionally, increased symptoms with certain exposures such as cold air, perfumes, smoke, dusts, pets and pet dander, and other allergens suggest a history of bronchial hyperresponsiveness. This patient has almost daily symptoms consistent with moderate persistent asthma. Based on NHLBI guidelines for asthma management, first-line treatment for persistent asthma of any severity is an inhaled steroid (ICS). As the severity increases, then the dosage of the ICS may be increased with other therapies added on. For those with moderate severity asthma, long acting beta-agonists may be added to low dose ICS for maintenance. However, long acting beta-agonists (LABAs) should not be used in asthma without the concomitant use of ICSs. In fact, some studies have suggested potential for harm in patients who received LABAs alone. Allergy testing might be helpful for those patients with uncontrolled asthma despite adequate maintenance therapy. Additionally, patients may report worsening of symptoms with certain exposures in different locations, and allergy testing might help the patient with allergen avoidance. In this patient, however, no history of worsening with exposures nor adequate maintenance therapy was given. Gastroesophageal reflux disease (GERD) may be a cause for worsening asthma symptoms. However, studies have shown that treatment of asymptomatic GERD does not improve symptoms. This patient did not endorse symptoms of GERD, and, thus, a proton pump inhibitor is unlikely to be helpful.
A 52-year-old female was evaluated for a lung mass three months ago. This was biopsied via bronchoscopy, and pathology was consistent with lung adenocarcinoma. After further evaluation, she was determined to have Stage IIIB lung carcinoma. She has completed her first cycle of combined chemotherapy and radiation therapy. Today, she complains of a one-week history of right lower extremity edema and pain, as well as a sudden onset of left-sided pleuritic chest pain associated with shortness of breath. Her vitals are as follows: HR 120 bpm, RR 24, oxygen saturations 92% RA, blood pressure 125/80. Duplex ultrasonography of the right leg reveals an occlusive thrombus of the right common femoral vein. Serum creatinine level is 2.3. Which of the following is the next appropriate action? Start LMWH treatment. Start unfractionated heparin treatment. Obtain chest CT angiogram. Start infusion of thrombolytic agent. Obtain V/Q scan.
Topic Pulmonary Medicine Answer B. Start unfractionated heparin treatment. Explanation This patient has clear evidence of venous thromboembolic (VTE) disease with a DVT and high clinical probability of a pulmonary embolism. Also, this patient is at substantial continued risk for VTE because of her malignancy. Initiate anticoagulation immediately before any further testing is done. Unfractionated and LMW heparins have been shown to decrease the risk of PE compared to warfarin and are recommended. Unfractionated heparin does not rely on normal renal clearance and is appropriate, but LMW heparins are renally cleared and could accumulate unpredictably in this patient with decreased renal function. Further studies, such as a chest CT angiogram and V/Q scan are not needed in this case as diagnosis has been made. Also, with impaired renal function, the contrast associated with a chest CTA would be a relative contraindication for ordering this test. This patient has relatively stable hemodynamics with only mild hypoxemia. Thrombolytics are not appropriate for this patient.
A 29-year-old female is 32 weeks pregnant with her first pregnancy. She has a history of mild intermittent asthma for which she uses an albuterol inhaler occasionally. She presents to your office with a 5-day history of increased dyspnea associated with dry cough and some intermittent wheezing. She denies any reflux symptoms, fever, nasal drainage, or recent sick contacts. On exam, she is slightly tachypneic with a respiratory rate of 20; the remainder of the vital signs are normal. Lung exam reveals mild expiratory wheezing with slightly prolonged expiratory phase. Which of the following is true regarding asthma in pregnancy? a) Budesonide is the only inhaled steroid with a Class X rating in regard to safety in pregnancy. b) The management principles for asthma are the same in pregnant and non-pregnant patients. c) Approximately 80% of patients with asthma will have symptoms that worsen with pregnancy. d) Oral corticosteroids would be contraindicated in patients with asthma exacerbation and pregnancy. e) Controlled asthma during pregnancy is associated with a higher risk of preeclampsia.
Topic Pulmonary Medicine Answer B. The management principles for asthma are the same in pregnant and non-pregnant patients. Explanation Worsening of asthma during pregnancy is very common, but 80% do not worsen. A simple rule is that 1/3 of patients will have asthma that gets worse, 1/3 that gets better, and 1/3 where symptoms remain stable. In patients with uncontrolled asthma, the management principles are the same in both pregnant and non-pregnant patients. It is important to manage asthma aggressively as uncontrolled asthma is associated with a higher risk of preterm delivery, low birth weight, and complications such as preeclampsia. Inhaled steroids would be the mainstay of treatment for persistent asthma. Although budesonide is the only agent in this class with a Class B safety rating, it is considered safe to use other inhaled steroids, particularly if a patient is already controlled on one prior to pregnancy. Oral corticosteroids may be required for severe asthma exacerbations. While they carry side effects, such as hyperglycemia, they should be used if the clinical situation warrants it as the risk to the fetus is high with severe asthma exacerbations.
A 26-year-old female is admitted to the ICU after an emergency exploratory laparotomy for a ruptured ectopic pregnancy. She received 16 units of packed cells in the OR, and a Swan-Ganz catheter was placed by anesthesia because they heard rales and were concerned about volume overload. On admission, she is in shock with a blood pressure of 70/30 and a hemoglobin of 5.0 g/dL. She is on mechanical ventilation in CMV mode with FiO2 60%, PEEP 10 cm, rate 20, and tidal volume 700 cc. Two additional units of packed cells were given on arrival to the ICU; the blood pressure came up to 100/60, and the heart rate is 120/minute. Post-transfusion labs reveal a hemoglobin of 7.0 g/dL, and the Swan-Ganz readings show a cardiac output of 10 L/min, pulmonary arterial "wedge" pressure of 12 mmHg, and a systemic vascular resistance of 600 dyne-sec-cm-5 (low!). The arterial blood gas now is pO2 85 mmHg, pCO2 44 mmHg, and pH of 7.26. Which of the following therapies is the most appropriate next step in continuing the resuscitation of this patient? a) Start dopamine at 10 µg/kg/min to increase the systemic vascular resistance. b) Transfuse 2 units packed red blood cells. c) Increase FiO2 to 70%. d) Increase the rate of the ventilator to increase the minute ventilation and lower the pCO2. e) Start dobutamine at 10 µg/kg/min to increase the cardiac output.
Topic Pulmonary Medicine Answer B. Transfuse 2 units packed red blood cells. Explanation This patient is still only partially resuscitated with evidence for both hemorrhagic shock and the hyperdynamic phase of SIRS (systemic inflammatory response syndrome). Most important is the continued acidosis, which is predominantly a metabolic acidosis and presumably a lactic acidosis indicating inadequate tissue perfusion. The best way to continue the resuscitation is to improve oxygen delivery (DO2) to the tissues and monitor the ABG and lactate level to assess improvement. Oxygen delivery is determined by cardiac output, hemoglobin, and oxygen saturation. In this case, the CO2 is already high, and the oxygen saturation with a pO2 of 85 is at least 90%. Increasing the pO2 will not provide much more hemoglobin saturation. Therefore, the best way to improve oxygen delivery is to increase the hemoglobin through additional packed red cell transfusions. Increasing the minute ventilation to compensate for the metabolic acidosis will only mask the problem. Dobutamine will reduce the systemic vascular resistance even more, resulting in further hypotension. Dopamine will increase the blood pressure and cause further tachycardia with no improvement in oxygen delivery to the tissues.
A 62-year-old male presents to your office for follow-up. He has a history of COPD but has had no recent testing. He complains of chronic dyspnea on exertion walking on flat ground after 50 feet as well as shortness of breath with stairs. He has not previously been intubated and was hospitalized twice in the last six months for respiratory issues. His only respiratory-related medication included albuterol q6h as needed for dyspnea. He is an active smoker with a 50-pack-year history of smoking. Pulmonary Function Tests were performed: FEV 1 : 1.5L (49% predicted) FVC: 3.0L (64% predicted) FEV 1 /FVC: 50% Oximetry Testing: Resting, Room Air: > 95% Exercise, Room Air: > 93% Overnight, Room Air: > 91% Based on current evidence, which of the following would not be beneficial for this patient? a) Pulmonary rehabilitation b)Use of oxygen at 2 LPM daily with rest and exertion c) Annual influenza vaccine d) Daily use of an inhaled corticosteroid e) Smoking cessation
Topic Pulmonary Medicine Answer B. Use of oxygen at 2 LPM daily with rest and exertion Explanation The patient described has obstructive disease based on his PFTs. Based on his FEV 1 , his COPD disease severity would be categorized as severe. Multiple interventions have proven to be beneficial for these patients, with endpoints including improvements in survival, decreased number of exacerbations, reduced hospitalizations, and improved quality of life. Smoking cessation has been shown to significantly reduce the decline of lung function in a previous smoker. Although lung function will not substantially improve, the rate of decline can approximate that of a nonsmoker after one year of smoking cessation. Pulmonary rehabilitation has been shown to reduce hospitalizations, increase exercise endurance, and improve quality of life. It should be considered in all patients with symptomatic COPD. Inhaled corticosteroids have been shown to reduce the number of exacerbations in those patients with COPD with an FEV 1 < 50% predicted. The influenza vaccine should be considered in all patients with COPD regardless of severity. It has been shown to lead to a 52% reduction in hospitalizations for all episodes of influenza and pneumonia and a 70% reduction in deaths from all causes. Oxygen is the only modality proven to improve the long-term mortality in those patients with COPD who meet certain qualifications. The criteria to start continuous oxygen (ideally 24 hrs but 15 hrs minimum) include: Resting P a O 2 < 55, O 2 Sat < 88%, P a O 2 < 59 with evidence of cor pulmonale or erythrocytosis. The patient in this scenario does not meet these criteria based on oximetry testing, and thus the use of oxygen has not been shown to be beneficial in this population.
A 67-year-old male presents with complaints of shortness of breath. It has been occurring for years, but he has noticed it more over the last 6 months. He complains of exertional dyspnea not associated with any chest pain but does have some occasional dry cough. He smokes 1 pack per day and has been doing so since age 31. His only other significant past medical history includes hypertension and diabetes. His examination reveals a normal cardiac exam with distant breath sounds on lung auscultation without any adventitial sounds. Chest radiograph reveals mild hyperexpansion and mild cardiomegaly without any other major abnormalities. He is sent for pulmonary function tests: Actual: Predicted: FVC: 2.2 L 51% FEV1: 0.98 L 31% FEV1/FVC: 44% TLC: 7.51 L 125% DLCO: 9.03 mL/min/mmHg 25% The pattern seen on pulmonary function testing is most consistent with which of the following? a) Moderate obstructive airways disease without hyperinflation b) Very severe obstructive airways disease with hyperinflation c) Mixed restrictive and obstructive pattern d) Isolated reduction in diffusion capacity related to pulmonary vascular disease e) Moderate restrictive disease
Topic Pulmonary Medicine Answer B. Very severe obstructive airways disease with hyperinflation Explanation The patient has a reduction in the ratio of the forced expiratory volume of 1 second (FEV1 ) to the forced vital capacity (FVC). A ratio less than 70% is consistent with obstructive disease. The severity of obstruction is characterized by the FEV1 percent predicted. An FEV1 of: FEV1 > or equal to 80% predicted—mild obstruction FEV1 50-80% predicted—moderate obstruction FEV1 30-50% predicted—severe obstruction FEV1 < 30% predicted—very severe obstruction Increases in the total lung capacity (TLC) greater than 120% predicted are consistent with hyperinflation. TLC < 80% would be consistent with a restrictive disease pattern that this patient does not have. Clearly, there is more than just an isolated diffusion abnormality, but causes of an isolated DLCO include pulmonary vascular disease and anemia.
Which of the following is most important in distinguishing between ventilatory failure and a gas exchange defect in a patient with breathing difficulties? a) Pulse oximetry b) PaCO2 c) A-a gradient d) PaO2 levels e) Serum pH
Topic Pulmonary Medicine Answer C. A-a gradient Explanation Inadequate oxygen levels leading to hypoxia can occur from: 1) Defects in moving oxygen-containing air from the environment to the alveoli (hypoventilation) 2) Difficulties exchanging oxygen across the alveolar membranes 3) Compromises in circulating adequate oxygen-saturated blood cells to the tissues Ventilatory deficiencies and abnormal alveolar exchange both lead to a diminished PaO2 with an elevated PaCO2. Oxygen saturation by pulse oximetry measurement will be low and and serum pH will be low. Comparison of the oxygen in the alveoli and the oxygen in the arterial blood (the A-a gradient) can help differentiate an alveolar gas exchange problem from a ventilatory defect.
A 72-year-old male comes in for follow-up evaluation. He has a known history of emphysema. He complains of persistent shortness of breath with exertion, which he says is getting worse. He can only walk about half a block on flat ground prior to getting dyspneic. He is significantly impaired with steps. He smoked 1 pack per day for 45 years and still smokes about 1/2 pack per day. He denies any exertional chest pain, palpitations, or lower extremity edema. His current medications include tiotropium bromide q day, salmeterol/fluticasone combination discus bid, and albuterol MDI qid. On exam: P 100; RR 28; oxygen saturation on RA 89%; BP 110/65 Wt: 52 kg (decreased by 3 kg since last visit) Lungs: severely diminished breath sounds bilaterally, prolonged exhalation, no wheezing His PFTs are as follows: FVC: 2.05 L (65% predicted) FEV1: 0.87 (35% predicted) FEV1/FVC: 55% TLC: 4.12 L (125% predicted) DLCO: 11.2 (45% predicted) Which of the following should be offered to this patient next? a) Smoking cessation program. b) Oxygen determination with rest, exercise, and nocturnally. c) All of the choices are correct. d) Referral to pulmonary rehabilitation. e) Influenza vaccine annually.
Topic Pulmonary Medicine Answer C. All of the choices are correct. Explanation This patient has COPD and most likely emphysema, which would be categorized as severe based on his FEV1. He also has significant exertional dyspnea as well as signs of hyperinflation on PFTs and muscle wasting, which is a component of advanced emphysema. He is already being treated with the conventional medications for COPD including long-acting beta-agonists and anticholinergics. In addition, inhaled steroids are indicated for those with moderate-to-severe COPD. All patients with COPD, regardless of severity, should be offered the influenza vaccine annually. This patient appears to have resting room air hypoxemia, which likely worsens with exertion. Oxygen has been the only specific therapy that has been shown to decrease mortality in patients with COPD, particularly if testing reveals desaturations below certain thresholds and the patient uses it for greater than 14 hours/day. Pulmonary rehabilitation has been proven to increase exercise capacity, decrease hospital admissions, and decrease dyspnea scores. It should be considered in all patients with moderate COPD or worse. All patients should be counseled regarding smoking cessation, especially those with impaired lung function. If a patient does quit smoking, the rate of decline of lung function can approximate the rate of decline for a nonsmoker after 1 year of smoking cessation.
You are presented a patient with respiratory problems. Her PFT patterns are as follows: Reduced expiratory flows Increased total lung capacity Reduced vital capacity Mild increase in DLCO Significant response to bronchodilator Based on the above results, which of the following diagnoses is most likely? a) Sarcoidosis b) Interstitial lung disease c) Asthma d) COPD e) Emphysema
Topic Pulmonary Medicine Answer C. Asthma Explanation Note the clues here: 1) significant response to bronchodilator; few of the diagnoses will significantly improve with this. Emphysema will not respond to bronchodilators; COPD may have a little improvement but not marked; interstitial lung disease and sarcoid will not be affected. 2) For the DLCO, only asthma has a potential to have a mild increase in DLCO; all the rest will be normal or reduced. Just about all will have reduced expiratory flows except for interstitial lung disease, which can have reduced flows but commonly will have slightly increased values to normal. Total lung capacity and vital capacity are not much help, since asthma will have similar patterns as compared to COPD and emphysema for these 2 values. Hyperinflation is common in both asthma and emphysema. So, key elements to look for to differentiate: Asthma will have mild increase in DLCO and respond to bronchodilators. COPD will have a normal or reduced DLCO and normal to slight increase in total lung capacity. Emphysema will have decreased DLCO with increased total lung capacity (hyperinflation). Finally, interstitial lung diseases will have reduced DLCO with total lung capacity reduction as well
Your receptionist asks you to see her 17-year-old daughter. The daughter has had a fever and cough for 2 days and is not better with over-the-counter medications. Her cough has become productive in the past day, and last night she coughed most of the night. Her fevers have ranged up to 101.5° F. She feels bad and complains of generalized body aches. PAST MEDICAL HISTORY: Negative SOCIAL HISTORY: Lives with her mother and 2 brothers, ages 12 and 10 Attends high school and makes As and Bs, except a D in Art Denies smoking or drinking FAMILY HISTORY: Mother 40 Father 42, healthy 2 brothers, healthy REVIEW OF SYSTEMS: Essentially negative PHYSICAL EXAMINATION: BP 100/60, P 90, Temp 100° F, RR 14 HEENT: PERRLA, EOMI, wears glasses TMs clear Throat clear; non-erythematous Neck: Supple no masses Heart: RRR with ejection click; no murmurs, rubs, or gallops Lungs: Coarse crackles heard at the left base Abdomen: Bowel sounds present; no hepatosplenomegaly Extremities: No cyanosis, clubbing, or edema Skin: No rashes LABORATORY: CXR: Left lower lobe infiltrate Based on your findings, which of the following is the best antibiotic choice for her? a) Amoxicillin-clavulanate 850 mg PO bid for 10 days b) Cefuroxime 250 mg PO bid for 10 days c) Azithromycin 250 mg two tabs PO today, then 1 tab PO q day x 4 days d) Amoxicillin 500 mg tid for 10 days e) Levofloxacin 500mg q day for 10 days
Topic Pulmonary Medicine Answer C. Azithromycin 250 mg two tabs PO today, then 1 tab PO q day x 4 days Explanation This patient could have Streptococcus pneumoniae, Mycoplasma pneumoniae, or Chlamydophila pneumoniae. Azithromycin and levofloxacin are the only agents listed that are effective against all 3 organisms. The problem is that she is 17. You must be cautious when giving a quinolone to a patient under the age of 18 except in special circumstances (anthrax, cystic fibrosis patients, etc.) for skeletal maturity concerns. The beta-lactam antibiotics listed are great for pneumococcus but won't get the atypicals. In a young patient with uncomplicated pneumonia, look for a macrolide or doxycycline as an acceptable choice.
A 40-year-old female comes to your office complaining of increased dyspnea and coughing accompanied by fever and night sweats. She is well known to you and has had stable asthma for the past 10 years, with good control achieved with her inhaled corticosteroid and occasional use of her beta-2 agonist inhaler. This is her third visit to your office in the past 2 months for "exacerbation of her asthma." CBC indicates a mildly elevated WBC count with 15% eosinophils. Which of the following would a chest x-ray most likely reveal? a) Right upper lobe cavity b) Segmental atelectasis c) Bilateral peripheral infiltrates d) No infiltrates
Topic Pulmonary Medicine Answer C. Bilateral peripheral infiltrates Explanation This presentation of a 40-year-old woman with a history of fever, night sweats, asthma exacerbation, and new eosinophilia is classic for chronic eosinophilic pneumonia. The CXR is classically referred to as the "photographic negative" of pulmonary edema. ABPA would be suggested by a history of coughing brownish sputum and the subsequent mucous plugging that can lead to segmental atelectasis on chest x-ray. There is always a possibility of pulmonary tuberculosis (right upper lobe cavity), but the eosinophilia would not be a hallmark.
What is the most likely fungal lung disease in a patient from Arkansas or Missouri with a draining leg lesion? a) Aspergillosis b) Histoplasmosis c) Blastomycosis d) Streptococcus pneumoniae e) Coccidioidomycosis
Topic Pulmonary Medicine Answer C. Blastomycosis Explanation Blastomycosis would present with lung and skin/bone lesions in Arkansas/Missouri. Note that histoplasmosis is very common also but does not have the draining skin lesions. Aspergillosis would be seen in immunocompromised and does not present with draining skin lesions. Coccidioidomycosis is seen in the desert Southwest but can have bony lesions that might drain.
A 60-year-old man has had a cough with production of sputum for the past month. He is afebrile with decreased breath sounds at the right lung base. A chest radiograph reveals an area of consolidation in the right lower lobe with air bronchograms. He is given antibiotic therapy; but when he is seen 2 months later, the chest radiograph is unchanged, and his cough continues. Sputum is clear but has a "salty" taste. Which of the following is the most likely diagnosis? Lipoid pneumonia from aspiration Obstructive pneumonia from squamous cell carcinoma Bronchoalveolar cell carcinoma Unresolved Mycoplasma pneumoniae Tuberculosis
Topic Pulmonary Medicine Answer C. Bronchoalveolar cell carcinoma Explanation This case details a patient with a clear pulmonary malignancy. But you are given additional information: sputum with a "salty taste." You are asked to associate this information with bronchoalveolar cell carcinoma, a subtype of adenocarcinoma that is often accompanied by bronchorrhea and a salty taste. The other choices are not associated with a classic salty taste. This is a straightforward recall question, and on occasion, the Boards will throw you a few of these.
A 60-year-old woman who has been on mechanical ventilation for 1 week due to ARDS from a pneumococcal pneumonia is slowly being weaned. Clinically, she is doing well and you are pleased with her progress. MEDICATIONS: Day 8 of ceftriaxone PHYSICAL EXAMINATION: HEENT: Pupils responsive and equal Mild thrush of her oral mucosa Neck: Supple, no masses Heart: RRR without murmurs, rubs, or gallops Lungs: Still with basilar crackles right greater than left Abdomen: Positive bowel sounds, tolerating tube feeds well; no masses Extremities: No cyanosis, clubbing, or edema LABORATORY: CBC shows a mild increase in WBC to 11,000 from 9,500 yesterday with 80% lymphs Tracheal aspirate culture from 2 days ago returns today and shows Pseudomonas aeruginosa sensitive only to amikacin, piperacillin/tazobactam, and ceftazidime AST 25 ALT 26 Bilirubin 0.2 mg/dL Creatinine 0.5 mg/dL BUN 10 mg/dL CXR: Slow improvement from admission; no new infiltrates Based on clinical evaluation and laboratory results, which of the following is the most appropriate next step? a) Switch antibiotic coverage to piperacillin/tazobactam alone. b) Add amikacin to ceftriaxone. c) Continue current therapy. d) Switch antibiotics to piperacillin/tazobactam + amikacin. e) Perform bronchoscopy and then start piperacillin/tazobactam + amikacin.
Topic Pulmonary Medicine Answer C. Continue current therapy. Explanation Clinically she is doing well. You are weaning her off the vent; her physical examination is stable; her laboratory is stable (never base antibiotic therapy on a minor bump in WBC). The sputum results are not unusual for a patient in the Intensive Care Unit; they will frequently become colonized with gram-negative organisms—particularly Pseudomonas. Never change therapy based on just a tracheal aspirate; you must have some other change in exam or laboratory that is significant for you to consider treating the organism found on a tracheal aspirate.
A 21-year-old woman is brought to your office, accompanied by her parents, after a "near syncope" event at home while they were carrying the daughter's trunk of dirty laundry into the house on her arrival from college. The daughter admits to feeling lightheaded while carrying the laundry and says that she is often short of breath climbing the 2 flights of stairs to her dormitory room. Upon further questioning, she admits to substernal chest pain accompanying her exertional dyspnea, but states that it isn't important because "I'm too young to have a heart attack." She denies fever, cough, hemoptysis, wheezing, orthopnea, or ankle swelling. She is on no medications and denies smoking or illicit drugs. She is sexually active but has never used oral contraceptives. On physical exam, she is afebrile with a heart rate of 90 and a blood pressure of 110/60. She is of average build and in no apparent distress. Neurological exam is normal. Her lungs are clear on auscultation. Her cardiovascular exam is remarkable for a parasternal heave and an increase in the intensity of P2. You perform a chest x-ray, which is normal, and an ECG, which reveals normal sinus rhythm with right axis deviation. CBC and basic chemistries are normal. Pregnancy test is negative. Which of the following is the most appropriate procedure for further evaluation of this patient? a) Pulmonary function tests b) Exercise stress test c) Echocardiogram d) V/Q scan e) Right heart catheterization
Topic Pulmonary Medicine Answer C. Echocardiogram Explanation Look for the young woman with exertional dyspnea, chest pain, and has physical exam findings consistent with right heart strain. The next step in the workup is an echocardiogram to document the existence of pulmonary hypertension and exclude significant mitral stenosis. After echocardiographic documentation of pulmonary hypertension, the workup would proceed in the absence of any discernible parenchymal lung disease to V/Q scan to look for thromboembolic disease. A pulmonary angiogram would be helpful in evaluating any perfusion defects seen on V/Q. The role of right heart catheterization is further down the road when you are considering vasodilator therapy in the patient with primary pulmonary hypertension. We do not suspect cardiac ischemia. Pulmonary function tests (particularly diffusion capacity) are helpful in assessing severity in documented cases of pulmonary hypertension, but have little use in the initial workup of the patient unless you suspect secondary pulmonary hypertension from underlying COPD.
You are seeing a 29-year-old with documented tuberculosis. He is currently on INH, rifampin, PZA, and ethambutol. He comes in because he has noted that he cannot see colors as well. He is an interior decorator and noted that last week he tried to put two shades of teal together, and he was embarrassed when he realized what he had done. PAST MEDICAL HISTORY: Essentially negative Tuberculosis exposure was from his mother, who works in a nursing home SOCIAL HISTORY: Smokes 1 ppd for 10 years Drinks alcohol at night; quit when started on his anti-TB meds Lives with his mother Not sexually active FAMILY HISTORY: Mother 55, with cavitary TB diagnosed 4 months ago Father died 10 years ago of massive stroke; age 60 REVIEW OF SYSTEMS: Besides the vision changes, no other problems PHYSICAL EXAMINATION: Essentially normal Snellen office chart shows 20/30 vision in both eyes Referral to ophthalmologist for extensive eye examination reveals loss of color discrimination. Which of the following is the most likely etiology for his change in vision? a) Rifampin toxicity b) Combination of ethambutol and INH toxicity c) Ethambutol toxicity d) INH toxicity e) Tuberculous involvement of the retina
Topic Pulmonary Medicine Answer C. Ethambutol toxicity Explanation Ethambutol is generally not hepatotoxic, but will cause problems with visual acuity, particularly color perceptions. The other agents are not associated with this problem.
A 28-year-old male smoker presents to the emergency department with complaints of a swollen and painful left leg. He states he was in good health until 1 week ago when he began to notice a dull pain in his leg. Since then, he has experienced swelling and redness. He denies trauma to the leg. Doppler ultrasound reveals a left popliteal thrombosis. Which of the following is the most likely cause of his DVT? a) Antithrombin III deficiency b) Homocysteinemia c) Factor V Leiden gene mutation (activated protein C resistance) d) Protein S deficiency e) Occult malignancy
Topic Pulmonary Medicine Answer C. Factor V Leiden gene mutation (activated protein C resistance) Explanation This question asks about basic epidemiology: What is the most common cause of venous thromboembolism in a young man aged 28 years? Even though he is a smoker, he is too young to have a common malignancy. The remaining choices are inherited causes of a hypercoagulable state. Factor V Leiden mutation is the most common cause (40-50% of cases) of inherited thrombophilia followed by deficiencies of protein C, S, and antithrombin III, and the prothrombin gene mutation. Aberrant homocysteine levels are now believed to be a marker of clotting, not a cause of it.
A 30-year-old male with history of asthma comes to the emergency department complaining that he "can't breathe." He is diaphoretic, 140/90, HR 128, RR 30. On physical exam the following is noted: He has an "Inspiratory Fall in Blood Pressure." He is leaning forward using his accessory muscles. Hyperresonance and diffuse wheezes are found on chest exam. ABG on room air: pH 7.39 pCO 2 45 mmHg pO 2 60 mmHg Which of the following is the most appropriate next step in management? a) Antibiotic therapy b) Magnesium sulfate IV c) Intubation and mechanical ventilation d) Noninvasive mask ventilation e) 40% F i O 2 by face mask
Topic Pulmonary Medicine Answer C. Intubation and mechanical ventilation Explanation This patient presents with a clinical history of asthma and the signs of status asthmaticus. The arterial blood gas does not show the expected respiratory alkalosis characteristic of early asthmatic exacerbations. Instead, the normal blood gas indicates that the patient is tiring and will need ventilatory support. Noninvasive positive pressure ventilation (NPPV) has been used in the treatment of status asthmaticus, but its use is not yet standard of care due to a lack of prospective studies. In addition, a tiring patient may ultimately cease respirations, and NPPV offers nothing to assist these patients if respiratory arrest develops. Intubation and careful mechanical ventilation are recommended.
A 52-year-old female initially complained of cough and fever. She was seen in the emergency department and was noted to have significant respiratory distress. Her O2 sat was 84% on 100% non-rebreather mask. Due to continued respiratory distress, the patient was intubated and placed on mechanical ventilation. Her initial ABG post-intubation is: pH 7.28, PaO2 59, PaCO2 54, O2 sat 89%, FiO2 100%. Mechanical ventilator settings: AC mode; rate 20; FiO2 100%; tidal volume 550; PEEP 8 Vitals: T 39.1° C (102.38° F); P 120; BP 100/44; RR 24 Her CXR reveals diffuse bilateral infiltrates. Echocardiogram reveals an estimated LVEF of 55% without any significant valvular abnormalities. Which of the following has been shown to improve mortality in those with this condition? a) Placement of a right heart catheter to assess cardiac parameters b) Initiation of systemic corticosteroids after 5 days of ventilatory support c) Maintenance of tidal volumes at 6 mL/kg of ideal body weight d) Use of nitric oxide e) Maintenance of tidal volumes at 12 mL/kg/hour
Topic Pulmonary Medicine Answer C. Maintenance of tidal volumes at 6 mL/kg of ideal body weight Explanation This patient has pneumonia, which has progressed to acute respiratory distress syndrome (ARDS). The clinical definition for ARDS includes: Acute onset of diffuse bilateral infiltrates PaO2 ratio < 200 (< 300 for acute lung injury [ALI]) No evidence of cardiogenic pulmonary edema Mechanical ventilatory support is almost always necessary for patients with ARDS. This inflammatory condition has been extensively studied, and outcomes continue to improve. However, the only proven intervention to reduce mortality is a lung protective strategy. Ventilatory support with a goal tidal volume (TV) of 6 mL/kg of ideal body weight and limitation of plateau pressures < 30 cm (protective group) was compared to those with goal TV of 12 mL/kg with no limit to plateau pressures (traditional group). Those in the protective group had a significant reduction in mortality. Associated with a lung-protective strategy was worsening ventilation, but this permissive hypercapnia still resulted in better outcomes. The use of the right heart catheter in the ICU has diminished significantly. Many studies have shown that use of these catheters and the information they provide does not lead to better outcomes, including conditions such as ARDS. Nitric oxide is a specific pulmonary vasodilator that is delivered by inhalation gas. It has been shown to improve oxygenation temporarily. However, no mortality benefit has ever been shown, and it may, in fact, be associated with worse outcomes. Systemic steroids have not improved mortality but may improve lung injury scores if initiated early in the course of ARDS
You are seeing a 70-year-old man with severe COPD for follow-up. He quit smoking about 5 years ago, but his health has continued to deteriorate. He is at the point now where he cannot ambulate in his home without getting severely short of breath. He wants to know if there is anything else that he can do to improve his health status. You explain to him that supplemental oxygen may be beneficial to him, but you will have to do some laboratory studies to demonstrate to Medicare that they should pay for this. He agrees to the testing. PAST MEDICAL HISTORY: History of coronary artery disease; status-post 4 vessel CABG 5 years ago History of gout History of MI in 1992 Multiple hospitalizations for COPD exacerbations—about once a year on average in the last 10 years Morbid obesity SOCIAL HISTORY: Lives with his new wife of 2 years, Bambi, a 28-year-old dancer Drinks 2 glasses of red wine every night Quit smoking 5 years ago; before that, he smoked 2 packs/day for 50+ years FAMILY HISTORY: No change from the last 20 H&Ps you've done; documented well for the chart REVIEW OF SYSTEMS: Occasional headache Occasional sore throat Dyspnea on exertion at 5 feet Stable exertional chest pain; usually relieved with one nitroglycerin or rest Cough: productive cough every morning of every day; no change in character or frequency No nausea or vomiting Increased difficulty initiating his urine stream PHYSICAL EXAMINATION: BP 126/67, HR 86, RR 28, Temp 97.9° F MS: Oriented x 3 General: Obese man in no distress at rest; but when you saw him walk in from the waiting room he was markedly distressed. HEENT: Left cataract Throat clear; dentures Neck: No masses, no bruits Heart: RRR without murmurs, rubs, or gallops Lungs: Chronic crackles throughout; no change from previous examinations; prolonged expiratory phase noted as usual Abdomen: +BS, soft, nontender Neuro: CN grossly intact; reflexes equal and symmetrical Motor: Full strength throughout with normal muscle tone and bulk Sensory: Unremarkable LABORATORY: ABG: 7.5 PCO2 = 50; PaO2 = 50; Oxygen saturation 85% Based on your findings, which of the following should you recommend? a) Supplemental oxygen is not indicated based on his laboratory values. b) Supplemental oxygen is not indicated based on his physical examination. c) Supplemental oxygen should be worn 24 hours a day by this patient. d) Supplemental oxygen worn intermittently would be better than continuous oxygen therapy because of the concern that his respiratory drive will be too suppressed on continuous oxygen therapy. e) Supplemental oxygen may be indicated, but you need more information.
Topic Pulmonary Medicine Answer C. Supplemental oxygen should be worn 24 hours a day by this patient. Explanation Only objective data from an arterial blood gas is acceptable to Medicare for supplemental oxygen. History and physical examination will not help get this funded. He needs to meet one of the following criteria: Resting PaO2 ≤ 55 mmHg, or O2 saturation ≤ 88%, or PaO2 ≤ 59 mmHg (O2 sat ≤ 89%) with evidence of cor pulmonale Evidence of cor pulmonale in these considerations is: Clinical evidence of right heart failure Pulmonale on ECG (P wave height > 2.5 mm in II, III, and AVF) Hct > 56 (due to polycythemia from chronic hypoxia 2° cor pulmonale) You can tell him that being on supplemental oxygen is likely to improve his symptoms and there is a possibility that, in several months, he may not need the oxygen anymore. Medicare guidelines require you to bring him back between 61 and 90 days and retest him. Note that the use of supplemental oxygen does not depress the respiratory drive and cause an elevated CO2. This is due to several factors, including an easing of pulmonary vasoconstriction, which leads to perfusion of previously underperfused spaces and possibly due to ventilation of previous dead space.
A 72-year-old male with a history of coronary artery disease and systolic cardiomyopathy presents with a 2-week history of progressive shortness of breath associated with a dry cough. He also complains of 2+ lower extremity edema. On lung exam, there are decreased breath sounds throughout the lower 2/3 of his right lung with dullness to percussion. Chest x-ray reveals a moderate pleural effusion. The patient undergoes bedside thoracentesis with the removal of 800 mL of clear, straw-colored fluid. Studies sent: Pleural fluid: Protein 1.3 LDH: 130 Culture: No growth Serum: Protein 4.2 LDH: 290 This improves the patient's symptoms, but a repeat chest x-ray four days later reveals recurrent pleural fluid. What should be done next for this patient? a) Repeat thoracentesis b) Thoracic surgery consult for pleurodesis c) More aggressive treatment for congestive heart failure with diuretics d) Placement of large bore chest tube for continued drainage
Topic Pulmonary Medicine Answer C. More aggressive treatment for congestive heart failure with diuretics Explanation This patient has an effusion most likely related to congestive heart failure. Based on Light's criteria, this patient has a transudative effusion. Light's criteria for an exudative effusion are: Pleural fluid protein/serum protein ratio greater than 0.5 Pleural fluid LDH/serum LDH ratio greater than 0.6 Pleural fluid LDH greater than 2/3 the upper limits of normal of the serum LDH If all negative, then classified as a transudate The most common etiology of a transudative effusion is congestive heart failure. CHF-related pleural effusions are: 65% bilateral, 25% right unilateral, and 10% left unilateral. Other causes of transudative effusions include hepatic hydrothorax, nephrotic syndrome, and hypoalbuminemia. The most effective treatment for this patient is more aggressive management of his congestive heart failure that may include diuretics. Placement of a chest tube or repeat thoracentesis will unlikely be beneficial because pleural fluid will continue to re-accumulate without correction of the underlying cause. Pleurodesis involves evacuation of the pleural space and instillation of a sclerosing agent such as talc or doxycycline. In the setting of pleural effusions due to CHF, this treatment is not very effective and would not be indicated.
A 40-year-old female patient comes to your office with complaints of a "flu-like" illness that began 4 days ago. She has just returned from Ohio, where she remembers there was a flu going around the neighborhood. While in Ohio, she helped her parents and neighbors shovel out an old barn that had been used as a bird roost. Which of the following would you recommend? a) Empiric trial of amphotericin B b) Cultures for Histoplasma capsulatum c) No testing or therapy d) Acute and convalescent serologic testing and treatment with amphotericin B e) A histoplasmin skin test
Topic Pulmonary Medicine Answer C. No testing or therapy Explanation This patient has acute histoplasmosis from the neighborhood barn-cleaning party. The neighborhood outbreak is not uncommon when there is land cleared or bulldozed. The key is that histoplasmosis is usually a self-limited disease for which we do not consider treatment unless there is evidence for progression to more disseminated disease. Some would consider treatment if symptoms persist more than 4 weeks, but before then treatment is not indicated by current guidelines. Treatment for mild disease that has persisted longer than 4 weeks would be with itraconazole, not amphotericin B, which is reserved for severe or disseminated disease. Accurate diagnosis relies on the history of exposure. Histoplasmin skin tests are useless, because many people will test positive after exposure without having the disease. Cultures are not very sensitive and may take weeks to grow. A positive culture from sputum or tissue is diagnostic, however. Serologic testing has variable sensitivity but would be the best test to diagnose her definitively; however, amphotericin B is not indicated.
A 65-year-old male insulation worker has a chest radiograph that shows bilateral pleural plaques along the lateral chest walls. He was exposed to asbestos intermittently during his occupational pursuits. He is asymptomatic, and his physical exam is unremarkable. He does not smoke. What is the most appropriate next step in patient care? a) CT scan of the chest to determine if plaques contain calcium b) Pulmonary function tests to determine if plaques are causing restriction c) Observation and reassurance d) Tuberculin skin test with standard strength PPD e) Bronchoscopy with biopsy to check for pulmonary fibrosis
Topic Pulmonary Medicine Answer C. Observation and reassurance Explanation Pleural plaques indicate significant exposure to asbestos. Their presence does not cause impairment in pulmonary function. The presence or absence of calcium in the plaques has no clinical significance. Bronchoscopy with biopsy is not indicated in this otherwise asymptomatic patient with an otherwise normal chest radiograph. Tuberculin skin test with standard-strength PPD is imperative in the patient with silicosis, but the risk of active tuberculosis disease is not known to be increased in asbestos exposure. Reassurance that the plaques are benign and continued observation of the patient (yearly chest radiographs) are appropriate since he still has potential for developing bronchogenic cancer of the airways in the future.
A 65-year-old male presents to the emergency department with acute shortness of breath. He complains of pain in his right chest that is worse with deep inspiration. He also reports some lightheadedness but no syncope. These symptoms started to occur 2 hours prior to presentation and awoke him from sleep. He does report that he recently returned from an overseas airplane trip that lasted 6 hours. He denies any lower extremity edema. On exam: P 125; BP 130/45; T 37.1° C (98.78° F); RR 24; Oxygen sat 88% on room air He is in mild distress without accessory muscle use. His lungs are clear to auscultation. Cardiac exam reveals tachycardia with regular rhythm, no murmurs. The remainder of his exam is unremarkable. ECG reveals sinus tachycardia with right bundle-branch block. A CXR is read as: "No significant cardiopulmonary process." Which of the following should be done next? a) Start empiric antibiotics for pneumonia. b) Discharge home with NSAIDs with follow-up with primary care physician in 1 week. c) Obtain chest CT angiogram. d) Start low-dose aspirin orally. e) Start PO warfarin at 10 mg daily.
Topic Pulmonary Medicine Answer C. Obtain chest CT angiogram. Explanation This patient has had recent immobilization with prolonged air travel, sudden onset of shortness of breath and pleuritic chest pain, and now tachycardia and hypoxemia. These are classic signs and symptoms of a pulmonary embolus, and this should be explored further. Chest CT angiogram is considered the gold standard for diagnosis of pulmonary emboli if there is no contraindication to obtaining the study. When there is a high clinical suspicion for pulmonary emboli, early administration of anticoagulants should be considered without delay. However, treatment should be started with intravenous unfractionated heparin or low-molecular-weight heparins. Although warfarin is used for maintenance treatment for venous thromboembolic disease, there is a delay before an appropriate level of anticoagulation is achieved. Although coronary ischemia certainly must be considered, the ECG does not reveal any ischemic changes. Therefore, aspirin would not be the next best step. The CXR does not have any evidence of pneumonia, so starting antibiotics wouldn't be indicated. Discharge home without further investigation would be inappropriate given this patient's presentation.
Yesterday a 20-year-old male presented with adult respiratory distress syndrome due to shock, and he required mechanical ventilation and positive end-expiratory pressure (PEEP). His condition gradually improves. Which of the following would not be a criterion for cessation of mechanical ventilation? a) Tidal volume 4 to 5 b) mL/kg c) FiO2 35% d) PEEP of 10 cm H2O e) Vital capacity greater than 15 mL/kg
Topic Pulmonary Medicine Answer C. PEEP of 10 cm H2O Explanation PEEP should be less than 5 cm H2O before discontinuation of mechanical ventilation and extubation can be considered. Vital capacity should be greater than 10 to 15 mL/kg. Tidal volume of 4 to 5 mL/kg is acceptable for removal from the ventilator. FiO2 should be less than 40% before discontinuation of mechanical ventilation. P(A-a) O2 at an FiO2 of 100%, less than 350 mmHg would also indicate increased probability of successful weaning off the ventilator and extubation.
Spirometry and peak flow measurement are 2 methods for evaluating airway status in pulmonary patients. When considering the data available from these two modalities, the value obtained with a peak flow meter corresponds most directly to which spirometric equivalent? a) Total lung capacity (TLC) b) Forced vital capacity (FVC) c) The first few milliseconds of airflow d) Forced expiratory volume in 1 second (FEV1)
Topic Pulmonary Medicine Answer C. The first few milliseconds of airflow Explanation Spirometry is a useful tool to evaluate pulmonary and airway function. Peak flow meters are used primarily in the management of asthma symptoms. Values obtained from peak flow meters correspond to the first few milliseconds of airflow. In the absence of chronic lung changes, this value indirectly relates to FEV1. This relationship does not hold up well for patients with COPD. Peak flow monitors are therefore practically useful only in the management of reactive airway disease.
An 80-year-old male is admitted to the hospital with dyspnea and a large right-sided pleural effusion. He is afebrile, complains of a cough that is productive of whitish-clear sputum and reports about a 10-lb. weight loss over the previous 6 months. He denies fever, chills, night sweats, hemoptysis, or chest pain. He has a 50-pack/year history of smoking but quit recently as a 50th anniversary present for his wife. He worked as an accountant and worked in the Brooklyn Naval Yard in World War II but denies any asbestos exposure. On physical exam he is noted to be afebrile, without adenopathy or skin lesions. Breath sounds are diminished at the right side, and he has dullness to percussion posteriorly to the inferior border of the scapula. There is no clubbing, cyanosis, or edema; his nails and fingertips on his right hand are discolored from nicotine. CBC: WBC count of 5,000 with 50% neutrophils, 3% bands, 24% lymphs, 17% monocytes, and 6% eosinophils Serum chemistries indicate: Glucose 84 mg/dL; protein 7.8 g/dL; LDH 162 U/L Review of the chest x-rays (PA, lateral, decubitus) shows a large, free-flowing effusion with no discernible underlying lung or mediastinal pathology. Sputum Gram stain has a few WBCs, no organisms Sputum cytology: Negative for malignant cells You perform a right-sided thoracentesis and remove almost 1 liter of dark, straw-colored fluid. Pleural fluid is sent for routine studies: Cell count: RBCs 8100μL WBC 3600/μL Differential: 88% lymphocytes 1% neutrophils 11% monocytes Fluid Chemistries: Glucose 45 mg/dL Protein 5.9 mg/dL LDH 332 U/L Cytology: Negative Which of the following is the most likely etiology of this patient's pleural effusion? a) Lymphoma b) Bronchogenic carcinoma c) Tuberculosis d) Malignant mesothelioma e) Parapneumonic effusion
Topic Pulmonary Medicine Answer C. Tuberculosis Explanation This effusion is clearly exudative, hemorrhagic, and with a lymphocytic predominance. There were no mesothelial cells noted. Tuberculosis pleuritis is common in elderly patients and should be a leading consideration in this case. Confirmation via fluid AFB stain and culture and/or pleural biopsy with culture should be performed. The absence of parenchymal or nodal disease leads us away from suspecting parapneumonic, lymphoma, or bronchogenic carcinoma. The negative pleural cytology rules out malignant mesothelioma. On the Board examination, a series of pleural effusions are presented to you.
A 30-year-old male is brought to the emergency department after being found unresponsive at home with a syringe in his arm. The patient is unresponsive, blood pressure is 120/60, and pulse is 100. His pupils are very small and unreactive. He appears cyanotic, and his respiratory rate is 8/minute. Arterial blood gas on room air shows a pH 7.22, PCO2 of 72 and a PO2 of 50. Which of the following is responsible for this patient's hypoxemia? a) Alveolar hypoventilation plus low V/Q b) Low V/Q ratio c) Venous blood was sampled, not arterial d) Alveolar hypoventilation alone e) Alveolar hypoventilation plus right-to-left shunt
Topic Pulmonary Medicine Answer D. Alveolar hypoventilation alone Explanation The differential diagnosis of arterial hypoxemia includes all of the choices plus decreased diffusion and high altitude (low FiO2). While this patient is at risk for aspiration pneumonia, pulmonary edema, and other underlying pulmonary pathology, a quick calculation of the A-a gradient on room air reveals a normal A-a gradient. (150 - (50 + 72/0.8) = A-a gradient of 10. The A-a gradient is increased in all causes of hypoxemia except hypoventilation and high altitude. We were not given any information that this patient was found on the summit of a 10,000 foot mountain; therefore, alveolar hypoventilation due to the presumed narcotic injection is the physiological mechanism for hypoxemia in this patient.
A 65-year-old male with poor general health status was evaluated in your office for progressive complaints of shortness of breath and decreasing exercise tolerance. As part of the workup, pulmonary function testing was ordered. The results of the testing revealed a significant decrease in the total lung capacity, functional reserve capacity, and the residual volume. The carbon monoxide diffusional capacity was found to be normal. Which of the following diagnoses would be most consistent with these findings? a) Eosinophilic pneumonia b) Silica pneumoconiosis c) Procainamide-induced pneumonitis d) Ankylosing spondylitis e) Pulmonary sarcoidosis
Topic Pulmonary Medicine Answer D. Ankylosing spondylitis Explanation Decreases in the total lung capacity, functional reserve capacity, and the residual volume are consistent with restrictive lung disease. This group of disorders occurs with both parenchymal processes and with conditions that cause limitations in chest wall mobility. A normal carbon monoxide diffusion test suggests a functional parenchyma and implicates chest wall mobility as the cause of the compromise. Of the choices provided, silica pneumoconiosis, drug (procainamide) induced pneumonitis, sarcoidosis, and eosinophilic pneumonia lead to parenchymal damage and restrictive changes. Kyphoscoliosis is the most common reason for restrictive lung disease arising from chest wall limitations, but several rheumatologic processes can also interfere. Ankylosing spondylitis can lead to both parenchymal and mobility limitations, but is the choice most likely to exhibit a normal diffusional capacity.
A 15-year-old who hunts ducks regularly in northern Louisiana and southern Arkansas. She comes to you with a 2-month history of cough and recent development of hemoptysis. On talking to her, you discover that she has lost 8 pounds in the last month. Past Medical History: Negative Social History: Lives in with her mother and stepfather Denies use of alcohol No illicit drug use Family History: Father died at age 45 of lung cancer Grandmother died at age 46 of lung cancer Physical Examination: BP 100/70, P 95, Temp 100° F, RR 25, Weight 110 lbs. HEENT: PERRLA, EOMI, TMs: Clear, Throat: Clear Neck: Supple Heart: RRR without murmurs, rubs, or gallops Lungs: Scattered crackles in the bases Abdomen: No hepatosplenomegaly Extremities: Draining pustular lesion noted on her left arm; she reports that it has been there for 1 month; puts "poultice" on it and it gets better, then it gets worse again. CXR: Possible mass noted at left base Laboratory: Gram stain of left arm lesion's fluid: Mixed bacterial flora KOH of left arm lesion's fluid: Small budding yeasts Acid-fast smear of left arm lesion's fluid: No organisms seen Based on your findings, which of the following is the most likely diagnosis? a) Histoplasmosis b) Lung carcinoma c) Coccidioidomycosis d)Blastomycosis e) Tuberculosis
Topic Pulmonary Medicine Answer D. Blastomycosis Explanation The combination of lung mass, draining cutaneous lesion, and the finding of budding yeast should point you toward either blastomycosis or coccidioidomycosis. Geography will narrow this down to blastomycosis, which occurs in the southeast and central U.S. bordering the Ohio and Mississippi River basins. Coccidioidomycosis occurs in the southwest states from west Texas to California. Histoplasmosis will not usually cause skin lesions as are seen with blastomycosis or coccidioidomycosis. Treatment would generally be with oral itraconazole for about 6 months. If she was severely ill, you would use amphotericin B.
A 26-year-old woman presents with complaints of shortness of breath. She states that she first noticed the shortness of breath about 5 months ago. It mainly occurs when she is exercising, and she is unable to jog anymore because of shortness of breath. Recently she has been awakened at night with shortness of breath and has the feeling that she can't catch her breath. She has not had any fever, cough, or sputum production. She denies orthopnea. Her physical examination is significant for the following: Heart: RRR without murmurs, rubs, or gallops Lungs: Clear to auscultation Abdomen: Benign Extremities: No edema, clubbing, or cyanosis CXR is normal. Office spirometry is normal for age. ABG on room air: pH 7.40, PaCO2, 42 mmHg, and PaO2 of 93 mmHg Which of the following diagnostic tests would you perform next? a) Flow-volume loop b) High-resolution CT of the chest c) Echocardiography d) Bronchoprovocation (methacholine challenge test) e) Full pulmonary function tests with diffusing capacity determination
Topic Pulmonary Medicine Answer D. Bronchoprovocation (methacholine challenge test) Explanation She has a normal cardiac examination as well as a normal pulmonary examination. The most likely etiology of dyspnea under the age of 40 years is asthma, especially if it is episodic. It is not unusual for office spirometry to be normal between episodes. The best test is the methacholine bronchoprovocation test, which will reveal hyperreactive airways—the key to diagnosing asthma. The echo would be good if pulmonary hypertension was suspected with a right parasternal heave and an accentuated 2nd heart sound. Interstitial lung disease would be more likely in an older patient, and the high resolution CT scan would be useful then. If stridor was present, the flow-volume loop could be helpful for upper airway obstruction. The normal ABG would make the diffusing capacity determination essentially unhelpful.
A 40-year-old female comes to your office complaining of increased dyspnea and coughing brownish sputum. She is well known to you, and has had stable asthma for the past 5 years with good control achieved with her inhaled corticosteroid and occasional use of her beta2-agonist inhaler. This is her third visit to your office in the past 2 months for exacerbation of her asthma. At this visit you do a chest x-ray, which shows patchy pulmonary infiltrates, and blood work that indicates a mildly elevated WBC count with eosinophilia on the differential. Which of the following is not consistent with a diagnosis of allergic bronchopulmonary aspergillosis (ABPA)? a) Marked elevation in serum IgE level b) Sputum culture positive for Aspergillus fumigatus c) Immediate skin test reaction (Type I, wheal and flare) to Aspergillus fumigatus d) Delayed (Type IV, cell mediated) skin test reaction to Aspergillus fumigatus e) Expectoration of brown mucous plugs and airway casts
Topic Pulmonary Medicine Answer D. Delayed (Type IV, cell mediated) skin test reaction to Aspergillus fumigatus Explanation All of the other choices are considered minor or major criteria for the diagnosis of ABPA, except that there is no Type IV, cell-mediated skin reaction. There are Type I and Type III (erythema and induration) skin test reactions. ABPA should be a consideration in any asthmatic who clinically worsens with repeated attacks despite adherence to therapy.
A 29-year-old woman has an x-ray film of the chest that shows bilateral prominent pulmonary arteries without pulmonary congestion. She has fatigue and chest discomfort with exertion. Physical examination shows a large a wave in the jugular pulse, clear lung fields, a systolic lift along the left sternal border, and a split S2 audible at the left sternal border and apex, with a loud second component. No cardiac murmur is present. Which of the following tests is most likely to provide the correct diagnosis? a) Pulmonary function with flow-volume loops b) Bronchoscopy with bronchoalveolar lavage c) CT angiogram with PE protocol d) Echocardiogram e) High-resolution CT of the chest
Topic Pulmonary Medicine Answer D. Echocardiogram Explanation This patient has symptoms of pulmonary hypertension. Primary pulmonary hypertension (PPH) is a rare disease of unknown cause. Secondary pulmonary hypertension is the result of other types of lung or heart disease. Recent advances in the treatment of PPH make identification and recognition imperative. When PPH is suspected, an echocardiogram should be done. This test will provide a rough estimate of resting pulmonary artery pressures. Mean pulmonary artery pressures greater than 20 mmHg at rest or 30 mmHg during exercise establishes a diagnosis of pulmonary hypertension. The diagnosis of PPH requires the absence of coexisting pulmonary or cardiac disease, and the echocardiogram helps determine if cardiac disease is present. Once PPH is suggested, perform right-heart catheterization to determine the response to vasodilators and plan appropriate long-term therapy.
A 65-year-old man presents with fever, chills, cough, and dyspnea for 3 days duration. This is associated with right-sided pleuritic chest pain as well as fatigue and mild nausea. His past medical history is only notable for hypertension for which he takes a thiazide diuretic. He has a 15-pack-year history of smoking but quit 10 years ago. On physical exam, his vitals are: T 102.4° F, RR 24, BP 100/45, O2 sat 92%. Lung exam reveals dullness to percussion with diminished breath sounds from mid-right lung to base. The remainder of the exam is not pertinent. PA/Lat and decubitus chest radiograph films reveal a free flowing moderate-size pleural effusion. Thoracentesis is performed using ultrasound guidance and sent for multiple studies. Which of the following findings on pleural fluid studies would necessitate complete pleural fluid drainage with tube thoracostomy? a) Pleural pH of 7.42 b) White blood cell count of 5,000 c) LDH level of 100 d) Growth of Streptococcus pneumoniae e) Pleural fluid protein/serum protein ratio less than 0.5
Topic Pulmonary Medicine Answer D. Growth of Streptococcus pneumoniae Explanation Pleural fluid with growth of organisms is consistent with a diagnosis of empyema. In this case, the pleural space should be evacuated to prevent complications related to an infection. Other reasons to consider tube thoracostomy or a similar procedure to allow complete evacuation of a pleural effusion include: gross pus, low pH (< 7.25), low glucose, and high LDH > 1,000. Based on Light's criteria, a low LDH and low pleural fluid to serum protein ratio are consistent with transudative pleural effusions. An elevated WBC is typically seen with most exudative effusions. However, empyema and complicated parapneumonic effusions, for which drainage is usually required, typically have WBC elevations > 50,000.
A 70-year-old man with COPD has had worsening shortness of breath for the last 6 months. He cannot move from his chair to his refrigerator without stopping several times along the way to catch his breath. A resting ABG is obtained and reveals the following: pH: 7.46 PCO2: 50 PaO2: 50 HCO3: 32 O2 sat: 84% Based on his history and laboratory results, what do you recommend? a) He should be placed on oxygen therapy during the daytime. b) He should be placed on oxygen therapy when he has exertion. c) He should not be placed on oxygen therapy except for episodes of shortness of breath. d) He should be placed on oxygen therapy 24 hours/day. e) He cannot be placed on oxygen therapy because his ventilatory drive will be impaired.
Topic Pulmonary Medicine Answer D. He should be placed on oxygen therapy 24 hours/day. Explanation He qualifies for 24-hour continuous oxygen therapy because his PaO2 is less than 55. Continuous (not intermittent or exertional) oxygen therapy is the only thing shown to prolong survival in these patients. There is a common misconception that chronic CO2 retainers have an impaired ventilatory drive that is O2 regulated rather than CO2 regulated. This led practitioners to withhold supplemental O2 out of concern that giving the patient a high PaO2 would depress the respiratory drive and cause increased CO2 retention. This is not true. Oxygen should be given to achieve a 90% O2 saturation.
A 25-year-old pregnant woman presents at 22 weeks for evaluation with dyspnea and pleuritic type chest pain. She says that she has had progressive left leg swelling for the past 6 days. A room air ABG is obtained and shows: pH 7.48, PaCO2 23, PaO2 80. What is the best diagnostic test to do at this point? a) Pulmonary angiography b) D-dimer c) V/Q scan d) Bilateral venography e) Duplex ultrasonography
Topic Pulmonary Medicine Answer E. Duplex ultrasonography Explanation She is pregnant, she has all the symptoms of DVT, right? The duplex ultrasound is the best test from the list. It won't require an invasive test like V/Q, but note—a V/Q is safe during pregnancy if needed. If V/Q results were indeterminate, an additional study would still be needed for diagnosis. The other radiologic tests are not safe in pregnancy. The D-dimer test would not be helpful—if it is positive, you still have to do some study to evaluate the leg; if it is negative, you can't ignore the swollen leg. The D-dimer is useful in a low-probability setting to rule out disease; it is not helpful in a high probability setting, especially in the face of a likely DVT.
A 45-year-old woman with lymphoma has been neutropenic for 10 days and develops nodules in her lungs on CXR. Initially she is treated with broad-spectrum antibiotics. Amphotericin B is added on day 7, which is also the day the nodules are seen on CXR. She has been having continued fevers and now is developing respiratory distress. A biopsy of her lung is shown in this figure: It is a methenamine silver stain. The histopathology shows numerous extracellular yeasts within an alveolar space. The yeasts show a narrow based budding and are of different sizes. A different stain would accentuate the capsule of this organism. Based on your findings at lung biopsy, which of the following studies should you order next? a) Lumbar puncture b) Ultrasound of her liver c) Upper endoscopy d) Nothing; continue treatment with amphotericin B e) MRI of the mandible Topic
Topic Pulmonary Medicine Answer D. Nothing; continue treatment with amphotericin B Explanation She has cryptococcal lung disease, and there is a high risk of dissemination to the central nervous system with this organism. The patient is neutropenic and developing respiratory failure. Treatment would be to cover for disseminated disease including cryptococcal meningitis.So at this juncture, a lumbar puncture is not needed. You aren't really going to have to differentiate, say, a Histoplasma from a Cryptococcus, but you will need to know the more common organisms that an immunocompromised person gets. A person with lymphoma, like this patient, is at increased risk for cryptococcal infection, Coccidioides infection, and Aspergillus infection. If we had said she was from the San Joaquin Valley—then it is a no-brainer, right? Aspergillus, remember, is branching. If they show you Cryptococcus, they will likely show it to you with the nice capsule that you remember from 2nd year medical school microbiology.
Your next patient is an 18-year-old woman who is HIV-infected. Her most recent CD4 count was 10. She is moving to your area from Iowa. She says she had a TB skin test 2 years ago and some other skin tests, all of which were read as 0 mm. She remembers that the doctor there told her to tell people the number was "0 millimeters" and not "negative." PAST MEDICAL HISTORY: She has not required hospitalization in 3 years; at that time, she was hospitalized for Pneumocystis pneumonia, which was when she was diagnosed with HIV and found to have AIDS. MEDICATIONS: Trizivir one PO bid, which she has been on for 3 months (she is now adherent, though she says in the past she had not been) Bactrim DS one PO M, W, F Azithromycin q week SOCIAL HISTORY: Lives with her boyfriend, a welder She works as a waitress at the local IHOP Smokes 3 packs/day cigarettes Doesn't drink FAMILY HISTORY: Unknown; ran away from home at age 13 REVIEW OF SYSTEMS: Occasional night sweats Low-grade fevers every 3-5 days Sore throat on occasion Cough daily; especially in the morning Loose stools daily; normal for her is 4-5 bowel movements daily; no blood Vomiting on occasion No rash Decreased appetite PHYSICAL EXAMINATION: Fairly well-appearing woman in no distress BP 110/70, P 90, RR 14, Temp 99° F, Ht 5' 5", Wt 110 HEENT: PERRLA, EOMI TMs clear Throat clear Neck: Supple; no masses Heart: RRR with no murmurs, rubs, or gallops Lungs: Scattered rhonchi at bases; cleared with cough Abdomen: Bowel sounds present; liver span 10 cm; no spleen palpated Extremities: No cyanosis, clubbing, or edema Skin: Facial acne; no other rashes LABORATORY: WBC: 2,400 with 70% lymphs, 20% neutrophils Hgb: 12.5 mg/dL; MCV 105 Platelets: 450,000 Electrolytes: Normal Albumin: 3.4 mg/dL AST: 30 U/L ALT: 25 U/L Total bilirubin: 0.4 mg/dL Viral load: < 50 copies/mL CD4: 50 She is due for her tuberculosis screening; which of the following do you recommend? a) PPD containing 5 TU of tuberculin with 2 controls (mumps and Candida) b) PPD containing 250 TU of tuberculin without controls 2-step boosted PPD with 5 TU of tuberculin (place one today and repeat in 2 weeks) c) PPD containing 5 TU of tuberculin without controls d) PPD containing 250 TU of tuberculin with 2 controls (mumps and Candida)
Topic Pulmonary Medicine Answer D. PPD containing 5 TU of tuberculin without controls Explanation The use of "control" testing is no longer recommended by the American Thoracic Society or the CDC for any patient routinely. Therefore, the use of controls should be discouraged. Even though she is severely immunocompromised, a standard PPD is still appropriate. Never, ever pick the 250 TU on a test. There is no reason to use it, so you can effectively mark out those answers when you see them listed on a test! The 2-step (or boosted) PPD has no place in routine practice; it is generally reserved for people who are repeatedly tested, such as health care workers.
A 32-year-old female comes to your office to establish primary care. Her past medical history is only notable for asthma, diagnosed as a teenager. She reports symptoms nearly daily that can range from dry cough, worse at night, and exertional dyspnea. She denies any symptoms consistent with reflux disease but does endorse occasional chest tightness and burning, particularly when she tries to jog outside in the morning. She smokes 1/2 ppd and has been doing so for 11 years. She denies any pets. Family history is notable for a brother with asthma. Her medications include theophylline, salmeterol inhaler once daily, and albuterol nebulizer three times a day. Which of the following regarding asthma is true? a) A negative methacholine challenge test does not rule out the diagnosis of asthma. b) Proton pump inhibitors are helpful in asthma control if a patient has asymptomatic GERD. c) Inhaled corticosteroids should be considered only in patients with persistent asthma who have severe nocturnal symptoms. d) Patients may have pulmonary function tests that are normal. e) Prior to exercise or during periods of acute shortness of breath, long-acting beta-agonists should be used.
Topic Pulmonary Medicine Answer D. Patients may have pulmonary function tests that are normal. Explanation Based on NHLBI guidelines for the management of asthma, all patients with asthma that is categorized as persistent should be considered for maintenance therapy. First line medications are inhaled corticosteroids with the dose recommendations based on the degree of severity. Short-acting beta-agonists may be helpful for exercise-induced asthma and may be used as rescue inhalers in periods of acute shortness of breath. Long-acting beta-agonists (LABAs), such as salmeterol or formoterol, should not be used for these purposes. In fact, LABAs have been shown in some studies to be associated with a higher risk of death, particularly if not used in conjunction with inhaled steroids. Furthermore, treatment with LABAs without concomitant use of inhaled steroids is not advised. Although treatment of symptomatic GERD may help control asthma symptoms, multiple studies have shown that treatment of asymptomatic GERD did not alter the course of asthma in terms of symptoms, exacerbations, or dyspnea scores. Asthma is characterized as episodic airflow obstruction that is fully or partially reversible. Therefore, patients may not manifest any objective signs of obstruction when pulmonary function testing is performed, but this doesn't exclude the diagnosis of asthma. Methacholine challenge testing evaluates the tendency of airways to cause bronchoconstriction in response to antigens such as methacholine. The patient inhales escalating doses of methacholine, and the dose that causes a 20% reduction in FEV1 is graphed. If there is not a reduction of the FEV1 by at least 20%, then the test is negative, which essentially rules out asthma.
A 32-year-old man presents with pleural effusion and cough. He was released from prison 4 months ago and has been living on the streets of Miami for the past 2 months. You are highly suspicious of tuberculosis. His sputum smear x 3 is negative for AFB. Which of the following is more likely to be diagnostic for tuberculosis? a) Acid-fast smear of pleural fluid b) CT scan of the chest c) Repeat sputums d) Pleural biopsy e) ACE levels
Topic Pulmonary Medicine Answer D. Pleural biopsy Explanation Pleural biopsy will have the highest yield of the tests listed. Repeating sputums is also a good idea, and it may be that they were just poor specimens. Pleural effusion acid-fast smears are rarely positive. CT of the chest is not going to give you any diagnostic information, and ACE levels are of no use.
A 45-year-old man with interstitial pulmonary fibrosis (IPF) returns to your office for routine follow-up after receiving his diagnosis 2 months prior. He continues on prednisone and cyclophosphamide and says, "I feel great!" He states he is certain he will be one of the few who does well with this diagnosis because he has had a "dramatic response." Physical exam is unchanged. Repeat HRCT shows persistent ground-glass opacities. Which of the following is associated with physiologic improvement of IPF after treatment? a) Development of diabetes secondary to steroid use b) Subjective improvement in dyspnea c) Improvement in lung pathology d) Reduction of the A-a gradient with exercise e) Improvement of the ground glass on HRCT
Topic Pulmonary Medicine Answer D. Reduction of the A-a gradient with exercise Explanation Recall that all patients who are treated with steroids usually feel better, including those with IPF. Objective improvement in IPF is made by observing the A-a gradient with exercise. A reduction in the gradient after the prescription of steroids or other immune modulators suggests a true response to treatment. Even so, evidence is lacking on whether response to treatment translates to a better outcome. The other answer choices do not indicate a response to treatment.
A 59-year-old man is seen in follow-up. He was initially evaluated for cavitating pulmonary nodules. He has a history of severe, deforming rheumatoid arthritis for 20 years that has required surgical interventions. In the past he has been on azathioprine, sulfasalazine, hydroxychloroquine, and methotrexate. None has been effective recently. He has been taking prednisone (varies from 5 mg to 20 mg a day) for many years. Four months ago, etanercept was begun, and his arthritic pain resolved almost completely. However, over the past 7-8 months he has developed a progressively worsening cough. It has been productive on occasion, but he denies any blood or blood-tinged sputum. Over the last 2 months, he has had increasing shortness of breath with exertion, to the point that he now cannot walk a block without being short of breath. PAST MEDICAL HISTORY: Diabetes mellitus, adult onset at age 40; probably associated with steroids Positive PPD 30 years ago and he took INH for 1 year SOCIAL HISTORY: Worked as a locksmith for 20 years before having to retire for disability Stopped smoking cigarettes 30 years ago; previously smoked 1 pack/day for 10 years Doesn't drink alcohol FAMILY HISTORY: Doesn't know; he was adopted REVIEW OF SYSTEMS: No fever No chills No sweats No chest pain No weight loss No change in vision No appetite changes PHYSICAL EXAMINATION: BP 130/82, P 69, RR 17, T 99° F, appears comfortable at rest HEENT: PERRLA, EOMI, developing cataract in left eye TMs clear Throat clear Neck: Supple, no masses Heart: RRR without murmurs, rubs, or gallops Lungs: Diffuse crackles at left base; no wheezes Abdomen: Bowel sounds present; no hepatosplenomegaly; no masses Extremities: Chronic, symmetric, deforming polyarthritis with moderate synovitis of metacarpophalangeal joints Joints were cool and without effusions Skin was without evidence of vasculitis or nodules LABORATORY: Sputum: Negative for acid-fast bacilli x 3 Liver functions: AST 30 U/L; ALT 28 U/L; total bilirubin 0.2 mg/dL; alkaline phosphatase 200 U/L; GGT 20 U/L; albumin 3.5 mg/dL Renal function: BUN 10 mg/dL; creatinine 0.5 mg/dL Urinalysis: Normal for age Pulmonary function testing: Predicted Actual % Predicted FVC (L) 3.8 1.9 50 FEV1 (L) 2.6 1.5 58 FEV1/FVC (%) 70 80 114 DLCO mL/min/mmHg 26.02 8.63 33 Which of the following do the pulmonary function findings indicate? a) Severe obstructive lung disease, with no restrictive defect and marked decrease in diffusing capacity b) Severe obstructive disease with mild restrictive defect and normal diffusing capacity c) Restrictive ventilatory defect and normal diffusing capacity d) Restrictive ventilatory defect and marked decrease in diffusing capacity e) Restrictive ventilatory defect only
Topic Pulmonary Medicine Answer D. Restrictive ventilatory defect and marked decrease in diffusing capacity Explanation Note that FVC and FEV1 are reduced in parallel—this suggests a restrictive ventilatory defect. The marked decrease in diffusing capacity suggests loss of capillary surface area, which may be seen in pulmonary fibrosis, emphysema, or pulmonary vascular disease.
A 60-year-old woman with lung cancer presents with new onset of anisocoria. Her right pupil is much smaller and rounder than her left pupil. You dim the lights in the examination room and note that the left pupil has dilated much more. On examination, her right pupil responds briskly to light. She has mild ptosis on the right and anhidrosis on the right as well. Her right pupil constricts with pilocarpine administration, and dilates with atropine. However, when you apply 4% cocaine, it minimally dilates. Where is her likely abnormality located? a) Left occipital lobe b) Left optic nerve c) Left sympathetic chain d) Right sympathetic chain e) Right optic nerve
Topic Pulmonary Medicine Answer D. Right sympathetic chain Explanation The symptoms are consistent with sympathetic denervation of her right eye—the "Horner" pupil. Most commonly, a pulmonary neoplasm of the superior sulcus will produce this lesion. Usually it is associated with ipsilateral ptosis and anhidrosis. Pupillary light responses should be normal, as should the response to agents that dilate and constrict the pupil. However, since the sympathetic chain is not working, cocaine cannot cause local release of sympathomimetic substances and is a poor dilator.
A 65-year-old man presents to his physician's office with a 6-month history of wheezing, dyspnea on exertion, and daily sputum production. He is a 50-pack-year smoker but is otherwise healthy. Examination shows markedly decreased breath sounds with mild wheezing at the end of expiration. Spirometry is performed: FVC is 2.3 L. FEV1 is 1.5 L (60% of predicted). The FEV1 /FVC ratio is 65%. Which of the following interventions will be most effective for improving this patient's long-term survival? a) Inhaled corticosteroids b) Inhaled ipratropium c) Long-term oral corticosteroids d) Smoking cessation
Topic Pulmonary Medicine Answer D. Smoking cessation Explanation Smoking cessation is the single most effective and cost-effective intervention for slowing the progression of COPD. After 1 year of smoking cessation, the rate of decline of lung function can approximate the rate of decline of a nonsmoker. Counseling patients to stop smoking, even if only a brief encounter, can be an effective intervention. Additionally, it has been shown that multimodality efforts toward smoking cessation, including counseling, ongoing support, nicotine replacement, and pharmacology are more effective in combination than individually. Inhaled corticosteroids and ipratropium may reduce hospitalization rates in patients with moderate-to-severe COPD, but they failed to slow the rate of decline of FEV1 in large randomized controlled trials. It is unlikely that either of these medications has a positive impact on long-term survival in stable COPD patients. Oral corticosteroids do not benefit most patients with COPD and can cause severe adverse effects. To date, no pharmacologic intervention for COPD has been shown to reduce mortality.
A 60-year-old man with COPD on chronic bronchodilator therapy and theophylline visits a local "doc-in-the-box" and receives a prescription for an antibiotic for "bronchitis." He begins his medication but 3 days later returns to you with nausea, vomiting, and tremulousness. Which of the following is the likely explanation for his new symptoms? a) He stopped taking his theophylline. b) The antibiotic he was given was amoxicillin. c) The antibiotic he was given was cefazolin. d) The antibiotic he was given was ciprofloxacin. e) The antibiotic he was given was gatifloxacin.
Topic Pulmonary Medicine Answer D. The antibiotic he was given was ciprofloxacin. Explanation Ciprofloxacin when given to a patient on theophylline can raise the theophylline levels into the toxic range. Therefore, never use both drugs in combination. Gatifloxacin and the newer quinolones do not have this effect. Amoxicillin and cefazolin would not cause this problem. Stopping the theophylline would not cause him to have nausea, vomiting, and tremulousness. Other drugs that increase theophylline levels include cimetidine, erythromycin, propranolol, and tetracycline.
If prophylaxis is not given, which of the following patients is considered at highest risk for pulmonary embolism? a) A 35-year-old woman undergoing hysterectomy b) A 30-year-old man undergoing tonsillectomy c) A 90-year-old man undergoing cataract surgery d) A 50-year-old woman undergoing hysterectomy e) A 50-year-old man undergoing knee replacement
Topic Pulmonary Medicine Answer E. A 50-year-old man undergoing knee replacement Explanation Knee replacement surgery has the highest risk—approaching 70% if prophylaxis is not given. This is closely followed by hip replacement surgery. Other very high-risk surgeries include extensive pelvic/abdominal surgery for cancer. Moderate-risk include patients undergoing any other surgery who are older than 40. CHF and those with pneumonias also are at moderate risk. Lower-risk procedures include all immobilized patients and patients under age 40 undergoing thoracic, gynecologic, or abdominal surgery. Cataract surgery has very low risk. So, on the Boards, look out for the person who is having or has had a knee or hip replacement and who presents with shortness of breath!
You are seeing a 46-year-old man who was raised in rural Mississippi. On a routine CXR (done by another physician), a 5-mm nodule was found in his left upper lobe. The nodule has some calcifications in it. He does not smoke. He has never had a CXR before. PAST MEDICAL HISTORY: Negative SOCIAL HISTORY: Teaches 9th grade Art at the local high school Lives alone Quit smoking 20 years ago FAMILY HISTORY: Mother 72; MI 2 years ago; doing well now Father 66; MI 1 year ago; doing well Brother 42; healthy REVIEW OF SYSTEMS: No weight loss No night sweats No fever No chills No cough No rashes No travel history PHYSICAL EXAMINATION: BP 120/70, P 90, RR 13, T 98.6° F HEENT: PERRLA, EOMI TMs clear Throat clear Neck: Supple Heart: RRR without murmurs, rubs, or gallops Lungs: Clear to auscultation Abdomen: Bowel sounds present; no hepatosplenomegaly Extremities: No cyanosis, clubbing, or edema Skin: No masses or rashes LABORATORY: CXR: 5-mm calcified nodule seen in mid-left upper lung field Which of the following should you do next to work up this nodule? a) Repeat CXR in 1 month. b) Place PPD. c) No further workup is needed. d) V/Q scan. e) CT of chest to evaluate the nodule.
Topic Pulmonary Medicine Answer E. CT of chest to evaluate the nodule. Explanation Calcified nodules are usually benign; however, increasing evidence has shown that almost all nodules need an initial CT scan. Then if things look OK, serial CT scans to follow up. Even though this patient is low-risk and this is likely old histoplasmosis, it warrants workup. This has been a change in thinking with the goal being not to miss any resectable lung cancers. For this patient, if the 5-mm nodule appears to be consistent with a benign lesion, repeat CT would be done in 6 to 12 months; if this shows no change, then it would be safe to stop surveillance. If changed, resection is warranted. Of note is that if he had a "popcorn" calcified pattern, this likely would be a hamartoma, but you would still check a CT scan and follow as indicated to be sure. Solitary nodules without calcifications are also watched with serial CT scans. If the patient is higher-risk (smoker, family history, etc.), these scans may be repeated in 3 months if the initial scan appears non-cancerous. Many would just proceed with excision, particularly if there are suspicions of malignancy.
A 27-year-old young woman came to the Emergency Department complaining of pleuritic chest pain of several hours duration. She was not a smoker but gave a history of using birth control pills. Her chest x-ray and physical exam were normal except for splinting with deep inspirations. Arterial blood gas showed pH 7.45, PaCO2 31 mmHg, HCO3 21 mEq/L, PaO2 83 mmHg (breathing ambient air; PB 747 mmHg). What should be the next step in management? a) Discharge with pain medication. b) Consider SSRI for panic attacks. c) Start PO fluoroquinolone for treatment of pneumonia. d) Start bicarbonate infusion. e) Diagnostic imaging to exclude pulmonary embolus.
Topic Pulmonary Medicine Answer E. Diagnostic imaging to exclude pulmonary embolus. Explanation This young woman's PaO2 often is judged "normal," and so an abnormality in oxygen transfer may be missed. The calculated PaO2 is 108.25mmHg. Her P(A-a)O2 was elevated at 25 mmHg (108 minus 83), indicating a state of V-Q imbalance, and therefore some parenchymal lung disease or abnormality. With her history of using oral contraceptives and symptoms, consideration for the diagnosis of pulmonary embolism must be made. Hence, risk stratification and imaging, such as chest CTA or V/Q scan, should be ordered. Discharge with a pain medication or treatment with an SSRI for panic attacks would not be indicated given the elevated A-a gradient and may miss an important diagnosis. Bicarbonate infusion has a limited role in the setting of metabolic acidosis. Furthermore, this patient has a normal pH, so infusion of bicarbonate likely will not be beneficial and may in fact be harmful. There is no evidence of pneumonia on chest radiograph, so antibiotics for this specific indication would not be appropriate.
A 30-year-old woman comes to your office complaining of shortness of breath. She states that she first noticed the shortness of breath 6 months ago. She is fine at rest but cannot go jogging any longer due to shortness of breath, and she is awakened at night once or twice a week with a feeling that she can't catch her breath. She denies fever, cough, sputum production, orthopnea, or chest pain. On exam, she is a well-appearing female in no apparent distress. The cardiovascular exam is normal, and auscultation of the lungs is normal. There is no leg edema, clubbing, or cyanosis. You perform routine blood work that indicates a normal CBC and chemistries. A chest x-ray is normal, and spirometry done in the office is normal. An arterial blood gas on room air shows pH 7.41, pCO2 40 mmHg, and pO2 of 90 mmHg. Which of the following diagnostic tests would you perform next? a) Full pulmonary function tests with diffusing capacity determination b) Flow-volume loop c) High-resolution CT (HRCT) of the chest d) Echocardiography e) Exercise challenge test
Topic Pulmonary Medicine Answer E. Exercise challenge test Explanation With a normal cardiovascular and pulmonary exam, the most likely cause of the episodic dyspnea in this patient under 40 years of age is exercise-induced bronchospasm (EIB). Because this bronchospasm is episodic and a reversible airways obstruction, spirometry performed while the patient is asymptomatic may well be normal. An exercise challenge test is the most direct way to establish a diagnosis of EIB. This usually involves 6 to 8 minutes of ergometer or treadmill exercise, sufficient to raise the heart rate to 85% of the predicted maximum. A test is generally considered positive if the FEV1 falls by 10% or more, although a fall of 15% is more diagnostic. Alternatively, surrogate tests to assess bronchial hyperresponsiveness (e.g., cold air hyperventilation, methacholine or histamine inhalation challenge) may be performed in specialized laboratories but do not always correlate with the presence of EIB. If the patient had rales on the exam and was older, the possibility of an interstitial lung disease would have to be entertained, and an HRCT of the chest would be indicated. Remember: 10% of patients with interstitial lung disease will have a normal chest x-ray at the time of presentation. If the patient gave a history of chest pain or syncope with exertion and physical exam revealed a right parasternal heave and an accentuated P2, then the diagnosis of pulmonary hypertension by echocardiography would be warranted. If an upper airway obstruction was suggested by the history or the patient was found to have stridor, a flow-volume loop would be a good screening test for upper airway obstruction. A normal ABG with a normal A-a gradient would make a diffusing capacity limitation unlikely.
A 40-year-old woman with sarcoidosis that is under fairly good control comes in today for a routine checkup and is noted to have a BP of 150/95. In reviewing her file, you note that the last time she came in she also had an elevated BP, and you had recommended dietary changes as well as weight loss. She has attempted these but evidently will require a pharmacologic agent to control her hypertension. Her current routine laboratory studies are normal. Which of the following is the best agent for her to use? a) Thiazide diuretic b) Loop diuretic c) Atenolol d) Calcium channel blocker e) Fosinopril
Topic Pulmonary Medicine Answer E. Fosinopril Explanation Think about it—what is frequently elevated in sarcoid patients? ACE-levels! So, if they have hypertension, the first drug to try is an ACE-inhibitor.
What is eosinophilic granulomatosis in association with lytic bone lesions, diabetes insipidus, and exophthalmus called? a) Maple-syrup lung disease b) Treacher-Collins syndrome c) Löffler syndrome d) Hannaman-Cross syndrome e) Hand-Schüller-Christian syndrome
Topic Pulmonary Medicine Answer E. Hand-Schüller-Christian syndrome Explanation Just something stupid to memorize. Actually look for it on the Boards. They are likely to give you the lytic bone lesions, diabetes insipidus, and exophthalmus and ask you what you would look for on lung biopsy—eosinophilic granulomatosis. Treacher-Collins syndrome is associated with malar hypoplasia and a cleft in the zygoma. Loeffler's syndrome is associated with pulmonary infiltrates and peripheral eosinophilia usually due to Ascaris or other worm migrating through the lung. Maple-syrup lung disease and Hannaman-Cross syndrome do not exist.
A 30-year-old female comes to your office with complaints of dyspnea, cough, and wheezing for the past 4 months. She reports that initially the symptoms were related only to exercise, which forced her to stop her aerobic workouts. Now she has daily symptoms even at rest and awakens several nights a week with wheezing. She is a nonsmoker. She denies any particular environmental factors and had no respiratory problems until now. On exam, she has a very mild end-expiratory wheeze. Office spirometry reveals a reduced FEV1 and FEV1/FVC at 70% predicted. Which of the following is the best initial treatment plan? a) Inhaled beta2-agonist, every 4-6 hours b) Inhaled cromolyn, 4 times a day c) Low-dose inhaled corticosteroid alone d) Sustained release theophylline, twice a day e) Inhaled corticosteroid plus long-acting beta-agonist
Topic Pulmonary Medicine Answer E. Inhaled corticosteroid plus long-acting beta-agonist Explanation This patient has asthma that would be characterized as moderate-persistent by the National Asthma Education and Prevention Program Guidelines. As such, she should be on daily medication with an inhaled corticosteroid and a long-acting inhaled beta-agonist; alternatively, you could prescribe medium doses of an inhaled corticosteroid alone. The emphasis on utilizing antiinflammatory therapies (inhaled corticosteroids) was a major focus on the NAEPP guidelines for any asthma patient characterized as mild-persistent or greater. This was to overcome the over-reliance on symptomatic relief with beta2-agonist inhalers and the rising number of asthma deaths (most clutching their beta-agonist inhalers and on no steroids!). A beta2-agonist inhaler must be available for symptomatic relief with the understanding that more frequent use of the beta-agonist inhaler indicates a need for stepped-up antiinflammatory therapy (higher-dose inhaled steroids, PO prednisone). Sustained release theophylline may help with nocturnal symptoms but has no antiinflammatory action. Inhaled cromolyn alone can take weeks to start working and is best used in atopic asthma in children.
A 33-year-old woman in your clinic has a chest CT (see below). She has mild, worsening airflow limitation on serial spirometry. She has increasing exertional dyspnea that is now significantly limiting her activity. She had a small, spontaneous pneumothorax last year that resolved without intervention. Alpha-1 antitrypsin levels were normal when checked at that time. She has never smoked, but her spouse does smoke. The most likely diagnosis is which of the following? a) Advanced emphysema from secondhand smoke b) Idiopathic pulmonary fibrosis Asthma c) Alpha-1-antitrypsin deficiency with advanced emphysema d) Lymphangioleiomyomatosis (LAM)
Topic Pulmonary Medicine Answer E. Lymphangioleiomyomatosis (LAM) Explanation This woman has sporadic lymphangioleiomyomatosis (LAM), a rare cystic lung disease caused by abnormal proliferation of smooth muscle cells that affects premenopausal women (average age 35) almost exclusively. Progressive exertional dyspnea with worsening airflow limitation, recurrent pneumothoraces, and chylous pleural effusions are the common clinical manifestations. 30-50% of women with sporadic LAM also have renal angiomyolipomas that are usually clinically silent. Tuberous sclerosis complex-associated LAM, another form of the condition, includes tumors in the brain, skin, and other organs, causing seizures and intellectual delay. The diagnosis of LAM can be made by high-resolution CT without biopsy in classic cases; lung biopsy (positive for the HMB-45 immunohistochemistry assay) may be required in others. LAM's major mimics on HRCT are pulmonary Langerhans cell histiocytoma and emphysema. In a trial, enrolling 89 women, those randomized to sirolimus had stable or improved lung function (FEV 1 and DLCO) and improved respiratory symptoms over 12 months of treatment, compared to those taking placebo. After stopping the drug, though, decline in lung function resumed. In an observational series, 12 women with chylous effusions due to LAM, who were treated with sirolimus, had marked improvement or resolution of their effusions. Lung transplantation is an important treatment option for severe cases of LAM. Early referral to a transplant center for those with severe disease seems prudent. Alpha-1-antitrypsin deficiency is a genetic disorder and not acquired—so it is ruled out by a previously normal Alpha-1-antitrypsin level. Idiopathic pulmonary fibrosis has a honeycombing and denser fibrotic changes on CT.
A 60-year-old male is sent to you by his orthopedist for pre-op clearance prior to elective hip arthroplasty. The patient has a history of asthma and is a nonsmoker, though admits to smoking 1 pack a day when he was in his 20s. He is on a steroid inhaler twice a day and reports that he uses his beta-agonist inhaler only once or twice a week. On physical exam, he is well appearing except for a limp in his right leg. His respiratory rate is 16/minute. Examination of the chest is normal with only a few scattered end-expiratory wheezes. The remainder of the physical examination is unremarkable. A peak flow done in the office is 90% of his predicted value. Which of the following is/are the most appropriate step(s) to take prior to your clearing the patient for surgery? a) You refuse to clear him for any elective surgery. b) Spirometry and arterial blood gasses with a carboxyhemoglobin level. c) Spirometry before surgery. d) Pulmonary consultation. e) No pulmonary function tests prior to surgery.
Topic Pulmonary Medicine Answer E. No pulmonary function tests prior to surgery. Explanation Preoperative PFTs and blood gasses are recommended for patients with underlying lung disease who are to undergo operative procedures that are in proximity to the diaphragm. An FEV1 less than 1 liter and arterial blood gases revealing hypoxemia and/or respiratory acidosis would place the patient at a higher risk for postoperative pulmonary complications (prolonged mechanical ventilation, pneumonia, atelectasis). This patient requires no further testing because his asthma is stable and the procedure is not in proximity to the diaphragm.
A 55-year-old male mail carrier is referred to your office from the hospital emergency department. He had been seen there after a fall on the ice, which left him with right-side pain. In the emergency department, he had a chest x-ray done to look for rib fractures. No rib fractures were found, but a 4-mm smooth, well-demarcated peripheral lesion was seen in the left lower lobe. He is a non-smoker and reports to be in good health. He denies any occupational exposures and enjoys the long walks while delivering the mail. His father is alive with hypertension; his mother died of breast cancer. He had a prior chest x-ray 1 year ago for a "rule-out" pneumonia office visit. Which of the following is the most appropriate next step? a) Thoracic surgery. b) Transthoracic needle biopsy. c) Bronchoscopy. d) Order sputums for AFB and fungus. e) Obtain prior chest x-rays for comparison.
Topic Pulmonary Medicine Answer E. Obtain prior chest x-rays for comparison. Explanation When a lung lesion is first identified in a patient, it is imperative to determine whether the lesion is, in fact, new or old. If the lesion is old, you want to evaluate the stability of the lesion (no appreciable growth over 2 years.) Therefore, the first step is to obtain any old films. If there are no prior films, the next step is a high-resolution CT scan of the chest to evaluate the presence of disease. Or, if the old films are suspicious for growth, the CT scan is also indicated. Generally, in most cases your next step after looking at old plain films is to proceed with a high-resolution CT scan. If abnormalities are demonstrated, then bronchoscopy is indicated (if the lesion is amenable) to look for endobronchial involvement and rule out an infectious etiology for the lesion. If bronchoscopy is not helpful, there is some controversy as to whether it is best to perform transthoracic needle biopsy or just go to surgical removal of the lesion. The decision involves whether your patient would tolerate a thoracotomy and whether a non-diagnostic transthoracic needle biopsy (atypia, inflammatory Schmutz) would just mandate a thoracotomy anyway.
A 43-year-old female is referred to your office for an abnormal chest CT. About 3 weeks ago, she had RUQ abdominal pain and underwent an abdominal CT. She was diagnosed with cholecystitis and underwent a laparoscopic cholecystectomy. An incidental finding of an oval-shaped right lower lobe lung nodule was noted. It is characterized as calcified and about 3 x 4 mm in size. A chest CT was then performed that confirmed these findings without any other abnormalities. The patient has a PMH notable for hypertension for which she takes an ACE inhibitor. She is a lifelong nonsmoker and works in an office environment. There is no family history of malignancy. What would you recommend for this patient? a) Referral to a thoracic surgeon for surgical resection of the nodule. b) Obtain a PET scan. c) CT guided biopsy of nodule. d) Repeat chest CT scan every 6 months for a total of 2 years to ensure stability. e) Reassurance with no further follow-up.
Topic Pulmonary Medicine Answer E. Reassurance with no further follow-up. Explanation This patient has the incidental finding of a very small lung nodule. She has no clear risk factors for malignancy, including age < 50, nonsmoker, and no personal or family history of malignancy. Furthermore, the nodule is calcified, which strongly suggests chronicity and a very low likelihood of malignancy. Finally, this nodule is very small. Current guidelines would support optional follow-up for a nodule < 4 mm in a low-risk patient. None of the options of an intervention to either remove or diagnose the nodule would be appropriate. A PET scan would only be indicated for nodules > 8 mm in size in those with moderate-to-high risk factors for malignancy. Repeat chest CT scan in 12 months could be considered for patient reassurance but more frequent surveillance would not be appropriate.
A 50-year-old man was placed on mechanical ventilation yesterday due to a pulmonary embolism that resulted in respiratory failure. It appears he now has developed ARDS. You have managed his ventilator settings adequately and have told the family that he will likely require prolonged mechanical ventilation before he starts to improve. He appears to be stable at 24 hours out. PAST MEDICAL HISTORY: Negative SOCIAL HISTORY: Negative FAMILY HISTORY: Negative REVIEW OF SYSTEMS: Negative (this is a rather boring case for a change) PHYSICAL EXAMINATION: HEENT: PERRLA, EOMI TMs clear Intubated Neck: No masses Heart: RRR with no murmurs, rubs, or gallops Lungs: Coarse BS without other focal findings Abdomen: Bowel sounds present; no hepatosplenomegaly Extremities: No cyanosis, clubbing, or edema Knowing that he will be on the ventilator for a prolonged period of time, which of the following would be an appropriate nutritional intervention? a) Continue NPO status another 24 hours. b) Start parenteral TPN feeds. c) Start peripheral TPN feeds. d) Continue NPO status until you are assured he is stable. e) Start enteral feeding.
Topic Pulmonary Medicine Answer E. Start enteral feeding. Explanation Enteral feeds are preferred over TPN because enteral feeds will maintain the intestinal epithelium and its natural defenses against bacteria. There is no reason to make this patient NPO. Pulmonary ventilation, alveolar gas exchange, or a combination of these 2 elements can affect respiratory compromise
A 45-year-old hospital orderly complains of progressive shortness of breath with ambulation for 2 weeks. Over the past week, he has also developed a nonproductive cough that is worse at night. He is otherwise healthy and does not smoke or use drugs. Lung exam: Symmetric excursions, decreased resonance to percussion, and reduced tactile fremitus at the left base CXR: Left pleural effusion TB skin test: 13 mm Induced sputum shows no acid-fast organisms Sputum NAA is pending. Thoracentesis: pH 7.20 Glucose 67 mg/dL LDH 1,162 IU/L Protein 6.5 g/dL Cells 133 cells/mm3 (30% neutrophils, 60% lymphocytes) Acid-fast smears show no organisms. Which of the following is the most appropriate next step in patient care? a) Intravenous ceftriaxone. b) No treatment and follow-up repeat CXR in 6 weeks. c) Oral prednisone. d) MRI of the chest with thin cuts through the diaphragm. e) Start isoniazid, rifampin, pyrazinamide, and ethambutol.
Topic Pulmonary Medicine Answer E. Start isoniazid, rifampin, pyrazinamide, and ethambutol. Explanation This question tests your knowledge of pleural tuberculosis. The case includes a health care worker with a TB skin test of 12 mm, which is positive (10 mm is the cut-off for a negative test in health care workers). Therefore, at minimum, this man has latent tuberculosis. Next, you must consider how the pleural effusion relates to his positive TB skin test. You are asked to recall that tuberculosis can variably present, and one manifestation is isolated disease of the pleural space. The pleural fluid has a slightly low glucose, lymphocyte predominance, and a pleural fluid LDH value of > 2/3 the normal value of serum (qualifying the fluid as an exudate); hence, the pleural fluid is highly suggestive of tuberculosis. The fact that the acid-fast smears do not show organisms does not exclude tuberculosis and is a rather common occurrence. This patient with a positive TB skin test and a pleural effusion consistent with tuberculosis should be evaluated next according to his risk for true tuberculosis. If he is truly at high risk for TB, he should be treated while undergoing further testing and awaiting cultures. If he is determined to be low risk, he can be observed. This patient would be high risk because he is a health care worker. Therefore, the correct answer is to start 4-drug therapy against tuberculosis. Further workup should include a repeat thoracentesis with multiple samples submitted for AFB smears and cultures and a pleural fluid ADA level. If the ADA level cannot be performed, a pleural biopsy is needed to send tissue for pathology and mycobacterial culture. Ceftriaxone is ineffective against tuberculosis. Prednisone is used in tuberculosis only in cases of CNS disease and TB pericarditis.
A 40-year-old distance runner presents with a new diagnosis of lymphoma. She has been doing well but presents with new-onset pleural effusion. A pleural tap is done and shows white-appearing exudative fluid. Which of the following is causing the fluid to be white? a) The WBC of the fluid is > 1,000, and there are clumped WBCs causing the whitish color. b) The rheumatoid factor of the fluid is high, and the RF causes a whitish color. c) The triglyceride level of the fluid is < 50 mg/dL, and cholesterol causes the whitish color. d) The calcium level of the fluid is > 10 mg/dL, and there are calcium globules causing the whitish color. e) The triglyceride level of the fluid is > 115 mg/dL, and there are chylomicrons causing the whitish color
Topic Pulmonary Medicine Answer E. The triglyceride level of the fluid is > 115 mg/dL, and there are chylomicrons causing the whitish color. Explanation She has a chylous effusion that is associated with leakage of thoracic duct lymph. It is most commonly seen in trauma, lymphoma, mediastinal cancer/fibrosis, and lymphangioleiomyomatosis. Pseudochylous effusions are associated with chronic inflammatory processes such as tuberculosis or rheumatoid arthritis lung disease and have a triglyceride level < 50 mg/dL but an elevated cholesterol level of > 250. In this patient, the white effusion is due to chylomicrons with elevated triglycerides.
Question: True or false: All patients with chronic obstructive pulmonary disease should be prescribed a short-acting bronchodilator?
True.